Chemistry

Ace your homework & exams now with Quizwiz!

(see pic) 9. Quality control results for uric acid are as follows: Results should be reported from: A. Run 1 only B. Runs 1 and 2 C. Runs 1, 2, and 3 D. Runs 1, 2, 3, and 4

9. C Although no single result exceeds the 2s limit, the 41s rule is broken on Run 4. This means that both QC1 and QC2 exceeded +1s on Run 3 and Run 4

412. In what form must a drug be in order to elicit a pharmacologic response? A. Free B. Bound to albumin C. Bound to globulins D. Bound to fatty acids

A. Free

312. In iron-deficiency anemia, what would be the expected percent saturation of transferrin with iron? A. Less than 15 B. Between 30 and 40 C. Between 40 and 50 D. Greater than 55

A. Less than 15

1. Which of the following lamps provides a continuous spectrum of radiant energy in the visible, near IR, and near UV regions of the spectrum? A. Tungsten-filament B. Hydrogen C. Deuterium D. Mercury vapor

A. Tungsten-filament

322. Which of the following is not a colligative property of solutions? A. pH B. Freezing point C. Osmotic pressure D. Vapor pressure

A. pH

189. Ketone bodies are formed because of an excessive breakdown of fatty acids. Of the following metabolites, which may be classified as a ketone body? A. Pyruvic acid B. (3-Hydroxybutyric acid C. Lactic acid D. Oxaloacetic acid

B. (3-Hydroxybutyric acid

375. Diabetes insipidus is associated with depressed secretion of which of the following hormones? A. Prolactin B. Antidiuretic hormone C. Growth hormone D. Oxytocin

B. Antidiuretic hormone

73. When using EMIT, the enzyme is coupled to A. Antibody B. Antigen C. Substrate D. Coenzyme

B. Antigen

275. Which of the following methods employs a reaction where bilirubin is oxidized to colorless biliverdin? A. Bilirubinometer B. Bilirubin oxidase C. High-performance liquid chromatography D. Jendrassik-Grof

B. Bilirubin oxidase

112. Which of the following is an oncofetal antigen that is elevated in nonmucinous epithelial ovarian cancer? A. CA549 B. CA 125 C. CA 19-9 D. CA 15-3

B. CA 125

402. Identification of the urinary metabolite benzoylecgonine would be useful in determining exposure to which of the following drugs? A. Codeine B. Cocaine C. Amphetamine D. Propoxyphene

B. Cocaine

81. Which of the following reagents can be used to measure protein in cerebrospinal fluid? A. Biuret B. Coomassie brilliant blue C. Ponceau S D. Bromcresol green

B. Coomassie brilliant blue

282. Which of the following characterizes Crigler-Najjar syndrome? A. Inability to transport bilirubin from the sinusoidal membrane to the microsomal region B. Deficiency of the enzyme system required for conjugation of bilirubin C. Inability to transport bilirubin glucuronides to the bile canaliculi D. Severe liver cell damage accompanied by necrosis

B. Deficiency of the enzyme system required for conjugation of bilirubin

217. The properties of enzymes are correctly described by which of the following statements? A. Enzymes are stable proteins. B. Enzymes are protein catalysts of biological origin. C. Enzymes affect the rate of a chemical reaction by raising the activation energy needed for the reaction to take place. D. Enzyme activity is not altered by heat denaturation.

B. Enzymes are protein catalysts of biological origin.

338. In acute diabetic ketoacidosis, which of the following laboratory findings would be expected? A. Fasting blood glucose elevated, pH elevated, ketone bodies present B. Fasting blood glucose elevated, pH low, ketone bodies present C. Fasting blood glucose elevated, pH normal, ketone bodies absent D. Fasting blood glucose decreased, pH low, ketone bodies absent

B. Fasting blood glucose elevated, pH low, ketone bodies present

163. Which glucose method can employ a polarographic oxygen electrode? A. Hexokinase B. Glucose oxidase C. Glucose dehydrogenase D. o-Toluidine

B. Glucose oxidase

389. The presence of a very high titer for antithyroglobulin antibodies and the detection of antithyroid peroxidase antibodies is highly suggestive of what disorder? A. Pernicious anemia B. Hashimoto thyroiditis C. Multinodular goiter D. Thyroid adenoma

B. Hashimoto thyroiditis

290. What is the normal renal threshold of sodium (measured in millimoles per liter)? A. 80-85 B. 90-110 C. 110-130 D. 135-148

C. 110-130

358. A 30-year-old woman is admitted to the hospital. She has truncal obesity, buffalo humpback, moon face, purple striae, hypertension, hyperglycemia, increased facial hair, acne, and amenorrhea. The physician orders endocrine testing. The results are as follows: Urine free cortisol—increased Serum cortisol (8 A.M.)—increased Plasma ACTH—decreased Dexamethasone suppression test: Overnight low dose—no suppression of serum cortisol High dose—no suppression of serum cortisol What is the most probable diagnosis? A. Pituitary adenoma B. Ectopic ACTH lung cancer C. Adrenocortical carcinoma D. Addison disease

C. Adrenocortical carcinoma

422. Which of the following serum components is able to alter the free drug level in plasma? A. Creatinine B. Urea C. Albumin D. Calcium

C. Albumin

78. Proteins may become denatured when subjected to mechanical agitation, heat, or extreme chemical treatment. How are proteins affected by denaturation? A. Alteration in primary structure B. Alteration in secondary structure C. Alteration in tertiary structure D. Increase in solubility

C. Alteration in tertiary structure

123. In the Berthelot reaction, what contaminant will cause the urea level to be falsely elevated? A. Sodium fluoride B. Protein C. Ammonia D. Bacteria

C. Ammonia

370. When do the highest levels of gonadotropins occur? A. During the follicular phase of the menstrual cycle B. During the luteal phase of the menstrual cycle C. At the midpoint of the menstrual cycle D. Several days prior to ovulation

C. At the midpoint of the menstrual cycle

229. Which of the following disorders is not associated with an elevation of serum creatine kinase? A. Cerebrovascular accidents B. Hypothyroidism C. Bone disease D. Intramuscular injection

C. Bone disease

103. Portal cirrhosis is a chronic disease of the liver. As observed on an electrophoretic serum protein pattern, what is a predominant characteristic of this disease? A. Monoclonal band in the gammaglobulin region B. Polyclonal band in the gammaglobulin region C. Bridging effect between the beta- and gamma-globulin bands D. Increase in the alpha2-globulin band

C. Bridging effect between the beta- and gamma-globulin bands

89. Which of the following is an anionic dye that binds selectively with albumin? A. Amido black B. Ponceau S C. Bromcresol green D. Coomassie brilliant blue

C. Bromcresol green

245. When an AMI occurs, in what order (list first to last) will the enzymes aspartate aminotransferase (AST), creatine kinase (CK), and lactate dehydrogenase (LD) become elevated in the serum? A. AST, LD, CK B. CK,LD,AST C. CK,AST, LD D. LD, CK,AST

C. CK,AST, LD

131. From what precursor is creatinine formed? A. Urea B. Glucose C. Creatine D. Uric acid

C. Creatine

393. Zollinger-Ellison syndrome is characterized by an elevated blood level of which of the following? A. Trypsin B. Pepsin C. Gastrin D. Cholecystokinin-pancreozymin

C. Gastrin

166. Which of the following glucose methods should not be used during the administration of an oral xylose absorption test? A. Glucose oxidase—colorimetric B. Glucose oxidase—polarographic C. Glucose dehydrogenase D. Hexokinase

C. Glucose dehydrogenase

51. Which type of elution technique may be used in high-performance liquid chromatography? A. Amphoteric B. Isoelectric C. Gradient D. Ion exchange

C. Gradient

101. Which of the following is not characteristic of multiple myeloma? A. Monoclonal band in the gamma region B. Hypercalcemia C. Hyperalbuminemia D. Hyperglobulinemia

C. Hyperalbuminemia

350. Secretion of hormones by the anterior pituitary may be controlled by the circulating levels of hormones from the respective target gland, as well as hormones secreted by what organ? A. Posterior lobe of the pituitary gland B. Intermediate lobe of the pituitary gland C. Hypothalamus D. Adrenal medulla

C. Hypothalamus

205. Which of the following techniques can be used to quantify apolipoproteins? A. Spectrophotometric endpoint B. Ion-selective electrode C. Immunonephelometric assay D. Refractometry

C. Immunonephelometric assay

233. Which of the following statements correctly describes alkaline phosphatase? A. Decreased in Paget disease B. Decreased in third trimester of a normal pregnancy C. Increased in obstructive jaundice D. Primarily found in cardiac muscle

C. Increased in obstructive jaundice

29. Which of the following is false about ion-selective electrode analysis of sodium? A. Uses a glass membrane B. Errors occur from protein buildup on the membrane. C. Membrane coated with valinomycin D. Principle based on potentiometry

C. Membrane coated with valinomycin

221. To what class of enzymes does lactate dehydrogenase belong? A. Isomerases B. Ligases C. Oxidoreductases D. Transferases

C. Oxidoreductases

42. What dye may be used for staining protein bands following electrophoresis? A. Fat red 7B B. Sudan blackB C. Ponceau S D. Oil redO

C. Ponceau S

306. Secondary hyperparathyroidism is often the result of A. Vitamin C deficiency B. Liver disease C. Renal disease D. Thyroid disease

C. Renal disease

415. Which of the following drugs is used as an immunosuppressant in organ transplantation, especially in liver transplants? A. Methotrexate B. Amiodarone C. Tacrolimus D. Paroxetine

C. Tacrolimus

27. When a pH-sensitive glass electrode is not actively in use, in what type of solution should it be kept? A. Tap water B. Physiologic saline solution C. The medium recommended by the manufacturer D. A buffer solution of alkaline pH

C. The medium recommended by the manufacturer

403. Of the following specimens, which would be appropriate for determining exposure to lead? A. EDTA plasma B. Serum C. Whole blood D. Cerebrospinal fluid

C. Whole blood

247. If elevated, which laboratory test would support a diagnosis of congestive heart failure? A. Homocysteine B. Troponin C. Albumin cobalt binding D. B-type natriuretic peptide

D. B-type natriuretic peptide

107. In which of the following disorders would the maternal serum level of aj-fetoprotein not be elevated? A. Neural tube defect B. Spinabifida C. Fetal distress D. Down syndrome

D. Down syndrome

197. The surfactant/albumin ratio by fluorescence polarization is performed to assess what physiological state? A. Hyperlipidemia B. Coronary artery disease C. Hemolytic disease of the newborn D. Fetal lung maturity

D. Fetal lung maturity

186. The kinetic methods for quantifying serum triglyceride employ enzymatic hydrolysis. The hydrolysis of triglyceride may be accomplished by what enzyme? A. Amylase B. Leucine aminopeptidase C. Lactate dehydrogenase D. Lipase

D. Lipase

68. In addition to performing hemoglobin electrophoresis, a solubility test may be performed to detect the presence of what hemoglobin? A. A, B. C C. F D. S

D. S

160. Which of the following analytes would not commonly be measured when monitoring complications of diabetes mellitus? A. Serum urea nitrogen B. Urinary albumin C. Serum creatinine D. Serum bilirubin

D. Serum bilirubin

378. Which of the following functions as an inhibiting factor for somatotropin release? A. Gonadotropin-releasing hormone B. Growth hormone-releasing hormone C. Somatomedin D. Somatostatin

D. Somatostatin

96. Which of the following is an acute-phase reactant protein able to inhibit enzymatic proteolysis and having the highest concentration of any of the plasma proteolytic inhibitors? A. C-reactive protein B. Haptoglobin C. ct2-Macroglobulin D. cq-Antitrypsin

D. cq-Antitrypsin

58. Which of the following is a characteristic of obstructive jaundice? A. The ratio of direct to total bilirubin is greater than 1:2 B. Conjugated bilirubin is elevated, but unconjugated bilirubin is normal C. Urinary urobilinogen is increased D. Urinary bilirubin is normal

58. A Obstruction prevents conjugated bilirubin from reaching the intestine, resulting in decreased production, excretion, and absorption of urobilinogen. Conjugated bilirubin regurgitates into sinusoidal blood and enters the general circulation via the hepatic vein. The level of serum direct (conjugated) bilirubin becomes greater than unconjugated bilirubin. The unconjugated form is also increased because of accompanying necrosis, deconjugation, and inhibition of UDP-glucuronyl transferase

58. The most commonly used detector for clinical gas-liquid chromatography (GLC) is based upon: A. Ultraviolet light absorbance at 254 nm B. Flame ionization C. Refractive index D. Thermal conductance

58. B Volatile solutes can be detected in GLC using flame ionization, thermal conductivity, electron capture, and mass spectroscopy. In flame ionization, energy from a flame is used to excite the analytes as they elute from the column. The flame is made by igniting a mixture of hydrogen, carrier gas, and air. Current is produced when an outer shell electron is ejected from the excited analyte

58. Which isoenzyme of ALP migrates farthest toward the anode when electrophoresed at pH 8.6? A. Placental B. Bone C. Liver D. Intestinal

58. C Liver ALP isoenzymes migrate farthest toward the anode, but fast and slow variants occur. The slow liver ALP band is difficult to distinguish from placental and bone ALP. The order from cathode to anode is: - Renal→Intestinal→Bone→Placental→Liver + Improved separation of bone and liver isoenzymes can be achieved by incubating the serum with neuraminidase prior to electrophoresis. The enzyme reduces the sialic content of the bone isoenzyme, causing it to migrate at a slower rate.

58. Which of the following diseases is caused by a deficiency of sphingomyelinase? A. Gaucher disease B. Fabry disease C. Niemann-Pick disease D. Tay-Sachs disease

58. C The diseases mentioned result from inborn errors of lipid metabolism (lipidoses) caused by deficiency of an enzyme needed for lipid degradation. Specific lipids accumulate in the lysosomes. Niemann-Pick disease results from a deficiency of sphingomyelinase; Gaucher disease from β-glucocerebrosidase; Fabry disease (sex linked) from α-galactosidase A; and Tay-Sachs from N acetylglucosaminidase A.

59. Which of the following would cause an increase in only the unconjugated bilirubin? A. Hemolytic anemia B. Obstructive jaundice C. Hepatitis D. Hepatic cirrhosis

59. A Conjugated bilirubin increases as a result of obstructive processes within the liver or biliary system or from failure of the enterohepatic circulation. Hemolytic anemia (prehepatic jaundice) presents a greater bilirubin load to a normal liver, resulting in increased bilirubin excretion. When the rate of bilirubin formation exceeds the rate of excretion, the unconjugated bilirubin rises.

59. Which method is considered the candidate reference method for triglyceride measurement? A. Glycerol kinase-ultraviolet B. CDC modification of van Handel and Zilversmit C. Hantzsch condensation D. Glycerol kinase coupled to peroxidase Chemistry/Apply principles of basic laboratory

59. B Enzymatic methods for triglyceride measurement are widely used because they eliminate the need for extraction and saponification. However, they are subject to positive interference from endogenous glycerol and variations in the efficiency of lipase, which can result in under- or overestimation of triglycerides.

59. What type of detector is used in high-performance liquid chromatography with electrochemical detection (HPLC-ECD)? A. Calomel electrode B. Conductivity electrode C. Glassy carbon electrode D. Polarographic electrode

59. C HPLC-ECD uses a glassy carbon measuring electrode and a silver-silver chloride reference. The analyte is oxidized or reduced by holding the glassy carbon electrode at a positive voltage (oxidization) or negative voltage (reduction). The resulting current flow is directly proportional to concentration. Phenolic groups such as catecholamines can be measured by HPLC-ECD.

6. Which of the following statements is true? A. Apoenzyme + prosthetic group = holoenzyme B. A coenzyme is an inorganic molecule required for activity C. Cofactors are as tightly bound to the enzyme as prosthetic groups D. All enzymes have optimal activity at pH 7.00

6. A A coenzyme is an organic molecule required for full enzyme activity. A prosthetic group is a coenzyme that is tightly bound to the apoenzyme and is required for activity. Cofactors are inorganic atoms or molecules needed for full catalytic activity. Pyridoxyl- 5´-phosphate is a prosthetic group for ALT and AST.

6. SITUATION: A technologist is performing an enzyme assay at 340 nm using a visible-range spectrophotometer. After setting the wavelength and adjusting the readout to zero %T with the light path blocked, a cuvette with deionized water is inserted. With the light path fully open and the 100%T control at maximum, the instrument readout will not rise above 90%T. What is the most appropriate first course of action? A. Replace the source lamp B. Insert a wider cuvette into the light path C. Measure the voltage across the lamp terminals D. Replace the instrument fuse Chemistry/Select course of action/

6. A Visible spectrophotometers are usually supplied with a tungsten or quartz halogen source lamp. Tungsten lamps produce a continuous range of wavelengths from about 320-2,000 nm. Output increases as wavelength becomes longer peaking at around 1,000 nm, and is poor below 400 nm. As the lamp envelope darkens with age, the amount of light reaching the photodetector at 340 nm becomes insufficient to set the blank reading to 100%T. Quartz halogen lamps produce light from 300 nm through the infrared region. Deuterium or hydrogen lamps produce ultraviolet-rich spectra optimal for ultraviolet (UV) work. Mercury vapor lamps produce a discontinuous spectrum that includes a high output at around 365 nm that is useful for fluorescent applications. Xenon lamps generate a continuous spectrum of fairly uniform intensity from 300-2,000 nm, making them useful for both visible and UV applications.

6. Select the main estrogen produced by the ovaries and used to evaluate ovarian function. A. Estriol (E3 ) B. Estradiol (E2 ) C. Epiestriol D. Hydroxyestrone

6. B E2 is the major estrogen produced by the ovaries and gives rise to both estrone (E1) and E3. E2 is used to evaluate both ovarian function and menstrual cycle dysfunction

6. The modification of diet in renal disease (MDRD) formula for calculating eGFR requires which four parameters? A. Urine creatinine, serum creatinine, height, weight B. Serum creatinine, age, gender, race C. Serum creatinine, height, weight, age D. Urine creatinine, gender, weight, age

6. B The National Kidney Foundation recommends screening for chronic kidney disease using the estimated glomerular filtration rate (eGFR) because of the high frequency of sample collection errors associated with measuring creatinine clearance. The eGFR should be calculated according to the MDRD formula, and reported along with the serum or plasma creatinine

6. What is the PCO2 if the dCO2 is 1.8 mmol/L? A. 24 mm Hg B. 35 mm Hg C. 60 mm Hg D. 72 mm Hg

6. C Dissolved CO2 is calculated from the measured PCO2 × 0.0306, the solubility coefficient for CO2 gas in blood at 37°C. dCO2 = PCO2 × 0.03 Therefore, PCO2 = dCO2 /0.03 PCO2 = 1.8 mmol/L ÷ 0.03 mmol/ L per mm Hg = 60 mm Hg

6. Convert 2.0 mEq/L magnesium (atomic weight = 24.3) to milligrams per deciliter. A. 0.8 mg/dL B. 1.2 mg/dL C. 2.4 mg/dL D. 4.9 mg/dL

6. C To convert from milliequivalent per liter to milligrams per deciliter, first calculate the milliequivalent weight (equivalent weight expressed in milligrams), which is the atomic mass divided by the valence. Because magnesium is divalent, each mole has the charge equivalent of 2 mol of hydrogen. Then, multiply the milliequivalent per liter by the milliequivalent weight to convert to milligrams per liter. Next, divide by 10 to convert milligrams per liter to milligrams per deciliter. Milliequivalent weight Mg = 24.3 ÷ 2 = 12.15 mg/mEq 2.0 mEq/L × 12.15 mg/mEq = 24.3 mg/L 24.3 mg/L × 1.0 L/10.0 dL = 2.4 mg/dL

60. Which of the following statements regarding ALP is true? A. All isoenzymes of ALP are antigenically distinct and can be identified by specific antibodies B. Highest serum levels are seen in intrahepatic obstruction C. Elevated serum ALP seen with elevated GGT suggests a hepatic source D. When jaundice is present, an elevated ALP suggests acute hepatitis

60. C ALP isoenzymes can result from different genes or from modification of a common gene product in the tissues. Some differ mainly in carbohydrate content and cannot be identified by immunologic methods. Highest levels of ALP are seen in Paget's disease of bone, where ALP can be as high as 25 times the URL

60. Which of the following enzymes is common to all enzymatic methods for triglyceride measurement? A. Glycerol phosphate oxidase B. Glycerol phosphate dehydrogenase C. Glycerol kinase D. Pyruvate kinase

60. C All enzymatic triglyceride methods require lipase to hydrolyze triglycerides, and glycerol kinase to phosphorylate glycerol, forming glycerol-3-phosphate. The most common method couples glycerol kinase with glycerol phosphate oxidase and peroxidase. 1. Triglyceride + H2O Lipase glycerol + fatty acids 2. Glycerol + ATP GK glycerol-3-phosphate + ADP 3. Glycerol-3-phosphate + O2 GPO dihydroxyacetone phosphate + H2O2 H2O2 + phenol + 4-aminophenazone Px quinoneimine dye + H2O GK = glycerol kinase; GPO = glycerol phosphate oxidase; Px = peroxidase

60. Which form of hyperbilirubinemia is caused by an inherited absence of UDP-glucuronyl transferase? A. Gilbert's syndrome B. Rotor syndrome C. Crigler-Najjar syndrome D. Dubin-Johnson syndrome

60. C Crigler-Najjar syndrome is a rare condition that occurs in two forms. Type 1 is inherited as an autosomal recessive trait and causes a total deficiency of UDP-glucuronyl transferase

61. Select the chemical that is used in most HPLC procedures to decrease solvent polarity. A. Hexane B. Nonane C. Chloroform D. Acetonitrile

61. D All of the compounds mentioned have nonpolar properties. Because most HPLC is reverse phase (a polar solvent is used), hexane and nonane are too nonpolar. Acetonitrile is more polar and less toxic than chloroform and along with methanol is a common polarity modifier for HPLC

62. Which of the following conditions is associated with hypernatremia? A. Diabetes insipidus B. Hypoaldosteronism C. Burns D. Diarrhea

62. A Diabetes insipidus results from failure to produce ADH. Because the collecting tubules are impermeable to water in the absence of ADH, severe hypovolemia and dehydration result. Hypovolemia stimulates aldosterone release, causing sodium reabsorption, which worsens the hypernatremia. Burns, hypoaldosteronism, diarrhea, and diuretic therapy are common causes of hyponatremia.

62. Which statement best characterizes serum bilirubin levels in the first week following delivery? A. Serum bilirubin 24 hours after delivery should not exceed the upper reference limit for adults B. Jaundice is usually first seen 48-72 hours postpartum in neonatal hyperbilirubinemia C. Serum bilirubin above 5.0 mg/dL occurring 2-5 days after delivery indicates hemolytic or hepatic disease D. Conjugated bilirubin accounts for about 50% of the total bilirubin in neonates

62. B Bilirubin levels may reach as high as 2-3 mg/dL in the first 24 hours after birth owing to the trauma of delivery, such as resorption of a subdural hematoma

64. Cholesterol esterase is used in enzymatic assays to: A. Oxidize cholesterol to form peroxide B. Hydrolyze fatty acids bound to the third carbon atom of cholesterol C. Separate cholesterol from apoproteins A-I and A-II by hydrolysis D. Reduce NAD+ to NADH

64. B Approximately two-thirds of the serum cholesterol has a fatty acid esterified to the hydroxyl group of the third carbon atom of the cholesterol molecule. Cholesterol esterase hydrolyzes fatty acids and is required because cholesterol oxidase cannot utilize esterified cholesterol as a substrate.

64. Which of the following buffers is used in the IFCC recommended method for ALP? A. Glycine B. Phosphate C. 2-Amino-2-methyl-1-propanol D. Citrate

64. C The Szasz modification of the Bowers-McComb method measures the hydrolysis of p-nitrophenyl phosphate, and continuously monitors the formation of p-nitrophenol at 405 nm. AMP buffer chelates phosphorus, preventing product inhibition; Zn+2 and Mg+2 are added to the substrate to activate ALP. HEDTA is used to chelate the excess Zn+2, which is inhibitory at high concentrations

64. A lab measures total bilirubin by the Jendrassik-Grof bilirubin method with sample blanking. What would be the effect of moderate hemolysis on the test result? A. Falsely increased due to optical interference B. Falsely increased due to release of bilirubin from RBCs C. Falsely low due to inhibition of the diazo reaction by hemoglobin D. No effect due to correction of positive interference by sample blanking

64. C The sample blank measures the absorbance of the sample and reagent in the absence of azobilirubin formation and corrects the measurement for optical interference caused by hemoglobin absorbing the wavelength of measurement.

65. Which of the following conditions is associated with hyponatremia? A. Diuretic therapy B. Cushing's syndrome C. Diabetes insipidus D. Nephrotic syndrome

65. A Diuretics lower blood pressure by promoting water loss. This is accomplished by causing sodium loss from the proximal tubule and/or loop. Addison's disease, syndrome of inappropriate ADH release, burns, diabetic ketoacidosis, hypopituitarism, vomiting, diarrhea, and cystic fibrosis also cause hyponatremia. Cushing's syndrome causes hypernatremia by promoting sodium reabsorption in the collecting tubule in exchange for potassium

65. Which of the following reagents is used in the direct HDL cholesterol method? A. Sulfated cyclodextrin B. Magnesium sulfate and dextran sulfate C. Anti-apoA-I D. Manganese heparin

65. A The direct HDL cholesterol method most commonly employed uses cholesterol esterase and oxidase enzymes conjugated to polyethylene glycol. In the presence of sulfated cyclodextrin, the enzymes do not react with non-HDL cholesterol molecules. Anti-apoA-I binds to HDL and is not used in HDL assays.

65. Which reagent is used in the Jendrassik-Grof method to solubilize unconjugated bilirubin? A. 50% methanol B. N-butanol C. Caffeine D. Acetic acid

65. C A polarity modifier is required to make unconjugated bilirubin soluble in diazo reagent. The Malloy-Evelyn method uses 50% methanol to reduce the polarity of the diazo reagent

65. What is the confirmatory method for measuring drugs of abuse? A. HPLC B. Enzyme-multiplied immunoassay technique (EMIT) C. Gas chromatography with mass spectroscopy (GC-MS) D. TLC Chemistry/

65. C GC-MS determines the mass spectrum of the compounds eluting from the analytic column. Each substance has a unique and characteristic spectrum of mass fragments. This spectrum is compared to spectra in a library of standards to determine the percent match. A match of greater than 95% is considered confirmatory

66. The fragments typically produced and analyzed in methods employing mass spectroscopy are typically: A. Of low molecular size ranging from 10-100 daltons B. Cations caused by electron loss or proton attachment C. Anions caused by bombarding the molecule with an electron source D. Neutral species formed after excited molecules form a stable resonance structure

66. B In almost all MS applications, cations of the molecule are measured. Cations can be formed by various methods, the most common of which is electron bombardment (electron ionization). The energy transferred to the molecule causes ejection of an outer shell electron. MS can analyze sizes from trace metals through macromolecules. Proteins are measured following conversion to cations by ionization procedures such as matrix-assisted laser desorption ionization (MALDI) in which energy from a nitrogen laser causes transfer of a proton from the matrix (an acid) to the protein.

66. In which condition is the measurement of acid phosphatase clinically useful? A. Measuring the prostatic isoenzyme to screen for prostate cancer B. Measuring the enzyme in a vaginal swab extract C. The diagnosis of hemolytic anemia D. As a marker for bone regeneration

66. B The PSA test is clinically more sensitive than prostatic acid phosphatase in detecting prostatic cancer. The clinical use of prostatic acid phosphatase is confined to the investigation of sexual assault. Acid phosphatase activity > 50 IU/L establishes the presence of seminal fluid in the vaginal sample

66. What do "direct" or homogenous methods for LDL cholesterol assay have in common? A. They are inaccurate when plasma triglyceride is above 250 mg/dL B. All use a detergent to facilitate selective reactivity with reagent enzymes C. All use monoclonal antibodies to apo A1 and C D. All are free of interference from abnormal lipoproteins

66. B The direct LDL cholesterol assays are all detergent based methods. One commonly used method employs a polyanionic detergent to release cholesterol from HDL, chylomicrons, and VLDL. The detergent binds to LDL and blocks its reaction with the esterase and oxidase enzymes in the reagent. Cholesterol oxidase oxidizes the non-LDL cholesterol, forming H2O2, and peroxidase catalyzes the oxidation of an electron donor by the H2O2,which does not result in color formation

66. Which statement about colorimetric bilirubin methods is true? A. Direct bilirubin must react with diazo reagent under alkaline conditions B. Most methods are based upon reaction with diazotized sulfanilic acid C. Ascorbic acid can be used to eliminate interference caused by Hgb D. The color of the azobilirubin product is independent of pH

66. B Unconjugated bilirubin is poorly soluble in acid, and therefore, direct bilirubin is assayed using diazotized sulfanilic acid diluted in weak HCl

66. Which of the following conditions involving electrolytes is described correctly? A. Pseudohyponatremia occurs only when undiluted samples are measured B. Potassium levels are slightly higher in heparinized plasma than in serum C. Hypoalbuminemia causes low total calcium but does not affect Cai D. Hypercalcemia may be induced by low serum magnesium

66. C When serum albumin is low, the equilibrium between bound and Cai is shifted, producing increased Cai. This inhibits release of PTH by negative feedback until the Cai level returns to normal. Potassium is released from platelets and leukocytes during coagulation, causing serum levels to be higher than plasma. Pseudohyponatremia is a measurement error caused by diluting samples containing excessive fat or protein. The colloids displace plasma water, resulting in less electrolytes being delivered into the diluent

67. Which definition best describes the catalytic activity of amylase? A. Hydrolyzes second α 1-4 glycosidic linkages of starch, glycogen, and other polyglucans B. Hydrolyzes all polyglucans completely to produce glucose C. Oxidatively degrades polysaccharides containing glucose D. Splits polysaccharides and disaccharides by addition of water

67. A Amylase in humans is a hydrolase that splits the second α 1-4 glycosidic bonds of polyglucans forming maltose. There are two major types of amylase: P-type derived from the pancreas and S-type derived from the salivary glands. These can be differentiated by both electrophoresis and immunoassay

67. Which of the following laboratory results is usually associated with cystic fibrosis? A. Sweat chloride greater than 60 mmol/L B. Elevated serum sodium and chloride C. Elevated fecal trypsin activity D. Low glucose

67. A Cystic fibrosis causes obstruction of the exocrine glands including the sweat glands, mucus glands, and pancreas. Newborns with pancreatic involvement demonstrate fecal trypsin deficiency, which may be detected by a low fecal chymotrypsin or immunoreactive trypsin result. However, these tests require confirmation. Serum sodium and chloride levels are low. More

67. Lipoprotein (a), or Lp(a), is significant when elevated in serum because it: A. Is an independent risk factor for atherosclerosis B. Blocks the clearance of VLDLs C. Displaces apo-AI from HDLs D. Is linked closely to a gene for obesity

67. A Lp(a) is a complex of apo-B100 and protein (a) formed by a disulfide bridge. The complex is structurally similar to plasminogen and is thought to promote coronary heart disease by interfering with the normal fibrinolytic process. Lp(a) is measured by immunoassay; however, the measurement will vary depending on the type of antibodies used and their epitope specificity.

67. What component is used in a GC-MS but not used in an LC-MS? A. Electron source B. Mass filter C. Detector D. Vacuum

67. A The mass spectrometer requires a sample that is suspended in a gas phase, and therefore, the sample from a GC can be directly injected into the mass spectrometer. While chemical ionization of the sample is possible, most GC-MS instruments utilize electron ionization. Electrons are produced by applying 70 electron volts to a filament of tungsten or rhenium under vacuum. The electrons collide with the neutral molecules coming from the GC, splitting them into fragments. The array of fragments is a unique identifier of each molecule

67. Which statement regarding the measurement of bilirubin by the Jendrassik-Grof method is correct? A. The same diluent is used for both total and direct assays to minimize differences in reactivity B. Positive interference by Hgb is prevented by the addition of HCl after the diazo reaction C. The color of the azobilirubin product is intensified by the addition of ascorbic acid D. Fehling's reagent is added after the diazo reaction to reduce optical interference by hemoglobin

67. D The Jendrassik-Grof method uses HCl as the diluent for the measurement of direct bilirubin because unconjugated bilirubin is poorly soluble at low pH. Total bilirubin is measured using an acetate buffer with caffeine added to increase the solubility of the unconjugated bilirubin

68. What process is most often used in LC-MS to introduce the sample into the mass filter? A. Electrospray ionization B. Chemical ionization C. Electron impact ionization D. Fast atom bombardment

68. A HPLC instruments use solvent rather than gas to separate molecules. The sample is converted into a gaseous state by electrospray ionization before it enters the mass filter. Electrospray ionization uses a small-bore tube that forms a 1-4 μ nozzle at the mass filter inlet and which is charged by several kilovolts. The sample enters the tube along with inert drying gas. The tube is heated to help evaporate solvent, but unlike electron impact used in GC-MS, the ionizer is not under vacuum. When a droplet of the sample reaches the nozzle, it becomes highly charged. The size of the droplet is decreased owing to evaporation. This causes the charge density to become excessive, and the droplets break apart. The tiny charged droplets repel each other and break apart again, forming a plume.

68. A neonatal bilirubin assay performed at the nursery by bichromatic direct spectrophotometry is 4.0 mg/dL. Four hours later, a second sample assayed for total bilirubin by the Jendrassik-Grof method gives a result of 3.0 mg/dL. Both samples are reported to be hemolyzed. What is the most likely explanation of these results? A. Hgb interference in the second assay B. δ-Bilirubin contributing to the result of the first assay C. Falsely high results from the first assay caused by direct bilirubin D. Physiological variation owing to premature hepatic microsomal enzymes

68. A The Jendrassik-Grof method is based upon a diazo reaction that may be suppressed by Hgb. Because serum blanking and measurement at 600 nm correct for positive interference from Hgb, the results may be falsely low when significant hemolysis is present

68. Which type of dietary fatty acid is not associated with an increase in serum LDL cholesterol production? A. Monounsaturated trans fatty acids B. Saturated fatty acids C. Monounsaturated cis fatty acids D. Monounsaturated trans Ω-9 fatty acids

68. C Polyunsaturated and cis monounsaturated fatty acids are not associated with increased production of LDL cholesterol. On the other hand, saturated and trans monounsaturated fatty acids are both associated with increased LDL. Cis fatty acids are those in which the H atoms belonging to the double-bonded carbons are on the same side of the molecule

68. When performing a sweat chloride collection, which of the following steps will result in analytical error? A. Using unweighed gauze soaked in pilocarpine nitrate on the inner surface of the forearm to stimulate sweating B. Collecting more than 75 mg of sweat in 30 minutes C. Leaving the preweighed gauze on the inside of the arm exposed to air during collection D. Rinsing the collected sweat from the gauze pad using chloride titrating solution

68. C The sweat chloride procedure requires the application of pilocarpine to stimulate sweating, and the use of iontophoresis (application of 0.16-mA current for 5 minutes) to bring the sweat to the surface. After iontophoresis, the skin on the inner surface of the forearm is washed with deionized water and dried, and a preweighed pair of 2-in.2 pads is taped to the skin. During the 30-minute collection of sweat, the gauze must be completely covered to prevent contamination and loss of sweat by evaporation.

69. Which electrolyte level best correlates with plasma osmolality? A. Sodium B. Chloride C. Bicarbonate D. Calcium

69. A Sodium and chloride are the major extracellular ions. Chloride passively follows sodium, making sodium the principal determinant of plasma osmolality.

69. SITUATION: A lipemic specimen collected from an adult after a 12-hour fast was assayed for total cholesterol, triglycerides, and HDL cholesterol using a direct HDL method. Following are the results: Total cholesterol = 220 mg/dL HDL cholesterol = 40 mg/dL Triglyceride = 420 mg/dL The physician requests an LDL cholesterol assay after receiving the results. How should the LDL cholesterol be determined? A. Dilute the specimen 1:10 and repeat all tests; calculate LDL cholesterol using the Friedewald equation B. Perform a direct LDL cholesterol assay C. Ultracentrifuge the sample and repeat the HDL cholesterol on the infranate. Use the new result to calculate the LDL cholesterol D. Repeat the HDL cholesterol using the manganese heparin precipitation method. Use the new result to calculate the LDL cholesterol

69. B An accurate LDL cholesterol can be reported, if the direct (detergent) method for LDL cholesterol is employed. These methods are not subject to interference by triglycerides at a concentration below 700 mg/dL

69. How soon following acute abdominal pain due to pancreatitis is the serum amylase expected to rise? A. 1-2 hours B. 2-12 hours C. 3-4 days D. 5-6 days Chemistry/Correlate

69. B Serum amylase usually peaks 2-12 hours following acute abdominal pain resulting from pancreatitis. Levels reach 2-6 times the URL and return to normal within 3-4 days. Urinary amylase peaks concurrently with serum but rises higher and remains elevated for up to 1 week.

69. In mass spectroscopy, the term base peak typically refers to: A. The peak with the lowest mass B. The peak with the most abundance C. A natural isotope of the molecular ion D. The first peak to reach the mass detector

69. B The base peak is typically the "molecular ion" or parent ion, meaning that it is the initial fragment made by releasing an electron. The cation thus formed has a charge of +1, and therefore, its m/z ratio is equal to its mass. The base peak is used for selective ion monitoring (SIM). It is the most abundant and most stable ion, and gives the best sensitivity for quantitative analysis

7. A method calls for extracting an acidic drug from urine with an anion exchange column. The pKa of the drug is 6.5. Extraction is enhanced by adjusting the sample pH to: A. 8.5 B. 6.5 C. 5.5 D. 4.5

7. A Extraction of a negatively charged drug onto an anion exchange (positively charged) column is optimal when more than 99% of the drug is in the form of anion. The extraction pH should be 2 pH units above the pKa of an acidic drug. When pH = pKa the drug will be 50% ionized, and when pH is greater than pKa the majority of drug is anionic

7. High serum total protein but low albumin is usually seen in: A. Multiple myeloma B. Hepatic cirrhosis C. Glomerulonephritis D. Nephrotic syndrome

7. A In multiple myeloma, synthesis of large quantities of monoclonal immunoglobulin by plasma cells often results in decreased synthesis of albumin. In glomerulonephritis and nephrotic syndrome, both total protein and albumin are low owing to loss of proteins through the glomeruli. In hepatic cirrhosis, decreased hepatic production of protein results in low total protein and albumin.

7. Which statement best describes the relationship between luteinizing hormone (LH) and folliclestimulating hormone (FSH) in cases of dysmenorrhea? A. Both are usually increased when there is pituitary adenoma B. Increases in both hormones and a decrease in estrogen signal a pituitary cause of ovarian failure C. Both hormones normally peak 1-2 days before ovulation D. In menopause, the LH level at the midcycle peak is higher than the level of FSH

7. C In women, serum or urine LH and FSH are measured along with estrogen and progesterone to evaluate the cause of menstrual cycle abnormalities and anovulation. Both hormones show a pronounced serum peak 1-2 days prior to ovulation and urine peak 20-44 hours before ovulation. Normally, the LH peak is sharper and greater than the FSH peak; however, in menopause, the FSH usually becomes higher than LH.

7. Which type of monochromator produces the purest monochromatic light in the UV range? A. A diffraction grating and a fixed exit slit B. A sharp cutoff filter and a variable exit slit C. Interference filters and a variable exit slit D. A prism and a variable exit slit

7. D Diffraction gratings and prisms both produce a continuous range of wavelengths. A diffraction grating produces a uniform separation of wavelengths. A prism produces much better separation of high-frequency light because refraction is greater for higher-energy wavelengths. Instruments using a prism and a variable exit slit can produce UV light of a very narrow bandpass. The adjustable slit is required in order to allow sufficient light to reach the detector to set 100%T

70. Which of the following statements regarding the diagnosis of pancreatitis is correct? A. Amylase and lipase are as predictive in chronic as in acute pancreatitis B. Diagnostic sensitivity is increased by assaying both amylase and lipase C. Measuring the urinary amylase:creatinine ratio is useful only when patients have renal failure D. Serum lipase peaks several hours before amylase after an episode of acute pancreatitis Chemistry/Correlate clinical and laboratory

70. B Amylase is not increased in all persons with pancreatitis and can be increased in several nonpancreatic conditions. Lipase adds both sensitivity and specificity to the diagnosis of acute pancreatitis. Plasma or serum lipase becomes abnormal within 6 hours, peaks at approximately 24 hours, and remains abnormal for about 1 week following an episode of acute pancreatitis.

70. Which formula is most accurate in predicting plasma osmolality? A. Na + 2(Cl) + BUN + glucose B. 2(Na) + 2(Cl) + glucose + urea C. 2(Na) + (glucose ÷ 18) + (BUN ÷ 2.8) D. Na + Cl + K + HCO3

70. C Calculated plasma osmolality is based upon measurement of sodium, glucose, and urea. Because sodium associates with a counter ion, two times the sodium estimates the millimoles per liter of electrolytes. Some laboratories multiply by 1.86 instead of 2 to correct for undissociated salts. Dividing glucose by 18 converts from milligrams per deciliter to millimoles per liter. Dividing blood urea nitrogen (BUN) by 2.8 converts from milligrams per deciliter BUN to millimoles per liter urea

70. Which method is the most useful when screening for errors of amino and organic acid metabolism? A. Two-dimensional thin-layer chromatography B. Gas chromatography C. Electrospray ionization tandem-mass spectroscopy D. Inductively charged coupled-mass spectroscopy Chemistry/Select instruments to perform test/Newborn

70. C While two-dimensional thin-layer chromatography can separate both amino and organic acids, it is not sufficiently sensitive for newborn screening. Electrospray ionization allows a small alcohol-extracted whole-blood sample to be analyzed by two mass spectrometers without prior separation by liquid or gas chromatography. Disorders of both organic and fatty acid metabolism are identified by the specific pattern of acylcarnitine ions produced. Amino acids are detected as amino species that have lost a carboxyl group during ionization, a process called neutral loss.

71. Which of the following conditions is associated with a high level of S-type amylase? A. Mumps B. Intestinal obstruction C. Alcoholic liver disease D. Peptic ulcers

71. A Both salivary and pancreatic amylases designated S-type and P-type, respectively, are present in normal serum. High amylase occurs in mumps, ectopic pregnancy, biliary obstruction, peptic ulcers, alcoholism, malignancies, and other nonpancreatic diseases. Isoenzymes can be separated by electrophoresis (S-type is faster than P-type), but more commonly immunoinhibition of S-type amylase is used to rule out mumps, malignancy, and ectopic pregnancy, which give rise to high S-type amylase.

71. In tandem-mass spectroscopy, the first mass filter performs the same function as: A. The ion source B. The chromatography column C. Extraction D. The vacuum system

71. B A tandem mass spectrometer uses two or more mass filters in sequence. The first filter functions as an ion trap. Once the sample is ionized, the filter selects molecular or parent ions of interest by excluding ions outside a specified size range. Therefore, it effectively separates the analyte(s) of interest from unwanted compounds. Tandem MS uses ESI to introduce the sample into the first mass filter, usually a quadrapole. The RF and DC voltages of the quadrapole are set to optimize the trajectory of the parent ions of interest and cause ejection of unwanted ions. The parent ions are drawn into a second mass filter where they are bombarded by argon atoms. The

72. SITUATION: A GC-MS analysis using nitrogen as the carrier gas shows an extensively noisy baseline. A sample of the solvent used for the extraction procedure, ethyl acetate, was injected and showed the same noise. Results of an Autotune test showed the appearance of a base peak at 16 with two smaller peaks at 17 and 18. These results indicate: A. The solvent is contaminated B. The carrier gas is contaminated C. There is electrical noise in the detector D. The ion source is dirty

72. B All of these situations are sources of baseline noise in GC-MS. However, the peak at 16 indicates the presence of oxygen in the carrier gas. Oxygen in the atmosphere also contains small quantities of two isotopes with molecular weights of 17 and 18 owing to one and two extra neutrons, respectively

73. Why is vacuum necessary in the mass filter of a mass spectrometer? A. Ionization does not occur at atmospheric pressure B. It prevents collision between fragments C. It removes electrons from the ion source D. It prevents contamination

73. B Vacuum is needed in the mass filter of the MS to prevent random collisions between ions that would alter their trajectory or time of flight. It is also needed in CG-MS instruments that use electron ionization. The vacuum prevents collision between the carrier gas molecules and the ions. In spectrometers that use electrospray ionization, chemical ionization, and laser desorption ionization (MALDI and SELDI TOF), the ion source is not under vacuum

73. Which of the following statements regarding amylase methods is true? A. Dilution of serum may result in lower than expected activity B. Methods generating NADH are preferred because they have higher sensitivity C. Synthetic substrates can be conjugated to p-nitrophenol (PNP) for a kinetic assay D. The reference range is consistent from method to method

73. C Many endogenous inhibitors of amylase, such as wheat germ, are found in serum. Diluted samples often show higher than expected activity caused by dilution of the inhibitor. Units of amylase activity vary widely depending upon the method of assay and calibration. Synthetic substrates such as maltotetrose or 4-nitrophenyl maltohepatoside can be used for kinetic assays. Maltotetrose

74. The reference method for lipase uses olive oil as the substrate because: A. Other esterases can hydrolyze triglyceride and synthetic diglycerides B. The reaction product can be coupled to NADH generating reactions C. Synthetic substrates are less soluble than olive oil in aqueous reagents D. Triglyceride substrates cause product inhibition

74. A Triglycerides may be hydrolyzed by nonspecific esterases in serum as well as lipase. Lipase acts only at an interface of oil and H2O and requires bile salts and colipase for activity. Colipase is a protein secreted by the pancreas

74. What method is used to introduce the sample into a mass spectrometer for analysis of a trace element? A. Electrospray ionization B. Laser desorption C. Inductively charged plasma (ICP) ionization D. Direct injection

74. C Mass spectrometers can be used to measure trace metals, but the atoms need to be vaporized and ionized like molecules before they enter the mass filter. This is done by introducing the sample into a very hot plasma (6,000-10,000°K) called a torch. The torch is made by circulating argon through inner and outer quartz tubes. The tubes are wrapped with a coil of wire that receives a radio frequency. This creates current flow through the wire and a magnetic field at the torch end. Argon atoms are excited by the current and magnetic field and ionize.

75. Which statement about the clinical utility of plasma or serum lipase is true? A. Lipase is not increased in mumps, malignancy, or ectopic pregnancy B. Lipase is not increased as dramatically as amylase in acute pancreatitis C. Increased plasma or serum lipase is specific for pancreatitis D. Lipase levels are elevated in both acute and chronic pancreatitis

75. A Lipase elevation is of greater magnitude (2-50 × N) and duration than amylase in acute pancreatitis. When the lipase method is optimized by inclusion of colipase and bile salts, the test is more sensitive and specific than serum amylase for detection of acute pancreatitis. However, lipase is also increased in peptic ulcers, renal insufficiency, and intestinal obstruction. Lipase levels are often low in chronic pancreatitis, and are low in cystic fibrosis

75. Which component is needed for a thermal cycler to amplify DNA? A. Programmable heating and cooling unit B. Vacuum chamber with zero head space C. Sealed airtight constant-temperature chamber D. Temperature-controlled ionization chamber Chemistry/Define fundamental characteristics

75. A The polymerase chain reaction for DNA amplification consists of three phases. Denaturation requires a temperature of 90°C-94°C and separates the double-stranded DNA. Annealing requires a temperature between 40°C-65°C and allows the primers to bind to the target base sequence. Extension requires a temperature of 72°C and allows the heat-stable polymerase to add complementary bases to the primer in the 5' to 3' direction

76. In real-time PCR, what value is needed in order to determine the threshold? A. Background signal B. Melt temperature C. Maximum fluorescence D. Threshold cycle

76. A In real-time PCR, the fluorescence of the reporter probe is proportional to the concentration of PCR products. For quantitation of PCR products, a well factor and background fluorescence must be determined. Well-factor values are analogous to cuvette blanks. They are used to correct the measurements from each well so that the same concentration of fluorescent dye gives the same signal intensity regardless of the well

78. In addition to velocity, what variable is also needed to calculate the relative centrifugal force (g force) of a centrifuge? A. Head radius B. Angular velocity coefficient C. Diameter of the centrifuge tube D. Ambient temperature in degrees Centigrade

78. A The relative centrifugal force (number times the force of gravity) is proportional to the square of the rotor speed in revolutions per minute and the radius in centimeters of the head (distance from the shaft to the end of the tube). RCF = s2 x r x 1.118 x 10-5 where s is the speed in RPM, r is the radius in cM, and 1.118 x 10-5 is a conversion constant

79. Which of the following situations is likely to cause an error when weighing with an electronic analytical balance? A. Failure to keep the knife edge clean B. Failure to close the doors of the balance before reading the weight C. Oxidation on the surface of the substitution weights D. Using the balance without allowing it to warm up for at least 10 minutes

79. B Electronic balances do not use substitution weights or knife edges to balance the weight on the pan. Instead, they measure the displacement of the pan by the weight on it using electromagnetic force to return it to its reference position. Regardless of the type of balance used, all need to be located on a firm weighing table free of vibration. Doors must be closed to prevent air currents from influencing the weighing, and the pan and platform must be clean and free of dust and chemical residue.

80. Which enzyme is measured in whole blood? A. Chymotrypsin B. Glucose-6-phosphate dehydrogenase C. Glycogen phosphorylase D. Lipase

80. B Glucose-6-phosphate dehydrogenase deficiency is the most common inherited RBC enzyme deficiency and is X linked. The enzyme is measured on a wholeblood hemolysate using glucose-6-phosphate as the substrate, and forms 6-phosphogluconate as NADP+ is converted to NADPH

9. Which set of results is most likely in an adult male with primary testicular failure? A. Increased LH, FSH, and decreased testosterone B. Decreased LH, FSH, and testosterone C. Decreased testosterone, androstenedione, and FSH D. Increased androstenedione, decreased testosterone, and normal FSH

9. A Primary testicular failure produces a picture that is hypergonadotropic. The LH and FSH are increased because the pituitary gland is normal and responds to decreased free testosterone. Androstenedione is an adrenal androgen and is unaffected. In testicular failure secondary to pituitary deficiency (hypogonadotropic testicular failure), the LH, FSH, and testosterone are low.

9. In addition to sodium bicarbonate, what other substance contributes most to the amount of base in the blood? A. Hemoglobin concentration B. Dissolved O2 concentration C. Inorganic phosphorus D. Organic phosphate

9. A The primary blood buffer bases preventing acidosis in order of concentration are bicarbonate, deoxyhemoglobin, albumin, and monohydrogen phosphate. At physiological pH, there is significantly more H2PO4 -1 than HPO4 -2, and phosphate is a more efficient buffer system at preventing alkalosis than acidosis. Since all of the blood buffer systems are in equilibrium, the pH can be calculated accurately from the concentration of bicarbonate and dissolved CO2 using the Henderson-Hasselbalch equation.

9. Which of the following dyes is the most specific for measurement of albumin? A. Bromcresol green (BCG) B. Bromcresol purple (BCP) C. Tetrabromosulfophthalein D. Tetrabromphenol blue

9. B Tetrabromphenol blue and tetrabromosulfophthalein are dyes that change pKa in the presence of protein. Although they have greater affinity for albumin than globulins, they are not sufficiently specific to apply to measurement of serum albumin. BCG and BCP are anionic dyes that undergo a spectral shift when they bind albumin at acid pH.

9. Which statement is true regarding the volume distribution (Vd) of a drug? A. Vd is equal to the peak blood concentration divided by the dose given B. Vd is the theoretical volume in liters into which the drug distributes C. The higher the Vd, the lower the dose needed to reach the desired blood level of drug D. The Vd is the principal determinant of the dosing interval

9. B The Vd of a drug represents the dilution of the drug after it has been distributed in the body. The Vd is used to estimate the peak drug blood level expected after a loading dose is given. The peak blood level equals the dose multiplied by f ÷ Vd. The Vd can be calculated by dividing the dose, Xo, by the initial plasma drug concentration, Co, (Vd = Xo/Co) or by dividing the clearance rate by K, the elimination rate constant (K = 0.693 divided by drug half-life).

9. Calculate the pH of a solution of 1.5 × 10-5 M NH4OH. A. 4.2 B. 7.2 C. 9.2 D. 11.2

9. C First, calculate the pOH of the solution. pOH = -Log [OH-] pOH = - Log 1.5 x 10-5 = 4.82 pH = 14 - pOH pH = 14 - 4.8 = 9.2

9. What is the primary clinical utility of measuring CEA? A. Diagnosis of liver cancer B. Diagnosis of colorectal cancer C. Screening for cancers of endodermal origin D. Monitoring for recurrence of cancer

9. D CEA is a glycoprotein that is secreted into plasma by various cancers of endodermal origin, including breast, lung, colorectal, and stomach cancer. However, it is present in only 40%-60% of such cancers, is present at low levels (<3.0 ng/mL) in normal adults, and is increased by causes other than cancer (e.g., smoking

9. Which statement regarding gestational diabetes mellitus (GDM) is correct? A. Is diagnosed using the same oral glucose tolerance criteria as in nonpregnancy B. Converts to diabetes mellitus after pregnancy in 60%-75% of cases C. Presents no increased health risk to the fetus D. Is defined as glucose intolerance originating during pregnancy

9. D Control of GDM reduces perinatal complications such as respiratory distress syndrome, high birth weight, and neonatal jaundice. Women at risk are usually screened between 24 and 28 weeks' gestation. The screening test can be performed nonfasting and consists of an oral 50-g glucose challenge followed by serum or plasma glucose measurement at 1 hour. A

202. Which of the following results would be the most consistent with high risk for coronary heart disease? A. 20 mg/dL HDL cholesterol and 250 mg/dL total cholesterol B. 45 mg/dL HDL cholesterol and 210 mg/dL total cholesterol C. 50 mg/dL HDL cholesterol and 180 mg/dL total cholesterol D. 55 mg/dL HDL cholesterol and 170 mg/dL total cholesterol

A. 20 mg/dL HDL cholesterol and 250 mg/dL total cholesterol

66. When performing electrophoresis at pH 8.6, which hemoglobin molecule migrates the fastest on cellulose acetate toward the anode? A. A! B. A2 C. F D. S

A. A!

8. In regard to bichromatic analysis, which of the following is false? A. Absorbance is measured at the spectral absorbance peak for a blank and the sample using the same wavelength. B. Eliminates background interferences C. Sample concentration determined from difference in two measured absorbances D. Functions as a reference blank for each sample

A. Absorbance is measured at the spectral absorbance peak for a blank and the sample using the same wavelength.

286. Which of the following disorders can be classified as a form of prehepatic jaundice? A. Acute hemolytic anemia B. Cirrhosis C. Dubin-Johnson syndrome D. Neoplasm of common bile duct

A. Acute hemolytic anemia

243. A 42-year-old male presents with anorexia, nausea, fever, and icterus of the skin and mucous membranes. He noticed that his urine had appeared dark for the past several days. The physician orders a series of biochemical tests. Based on the following test results, what is the most likely diagnosis? Serum alkaline phosphatase—slightly elevated Serum alanine aminotransferase— markedly elevated Serum aspartate aminotransferase— markedly elevated Serum gamma-glutamyltransferase— slightly elevated Serum total bilirubin—moderately elevated Urine bilirubin—positive Fecal urobilinogen—decreased A. Acute hepatitis B. Alcoholic cirrhosis C. Metastatic carcinoma of the pancreas D. Obstructive jaundice

A. Acute hepatitis

365. Because of infertility problems, a physician would like to determine when a woman ovulates. The physician orders serial assays of plasma progesterone. From these assays, how can the physician recognize when ovulation occurs? A. After ovulation, progesterone rapidly increases. B. After ovulation, progesterone rapidly decreases. C. Right before ovulation, progesterone rapidly increases. D. There is a gradual, steady increase in progesterone throughout the menstrual cycle.

A. After ovulation, progesterone rapidly increases.

106. In serum protein electrophoresis, when a barbital buffer of pH 8.6 is employed, what protein fraction will migrate the fastest toward the anode? A. Albumin B. Alpha!-globulin C. Beta-globulin D. Gamma-globulin

A. Albumin

264. Which of the following functions as a transport protein for bilirubin in the blood? A. Albumin B. Alpha]-globulin C. Beta-globulin D. Gamma-globulin

A. Albumin

352. Which of the following is the major mineralocorticoid? A. Aldosterone B. Cortisol C. Corticosterone D. Testosterone

A. Aldosterone

418. Nortriptyline is the active metabolite of which of the following drugs? A. Amitriptyline B. Desipramine C. Imipramine D. Doxepin

A. Amitriptyline

38. Which of the following techniques is based on electro-osmotic flow? A. Capillary electrophoresis B. Zone electrophoresis C. Iontophoresis D. Isoelectric focusing

A. Capillary electrophoresis

343. Which of the following statements about carbonic anhydrase (CA) is true? A. Catalyzes conversion of CC>2 and F^O to HHCO3 in red blood cells B. Causes shift to the left in oxygen dissociation curve C. Catalyzes formation of H2CO3 from CO2 and H2O in the tissues D. Inactive in renal tubular cells

A. Catalyzes conversion of CC>2 and F^O to HHCO3 in red blood cells

148. Which of the following defines the term "glycolysis"? A. Conversion of glucose into lactate or pyruvate B. Conversion of glucose to glycogen C. Breakdown of glycogen to form glucose D. Breakdown of lipids to form glucose

A. Conversion of glucose into lactate or pyruvate

244. To aid in the diagnosis of skeletal muscle disease, which of the following serum enzyme measurements would be of most use? A. Creatine kinase B. Alkaline phosphatase C. Aspartate aminotransferase D. Alanine aminotransferase

A. Creatine kinase

129. An increased serum level of which of the following analytes is most commonly associated with decreased glomerular filtration? A. Creatinine B. Uric acid C. Urea D. Ammonia

A. Creatinine

308. Which of the following reagents is used in a colorimetric method to quantify the concentration of serum calcium? A. Cresolphthalein complexone B. Lanthanum C. Malachite green D. Amino-naphthol-sulfonic acid

A. Cresolphthalein complexone

52. Which of the following statements best describes discrete analysis? A. Each sample-reagent mixture is handled separately in its own reaction vessel. B. Samples are analyzed in a flowing stream of reagent. C. Analyzer must be dedicated to measurement of only one analyte. D. It does not have random access capability.

A. Each sample-reagent mixture is handled separately in its own reaction vessel.

372. Which of the following hormones initiates its response by binding to cytoplasmic receptors? A. Estradiol B. Epinephrine C. Growth hormone D. Follicle-stimulating hormone

A. Estradiol

40. In serum protein electrophoresis, when a buffer solution of pH 8.6 is used, which of the following characterizes the proteins? A. Exhibit net negative charge B. Exhibit net positive charge C. Exhibit charge neutrality D. Migrate toward the cathode

A. Exhibit net negative charge

87. Which glycoprotein binds with hemoglobin to facilitate the removal of hemoglobin by the reticuloendothelial system? A. Haptoglobin B. Ceruloplasmin C. cxpAntitrypsin D. Fibrinogen

A. Haptoglobin

316. In what disorder would an increased percent saturation of transferrin be expected? A. Hemochromatosis B. Iron-deficiency anemia C. Myocardial infarction D. Malignancy

A. Hemochromatosis

318. Which of the following is not a typical finding in magnesium deficiency tetany? A. High serum phosphate level B. Normal serum calcium level C. Normal blood pH value D. Low serum potassium level

A. High serum phosphate level

428. Which of the following techniques is more commonly used to measure vitamins? A. High-performance liquid chromatography B. Spectrophotometry C. Nephelometry D. Microbiological

A. High-performance liquid chromatography

102. What technique is used to quantify specific immunoglobulin classes? A. Immunonephelometry B. Serum protein electrophoresis C. Isoelectric focusing D. Immunoelectrophoresis

A. Immunonephelometry

15. Which of the following represents a primary advantage of performing fluorometric over absorption spectroscopic methods of analysis? A. Increased specificity and increased sensitivity B. Increased specificity and decreased sensitivity C. Purity of reagents used not as critical D. Ease of performing assays

A. Increased specificity and increased sensitivity

209. A 46-year-old known alcoholic with liver damage is brought into the emergency department unconscious. In what way would you expect his plasma lipid values to be affected? A. Increased total cholesterol, triglyceride, LDL, and VLDL B. Increased total cholesterol and triglyceride, decreased LDL and VLDL C. Decreased total cholesterol, triglyceride, LDL, and VLDL D. Normal lipid metabolism, unaffected by the alcoholism

A. Increased total cholesterol, triglyceride, LDL, and VLDL

272. Which of the following does not accurately describe direct bilirubin? A. Insoluble in water B. Conjugated in the liver C. Conjugated with glucuronic acid D. Excreted in the urine of jaundiced patients

A. Insoluble in water

433. Which of the following is not associated with vitamin B12? A. Insoluble in water B. Intrinsic factor C. Schilling test D. Pernicious anemia

A. Insoluble in water

(see pic) 199. A 54-year-old male, with a history of type 2 diabetes mellitus for the past 8 years, is seen by his family physician. The patient indicates that during the past week he had experienced what he described as feeling lightheaded and faint. He also indicated that he became out of breath and had experienced mild chest pain when doing heavy yard work, but the chest pain subsided when he sat down and rested. The physician performed an ECG immediately, which was normal, and he ordered blood tests. The patient fasted overnight and had blood drawn the next morning. The laboratory test values follow. Based on the patient's test results, history, and symptoms, which of the laboratory values in the chart above does not support the patient's diagnosis? A. LDL cholesterol B. HDL cholesterol C. Hemoglobin Aic D. hs-CRP

A. LDL cholesterol

17. Which of the following may be associated with bioluminescence? A. Light emission produced due to enzymatic oxidation of a substrate B. Less sensitive than direct fluorescent assays C. Electron excitation caused by radiant energy D. Employs a radioactive label

A. Light emission produced due to enzymatic oxidation of a substrate

206. Which of the following may be described as a variant form of LDL, associated with increased risk of atherosclerotic cardiovascular disease? A. Lp(a) B. HDL C. ApoA-I D. ApoA-II

A. Lp(a)

434. Which of the following tissues is important in vitamin D metabolism? A. Skin B. Spleen C. Pancreas D. Thyroid

A. Skin

270. As the red blood cells disintegrate, hemoglobin is released and converted to the pigment bilirubin. Which organ is primarily responsible for this function? A. Spleen B. Kidneys C. Intestines D. Liver

A. Spleen

147. Which of the following carbohydrates is a polysaccharide? A. Starch B. Sucrose C. Lactose D. Glucose

A. Starch

178. Which of the following is characterized by a deficiency of glucose-6-phosphatase resulting in hepatomegaly, lactic acidosis, and severe fasting hypoglycemia? A. Type I—von Gierke disease B. Type II—Pompe disease C. Type III—Cori disease D. Type IV—Andersen disease

A. Type I—von Gierke disease

176. A patient has a urine uric acid level of 1575 mg/day. What effect will this have on the measured urine glucose level when the glucose oxidase/peroxidase method is employed? A. Urine glucose level will be falsely low. B. Urine glucose level will be falsely high. C. Urine glucose level will be accurate. D. Urine glucose level will exceed the linearity of the method

A. Urine glucose level will be falsely

276. What collective term encompasses the reduction products stercobilinogen, urobilinogen, and mesobilirubinogen? A. Urobilinogen B. Mesobilirubinogen C. Urobilin D. Bilirubin

A. Urobilinogen

65. Hemoglobin S is an abnormal hemoglobin that is characterized by a substitution of which amino acid? A. Valine for glutamic acid in position 6 on the beta chain B. Valine for glutamic acid in position 6 on the alpha chain C. Lysine for glutamic acid in position 6 on the beta chain D. Lysine for glutamic acid in position 6 on the alpha chain

A. Valine for glutamic acid in position 6 on the beta chain

359. Which of the following is the most common cause of the adrenogenital syndrome called congenital adrenal hyperplasia, and which test is used for its diagnosis? A. 17a-Hydroxylase deficiency; progesterone assay B. 21 -Hydroxylase deficiency; 17ct-hydroxyprogesterone assay C. 3[3-Hydroxysteroid dehydrogenaseisomerase deficiency; 17a-hydroxypregnenolone assay D. 11 (3-Hydroxylase deficiency; 11-deoxycortisol assay

B. 21 -Hydroxylase deficiency; 17ct-hydroxyprogesterone assay

355. As a screening test for Gushing syndrome, the physician wishes to see whether a patient exhibits normal diurnal rhythm in his or her cortisol secretion. At what time should the specimens be drawn for plasma cortisol determination? A. 6 A.M., 2 P.M. B. 8 A.M.,4 P.M. C. 12 noon, 6 P.M. D. 12 noon, 12 midnight

B. 8 A.M.,4 P.M.

259. The laboratory receives a request that assays for urinary aminolevulinic acid, porphobilinogen, uroporphyrin, and coproporphyrin are to be performed on a patient. Which of the following will not contribute to the integrity of the sample when these assays are performed on the same urine specimen? A. Refrigeration B. Addition of hydrochloric acid C. 24-hour urine collection D. Use of a brown bottle

B. Addition of hydrochloric acid

220. Which of the following enzymes does not belong to the class of enzymes known as the hydrolases? A. Alkaline phosphatase B. Aldolase C. Amylase D. Lipase

B. Aldolase

307. Which of the following reagents is used to determine the concentration of serum inorganic phosphate? A. Ehrlich's reagent B. Ammonium molybdate C. 8-Hydroxyquinoline D. Bathophenanthroline

B. Ammonium molybdate

231. Which enzyme is measured by methodologies that use small oligosaccharides and 4-nitrophenyl-glycoside for substrates? A. Lipase B. Amylase C. Creatine kinase D. Cholinesterase

B. Amylase

235. In acute pancreatitis, a significant increase in which serum enzyme would be expected diagnostically? A. Creatine kinase B. Amylase C. Alkaline phosphatase D. Aspartate aminotransferase

B. Amylase

353. Plasma renin activity (PRA) measurements are usually made by measuring which of the following using immunoassay? A. Angiotensinogen B. AngiotensinI C. Angiotensin II D. Angiotensin-converting enzyme

B. AngiotensinI

332. What is the specimen of choice for analysis of acid-base disturbances involving pulmonary dysfunction in an adult? A. Venous blood B. Arterial blood C. Capillary blood D. Urine

B. Arterial blood

222. Which of the following enzymes catalyzes the transfer of amino groups causing the interconversion of amino acids and a-oxoacids? A. Amylase B. Aspartate transaminase C. Alkaline phosphatase D. Lactate dehydrogenase

B. Aspartate transaminase

261. To quantify serum bilirubin levels, it is necessary that bilirubin couples with diazotized sulfanilic acid to form what complex? A. Verdobilirubin B. Azobilirubin C. Azobilirubinogen D. Bilirubin glucuronide

B. Azobilirubin

416. Which of the following is a commonly encountered xanthine that could potentially interfere with the determination of theophylline? A. Nicotine B. Caffeine C. Amphetamine D. Procainamide

B. Caffeine

228. Which of the following is false about myoglobin as it relates to acute myocardial infarction (AMI)? A. Measure serially B. Cardiac specific C. Initial increase occurs in 1-3 hours D. Doubling of initial value within 1-2 hours suggestive of AMI

B. Cardiac specific

33. The measurement of CO2 in blood by means of a PCO2 electrode is dependent on the A. Passage of H+ ions through the membrane that separates the sample and the electrode B. Change in pH because of increased carbonic acid in the electrolyte surrounding the electrodes C. Movement of bicarbonate across the membrane that separates the sample and the electrode D. Linear relationship between PCO2 in the sample and measured pH

B. Change in pH because of increased carbonic acid in the electrolyte surrounding the electrodes

191. Each lipoprotein fraction is composed of varying amounts of lipid and protein components. The beta-lipoprotein fraction consists primarily of which lipid? A. Fatty acid B. Cholesterol C. Phospholipid D. Triglyceride

B. Cholesterol

311. To what metal does ceruloplasmin firmly bind? A. Chromium B. Copper C. Zinc D. Iron

B. Copper

53. Which of the following chromatography systems may be described as having a stationary phase that is liquid absorbed on particles packed in a column and a liquid moving phase that is pumped through a column? A. Thin-layer B. High-performance liquid C. Ion-exchange D. Gas-liquid

B. High-performance liquid

174. As part of a routine physical, a fasting plasma glucose is performed on a 45-yearold male and the test result is 105 mg/dL. How should this individual be classified? A. Normal for his age B. Impaired fasting glucose C. Type 1 diabetes mellitus D. Type 2 diabetes mellitus

B. Impaired fasting glucose

48. Given the following information on a particular compound that has been visualized by means of thin-layer chromatography, calculate the /fyof the compound. Distance from origin to spot center = 48 mm Distance from spot center to solvent front = 93 mm Distance from origin to solvent front = 141 mm A. 0.29 B. 0.34 C. 0.52 D. 0.66

B. In thin-layer chromatography (TLC), the Rf (retention factor) describes the distance traveled by the solute (compound of interest) in relation to the distance traveled by the solvent (mobile phase). Measurements of the TLC plate are made from the origin or point of sample application to the center of the developed spot and from the origin to the solvent front. An Rf may be calculated by means of the following formula: Rf = (Distance from origin to spot center) / (Distance from origin to solvent front) Rf = 48mm / 141 mm = 0.34

413. An epileptic patient receiving phenytoin develops acute glomerulonephritis. What change, if any, would be expected in the patient's circulating drug level? A. Decrease in free drug B. Increase in free drug C. Increase in protein-bound drug D. No change in circulating drug level

B. Increase in free drug

172. What effect if any would be expected when the secretion of epinephrine is stimulated by physical or emotional stress? A. Decreased blood glucose level B. Increased blood glucose level C. Increased glycogen storage D. No effect on blood glucose or glycogen levels

B. Increased blood glucose level

309. Which of the following has an effect on plasma calcium levels? A. Sodium B. Inorganic phosphate C. Potassium D. Iron

B. Inorganic phosphate

410. Increased trough levels of aminoglycosides in the serum are often associated with toxic effects to which organ? A. Heart B. Kidney C. Pancreas D. Liver

B. Kidney

404. Free erythrocyte protoporphyrin (FEP) levels are useful as a screening method for exposure to which of the following metals? A. Zinc B. Lead C. Iron D. Mercury

B. Lead

194. Several malabsorption problems are characterized by a condition known as steatorrhea. Steatorrhea is caused by an abnormal accumulation of what substance in the feces? A. Proteins B. Lipids C. Carbohydrates D. Vitamins

B. Lipids

419. Which of the following is used in the treatment of manic depression? A. Potassium B. Lithium C. Calcium D. Chloride

B. Lithium

401. THC (A9-tetrahydrocannabinol) is the principal active component of what drug? A. Benzodiazepine B. Marijuana C. Morphine D. Codeine

B. Marijuana

46. Which of the following is not associated with isoelectric focusing? A. Continuous pH gradient B. Migration of proteins with net charge of zero C. Separation dependent on isoelectric point D. Zone electrophoresis

B. Migration of proteins with net charge of zero

248. A 4-year-old male child is brought to the pediatrician because the parents are concerned about the child's frequent falling, which results in bruising. The parents indicate that the child has difficulty running, walking, standing up, climbing stairs, and even sitting up straight. The child also appears somewhat weak. Which of the following results is not consistent with the most likely diagnosis? A. Moderately elevated AST B. Moderately elevated ALP C. Moderately elevated LD D. Markedly elevated CK

B. Moderately elevated ALP

2. Which of the following isolates light within a narrow region of the spectrum? A. Photomultiplier tube B. Monochromator C. Photovoltaic cell D. Detector

B. Monochromator

399. Heroin is synthesized from what drug? A. Diazepam B. Morphine C. Ecgonine D. Chlorpromazine

B. Morphine

400. After absorption, codeine is rapidly metabolized to what compound? A. Phencyclidine B. Morphine C. Methadone D. Propoxyphene

B. Morphine

203. A patient's total cholesterol is 300 mg/dL, his HDL cholesterol is 50 mg/dL, and his triglyceride is 200 mg/dL. What is this patient's calculated LDL cholesterol? A. 200 B. 210 C. 290 D. 350

B. Once the total cholesterol, triglyceride, and HDL cholesterol are known, LDL cholesterol can be quantified by using the Friedewald equation LDL cholesterol = Total cholesterol — (HDL cholesterol + Triglyceride/5) In this example, all results are in mg/dL: LDL cholesterol = 300 - (50 + 200/5) = 300 - (90) = 210 mg/dL

361. Which of the following tissues does not secrete steroid hormones? A. Ovaries B. Pituitary gland C. Testes D. Adrenal cortex

B. Pituitary gland

344. Which of the following statements best describes "base excess"? A. Primarily refers to carbonic acid concentration B. Positive values reflect metabolic alkalosis. C. Created through metabolism of carbohydrates D. Negative values represent a respiratory imbalance

B. Positive values reflect metabolic alkalosis.

292. The presence of only slightly visible hemolysis will significantly increase the serum level of which of the following electrolytes? A. Sodium B. Potassium C. Chloride D. Bicarbonate

B. Potassium

114. Which of the following disorders is not associated with an elevation of serum aj-fetoprotein? A. Testicular germ cell tumors B. Prostatic carcinoma C. Pancreatic carcinoma D. Gastric carcinoma

B. Prostatic carcinoma

109. Which of the following is not associated with carcinoembryonic antigen? A. Increased levels seen with malignancies of the lungs B. Quantified by using capillary electrophoresis C. Used to monitor treatment of colon cancer D. Glycoprotein in nature

B. Quantified by using capillary electrophoresis

207. In what way is the "normal" population reference interval for total cholesterol in America different from that of other clinical chemistry parameters (i.e., protein, sodium, BUN, creatinine, etc.)? A. Established units for total cholesterol are mg/dL; no other chemistry test has these units. B. Reference interval is artificially set to reflect good health even though Americans as a group have "normally" higher total cholesterol levels. C. Total cholesterol reference interval must be interpreted in line with triglyceride, phospholipid, and sphingolipid values. D. Total cholesterol reference interval is based on a manual procedure, whereas all other chemistry parameters are based on automated procedures.

B. Reference interval is artificially set to reflect good health even though Americans as a group have "normally" higher total cholesterol levels.

237. Which of the following statements is not associated with serum cholinesterase? A. Inhibited by organic insecticides B. Referred to as "true" cholinesterase C. Decreased level causes prolonged apnea after administration of succinyldicholine D. Acts on the substrate propionylthiocholine

B. Referred to as "true" cholinesterase

347. A 24-year-old drug abuser is brought into the emergency department unconscious. He has shallow breaths, looks pale, and is "clammy." Blood gases show the following results: pH = 7.29, PCO2 = 50 mmHg, HCOs = 25 mmol/L .What condition is indicated by these results? A. Metabolic alkalosis, partially compensated B. Respiratory acidosis, uncompensated C. A dual problem of acidosis D. An error in one of the blood gas measurements

B. Respiratory acidosis, uncompensated

18. Nephelometry is based on the measurement of light that is A. Absorbed by particles in suspension B. Scattered by particles in suspension C. Produced by fluorescence D. Produced by excitation of ground-state atoms

B. Scattered by particles in suspension

379. Which of the following is not associated with growth hormone? A. Somatotropin B. Secreted by posterior pituitary C. Hypersecretion results in acromegaly D. Affects lipid, carbohydrate, and protein metabolism

B. Secreted by posterior pituitary

287. The following laboratory results are determined on a patient with a suggested diagnosis of biliary obstruction: Serum total bilirubin—increased Serum conjugated bilirubin—normal Urine bilirubin—increased Fecal urobilin—decreased Which laboratory result is the least consistent with such a diagnosis? A. Serum total bilirubin B. Serum conjugated bilirubin C. Urine bilirubin D. Fecal urobilin

B. Serum conjugated bilirubin

56. Ion-exchange chromatography separates solutes in a sample based on the A. Solubility of the solutes B. Sign and magnitude of the ionic charge C. Adsorption ability of the solutes D. Molecular size

B. Sign and magnitude of the ionic charge

119. In the urea method, the enzymatic action of urease is inhibited when blood for analysis is drawn in a tube containing what anticoagulant? A. Sodium heparin B. Sodium fluoride C. Sodium oxalate D. Ethylenediaminetetra-acetic acid

B. Sodium fluoride

175. A cerebrospinal fluid specimen is sent to the lab at 9:00 P.M. for glucose analysis. The specimen is cloudy and appears to contain red blood cells. Which of the following statements is true? A. Glucose testing cannot be performed on the specimen. B. Specimen should be centrifuged and glucose assayed immediately. C. Specimen can be refrigerated as received and glucose assayed the next day. D. Specimen can be frozen as received and glucose assayed the next day

B. Specimen should be centrifuged and

310. A patient's serum inorganic phosphate level is found to be elevated but the physician cannot determine a physiological basis for this abnormal result. What could possibly have caused an erroneous result to be reported? A. Patient not fasting when blood was drawn B. Specimen was hemolyzed C. Effect of diurnal variation D. Patient receiving intravenous glucose therapy

B. Specimen was hemolyzed

(see pic) 214. Based on the following graph of velocity of an enzyme reaction versus substrate concentration, you are designing a new method to measure the activity of an enzyme of clinical interest. To formulate the new methodology so that enzyme activity is assessed using zero-order kinetics, which concentration of substrate should you initially determine experimentally? A. Substrate concentration a B. Substrate concentration b C. Substrate concentration c D. Substrate concentration d

B. Substrate concentration b

213. In the assay of an enzyme, zero-order kinetics are best described by which of the following statements? A. Enzyme is present in excess; rate of reaction is variable with time and dependent only on the concentration of the enzyme in the system. B. Substrate is present in excess; rate of reaction is constant with time and dependent only on the concentration of enzyme in the system. C. Substrate is present in excess; rate of reaction is constant with enzyme concentration and dependent only on the time in which the reaction is run. D. Enzyme is present in excess; rate of reaction is independent of both time and concentration of the enzyme in the system.

B. Substrate is present in excess; rate of reaction is constant with time and dependent only on the concentration of enzyme in the system.

162. Which of the following is not associated with hypoglycemia? A. Neuroglycopenia B. Symptoms occur with plasma glucose level of 60-70 mg/dL C. Decreased hepatic glucose production D. Diagnostic test is 72-hour fast

B. Symptoms occur with plasma glucose level of 60-70 mg/dL

1. Which of the following hormones is often decreased by approximately 25% in the serum of pregnant women who have a fetus with Down syndrome? A. Estriol (E3) B. Human chorionic gonadotropin (hCG) C. Progesterone D. Estradiol (E2)

1. A E3 is produced in the placenta and fetal liver from dehydroepiandrosterone derived from the mother and fetal liver. E3 is the major estrogen produced during pregnancy, and levels rise throughout gestation. Serum free E3 is often lower than expected for the gestational age in a pregnancy associated with Down syndrome

1. Which of the following tumor markers is classified as a tumor suppressor gene? A. BRCA-1 B. Carcinoembryonic antigen (CEA) C. Human chorionic gonadotropin (hCG) D. Nuclear matrix protein

1. A Tumor markers may be enzymes, hormones, receptors, oncofetal (glycoprotein) antigens, or oncogenes. BRCA-1 is located on the long arm of chromosome 17 and carries an 85% lifetime risk of breast or ovarian cancer when present. Its product functions in DNA repair and slows cell proliferation.

1. Which of the following represents the Henderson-Hasselbalch equation as applied to blood pH? A. pH = 6.1 + log HCO3-/PCO2 B. pH = 6.1 + log HCO3-/(0.03 × PCO2) C. pH = 6.1 + log dCO2/HCO3- D. pH = 6.1 + log (0.03 × PCO2)/HCO3-

1. B The Henderson-Hasselbalch equation describes the pH of a buffer comprised of a weak acid and its salt. pH = pKa + log salt/acid, where pKa is the negative logarithm of the dissociation constant of the acid. In this case, the salt is sodium bicarbonate and the acid is the dissolved CO2, which is equal to 0.03 (mmol/L per mm Hg) x PCO2. The pKa includes both the hydration and dissociation constant for dissolved CO2 in blood, 6.1 and is termed pK

1. Kjeldahl's procedure for total protein is based upon the premise that: A. Proteins are negatively charged B. The pKa of proteins is the same C. The nitrogen content of proteins is constant D. Proteins have similar tyrosine and tryptophan content

1. C Kjeldahl's method measures the nitrogen content of proteins as ammonium ion by back titration following oxidation of proteins by sulfuric acid and heat. It assumes that proteins average 16% nitrogen by weight. Protein in grams per deciliter is calculated by multiplying protein nitrogen by 6.25. The Kjeldahl method is a reference method for total protein that is used to assign a protein assay value to calibrators

1. Which of the following procedures can be used to detect proportional error in a new method for glucose? A. Compare the standard deviation of 40 patient samples to the hexokinase method B. Measure a mixture made from equal parts of normal and high-QC sera C. Add 5.0 mg of glucose to 1.0 mL of a serum of known concentration and measure D. Compare the mean of 40 normal samples to the hexokinase method

1. C Proportional error is percentage deviation from the expected result, and affects the slope of the calibration curve. It causes a greater absolute error (loss of accuracy) as concentration increases. It is measured by a recovery study in which a sample is spiked with known amounts of analyte. In the example, the concentration should increase by 500 mg/dL

1. An international unit (IU) of enzyme activity is the quantity of enzyme that: A. Converts 1 μmol of substrate to product per liter B. Forms 1 mg of product per deciliter C. Converts 1 μmol of substrate to product per minute D. Forms 1 μmol of product per liter

1. C The IU is a rate expressed in micromoles per minute. Activity is reported as IUs per liter (IU/L) or mIU/mL. The SI unit for enzyme activity is the katal (1 katal converts 1 mol of substrate to product in 1 second

1. In which of the following cases is qualitative analysis of the drug usually adequate? A. To determine whether the dose of a drug with a low therapeutic index is likely to be toxic B. To determine whether a patient is complying with the physician's instructions C. To adjust dose if individual differences or disease alter expected response D. To determine whether the patient has been taking amphetamines

1. D The purpose of therapeutic drug monitoring is to achieve a therapeutic blood drug level rapidly and minimize the risk of drug toxicity caused by overdose. Therapeutic drug monitoring is a quantitative procedure performed for drugs with a narrow therapeutic index (ratio of the concentration producing the desired effect to the concentration producing toxicity).

10. For drugs with first-order elimination, which statement about drug clearance is true? A. Clearance = elimination rate ÷ serum level B. It is most often performed by the liver C. It is directly related to half-life D. Clearance rate is independent of dose

10. A First-order elimination represents a linear relationship between the amount of drug eliminated per hour and the blood level of drug. For drugs following linear kinetics, clearance equals the elimination rate divided by the drug concentration in blood.

10. Which of the following effects results from exposure of a normal arterial blood sample to room air? A. PO2 increased PCO2 decreased pH increased B. PO2 decreased PCO2 increased pH decreased C. PO2 increased PCO2 decreased pH decreased D. PO2 decreased PCO2 decreased pH decreased

10. A The PO2 of air at sea level (21% O2) is about 150 mm Hg. The PCO2 of air is only about 0.3 mm Hg. Consequently, blood releases CO2 gas and gains O2 when exposed to air. Loss of CO2 shifts the equilibrium of the bicarbonate buffer system to the right, decreasing hydrogen ion concentration and blood becomes more alkaline

10. Which of the following enzymes is activated by calcium ions? A. CK B. Amylase C. ALP D. LD

10. B Most enzymes require metals as activators or cofactors. CK and ALP require Mg+2 for full activity, and amylase requires Ca+2. Metals required for activity should be components of the substrate used for enzyme analysis. The substrate must also contain anions required (e.g., Cl- for amylase) and should not contain inhibiting cations or anions (e.g., Zn+2 and Mn+2 for CK)

10. How many significant figures should be reported when the pH of a 0.060 M solution of nitric acid is calculated? A. 1 B. 2 C. 3 D. 4

10. B When zeros appear by themselves to the left of the decimal point, they are not significant. When they are to the left of the decimal point and are preceded by a number, they are significant. Zeros after the decimal point preceding a number are not significant.

10. Which of the following findings is characteristic of all forms of clinical hypoglycemia? A. A fasting blood glucose value below 55 mg/dL B. High fasting insulin levels C. Neuroglycopenic symptoms at the time of low blood sugar D. Decreased serum C peptide

10. C Clinical hypoglycemia can be caused by insulinoma, drugs, alcoholism, and reactive hypoglycemia. Reactive hypoglycemia is characterized by delayed or excessive insulin output after eating and is very rare. Fasting insulin is normal but postprandial levels are increased. High fasting insulin levels (usually > 6 μg/L) are seen in insulinoma, and patients with insulinoma almost always display fasting hypoglycemia, especially when the fast is extended to 48-72 hours.

10. When should progesterone be measured when evaluating an adult female for anovulation? A. At the onset of menses B. During the first 7 days of the menstrual cycle C. At the midcycle just after LH peaks D. At the end of the menstrual cycle

10. C Progesterone is often measured along with LH, FSH, estrogen, and prolactin to evaluate female infertility and dysmenorrhea. Progesterone is produced by the corpus luteum and levels are very low during the early follicular phase of the cycle. Progesterone is released by the corpus luteum following the LH surge that occurs 1-2 days prior to ovulation and is an indication that ovulation occurred. Low progesterone at midcycle indicates that ovulation did not occur. This is often the case in polyovarian cyst syndrome

10. SITUATION: A peak blood level for orally administered theophylline (therapeutic range 8-20 mg/L) measured at 8 a.m. is 5.0 mg/L. The preceding trough level was 4.6 mg/L. What is the most likely explanation of these results? A. Laboratory error made on peak measurement B. Specimen for peak level was collected from wrong patient C. Blood for peak level was drawn too soon D. Elimination rate has reached maximum

10. C Sample collection time is critical for accurate therapeutic drug monitoring. Blood for trough levels must be collected immediately before the next dose. Blood collection time for peak levels must not occur prior to complete absorption and distribution of drug. This usually requires 1-2 hours for orally administered drugs.

10. Which condition is a common cause of stray light? A. Unstable source lamp voltage B. Improper wavelength calibration C. Dispersion from second-order spectra D. Misaligned source lamp

10. C Stray light is caused by the presence of any light other than the wavelength of measurement reaching the detector. It is most often caused by second-order spectra, deteriorated optics, light dispersed by a darkened lamp envelope, and extraneous room light.

10. Interference from other reducing substances can be partially eliminated in the Jaffe reaction by: A. Measuring the product at 340 nm B. Measuring the product with an electrode C. Measuring the timed rate of product formation D. Performing a sample blank

10. C The Jaffe reaction is nonspecific; proteins and other reducing substances such as pyruvate, protein, and ascorbate cause positive interference. Much of this interference is reduced by using a timed rate reaction. Ketoacids react with alkaline picrate almost immediately, and proteins react slowly. Therefore, reading the absorbance at 20 and 80 seconds and using the absorbance difference minimizes the effects of those compounds

11. Which of the following statements is true? A. Cystatin C is measured immunochemically B. The calibrator used for cystatin C is traceable to the National Bureau of Standards calibrator C. Cystatin C assays have a lower coefficient of variation than plasma creatinine D. Enzymatic and rate Jaffe reactions for creatinine give comparable results

11. A Cystatin C can be measured by enzyme immunoassay, immunonephelometry, and immunoturbidimetry. However, there is no standardized calibrator as for creatinine, and therefore, results vary considerably from lab to lab.

11. Which of the following formulas for O2 content is correct? A. O2 content = %O2 saturation/100 × Hgb g/dL × 1.39 mL/g + (0.0031 × PO2) B. O2 content = PO2 × 0.0306 mmol/L/mm C. O2 content = O2 saturation × Hgb g/dL × 0.003 mL/g D. O2 content = O2 capacity × 0.003 mL/g

11. A Oxygen content is the sum of O2 bound to Hgb and O2 dissolved in the plasma. It is dependent upon the Hgb concentration and the percentage of Hgb bound to O2 (O2 saturation). Each gram of Hgb binds 1.39 mL of O2. The dissolved O2 is determined from the solubility coefficient of O2 (0.0031 mL per dL/mm Hg) and the PO2. O2 content = % Sat/100 × Hgb in g/dL × 1.39 mL/g + (0.0031 × PO2).

11. At pH 8.6, proteins are _________ charged and migrate toward the _________. A. Negatively, anode B. Positively, cathode C. Positively, anode D. Negatively, cathode

11. A Proteins are amphoteric owing to ionization of acidic and basic side chains of amino acids. When the pH of the solution equals the isoelectric point ( pI ), the protein will have no net charge and is insoluble. When the pH of the solution is above the pI, the protein will have a net negative charge. Anions migrate toward the anode (positive electrode

11. A person is suspected of having testicular cancer. Which type of hCG test would be most useful? A. Plasma immunoassay for intact hCG only B. Plasma immunoassay for intact hCG and the β-hCG subunit C. Plasma immunoassay for the free alpha and β-hCG subunits D. Urine assay for hCG β core

11. B In addition to testicular cancer, hCG is produced by trophoblastic tumors and choriocarcinomas. Some of these tumors secrete the β-subunit without intact hCG. This is especially true after treatment when hCG is used to monitor for recurrence. T

11. Which statement regarding glycated (glycosylated) Hgb (G-Hgb) is true? A. Has a sugar attached to the C-terminal end of the β chain B. Is a highly reversible aminoglycan C. Reflects the extent of glucose regulation in the 8- to 12-week interval prior to sampling D. Will be abnormal within 4 days following an episode of hyperglycemia

11. C G-Hgb results from the nonenzymatic attachment of a sugar such as glucose to the N-terminal valine of the β chain. The reaction is nonreversible and is related to the time-averaged blood glucose concentration over the life span of the RBCs. There are three G-Hgb fractions designated A1a, A1b, and Alc. Hemoglobin A1c makes up about 80% of glycated hemoglobin, and is used to determine the adequacy of insulin therapy. The time-averaged blood glucose is approximated by the formula (G-Hgb × 33.3) - 86 mg/dL, and insulin adjustments can be made to bring this level to within reference limits. Also, glycated protein assay (called fructosamine) provides similar data for the period between 2 and 4 weeks before sampling.

11. SITUATION: A patient breathing room air has the following arterial blood gas and electrolyte results: pH = 7.54 PCO2 = 18.5 mm Hg PO2 = 145 mm Hg HCO3 = Na = 135 mmol/L K = 4.6 mmol/L 18 mmol/L Cl = 98 mmol/L TCO2 = 20 mmol/L The best explanation for these results is: A. Blood for electrolytes was drawn above an IV B. Serum sample was hemolyzed C. Venous blood was sampled for arterial blood gases D. Blood gas sample was exposed to air

11. D A patient breathing room air cannot have an arterial PO2 greater than 105 mm Hg because alveolar PO2 is 110 mm Hg when breathing 20% O2. Exposure to air caused loss of CO2 gas and increased pH.

11. A female with severe excessive pubic and facial hair growth (hirsutism) should be tested for which of the following hormones? A. Estrogen and progesterone B. Chorionic gonadotropin C. Growth hormone D. Testosterone and dehydroepiandrosterone sulfate

11. D Excessive hair grown in females results from excessive androgen production, and is most commonly seen in polycystic ovarian syndrome, which produces high levels of ovarian-derived testosterone. It will also occur as a consequence of Cushing's syndrome and mild congenital adrenal hyperplasia

(see pic) 11. A linearity study is performed on a visible spectrophotometer at 650 nm and the following absorbance readings are obtained. The study was repeated using freshly prepared standards and reagents, but results were identical to those shown. What is the most likely cause of these results? A. Wrong wavelength used B. Insufficient chromophore concentration C. Matrix interference D. Stray light

11. D Stray light is the most common cause of loss of linearity at high-analyte concentrations. Light transmitted through the cuvette is lowest when absorption is highest. Therefore, stray light is a greater percentage of the detector response when sample concentration is high. Stray light is usually most significant when measurements are made at the extremes of the visible spectrum because lamp output and detector response are low.

11. Which statement about steady-state drug levels is true? A. The absorbed drug must be greater than the amount excreted B. Steady state can be measured after two elimination half-lives C. Constant intravenous infusion will give the same minima and maxima as an oral dose D. Oral dosing intervals give peaks and troughs in the dose-response curve

11. D When drugs are infused intravenously, both the distribution and elimination rates are constant. This eliminates the peaks and troughs seen in the dose-response curve. Peak and trough levels are characteristics of intermittent dosing regimens. The steady state is reached when drug in the next dose is sufficient only to replace the drug eliminated since the last dose

12. Which statement about methods for measuring LD is true? A. The formation of pyruvate from lactate (forward reaction) generates NAD+ B. The pyruvate-to-lactate reaction proceeds at about twice the rate as the forward reaction C. The lactate-to-pyruvate reaction is optimized at pH 7.4 D. The negative-rate reaction is preferred

12. B Although the rate of the reverse reaction (P →L) is faster, the L →P reaction is more popular because it produces a positive rate (generates NADH), is not subject to product inhibition, and is highly linear. The pH optimum for the forward reaction is approximately 8.8.

12. Electrophoretic movement of proteins toward the anode will decrease by increasing the: A. Buffer pH B. Ionic strength of the buffer C. Current D. Voltage

12. B Electrophoresis is the migration of charged molecules in an electric field. Increasing the strength of the field by increasing voltage (or current) increases migration. However, increasing ionic strength decreases the migration of proteins. Counterions (cations) in the buffer move with the proteins, reducing their electromagnetic attraction for the anode

12. Which set of results is most likely in a female with hypogonadotropic ovarian failure? A. Increased LH, FSH, and estrogen B. Decreased LH, FSH, and estrogen C. Decreased prolactin and estrogen D. Increased LH and FSH, and decreased estrogen

12. B Hypogonadotropic ovarian failure is the result of pituitary dysfunction. It may be caused by low levels of both LH and FSH, or it may be caused by high levels of prolactin as in prolactinoma, since prolactin will inhibit LHRH and result in low LH and FSH

12. What is the American Diabetes Association recommended cutoff value for adequate control of blood glucose in diabetics as measured by glycated hemoglobin? A. 5% B. 6.5% C. 9.5% D. 11%

12. B The ADA recommends that 6.5% be used as the cutoff for determining the adequacy of treatment for diabetes. A glycated hemoglobin test should be performed at the time of diagnosis and every 6 months thereafter if the result is < 6.5%. If the result is 6.5% or more, the treatment plan should be adjusted to achieve a lower level, and the test performed every 3 months until control is improved.

12. What is the pH of a buffer containing 40.0 mmol/L NaHC2O4 and 4.0 mmol/L H2C2O4? (pKa = 1.25) A. 1.35 B. 2.25 C. 5.75 D. 6.12

12. B The Henderson-Hasselbalch equation can be used to determine the pH of a buffer containing a weak acid and a salt of the acid.

12. The normal difference between alveolar and arterial PO2 (PAO2-PaO2 difference) is: A. 3 mm Hg B. 10 mm Hg C. 40 mm Hg D. 50 mm Hg

12. B The PAO2-PaO2 difference results from the low ratio of ventilation to perfusion in the base of the lungs. The hemoglobin in the blood coming from the base of the lung has a lower O2 saturation. This blood will take up O2 from the plasma of blood leaving well-ventilated areas of the lung, thus lowering the mixed arterial PO2

12. In which case would eGFR derived from the plasma creatinine likely give a more accurate measure of GFR than measurement of plasma cystatin C? A. Diabetic patient B. Chronic renal failure C. Post-renal transplant D. Chronic hepatitis

12. C Cystatin C is eliminated almost exclusively by the kidneys and plasma levels are not dependent on age, sex, or nutritional status. However, plasma levels are affected by some drugs, including those used to prevent renal transplant rejection. Increased plasma levels have been reported in chronic inflammatory diseases and cancer.

12. A patient treated for a germ cell tumor has a total and free β-hCG assay performed prior to surgery. The result is 40,000 mIU/mL. One week following surgery, the hCG is 5,000 mIU/mL. Chemotherapy is started, and the hCG is measured 1 week later and found to be 10,000 mIU/mL. What does this indicate? A. Recurrence of the tumor B. Falsely increased hCG owing to drug interference with the assay C. Analytical error with the test reported as 5,000 mIU/mL D. Transient hCG increase caused by chemotherapy

12. D Treatment of tumors with chemotherapy often causes a transient increase in the production of tumor markers as the drugs destroy tumor cells. The half-life of hCG is 24-36 hours; therefore, the expected decline 1 week postsurgery was consistent with the result of 5,000 mIU/mL

13. At pH 8.6, the cathodal movement of γ globulins is caused by: A. Electroendosmosis B. Wick flow C. A net positive charge D. Cathodal sample application

13. A Agarose and cellulose acetate contain fixed anions (e.g., acetate) that attract counterions when hydrated with buffer. When voltage is applied the cations migrate to the cathode, creating an osmotic force that draws H2O with them. This force, called electroendosmosis, opposes protein migration toward the anode and may cause some γ-globulins to be displaced toward the cathode

13. Which set of results for ER and PR is associated with the highest likelihood of a favorable response to treatment with estrogen-suppression therapy (tamoxifen)? A. ER positive, PR positive B. ER positive, PR negative C. ER negative, PR positive D. ER negative, PR negative Chemistry/Correlate clinical and

13. A Both ER and PR receptor assays are performed on breast tissue biopsies to determine the probability of response to tamoxifen. The PR receptor is produced from the ER receptor and expression of both predicts a positive response to the drug

13. Which condition produces the highest elevation of serum lactate dehydrogenase? A. Pernicious anemia B. Myocardial infarction C. Acute hepatitis D. Muscular dystrophy

13. A Serum LD levels are highest in pernicious anemia, reaching 10-50 times the upper reference limit (URL) as a result of intramedullary hemolysis. Moderate elevations (5-10 × URL) usually are seen in acute myocardial infarction, necrotic liver disease, and muscular dystrophy. Slight increases (2-3 × URL) are sometimes seen in obstructive liver disease

13. Which statement regarding measurement of Hgb A1c is true? A. Levels do not need to be done fasting B. Both the labile and stable Hgb A1c fractions are measured C. Samples should be measured within 2 hours of collection D. The assay must be done by chromatography

13. A Since Hgb A1C represents the average blood glucose 2-3 months prior to blood collection, the dietary status of the patient on the day of the test has no effect upon the results. Refrigerated whole-blood samples are stable for up to 1 week. Hgb A1C is assayed by cation exchange high-performance liquid chromatography or immunoassay (immunoturbidimetric inhibition) because both methods are specific for stable Hgb A1C, and do not demonstrate errors caused by abnormal hemoglobins, temperature of reagents, or fractions other than A1c.

13. SITUATION: Laboratory results on a patient from the emergency department are: glucose = Na = 155 mmol/L K = 1.2 mmol/L 1,100 mg/dL Cl = 115 mmol/L TCO2 = 3.0 mmol/L What is the most likely explanation of these results? A. Sample drawn above an IV B. Metabolic acidosis with increased anion gap C. Diabetic ketoacidosis D. Laboratory error measuring electrolytes caused by hyperglycemia

13. A These results are consistent with dilution of venous blood by intravenous fluid containing 5% dextrose and normal saline. The intravenous fluid is free of potassium and bicarbonate, accounting for the low level of these electrolytes (incompatible with life).

13. A solvent needed for HPLC requires a 20.0 mmol/L phosphoric acid buffer, pH 3.50, made by mixing KH2PO4 and H3PO4. How many grams of KH2PO4 are required to make 1.0 L of this buffer? Formula weights: KH2PO4 = 136.1; H3PO4 = 98.0; pKa H3PO4 = 2.12 A. 1.96 g B. 2.61 g C. 2.72 g D. 19.2 g

13. B The Henderson-Hasselbalch equation is used to calculate the ratio of salt to acid needed to give a pH of 3.50

13. The onset of menopause is usually associated with what hormone changes? A. Decreased estrogen, testosterone, and androgens B. Decreased estrogen, FSH, LH, and progesterone C. Decreased estrogen and progesterone, and increased LH and FSH D. Decreased estrogen and progesterone, normal LH and FSH

13. C In menopause, estrogen production decreases to where the menstrual cycle and ovarian follicle maturation stop. The decreased estrogen causes the pituitary release of LH and FSH. In menopause, the FSH at midcycle is higher than LH. The increased LH causes the ovaries to secrete testosterone and androgens

13. A decreased PAO2-PaO2 difference is found in: A. A/V (arteriovenous) shunting B. V/Q (ventilation/perfusion) inequality C. Ventilation defects D. All of these options

13. C Patients with A/V shunts, V/Q inequalities, and cardiac failure will have an increased PAO2-PaO2 difference. However, patients with ventilation problems have low alveolar PO2 owing to retention of CO2 in the airway. This reduces the PAO2-PaO2 difference

13. A sample of amniotic fluid collected for fetal lung maturity studies from a woman with a pregnancy compromised by hemolytic disease of the newborn (HDN) has a creatinine of 88 mg/dL. What is the most likely cause of this result? A. The specimen is contaminated with blood B. Bilirubin has interfered with the measurement of creatinine C. A random error occurred when the absorbance signal was being processed by the analyzer D. The fluid is urine from accidental puncture of the urinary bladder

13. D Creatinine levels in this range are found only in urine specimens. Adults usually excrete between 1.2 and 1.5 g of creatinine per day. For this reason, creatinine is routinely measured in 24-hour urine samples to determine the completeness of collection.

13. If the peak level is appropriate but the trough level too low at steady state, then the dose interval should: A. Be lengthened without changing the dose per day B. Be lengthened and dose rate decreased C. Not be changed, but dose per day increased D. Be shortened, but dose per day not changed

13. D Increasing the dose rate may result in peak drug levels in the toxic range. Decreasing the dosing interval will raise the trough level so that it is maintained in the therapeutic range. The trough level is affected by the drug clearance rate. If clearance increases, then trough level decreases.

13. Which of the following materials is best suited for verifying the wavelength calibration of a spectrophotometer? A. Neutral density filters B. Potassium dichromate solutions traceable to the National Bureau of Standards reference C. Wratten filters D. Holmium oxide glass

13. D Wavelength accuracy is verified by determining the wavelength reading that gives the highest absorbance (or transmittance) when a substance with a narrow natural bandpass (sharp absorbance or transmittance peak) is scanned. For example, didymium glass has a sharp absorbance peak at 585 nm. Therefore, an instrument should give its highest absorbance reading when the wavelength dial is set at 585 nm. Holmium oxide produces a very narrow absorbance peak at 361 nm; likewise, the hydrogen lamp of a UV spectrophotometer produces a 656-nm emission line that can be used to verify wavelength. Neutral density filters and dichromate solutions are used to verify absorbance accuracy or linearity. A Wratten filter is a widebandpass filter made by placing a thin layer of colored gelatin between two glass plates and is unsuitable for spectrophotometric calibration.

14. Which type of cancer is associated with the highest level of AFP? A. Hepatoma B. Ovarian cancer C. Testicular cancer D. Breast cancer

14. A AFP is increased in all persons with yolk sac tumors and over 80% of those with hepatoma. Levels above 1000 ng/mL are diagnostic of hepatoma. Ectopic AFP-secreting tumors are produced by ovarian, testicular, breast, GI, and bladder cancers, and these sources should be considered when 10-fold or higher elevations are seen in the absence of abnormal liver function. AFP

14. Which of the following statements is correct in assessing GH deficiency? A. Pituitary failure may involve one, several, or all adenohypophyseal hormones; but GH deficiency is usually found B. A normal random serum level of GH in a child under 6 years old rules out GH deficiency C. Administration of arginine, insulin, or glucagon will suppress GH release D. GH levels in the blood show little variation within a 24-hour period

14. A Because GH is the most abundant pituitary hormone, it may be used as a screening test for pituitary failure in adults. Pituitary hormone deficiencies are rare and are evaluated by measuring those hormones associated with the specific type of target organ dysfunction. GH secretion peaks during sleep, and pulsed increases are seen following exercise and meals.

14. If the steady-state drug level is too high, the best course of action is to: A. Decrease the dose B. Decrease the dose interval C. Decrease the dose and decrease the dose interval D. Change the route of administration

14. A Decreasing both dose and dosing interval will have offsetting effects on peak and trough blood levels. The appropriate dose can be calculated if the clearance or Vd and f are known. For example, the initial dose is calculated by multiplying the desired peak blood drug concentration by the Vd

14. The determination of the oxygen saturation of hemoglobin is best accomplished by: A. Polychromatic absorbance measurements of a whole-blood hemolysate B. Near infrared transcutaneous absorbance measurement C. Treatment of whole blood with alkaline dithionite prior to measuring absorbance D. Calculation using PO2 and total hemoglobin by direct spectrophotometry

14. A Measurement of oxyhemoglobin, deoxyhemoglobin (reduced hemoglobin), carboxyhemoglobin, methemoglobin, and sulfhemoglobin can be accomplished by direct spectrophotometry at multiple wavelengths and analysis of the absorptivity coefficients of each pigment at various wavelengths. The O2 saturation is determined by dividing the fraction of oxyhemoglobin by the sum of all pigments. This eliminates much of the error that occurs in the other methods when the quantity of an abnormal hemoglobin pigment is increased

14. Which analyte should be reported as a ratio using creatinine concentration as a reference? A. Urinary microalbumin B. Urinary estriol C. Urinary sodium D. Urinary urea

14. A Measurement of urinary microalbumin concentration should be reported as a ratio of albumin to creatinine (e.g., mg albumin per g creatinine). This eliminates the need for 24-hour collection in order to avoid variation caused by differences in fluid intake. A dry reagent strip test for creatinine is available that measures the ability of a creatinine-copper complex to break down H2O2 , forming a colored complex.

14. Why do many optical systems in chemistry analyzers utilize a reference light path? A. To increase the sensitivity of the measurement B. To minimize error caused by source lamp fluctuation C. To obviate the need for wavelength adjustment D. To reduce stray light effects

14. B A reference beam is used to produce an electrical signal at the detector to which the measurement of light absorption by the sample is compared. This safeguards against measurement errors caused power fluctuations that change the source lamp intensity. Although reference beams increase the accuracy of measurements, they do so at the expense of optical sensitivity since some of the incident light must be used to produce the reference beam

(see pic) 14. A procedure for cholesterol is calibrated with a serum-based cholesterol standard that was determined by the Abell-Kendall method to be 200.0 mg/dL. Assuming the same volume of sample and reagent are used, calculate the cholesterol concentration in the patient's sample from the following results. A. 123 mg/dL B. 172 mg/dL C. 232 mg/dL D. 314 mg/dL

14. B Cu = Au/As × Cs where Cu = concentration of unknown, Au = absorbance of unknown, As = absorbance of standard, and Cs = concentration of standard. Cu = 0.740/0.860 × 200 mg/dL = 172 mg/dL

14. Which stationary phase is used for the measurement of hemoglobin A1c by high performance liquid chromatography? A. Octadecylsilane (C18) B. Cation exchanger C. Anion exchanger D. Polystyrene divinylbenzene

14. B HPLC methods for measuring Hgb A1c are performed by diluting whole blood with an acid buffer that hemolyzes the sample. Normal hemoglobin A has a weak positive charge at an acidic pH and binds weakly to the resin. Glycated hemoglobin has an even weaker positive charge and is eluted before hemoglobin A. Abnormal hemoglobin molecules S, D, E, and C have a higher positive charge than hemoglobin A and are retained longer on the column. Elution is accomplished by increasing the ionic strength of the mobile phase. Cations in the buffer displace the hemoglobin pigments from the column

14. SITUATION: A plasma sample from a person in a coma as a result of an automobile accident gave the following results: Total CK 480 IU/L CK-MB 8 μg/L Myoglobin 800 μg/L Troponin I 0.02 μg/L What is the best interpretation of these results? A. The person had a heart attack that caused the accident B. The accident caused traumatic injury, but no heart attack occurred C. A heart attack occurred in addition to a stroke D. It is not possible to tell whether a heart attack occurred because of the extensive trauma

14. B The automobile accident caused both brain damage (coma) and muscle damage (myoglobin). The sandwich assay for MB uses antibodies to both the M and B subunits of CK-MB and therefore, is not subject to interference from CK-BB that could have resulted from brain injury. The CK relative index is 1.6, which is lower than would be expected if the CK-MB were derived from heart damage. Since the TnI is within normal limits, the slight increase in CK-MB is due to the gross release of CK from skeletal muscle.

14. In which condition is the LD most likely to be within normal limits? A. Hepatic carcinoma B. Pulmonary infarction C. Acute appendicitis D. Crush injury

14. C LD is increased slightly to moderately in most causes of liver disease. Smallest elevations are seen in obstructive jaundice and highest in hepatic carcinoma and toxic hepatitis, where levels can reach 10-fold the upper reference limit. LD is also increased in crush injury and muscular dystrophies due to skeletal muscle damage, and in pulmonary infarction owing to embolism formation. Amylase is increased in a majority of persons with acute appendicitis, but LD is not.

14. Which of the following conditions will prevent any migration of proteins across an electrophoretic support medium such as agarose? A. Using too high a voltage B. Excessive current during the procedure C. Loss of contact between a buffer chamber and the medium D. Evaporation of solvent from the surface of the medium

14. C Movement of proteins is dependent upon the presence of a salt bridge that allows current to flow via transport of ions to the electrodes across the support medium. If the salt bridge is not intact, there will be no migration, even if voltage is maintained across the electrodes. For agarose and cellulose acetate, heat causes evaporation of solvent from the buffer.

15. Correction of pH for a patient with a body temperature of 38°C would require: A. Subtraction of 0.015 B. Subtraction of 0.01% C. Addition of 0.020 D. Subtraction of 0.020

15. A The pH decreases by 0.015 for each degree Celsius above the 37°C. Because the blood gas analyzer measures pH at 37°C, the in vivo pH would be 0.015 pH units below the measured pH

15. Urea is produced from: A. The catabolism of proteins and amino acids B. Oxidation of purines C. Oxidation of pyrimidines D. The breakdown of complex carbohydrates

15. A Urea is generated by deamination of amino acids. Most is derived from the hepatic catabolism of proteins. Uric acid is produced by the catabolism of purines. Oxidation of pyrimidines produces orotic acid

15. Which statement best describes the level of GH in patients with pituitary adenoma associated with acromegaly? A. The fasting GH level is always elevated at least twofold B. Some patients will require a glucose suppression test to establish a diagnosis C. A normal fasting GH level rules out acromegaly D. Patients produce a lower concentration of insulin-like growth factor I (IGF-1) than expected from their GH level

15. B Approximately 90% of patients with acromegaly will have an elevated fasting GH level, but 10% will not. In addition, a single measurement is not sufficient to establish a diagnosis of acromegaly because various metabolic and nutritional factors can cause an elevated serum GH in the absence of pituitary disease.

15. SITUATION: A patient has the following electrolyte results: Na = 130 mmol/L K = 4.8 mmol/L Cl = 105 mmol/L TCO2 = 26 mmol/L Assuming acceptable QC, select the best course of action. A. Report these results B. Check the albumin, total protein, Ca, P, and Mg results; if normal, repeat the sodium test C. Request a new sample D. Recalibrate and repeat the potassium test

15. B The anion gap of this sample is < 4 mmol/L. This may result from laboratory error, retention of an unmeasured cation (e.g., calcium), or low level of unmeasured anion such as phosphorus or albumin. The sodium is inappropriately low for the chloride and bicarbonate and should be repeated if no biochemical cause is apparent

(see pic) 15. A glycerol kinase method for triglyceride calls for a serum blank in which normal saline is substituted for lipase in order to measure endogenous glycerol. Given the following results, and assuming the same volume of sample and reagent are used for each test, calculate the triglyceride concentration in the patient's sample. A. 119 mg/dL B. 131 mg/dL C. 156 mg/dL D. 180 mg/dL

15. B The serum blank absorbance is subtracted from the result for the patient's serum before applying the ratiometric formula to calculate concentration. Cu = [(Au - ASB)/As] × Cs where ASB = absorbance of serum blank = (0.750-0.100)/0.620 × 125 mg/dL = 131 mg/dL

15. Which component is required in a spectrophotometer in order to produce a spectral absorbance curve? A. Multiple monochromators B. A reference optical beam C. Photodiode array D. Laser light source

15. C There are two ways to perform spectral scanning for compound identification. One is to use a stepping motor that continuously turns the monochromator so that the wavelength aligned with the exit slit changes at a constant rate. A more practical method is to use a diode array detector. This consists of a chip embedded with as many as several hundred photodiodes. Each photodiode is aligned with a narrow part of the spectrum produced by a diffraction grating, and produces current proportional to the intensity of the band of light striking it (usually 1-2 nm in range). The diode signals are processed by a computer to create a spectral absorbance or transmittance curve

15. Evaluate the following chromatogram of a whole-blood hemolysate, and identify the cause and best course of action. A. Result is not reportable because hemoglobin F is present and interferes B. The result is not reportable because hemoglobin C is present and interferes C. The result is not reportable because labile hemoglobin A1c is present D. The result is reportable; neither hemoglobin F or C interfere

15. D The chromatogram is from a person with hemoglobin AC; however, hemoglobin C is completely separated from Hgb A1c and does not interfere. Hgb F is also present, but does not interfere unless its concentration is > 30%. Labile hemoglobin is formed initially when the aldehyde of glucose reacts with the N-terminal valine of the β globin chain. This Shiff base is reversible but is converted to Hgb A1c by rearrangement to a ketoamine. It is called labile A1c and produces a peak (LA1c) after HgF and before Hgb A1c. Therefore, it does not interfere

15. The LD pleural fluid:serum ratio for a transudative fluid is usually: A. 3:1 or higher B. 2:1 C. 1:1 D. 1:2 or less

15. D The lactate dehydrogenase activity of body fluids is normally less than serum, and a fluid to serum LD ratio greater than 1:2 is highly suggestive of an exudative process. Elevated lactate dehydrogenase in chest fluid is often caused by lung malignancy, metastatic carcinoma, Hodgkin's disease, and leukemia.

15. When should blood samples for trough drug levels be collected? A. 30 minutes after peak levels B. 45 minutes before the next dose C. 1-2 hours after the last dose D. Immediately before the next dose is given

15. D The trough concentration of a drug is the lowest concentration obtained in the dosing interval. This occurs immediately before the absorption of the next dose given. Trough levels are usually collected just before the next dose is given

16. The half-band width of a monochromator is defined by: A. The range of wavelengths passed at 50% maximum transmittance B. One-half the lowest wavelength of optical purity C. The wavelength of peak transmittance D. One-half the wavelength of peak absorbance

16. A Half-band width is a measure of bandpass made using a solution or filter having a narrow natural bandpass (transmittance peak). The wavelength giving maximum transmittance is set to 100%T (or 0 A). Then, the wavelength dial is adjusted downward, until a readout of 50%T (0.301 A) is obtained. Next, the wavelength is adjusted upward until 50%T is obtained. The wavelength difference is the half-band width. The narrower the half-band width, the better the photometric resolution of the instrument.

16. In which type of liver disease would you expect the greatest elevation of LD? A. Toxic hepatitis B. Alcoholic hepatitis C. Cirrhosis D. Acute viral hepatitis

16. A Liver disease produces an elevated LD-4 and LD-5. Levels may reach up to 10 times the URL in toxic hepatitis and in hepatoma. However, LD levels are lower in viral hepatitis (2-5 × URL), only slightly elevated in cirrhosis (2-3 × URL) and not significantly elevated in alcoholic liver disease

16. Blood sample collection time for peak drug levels: A. Varies with the drug, depending on its rate of absorption B. Is independent of drug formulation C. Is independent of the route of administration D. Is 30 minutes after a bolus intravenous injection is completed

16. A The peak concentration of a drug is the highest concentration obtained in the dosing interval. For oral drugs, the time of peak concentration is dependent upon their rates of absorption and elimination and is determined by serial blood measurements. Peak levels for oral drugs are usually drawn 1-2 hours after administration of the dose. For drugs given intravenously, peak levels are measured immediately after the infusion is completed

16. Select the anticoagulant of choice for blood gas studies. A. Sodium citrate 3.2% B. Lithium heparin 100 U/mL blood C. Sodium citrate 3.8% D. Ammonium oxalate 5.0%

16. B Heparin is the only anticoagulant that does not alter the pH of blood; heparin salts must be used for pH and blood gases. Solutions of heparin are air equilibrated and must be used sparingly to prevent contamination of the sample by gas in the solution

(see pic) 16. Which statement best describes the use of the Hgb A1C test? A. Should be used for monitoring glucose control only B. May be used for both diagnosis and monitoring C. Should be used only to monitor persons with type 1 diabetes D. May be used only to monitor persons with type 2 diabetes

16. B The ADA now recommends that the hemoglobin A1c test be used for both diagnosis and monitoring blood glucose levels. The cutpoint for diabetes is an A1c of 6.5. Persons with an A1c of 5.7%-6.4% are classified as being at high risk for diabetes within 5 years. An A1c between 4.0%-5.5% is defined as within normal limits

16. Hyperparathyroidism is most consistently associated with: A. Hypocalcemia B. Hypocalciuria C. Hypophosphatemia D. Metabolic alkalosis

16. C Hyperparathyroidism causes increased resorption of calcium and decreased renal retention of phosphate. Increased serum calcium leads to increased urinary excretion. The distal collecting tubule of the nephron reabsorbs less bicarbonate as well as phosphate, resulting in acidosis

16. A stat plasma lithium determined using an ion-selective electrode is measured at 14.0 mmol/L. Select the most appropriate course of action. A. Immediately report this result B. Check sample for hemolysis C. Call for a new specimen D. Rerun the lithium calibrators

16. C Lithium in excess of 2.0 mmol/L is toxic (in some laboratories 1.5 mmol/L is the upper therapeutic limit). A level of 14 mmol/L would not occur unless the sample were contaminated with lithium. This would most likely result from collection in a green-stoppered tube containing the lithium salt of heparin

16. A procedure for aspartate aminotransferase (AST) is performed manually because of a repeating error code for nonlinearity obtained on the laboratory's automated chemistry analyzer; 0.05 mL of serum and 1.0 mL of substrate are used. The reaction rate is measured at 30°C at 340 nm using a 1.0 cM light path, and the delta absorbance (-ΔA) per minute is determined to be 0.382. Based upon a molar absorptivity coefficient for NADH at 340 nm of 6.22 X 103 M-1 cM-1 L-1, calculate the enzyme activity in international units (IUs) per liter. A. 26 IU/L B. 326 IU/L C. 1228 IU/L D. 1290 IU/L

16. D An IU is defined as 1 μmol of substrate consumed or product produced per minute. The micromoles of NADH consumed in this reaction are determined by dividing the change in absorbance per minute by the absorbance of 1 μmol of NADH. Because 1 mol/L/cm would have an absorbance of 6.22 X 103 absorbance units, then 1 μmol/mL/cm would produce an absorbance of 6.22. Therefore, dividing the δA per minute by 6.22 gives the micromoles of NADH consumed in the reaction. This is multiplied by the dilution of serum to determine the micromoles per milliliter, and multiplied by 1,000 to convert to micromoles per liter.

16. Which of the following proteins migrates in the β region at pH 8.6? A. Haptoglobin B. Orosomucoprotein C. Antichymotrypsin D. Transferrin

16. D Transferrin, β lipoprotein, C3, and C4 are the dominant proteins in the β-globulin region. Haptoglobin and α2- macroglobulin are the principal proteins in the α2-fraction. α1-Antitrypsin, α1-lipoprotein, and α1-acid glycoprotein (orosomucoprotein) make up most of the α1-fraction. Immunoglobulins dominate the γ region. Plasma is not used for protein electrophoresis because fibrinogen will produce a band resembling a small monoclonal protein in the beta region

17. Which of the following is one advantage of high-resolution (HR) agarose electrophoresis over lower-current electrophoresis? A. High-resolution procedures detect monoclonal and oligoclonal bands at a lower concentration B. A smaller sample volume is used C. Results are obtained more rapidly D. Densitometric scanning of HR gels is more accurate Chemistry/

17. A HR agarose procedures use higher current and a cooling device to resolve 12 or more bands. Advantages include phenotyping of α1-antitrypsin (detection of Z and S variants), detection of β2 microglobulin in urine indicating tubular proteinuria (often associated with drug-induced nephrosis), and greater sensitivity detecting monoclonal gammopathies, immune complexes, and oligoclonal bands in CSF associated with multiple sclerosis. Its disadvantage is that densitometric scans of HR gels usually underestimate albumin.

17. Which of the following conditions will interfere with the measurement of LD? A. Slight hemolysis during sample collection B. Storage at 4°C for 3 days C. Storage at room temperature for 16 hours D. Use of plasma collected in heparin

17. A RBCs are rich in LD-1 and LD-2, and even slight hemolysis will falsely elevate results. Hemolytic, megaloblastic, and pernicious anemias are associated with LD levels of 10-50 times the URL. LD is stable for 2 days at room temperature or 1 week at 4°C; however, freezing causes deterioration of LD-5. The activity of LD is inhibited by EDTA, which binds divalent cations; serum or heparinized plasma should be used

17. When referring to quality control (QC) results, what parameter usually determines the acceptable range? A. The 95% confidence interval for the mean B. The range that includes 50% of the results C. The central 68% of results D. The range encompassed by ±2.5 standard deviations

17. A The acceptable range for quality control results is usually set at the 95% confidence interval. This is defined as the range between -1.96s and +1.96s. This means that we can expect a QC result to fall within this range 95 out of 100 times. For practical purposes, this is the same as ±2 s (95.4 out of 100 results should fall within ±2 s of the mean on the basis of chance).

17. A person has an elevated 24-hour urinary homovanillic acid (HVA) and vanillymandelic acid (VMA). Urinary metanephrines, chromogranin A, and neuron-specific enolase are also elevated but 5-hydroxyindoleacetic acid is within the reference range. What is the most likely diagnosis? A. Carcinoid tumors of the intestine B. Pheochromocytoma C. Neuroblastoma D. Pancreatic cancer

17. C Neuron-specific enolase is an isoenzyme containing two gamma polypeptides that are specific for nervous tissue and are found in neuroendocrine cells. Plasma levels are increased in neuroblastomas, carcinoid tumors, thyroid medullary carcinomas, and in some lung cancers and seminomas.

17. Which statement regarding the use of PTH is true? A. Determination of serum PTH level is the best screening test for disorders of calcium metabolism B. PTH levels differentiate primary and secondary causes of hypoparathyroidism C. PTH levels differentiate primary and secondary causes of hypocalcemia D. PTH levels are low in patients with pseudohypoparathyroidism

17. C Serum Cai is the best screening test to determine if a disorder of calcium metabolism is present, and will distinguish primary hyperparathyroidism (high Cai) and secondary hyperparathyroidism (low Cai). PTH levels are used to distinguish primary and secondary causes of hypocalcemia. Serum PTH is low in primary hypocalcemia (which results from parathyroid gland disease), but is high in secondary hypocalcemia (e.g., renal failure).

17. What is the maximum recommended storage time and temperature for an arterial blood gas sample drawn in a plastic syringe? (see pic for answers)

17. D Arterial blood gas samples collected in plastic syringes should be stored at room temperature because cooling the sample allows oxygen to enter the syringe. Storage time should be no more than 30 minutes because longer storage results in a significant drop in pH and PO2 and increased PCO2

17. Which could account for drug toxicity following a normally prescribed dose? A. Decreased renal clearance caused by kidney disease B. Discontinuance or administration of another drug C. Altered serum protein binding caused by disease D. All of these options

17. D Therapeutic drug monitoring is necessary for drugs that have a narrow therapeutic index. Individual differences alter pharmacokinetics, causing lack of correlation between dose and drug blood level. These include age, diet, ingestion with or without food, genetic factors, exercise, smoking, pregnancy, metabolism of other drugs, protein binding, and disease states

17. Which of the statements below about serum urea is true? A. Levels are independent of diet B. Urea is not reabsorbed by the renal tubules C. High BUN levels can result from necrotic liver disease D. BUN is elevated in prerenal as well as renal failure Chemistry

17. D Urea is completely filtered by the glomeruli but reabsorbed by the renal tubules at a rate dependent upon filtrate flow and tubular status. Urea levels are a sensitive indicator of renal disease, becoming elevated by glomerular injury, tubular damage, or poor blood flow to the kidneys (prerenal failure). Serum urea (and BUN) levels are influenced by diet and are low in necrotic liver disease

18. What is the recommended cutoff for the early detection of chronic kidney disease in diabetics using the test for microalbuminuria? A. >30 mg/g creatinine B. >80 mg/g creatinine C. >200 mg/g creatinine D. >80 mg/L

18. A Microalbuminuria is the excretion of small quantities of albumin in the urine. In diabetics, excretion of albumin that is within allowable limits for healthy persons may signal the onset of chronic kidney disease. The term microalbuminuria is defined as albumin excretion ≥ 30 mg/g creatinine but ≤ 300 mg/g creatinine. The use of the albumin to creatinine ratio is preferred to measures of albumin excretory rate (μg/min) because the latter is subject to error associated with timed specimen collection. ADA recommends the test be done annually for all type 2 diabetics and type 1 diabetics who have had the disease for > 5 years

18. SITUATION: An amylase result is 550 U/L. A 1:4 dilution of the specimen in NaCl gives 180 U/L (before mathematical correction for dilution). The dilution is repeated with the same results. The technologist should: A. Report the amylase as 550 U/L B. Report the amylase as 720 U/L C. Report the amylase as 900 U/L D. Dilute the sample 1:10 in distilled water and repeat

18. B A 1:4 dilution refers to 1 part serum and 3 parts diluent; the result is multiplied by 4 to determine the serum concentration. Serum may contain wheat germ gluten or other natural amylase inhibitors that, when diluted, result in increased enzyme activity. Serum for amylase should always be diluted with normal saline because chloride ions are needed for amylase activity.

18. A plasma sample is hemolyzed and turbid. What is required to perform a sample blank in order to correct the measurement for the intrinsic absorbance of the sample when performing a spectrophotometric assay? A. Substitute deionized water for the sample B. Dilute the sample 1:2 with a standard of known concentration C. Substitute saline for the reagent D. Use a larger volume of the sample

18. C A sample blank is used to subtract the intrinsic absorbance of the sample usually caused by hemolysis, icterus, turbidity, or drug interference. On automated analyzers, this is accomplished by measuring the absorbance after the addition of sample and a first reagent, usually a diluent. For tests using a single reagent, sample blanking can be done prior to the incubation phase before any color develops. Substituting deionized water for sample is done to subtract the absorbance of the reagent (reagent blanking). Diluting the sample with a standard (standard addition) may be done when the absorbance is below the minimum detection limit for the assay. Using a larger volume of sample will make the interference worse

18. A patient's BUN is 60 mg/dL and serum creatinine is 3.0 mg/dL. These results suggest: A. Laboratory error measuring BUN B. Renal failure C. Prerenal failure D. Patient was not fasting

18. C BUN is affected by renal blood flow as well as by glomerular and tubular function. When blood flow to the kidneys is diminished by circulatory insufficiency (prerenal failure), glomerular filtration decreases and tubular reabsorption increases due to slower filtrate flow. Because urea is reabsorbed, BUN levels rise higher than creatinine. This causes the BUN:creatinine ratio to be greater than 10:1 in prerenal failure

18. Which of the following conditions is associated with "β-γ bridging"? A. Multiple myeloma B. Malignancy C. Hepatic cirrhosis D. Rheumatoid arthritis

18. C Hepatic cirrhosis produces a polyclonal gammopathy associated with a high IgA level. This obliterates the valley between β and γ zones. Malignancy and rheumatoid arthritis produce polyclonal gammopathies classified as chronic inflammatory or delayed response patterns. Multiple myeloma produces a zone of restricted mobility usually in the γ, but sometimes in the β- or α2-region

18. A patient's blood gas results are as follows: pH = 7.26 dCO2 = 2.0 mmol/L HCO3 - = 29 mmol/L These results would be classified as: A. Metabolic acidosis B. Metabolic alkalosis C. Respiratory acidosis D. Respiratory alkalosis

18. C Imbalances are classified as respiratory when the primary disturbance is with PCO2 because PCO2 is regulated by ventilation. PCO2 = dCO2/0.03 or 60 mm Hg (normal 35-45 mm Hg). Increased dCO2 will increase hydrogen ion concentration, causing acidosis. Bicarbonate is moderately increased, but a primary increase in NaHCO3 causes alkalosis. Thus, the cause of this acidosis is CO2 retention (respiratory acidosis), and it is partially compensated by renal retention of bicarbonate

18. The best method of analysis for serum PTH involves using antibodies that detect: A. The amino-terminal fragment of PTH B. The carboxy-terminal end of PTH C. Both the amino-terminal fragment and intact PTH D. All fragments of PTH as well as intact hormone

18. C PTH is a polypeptide comprised of 84 amino acids. The biological activity of the hormone resides in the N-terminal portion of the polypeptide, but the hormone is rapidly degraded and produces N-terminal, middle, and C-terminal fragments. Fragments lacking the N-terminal portion are inactive.

18. In which of the following conditions is PSA least likely to be increased? A. Precancerous lesions of the prostate B. Postprostate biopsy C. Benign prostatic hypertrophy D. Post-digital rectal examination

18. D PSA is a serine protease responsible for liquefaction of the seminal fluid. PSA has been used successfully to monitor for recurrence and follow the response of patients to androgen-suppression therapy. Currently, it is one of the few FDA-approved tumor markers for cancer screening.

19. In the Oliver-Rosalki method for CK, adenosine monophosphate (AMP) is added to the substrate in order to: A. Inhibit adenylate kinase B. Block the oxidation of glutathione C. Increase the amount of ADP that is available D. Block the action of diadenosine pentaphosphate Chemistry/Apply principles of basic laboratory

19. A Positive interference in the Oliver-Rosalki method can occur when adenylate kinase is present in the serum from hemolysis or damaged tissue. Adenylate kinase hydrolyzes ADP, forming AMP and ATP (2 ADP AK AMP + ATP). This reaction is inhibited by adding AMP and diadenosine pentaphosphate (Ap5A) to the substrate

19. Which of the following conditions is cause for rejecting an analytical run? A. Two consecutive controls greater than 2 s above or below the mean B. Three consecutive controls greater than 1 s above the mean C. Four controls steadily increasing in value but less than ±1 s from the mean D. One control above +1 s and the other below -1 s from the mean

19. A Rejecting a run when three consecutive controls fall between 1 and 2 s or when a trend of four increasing or decreasing control results occurs would lead to frequent rejection of valid analytical runs. Appropriate control limits are four consecutive controls above or below 1 s (41s) to detect a significant shift, and a cusum result exceeding the ±2.7 s limit to detect a significant shift or trend. When controls deviate in opposite directions, the difference should exceed 4s before the run is rejected

19. SITUATION: A patient's biochemistry results are: ALT = 55 IU/L AST = 165 IU/L glucose = 87 mg/dL LD = 340 IU/L Na = 142 mmol/L K = 6.8 mmol/L Ca = 8.4 mg/dL Pi = 7.2 mg/dL Select the best course of action. A. Report results along with an estimate of the degree of hemolysis B. Repeat LD but report all other results C. Request a new sample D. Dilute the serum 1:2 and repeat AST and LD

19. A Results indicate a moderately hemolyzed sample. Because sodium, calcium, and glucose are not significantly affected, results should be reported along with an estimate of visible hemolysis. The physician may reorder affected tests of interest

19. A patient's blood gas results are: pH = 7.50 PCO2 = 55 mm Hg HCO3 - = 40 mmol/L These results indicate: A. Respiratory acidosis B. Metabolic alkalosis C. Respiratory alkalosis D. Metabolic acidosis

19. B A pH above 7.45 corresponds with alkalosis. Both bicarbonate and PCO2 are elevated. Bicarbonate is the conjugate base and is under metabolic (renal) control, while PCO2 is an acid and is under respiratory control. Increased bicarbonate (but not increased CO2) results in alkalosis; therefore, the classification is metabolic alkalosis, partially compensated by increased PCO2

19. Which of the following statements regarding PSA is true? A. Complexed PSA in plasma is normally less than free PSA B. Free PSA below 25% is associated with malignant disease C. A total PSA below 4 ng/mL rules out malignant disease D. A total PSA above 10 ng/mL is diagnostic of malignant disease

19. B In normal plasma, 55%-95% of the PSA is bound to protease inhibitors, primarily α1-antichymotrypsin, and the remainder is called free PSA. At a cutoff of 4 ng/mL commonly used for the upper reference limit, total PSA has a sensitivity of approximately 60%, and 22% of men with a PSA below 4 ng/mL have evidence of early prostate cancer on biopsy.

19. Which support medium can be used to determine the molecular weight of a protein? A. Cellulose acetate B. Polyacrylamide gel C. Agar gel D. Agarose gel

19. B Polyacrylamide gels separate by molecular sieving as well as charge. Sodium dodecyl sulfate (SDS) is a nonionic detergent that binds to proteins, neutralizing their charge. Polyacrylamide gel electrophoresis (PAGE) after treating with SDS separates proteins on the basis of molecular size. The smaller proteins become trapped in the pores of the gel and migrate more slowly.

19. Urinary urea measurements may be used for calculation of: A. Glomerular filtration B. Renal blood flow C. Nitrogen balance D. All of these options

19. C Because BUN is handled by the tubules, serum levels are not specific for glomerular filtration rate. Urea clearance is influenced by diet and liver function as well as renal function. Protein intake minus excretion determines nitrogen balance. A negative balance (excretion exceeds intake) occurs in stress, starvation, fever, cachexia, and chronic illness. Nitrogen balance = (Protein intake in grams per day ÷ 6.25) - (Urine urea nitrogen in grams per day + 4), where 4 estimates the protein nitrogen lost in the feces per day and dividing by 6.25 converts protein to protein nitrogen

19. Drugs rapidly infused intravenously usually follow which elimination model? A. One compartment, first order B. One compartment, logarithmic C. Biphasic or two compartment with serum level rapidly falling in the first phase D. Michaelis-Menton or concentration-dependent elimination

19. C Drugs rapidly infused intravenously follow a two-compartment model of elimination. The central compartment is the blood and tissues that are well perfused. The second consists of tissues for which distribution of drug is time dependent. In determining the loading dose, the desired serum concentration should be multiplied by the volume of the central compartment to avoid toxic levels

19. In addition to measuring blood glucose, Hgb A1c, and microalbumin, which test should be done on diabetic persons once per year? A. Urine glucose B. Urine ketones C. Plasma fructosamines D. Estimated glomerular filtration rate

19. D While urinary glucose can identify persons who may have diabetes, it is not sensitive enough to manage glucose control on a daily basis, and has been replaced by whole-blood glucose monitoring or continuous glucose monitoring

2. Which of the following statements describes a nonkinetic enzyme assay? A. Initial absorbance is measured followed by a second reading after 5 minutes B. Absorbance is measured at 10-second intervals for 100 seconds C. Absorbance is monitored continuously for 1 minute using a chart recorder D. Reflectance is measured from a xenon source lamp pulsing at 60 Hz Chemistry/Apply principles

2. A A kinetic assay uses several evenly spaced absorbance measurements to calculate the change in absorbance per unit time. A constant change in absorbance per unit of time occurs only when the rate of the reaction is zero order (independent of substrate concentration). Enzyme activity is proportional to rate only under zero-order conditions

2. Which of two instruments can be assumed to have the narrower bandpass? Assume that wavelength is accurately calibrated. A. The instrument giving the highest absorbance for a solution of 0.1 mmol/L NADH at 340 nm B. The instrument giving the lowest %T for a solution of nickel sulfate at 700 nm C. The instrument giving the highest %T reading for 1.0% v/v HCl at 350 nm D. The instrument giving the most linear plot of absorbance versus concentration

2. A Bandpass is defined by the range of wavelengths passed through the sample at the specified wavelength setting. It can be measured using any solution having a narrow absorbance peak (e.g., NADH at 340 nm). The instrument producing the purest monochromatic light will have the highest absorbance reading

2. In general, in which of the following situations is the analysis of a tumor marker most useful? A. Testing for recurrence B. Prognosis C. Screening D. Diagnosis

2. A Most tumor markers are expressed at very low levels so that the concentration in early malignancy overlaps that seen in normal individuals. This makes them ineffective for screening. Three exceptions are hCG in males for testicular cancer, calcitonin for thyroid medullary cancer, and prostate-specific antigen (PSA) for prostate cancer

2. Creatinine is considered the substance of choice to measure endogenous renal clearance because: A. The rate of formation per day is independent of body size B. It is completely filtered by the glomeruli C. Plasma levels are highly dependent upon diet D. Clearance is the same for both men and women

2. B Creatinine concentration is dependent upon muscle mass, but varies by less than 15% per day. Creatinine is not metabolized by the liver, or dependent on diet, and is 100% filtered by the glomeruli. It is not reabsorbed significantly but is secreted slightly, especially when filtrate flow is slow. Plasma creatinine and cystatin C are the two substances of choice for evaluating the glomerular filtration rate (GFR

2. Which of the following hormones promotes hyperglycemia? A. Calcitonin B. Growth hormone C. Aldosterone D. Renin

2. B Growth hormone and cortisol promote gluconeogenesis and epinephrine stimulates glycogenolysis. Excess thyroid hormone causes hyperglycemia by increasing glucagon and inactivation of insulin, thereby promoting both gluconeogenesis and glycogenolysis. An increase in any of these hormones can cause hyperglycemia. Calcitonin opposes the action of parathyroid hormone. Aldosterone is the primary mineralocorticoid hormone and stimulates sodium reabsorption and potassium secretion by the kidneys. Renin is released from the kidney due to ineffective arterial pressure and promotes activation of angiotensinogen and aldosterone secretion.

2. What is the PO2 of calibration gas containing 20.0% O2, when the barometric pressure is 30 in.? A. 60 mm Hg B. 86 mm Hg C. 143 mm Hg D. 152 mm Hg

2. C Convert barometric pressure in inches to mm Hg by multiplying by 25.4 (mm/in.). Next, subtract the vapor pressure of H2O at 37°C, 47 mm Hg, to give dry gas pressure. Multiply dry gas pressure by the %O2: 25.4 mm/in. × 30 in. = 762 mm Hg 762 mm Hg - 47 mm Hg (vapor pressure) = 715 mm Hg (dry gas pressure) 0.20 × 715 mm Hg = 143 mm Hg PO2

2. How many milliliters of glacial acetic acid are needed to prepare 2.0 L of 10.0% v/v acetic acid? A. 10.0 mL B. 20.0 mL C. 100.0 mL D. 200.0 mL

2. D The expression percent v/v refers to the volume of one liquid in mL present in 100.0 mL of solution. To calculate, multiply the percentage (as mL) by the volume required (mL), then divide by 100 (mL). (10.0 mL × 2000.0 mL) ÷ 100.0 mL = 200.0 mL To prepare 2.0 L of a 10.0% v/v solution of acetic acid, add approximately 1.0 L of deionized H2O to a 2.0-L volumetric flask. Add 200.0 mL of glacial acetic acid and mix. Then, add sufficient deionized H2O to bring the meniscus to the 2.0-L line and mix again.

20. Which fact must be considered when evaluating a patient who displays signs of drug toxicity? A. Drug metabolites (e.g., N-acetylprocainamide) may need to be measured as well as parent drug B. If the concentration of total drug is within therapeutic limits, the concentration of free drug cannot be toxic C. If the drug has a wide therapeutic index, then it will not be toxic D. A drug level cannot be toxic if the trough is within the published therapeutic range

20. A Altered drug pharmacokinetics may result in toxicity even when the dose of drug is within the accepted therapeutic range. Two common causes of this are the presence of unmeasured metabolites that are physiologically active, and the presence of a higher than expected concentration of free drug. Because only free drug is physiologically active, decreased binding protein or factors that shift the equilibrium favoring more unbound drug can result in toxicity when the total drug concentration is within the therapeutic range.

20. Which is normally the most abundant corticosteroid hormone secreted by the adrenal cortex? A. Cortisol B. Dehydroepiandrosterone C. Aldosterone D. Corticosterone

20. A Cortisol is the most abundant adrenal hormone, and abnormal levels have pronounced effects on carbohydrate and lipid metabolism. Cortisol is a 21-carbon steroid with a dihydroxyacetone group at C17 and hydroxyl group at C11 that account for its glucocorticoid potency

20. A 55-year-old male with early stage prostate cancer diagnosed by biopsy had his prostate gland removed (simple prostatectomy). His PSA prior to surgery was 10.0 ng/mL. If the surgery was successful in completely removing the tumor cells, what would the PSA result be 1 month after surgery? A. Undetectable B. 1-3 ng/mL C. Less than 4 ng/mL D. Less than 10 ng/mL

20. A If the tumor were confined to the prostate, the PSA would be undetectable 1 month following successful surgery, since there is no other tissue source of PSA. The half-life of PSA is 2.2-3.2 days, and the minimum detection limit of most assays is 0.2 ng/mL or lower

20. Which of the following stains is used for lipoprotein electrophoresis? A. Oil Red O B. Coomassie Brilliant Blue C. Amido Black D. Ponceau S

20. A Oil Red O and Sudan Black B stain neutral fats and are used to stain lipoproteins as well as fat in urine or stool. The other stains are used for proteins. Coomassie Brilliant Blue is more sensitive than Ponceau S or Amido Black, and all three stains have slightly greater affinity for albumin than globulins. In addition, silver nitrate may be used to stain CSF proteins because it has greater sensitivity than the other stains.

20. One of two controls within a run is above +2s and the other control is below -2s from the mean. What do these results indicate? A. Poor precision has led to random error (RE) B. A systematic error (SE) is present C. Proportional error is present D. QC material is contaminated

20. A When control results deviate from the mean in opposite directions, the run is affected by RE, which results from imprecision. An analytical run is rejected when two controls within the same run have an algebraic difference in excess of 4s (R4s). The R4S rule is applied only to controls within a run (Level 1 - Level 2), never across runs or days

20. BUN is determined electrochemically by coupling the urease reaction to measurement of: A. Potential with a urea-selective electrode B. The timed rate of increase in conductivity C. The oxidation of ammonia D. Carbon dioxide

20. B A conductivity electrode is used to measure the increase in conductance of the solution as urea is hydrolyzed by urease in the presence of sodium carbonate. Urea + H2O →2NH3 + CO2 2NH3 + 2H2O + Na2CO3→ 2NH4 ++CO3 -2 + 2NaOH

20. A blood sample is left on a phlebotomy tray for 4.5 hours before it is delivered to the laboratory. Which group of tests could be performed? A. Glucose, Na, K, Cl, TCO2 B. Uric acid, BUN, creatinine C. Total and direct bilirubin D. CK, ALT, ALP, AST

20. B Glucose in serum is metabolized by cells at a rate of about 7% per hour. Bilirubin levels will fall if the sample is exposed to sunlight. Transaminases should be measured within 4 hours and ALP within 2 hours if the sample is stored at room temperature. Uric acid, BUN, and creatinine are least likely to be affected

20. Which set of results is consistent with uncompensated respiratory alkalosis? A. pH 7.70 HCO3 30 mmol/L PCO2 25 mm Hg B. pH 7.66 HCO3 22 mmol/L PCO2 20 mm Hg C. pH 7.46 HCO3 38 mmol/L PCO2 55 mm Hg D. pH 7.36 HCO3 22 mmol/L PCO2 38 mm Hg

20. B Respiratory alkalosis is caused by hyperventilation, inducing low PCO2. Very often, in the early phase of an acute respiratory disturbance, the kidneys have not had time to compensate, and the bicarbonate is within normal limits. In answer A, the bicarbonate is high and PCO2 low; thus, both are contributing to alkalosis and this would be classified as a combined acid-base disturbance. In answer C, the pH is almost normal, and both bicarbonate and PCO2 are increased. This can occur in the early stage of a metabolic acid- base disturbance when full respiratory compensation occurs or in a combined acid-base disorder. In answer D, both bicarbonate and PCO2 are within normal limits (22-26 mmol/L, 35-45 mm Hg, respectively) as is the pH

20. Which substance is used in the CK assay to activate the enzyme? A. Flavin adenine dinucleotide (FAD) B. Imidazole C. N-acetylcysteine D. Pyridoxyl-5´-phosphate Chemistry/Apply principles

20. C In addition to Mg+2, CK requires a thiol compound to reduce interchain disulfide bridges and bind heavy metals that inactivate the enzyme. N-acetylcysteine is an activator of CK used for this purpose in the IFCC recommended method. Pyridoxyl-5´-phosphate is a prosthetic group of AST and ALT. FAD is a prosthetic group of glucose oxidase. Imidazole is used to buffer the CK reagent

20. Which statement regarding reflectometry is true? A. The relation between reflectance density and concentration is linear B. Single-point calibration can be used to determine concentration C. 100% reflectance is set with an opaque film called a white reference D. The diode array is the photodetector of choice

20. C Reflectometry does not follow Beer's law, but the relationship between concentration and reflectance can be described by a logistic formula or algorithm that can be solved for concentration. For example, K/S = (1 - R)2/2R, where K = Kubelka-Munk absorptivity constant, S = scattering coefficient, R = reflectance density. K/S is proportional to concentration. The white reference is analogous to the 100%T setting in spectrophotometry and serves as a reference signal. Dr = log R0/R1, where Dr is the reflectance density, R0 is the white reference signal, and R1 is the photodetector signal for the test sample

21. Bichromatic measurement of absorbance can correct for interfering substances if: A. The contribution of the interferent to absorbance is the same at both wavelengths B. Both wavelengths pass through the sample simultaneously C. The side band is a harmonic of the primary wavelength D. The chromogen has the same absorbance at both wavelengths

21. A In bichromatic photometry, the absorbance of sample is measured at two different wavelengths. The primary wavelength is at or near the absorbance maximum. An interfering substance having the same absorbance at both primary and secondary (side band) wavelengths does not affect the absorbance difference (Ad).

21. Which of the following statements regarding adrenal cortical dysfunction is true? A. Patients with Cushing's syndrome usually have hyperkalemia B. Cushing's syndrome is associated with glucose intolerance C. Addison's disease is associated with hypernatremia D. Addison's disease is caused by elevated levels of cortisol

21. B Patients with Cushing's syndrome have elevated levels of cortisol and other adrenal corticosteroids. This causes the characteristic cushingoid appearance that includes obesity, acne, and humpback posture. Osteoporosis, hypertension, hypokalemia, and glycosuria are characteristics

21. Which of the following is the reference method for measuring serum glucose? A. Somogyi-Nelson B. Hexokinase C. Glucose oxidase D. Glucose dehydrogenase

21. B The hexokinase method is considered more accurate than glucose oxidase methods because the coupling reaction using glucose-6-phosphate dehydrogenase (G-6-PD) is highly specific. The hexokinase method may be done on serum or plasma collected using heparin, EDTA, fluoride, oxalate, or citrate. The method can also be used for urine, cerebrospinal fluid, and serous fluids.

21. An HPLC assay for procainamide gives an internal standard peak that is 15% greater in area and height for sample 1 than sample 2. The technologist should suspect that: A. The column pressure increased while sample 2 was being analyzed B. Less recovery from sample 2 occurred in the extraction step C. The pH of the mobile phase increased during chromatography of sample 2 D. There was more procainamide in sample 1 than sample 2

21. B The internal standard compensates for variation in extraction, evaporation, reconstitution, and injection volume. The same amount of internal standard is added to all samples and standards prior to assay. Increased column pH or pressure usually alters retention time, and may not affect peak quantitation

21. When a therapeutic drug is suspected of causing toxicity, which specimen is the most appropriate for an initial investigation? A. Trough blood sample B. Peak blood sample C. Urine at the time of symptoms D. Gastric fluid at the time of symptoms

21. B When a drug is suspected of toxicity, the peak blood sample (sample after absorption and distribution are complete) should be obtained because it is most likely to exceed the therapeutic limit. If the peak level is above the upper therapeutic limit, then toxicity is confirmed, and the drug dose is lowered

21. Which of the following serum protein electrophoresis results suggests an acute inflammatory process? (see pic for answers)

21. C Acute inflammation is characterized by increased production of acute phase proteins. These include α1-antitrypsin, α1-acid glycoprotein, α1-antichymotrypsin, and haptoglobin. Albumin is slightly decreased. γ- and β-fractions are normal

21. SITUATION: A specimen for CK performed on an automated analyzer using an optimized Oliver-Rosalki method gives an error flag indicating substrate depletion. The sample is diluted 1:2 and 1:4 by the serial dilution technique and reassayed. After correcting for the dilution, the results are as follows: 1:2 Dilution = 3,000 IU/L 1:4 Dilution = 3,600 IU/L Dilutions are made a second time and assayed again but give identical results. What is the most likely explanation? A. The serum became contaminated prior to making the 1:4 dilution B. The wrong pipet was used to make one of the dilutions C. An endogenous competitive inhibitor is present in the serum D. An error has been made in calculating the enzyme activity of one of the two dilutions

21. C When a competitive inhibitor is present in the serum, a dilution of the sample will cause an increase in the reaction rate by reducing the concentration of the inhibitor. Dilution of serum frequently increases the activity of CK and amylase. The same effect will occur when a smaller volume of serum is used in the assay because less inhibitor will be present in the reaction mixture.

21. Which of the following will shift the O2 dissociation curve to the left? A. Anemia B. Hyperthermia C. Hypercapnia D. Alkalosis

21. D A left shift in the oxyhemoglobin dissociation curve signifies an increase in the affinity of Hgb for O2. This occurs in alkalosis, hypothermia, and in those hemoglobinopathies such as Hgb Chesapeake that increase the binding of O2 to heme. A right shift in the oxyhemoglobin dissociation curve lowers the affinity of Hgb for O2. This occurs in anemia due to increased 2,3-diphosphoglycerate (2,3-DPG), with increased body temperature, increased hydrogen ion concentration, hypercapnia (increased PCO2), and in some hemoglobinopathies, such as Hgb Kansas

21. Two consecutive controls are both beyond -2s from the mean. How frequently would this occur on the basis of chance alone? A. 1:100 B. 5:100 C. 1:400 D. 1:1,600

21. D QC results follow a Gaussian or normal distribution. Ninety-five percent of the results fall within ±2s of the mean; therefore, 2.5 out of 100 (1:40) are above +2s and 2.5 out of 100 are below -2s. The probability of two consecutive controls being beyond -2s is the product of their individual probabilities. 1/40 × 1/40 = 1/1,600 trials by chance

22. In which circumstance will the reporting of calculated oxygen saturation of hemoglobin based on PO2, PCO2, pH, temperature, and hemoglobin be in error? A. Carbon monoxide poisoning B. Diabetic ketoacidosis C. Patient receiving oxygen therapy D. Assisted ventilation for respiratory failure

22. A CO has about 200 times the affinity as O2 for hemoglobin and will displace O2 from hemoglobin at concentrations that have no significant effect on the PAO2. Consequently, calculated oxygen saturation will be erroneously high. Other cases in which the calculated O2Sat should not be used include any hemoglobinopathy that affects oxygen affinity and methemoglobinemia. The other situations above affect the O2 saturation of hemoglobin in a manner that can be predicted by the effect of pH, PO2, and PCO2 on the oxyhemoglobin dissociation curve.

22. Which of the following conditions is usually associated with an acute inflammatory pattern? A. Myocardial infarction (MI) B. Malignancy C. Rheumatoid arthritis D. Hepatitis

22. A MI produces a pattern of acute inflammation usually associated with tissue injury. This pattern results from production of acute phase proteins including α1-antitrypsin, α1-antichymotrypsin, and haptoglobin. It is also seen in early infection, pregnancy, and early nephritis. Malignancy, rheumatoid arthritis, and hepatitis are associated with a chronic inflammatory pattern. This differs from the acute pattern by the addition of a polyclonal gammopathy

22. For a drug that follows first-order pharmacokinetics, adjustment of dosage to achieve the desired blood level can be made using which formula? (see pic for answers)

22. A Most drugs follow first-order pharmacokinetics, meaning the clearance of drug is linearly related to the drug dose. The dose of such drugs can be adjusted by multiplying the ratio of the current dose to blood concentration by the desired drug concentration, provided the blood concentration is measured at steady state

22. Which of the following statements about cortisol in Cushing's syndrome is true? A. Twenty-four-hour urinary free cortisol is a more sensitive test than plasma total cortisol B. Patients with Cushing's disease show pronounced diurnal variation in serum cortisol C. Free cortisol is increased by a high-serum cortisol-binding protein concentration D. An elevated serum total cortisol level is diagnostic of Cushing's syndrome

22. A Serum cortisol can be increased by factors such as stress, medications, and cortisol-binding protein, and the cortisol level of normal patients will overlap those seen in Cushing's syndrome because of pulse variation. When cortisol levels become elevated, cortisol-binding protein becomes saturated, and free (unbound) cortisol is filtered by the glomeruli

22. Polarographic methods for glucose analysis are based upon which principle of measurement? A. Nonenzymatic oxidation of glucose B. The rate of O2 depletion C. Chemiluminescence caused by formation of adenosine triphosphate (ATP) D. The change in electrical potential as glucose is oxidized

22. B Polarographic glucose electrodes measure the consumption of O2 as glucose is oxidized. Glucose oxidase in the reagent catalyzes the oxidation of glucose by O2 under first-order conditions, forming hydrogen peroxide (H2O2).

22. The term R4S means that: A. Four consecutive controls are greater than ±1 standard deviation from the mean B. Two controls in the same run are greater than 4s units apart C. Two consecutive controls in the same run are each greater than ±4s from the mean D. There is a shift above the mean for four consecutive controls

22. B The R4s rule is applied to two control levels within the same run. The rule is violated when the algebraic difference between them (level 1 - level 2) exceeds 4s. The rule is never applied across different runs. The R4s rule detects random error (error due to poor precision

22. After staining a silica gel plate to determine the L/S ratio, the technologist notes that the lipid standards both migrated 1 cm faster than usual. The technologist should: A. Repeat the separation on a new silica gel plate B. Check the pH of the developing solvent C. Prepare fresh developing solvent and repeat the assay D. Reduce solvent migration time for all subsequent runs

22. C TLC plates migrate in solvent until the front comes to 1 cm of the top of the plate. Separation of lipids on silica gel is based upon adsorption. Higher Rf values indicate greater solubility of lipids in the developing solvent. This may be caused by evaporation of H2O, lowering the polarity of the solvent

22. SITUATION: A physician calls to request a CK on a sample already sent to the laboratory for coagulation studies. The sample is 2-hour-old citrated blood and has been stored at 4°C. The plasma shows very slight hemolysis. What is the best course of action and the reason for it? A. Perform the CK assay on the sample because no interferent is present B. Reject the sample because it is slightly hemolyzed C. Reject the sample because it has been stored too long D. Reject the sample because the citrate will interfere

22. D CK activity is lost with excessive storage, the most labile isoenzyme being CK-1. However, CK in serum is stable at room temperature for about 4 hours and up to 1 week at 4°C provided that an optimized method is used. Slight hemolysis does not interfere because CK is absent from RBCs.

23. Which of the following conditions is characterized by primary hyperaldosteronism caused by adrenal adenoma, carcinoma, or hyperplasia? A. Cushing's syndrome B. Addison's disease C. Conn's syndrome D. Pheochromocytoma

23. C Conn's syndrome is characterized by hypertension, hypokalemia, and hypernatremia with increased plasma and urine aldosterone and decreased renin. Cushing's syndrome results from excessive production of cortisol, and Addison's disease from deficient production of adrenal corticosteroids

23. Which of the following statements about fluorometry is accurate? A. Fluorometry is less sensitive than spectrophotometry B. Fluorometry is less specific than spectrophotometry C. Unsaturated cyclic molecules are often fluorescent D. Fluorescence is directly proportional to temperature Chemistry/Apply principles

23. C Increasing temperature results in more random collision between molecules by increasing their motion. This causes energy to be dissipated as heat instead of fluorescence. Temperature is inversely proportional to fluorescence. Fluorescence is more sensitive than spectrophotometry because the detector signal can be amplified when dilute solutions are measured. It is also more specific than spectrophotometry because both the excitation and emission wavelengths are characteristics of the compound being measured

23. A quantitative urine glucose was determined to be 160 mg/dL by the Trinder glucose oxidase method. The sample was refrigerated overnight. The next day, the glucose is repeated and found to be 240 mg/dL using a polarographic method. What is the most likely cause of this discrepancy? A. Poor precision when performing one of the methods B. Contamination resulting from overnight storage C. High levels of reducing substances interfering with the Trinder reaction D. Positive interference in the polarographic method caused by hematuria

23. C Urine often contains high levels of ascorbate and other reducing substances. These may cause significant negative bias when measuring glucose using a peroxidase-coupled method. The reductants compete with chromogen for H2O2.

23. Which would be consistent with partially compensated respiratory acidosis? A.pH PCO2 Bicarbonate increased increased increased B.pH PCO2 Bicarbonate increased decreased decreased C.pH PCO2 Bicarbonate decreased decreased decreased D.pH PCO2 Bicarbonate decreased increased increased

23. D Acidosis = low pH; respiratory = disturbance of PCO2; a low pH is caused by increased PCO2. In partially compensated respiratory acidosis, the metabolic component of the buffer system, bicarbonate, is retained. This helps to compensate for retention of PCO2 by titrating hydrogen ions. The compensatory component always moves in the same direction as the cause of the acid-base disturbance

23. For which drug group are both peak and trough measurements usually required? A. Antiarrhythmics B. Analgesics C. Tricyclic antidepressants D. Aminoglycoside antibiotics

23. D Aminoglycoside antibiotics cause damage to the eighth cranial nerve at toxic levels, resulting in hearing loss. When given at subtherapeutic doses, they fail to resolve infection. Most drugs falling in the other classes have a narrow peak-trough difference but are highly toxic when blood levels exceed the therapeutic range. Usually, these can be safely monitored by measuring trough levels

24. In most circumstances, when two controls within a run are both greater than ±2s from the mean, what action should be taken first? A. Recalibrate, then repeat controls followed by selected patient samples if quality control is acceptable B. Repeat the controls before taking any corrective action C. Change the reagent lot, then recalibrate D. Prepare fresh standards and recalibrate

24. A When a 22s rule is broken an SE is present and corrective action is required (repeating just the QC will not correct the problem). If

24. Which of the following statements about TLC for drug screening is true? A. Acidic drugs are extracted in an alkaline nonpolar solvent B. A drug is identified by comparing its Rf value and staining to standards C. Testing must be performed using a urine sample D. Opiates and other alkaloids are extracted at an acid pH

24. B TLC can be performed on urine, serum, or gastric fluid and qualitatively identifies most drugs. Each has a characteristic Rf, which is the ratio of the distance migrated by the drug to the solvent. The Rf of the sample must match the Rf of the drug standard. Extraction of drugs for TLC is highly pH dependent. The pH must be adjusted to reduce the solubility (ionization) of the drug in the aqueous phase. Usually, alkaline drugs (e.g., opiates) are extracted at pH 9.0 and acidic drugs (e.g., barbiturates) at pH 4.5.

24. Which condition results in metabolic acidosis with severe hypokalemia and chronic alkaline urine? A. Diabetic ketoacidosis B. Phenformin-induced acidosis C. Renal tubular acidosis D. Acidosis caused by starvation

24. C Metabolic acidosis can be caused by any condition that lowers bicarbonate. In nonrenal causes, the kidneys will attempt to compensate by increased acid excretion. However, in renal tubular acidosis (RTA), an intrinsic defect in the tubules prevents bicarbonate reabsorption. This causes alkaline instead of acidic urine. Excretion of bicarbonate as potassium bicarbonate (KHCO3) results in severe hypokalemia

24. What is the clinical utility of testing for serum prealbumin? A. Low levels are associated with increased free cortisol B. High levels are an indicator of acute inflammation C. Serial low levels indicate compromised nutritional status D. Levels correlate with glomerular injury in patients with diabetes mellitus

24. C Prealbumin (also called transthyretin) is a small protein with a half-life of only 2 days. Serum levels fall rapidly in patients with deficient protein nutrition. As a result, prealbumin is used to detect malnutrition and to measure the patient's response to dietary supplementation. The cutpoint used to identify nutritional deficiency in elderly patients is usually 11 mg/dL. Prealbumin is usually measured by immunonephelometry

24. Which of the following is the most common cause of Cushing's syndrome? A. Pituitary adenoma B. Adrenal hyperplasia C. Overuse of corticosteroids D. Ectopic adrenocorticotropic hormone (ACTH) production by tumors

24. C The most common cause of Cushing's syndrome is the administration of medications with cortisol or glucocorticoid activity. Excluding iatrogenic causes, approximately 60%-70% of Cushing's syndrome results from hypothalamic-pituitary misregulation and is called Cushing's disease.

24. Select the enzyme that is most specific for β-D-glucose. A. Hexokinase B. G-6-PD C. Phosphohexisomerase D. Glucose oxidase

24. D Glucose oxidase is the most specific enzyme reacting with only β-D-glucose. However, the peroxidase coupling reaction used in the glucose oxidase method is subject to positive and negative interference. Therefore, hexokinase is used in the reference method.

25. When establishing QC limits, which of the following practices is inappropriate? A. Using last month's QC data to determine current target limits B. Exclusion of any QC results greater than ±2s from the mean C. Using control results from all shifts on which the assay is performed D. Using limits determined by reference laboratories using the same method

25. B Data between ±2 and ±3s must be included in calculations of the next month's acceptable range. Elimination of these values would continuously reduce the distribution of QC results, making "out-of-control" situations a frequent occurrence. Generally, QC results greater than 3s are not used to calculate next month's mean

25. A patient's CK-MB is reported as 18 μg/L and the total CK as 560 IU/L. What is the CK relative index (CKI)? A. 0.10% B. 3.2% C. 10.0% D. 30.0%

25. B The CKI is an expression of the percentage of the total CK that is attributed to CK-MB. CKI = (CK-MB in μg/L)/(Total CK in IU/L) x 100 The reference range is 0%-2.5%. Values above 2.5% point to an increase in CK-MB from cardiac muscle.

25. The EMIT for drugs of abuse uses an: A. Antibody conjugated to a drug B. Enzyme conjugated to an antibody C. Enzyme conjugated to a drug D. Antibody bound to a solid phase

25. C In EMIT, enzyme-labeled drug competes with drug in the sample for a limited amount of reagent antibodies. When antibody binds to the enzyme-drug conjugate, it blocks the catalytic site of the enzyme. Enzyme activity is directly proportional to sample drug concentration because the quantity of unbound drug-enzyme conjugate will be highest when drug is present in the sample

25. Which substance is used to generate the light signal in electrochemiluminescence? A. Acridinium B. Luminol C. Dioxetane phosphate D. Ruthenium

25. D All of these substances are chemiluminescent. Dioxetane phosphate is excited by alkaline phosphatase. Acridinium and luminol are excited by hydrogen peroxide. In electrochemiluminesence, ruthenium is used to label antibody or antigen. Antigen-antibody complexes containing the ruthenium label are bound to paramagnetic particles via a strepavidin-biotin reaction. The paramagnetic particles are attracted to an electrode surface. The flowcell is washed with a solution containing tripropylamine (TPA) to remove unbound ruthenium label. At the electrode surface, the TPA is oxidized and the electrons excite the ruthenium, causing production of 620-nm light

25. Which aminoaciduria results in the overflow of branched chain amino acids? A. Hartnup's disease B. Alkaptonuria C. Homocystinuria D. Maple syrup urine disease

25. D Valine, leucine, and isoleucine accumulate due to branched-chain decarboxylase deficiency in maple syrup urine disease. These are transaminated to ketoacids that are excreted, giving urine a maple sugar odor. Alkaptonuria is caused by homogentisic acid oxidase deficiency, causing homogentisic aciduria.

(see pic for answers) 26. Which of the following assays has the poorest precision?

26. A Although calcium has the lowest s, it represents the assay with poorest precision. Relative precision between different analytes or different levels of the same analyte must be evaluated by the coefficient of variation (CV) because standard deviation is dependent upon the mean. CV = s × 100/Mean

26. In a nonmyocardial as opposed to a myocardial cause of an increased serum or plasma CK-MB, which would be expected? A. An increase in CK-MB that is persistent B. An increase in the percent CK-MB as well as concentration C. The presence of increased TnI D. A more modest increase in total CK than CK-MB Chemistry/Evaluate

26. A Plasma CK-MB becomes abnormal 4 hours postinfarction, peaks in 16-20 hours, and usually returns to normal within 48 hours. In some noncardiac causes of elevated plasma CK-MB such as muscular dystrophy, there is a persistent elevation of both total CK and CK-MB. TnI and TnT are cardiac-specific markers.

26. Light scattering when the wavelength is greater than 10 times the particle diameter is described by: A. Rayleigh's law B. The Beer-Lambert law C. Mie's law D. The Rayleigh-Debye law

26. A Rayleigh's law states that when the incident wavelength is much longer than the particle diameter, there is maximum backscatter and minimum right-angle scatter. The Rayleigh-Debye law predicts maximum right-angle scatter when wavelength and particle diameter approach equality. In nephelometry, the relationship between wavelength and diameter determines the angle at which the detector is located

26. Which statement about EMIT is true? A. Enzyme activity is inversely proportional to drug level B. Formation of NADH is monitored at 340 nm C. ALP is the commonly used conjugate D. Assay use is restricted to serum

26. B EMIT is a homogenous immunoassay, meaning that free antigen does not have to be separated from bound antigen. Most EMIT assays use a two-reagent system. Reagent A contains substrate (usually glucose-6-PO4), coenzyme (NAD+), and antibody to the drug. Reagent B contains enzyme-labeled drug (usually G-6-PD-drug) and buffer

26. Which glucose method is subject to falsely low results caused by ascorbate? A. Hexokinase B. Glucose dehydrogenase C. Trinder glucose oxidase D. Polarography Chemistry/Apply knowledge

26. C Although glucose oxidase is specific for β-D-glucose, the coupling (indicator) reaction is prone to negative interference from ascorbate, uric acid, acetoacetic acid, and other reducing agents

26. Serum protein and immunofixation electrophoresis are ordered on a patient. The former is performed, but there is no evidence of a monoclonal protein. Select the best course of action. A. Perform quantitative Ig G, A, M B. Perform the IFE on the serum C. Report the result; request a urine sample for protein electrophoresis D. Perform IFE on the serum and request a urine sample for IFE Chemistry/Evaluate

26. C An area of restricted mobility should be identified on serum protein electrophoresis before IFE is performed. About one out of four patients with multiple myeloma have monoclonal free λ or κ chains in urine only, and therefore, urine electrophoresis should be included in initial testing.

26. In which situation is the plasma or 24-hour urinary cortisol not consistent with the clinical picture? A. In pregnant patients B. In patients with a positive overnight dexamethasone suppression test C. In congenital adrenal hyperplasia D. In Cushing's syndrome caused by ectopic ACTH producing tumors

26. C Congenital adrenal hyperplasia (adrenogenital syndrome) results from a deficiency of an enzyme required for synthesis of cortisol. Approximately 90% of cases are caused by a deficiency of 21-hydroxylase blocking conversion of 17-α hydroxyprogesterone to 11-deoxycortisol. Most other cases are caused by 11-hydroxylase deficiency, which blocks conversion of 11-deoxycortisol to cortisol

26. In addition to phenylketonuria, maple syrup urine disease, and homocystinuria, what other aminoaciduria can be detected by tandem MS? A. Alkaptonuria B. Hartnup disease C. Citrullinemia D. Cystinuria

26. C Most states use electrospray ionization tandem-mass spectroscopy (MS/MS), which can detect over 20 inborn errors of metabolism from a single blood spot. Typically, this includes phenylketonuria, tyrosinemia, maple syrup urine disease, homocystinuria, citrullinemia, and argininosuccinate acidemia. The latter two are errors of the urea cycle

26. Which of the following disorders is associated with lactate acidosis? A. Diarrhea B. Renal tubular acidosis C. Hypoaldosteronism D. Alcoholism

26. D Lactate acidosis often results from hypoxia, which causes a deficit of nicotinamide adenine dinucleotide, the oxidized form (NAD+). This promotes the reduction of pyruvate to lactate, regenerating NAD+ needed for glycolysis. In alcoholic acidosis, oxidation of ethanol to acetaldehyde consumes the NAD+. In diabetes, lactate acidosis can result from depletion of Krebs cycle intermediates. D

27. Quantitative determination of Hgb A2 and Hgb F are best performed by: A. High-performance liquid chromatography B. Alkali denaturation C. Electrophoresis D. Direct bichromatic spectrophotometry

27. A Hgb A2 and Hgb F are often quantitated to diagnose persons with thalassemia. The method of choice is HPLC using cation exchange chromatography. Hemoglobins are eluted from the column in order of increasing positive charge using a sodium phosphate buffer to produce a gradient of increasing ionic strength. Hemoglobin F elutes from the column earlier than Hgb A2 because it is less positively charged.

27. Which of the following is the primary mechanism of compensation for metabolic acidosis? A. Hyperventilation B. Release of epinephrine C. Aldosterone release D. Bicarbonate excretion Chemistry/Apply knowledge

27. A In metabolic acidosis, the respiratory center is stimulated by chemoreceptors in the carotid sinus, causing hyperventilation. This results in increased release of CO2. Respiratory compensation begins almost immediately unless blocked by pulmonary disease or respiratory therapy. Hyperventilation can bring the PCO2 down to approximately 10-15 mm Hg.

27. Which statement regarding cloned enzyme donor immunoassay (CEDIA) is true? A. The enzyme used is glucose-6-phosphate dehydrogenase B. The enzyme donor and acceptor molecules are fragments of β-galactosidase C. Drug concentration is inversely related to fluorescence D. The antibody is covalently linked to the enzyme donor

27. B CEDIA is a homogenous enzyme immunoassay that is commonly used to measure drugs of abuse. Drug conjugated to a fragment of β-galactosidase that is catalytically inactive competes with drug in the sample for a limited number of antibodies to the drug. The fragment, called the enzyme donor (ED), and substrate (chlorophenol red-β-D-galactopyranose) are mixed with the sample. A

27. Which statement best describes the clinical utility of plasma or serum myoglobin? A. Levels greater than 100 μg/L are diagnostic of AMI B. Levels below 100 μg/L on admission and 2-4 hours postadmission help to exclude a diagnosis of AMI C. Myoglobin peaks after the cardiac troponins but is more sensitive D. The persistence of myoglobin > 110 μg/L for 3 days following chest pain favors a diagnosis of AMI

27. B Myoglobin is a heme-containing pigment in both skeletal and cardiac muscle cells. The upper limit of normal is approximately 90 μg/L for males and 75 μg/L for females. The plasma myoglobin is a sensitive marker for AMI. Over 95% of affected persons have a value higher than the cutoff (typically >110 μg/L).

27. Which of the following is a potential source of error in the hexokinase method? A. Galactosemia B. Hemolysis C. Sample collected in fluoride D. Ascorbic acid

27. B The hexokinase method can be performed on serum or plasma using heparin, EDTA, citrate, or oxalate. RBCs contain glucose-6-PO4 and intracellular enzymes that generate NADH, causing positive interference.

27. Which test is used to distinguish Cushing's disease (pituitary Cushing's) from Cushing's syndrome caused by adrenal tumors? A. Low-dose overnight dexamethasone suppression B. Petrosal sinus sampling C. Serum ACTH D. Twenty-four-hour urinary free cortisol

27. C Serum ACTH assays are very helpful in distinguishing the cause of Cushing's syndrome. Patients with adrenal tumors have values approaching zero. Patients with ectopic ACTH tumors have values greater than 200 pg/dL. Fifty percent of patients with Cushing's disease have high 8 a.m. ACTH levels (between 100-200 pg/dL).

27. Which statement regarding nephelometry is true? A. Nephelometry is less sensitive than absorption spectrophotometry B. Nephelometry follows Beer's law C. The optical design is identical to a turbidimeter except that a HeNe laser light source is used D. The detector response is directly proportional to concentration

27. D In nephelometry, the detector output is proportional to concentration (as opposed to turbidimetry where the detector is behind the cuvette). The detector(s) is (are) usually placed at an angle between 25° and 90° to the incident light, depending upon the application. Nephelometers, like fluorometers, are calibrated to read zero with the light path blocked, and sensitivity can be increased up to 1,000 times by amplification of the detector output or increasing the photomultiplier tube dynode voltage

28. Which is the most widely used screening test for Cushing's syndrome? A. Overnight low-dose dexamethasone suppression test B. Corticotropin-releasing hormone stimulation test C. Petrosal sinus sampling D. Metyrapone stimulation test

28. A Dexamethasone is a synthetic corticosteroid that exhibits 30-fold greater negative feedback on the hypothalamus than cortisol. When an oral dose of 1 mg of the drug is given to a patient at 11 p.m., the 8 a.m. serum total cortisol level should be below 5.0 μg/dL. Patients with Cushing's syndrome almost always exceed this cutoff

28. Which statement about glucose in cerebrospinal fluid (CSF) is correct? A. Levels below 40 mg/dL occur in septic meningitis, cancer, and multiple sclerosis B. CSF glucose is normally the same as the plasma glucose level C. Hyperglycorrhachia is caused by dehydration D. In some clinical conditions, the CSF glucose can be greater than the plasma glucose

28. A High glucose in CSF is a reflection of hyperglycemia and not central nervous system disease. The CSF glucose is usually 50%-65% of the plasma glucose. Low levels are significant and are most often associated with bacterial or fungal meningitis, malignancy in the central nervous system, and some cases of subarachnoid hemorrhage, rheumatoid arthritis, and multiple sclerosis

28. Which of the following plots is best for detecting all types of QC errors? A. Levy-Jennings B. Tonks-Youden C. Cusum D. Linear regression

28. A The Levy-Jennings plot is a graph of all QC results with concentration plotted on the y axis and run number on the x axis. The mean is at the center of the y axis, and concentrations corresponding to -2 and +2s are highlighted. Results are evaluated for multirule violations across both levels and runs. Corrective action for shifts and trends can be taken before QC rules are broken

28. The purpose of the nebulizer in an atomic absorption spectrophotometer that uses a flame is to: A. Convert ions to atoms B. Cause ejection of an outer shell electron C. Reduce evaporation of the sample D. Burn off organic impurities

28. A The atomizer of the atomic absorption spectrophotometer consists of either a nebulizer and flame or a graphite furnace. The nebulizer disperses the sample into a fine aerosol, distributing it evenly into the flame. Heat from the flame is used to evaporate water and break the ionic bonds of salts, forming ground state atoms. The flame also excites a small percentage of the atoms, which release a characteristic emission line

28. What is the typical time course for plasma myoglobin following an AMI? A. Abnormal before 1 hour; peaks within 3 hours; returns to normal in 8 hours B. Abnormal within 3 hours; peaks within 6 hours; returns to normal in 18 hours C. Abnormal within 2 hours; peaks within 12 hours; returns to normal in 36 hours D. Abnormal within 6 hours; peaks within 24 hours; returns to normal in 72 hours

28. C After AMI, myoglobin usually rises above the cutoff within 1-2 hours, peaks within 8-12 hours and returns to normal within 36 hours. Typically, levels reach a peak concentration that is 10-fold the upper reference limit. Since myoglobin is the first marker to become abnormal after an AMI, it should be measured on admission and if negative, again 2 hours later.

28. A technologist is asked to use the serum from a clot tube left over from a chemistry profile run at 8 a.m. for a stat ionized calcium (Cai) at 11 a.m. The technologist should: A. Perform the assay on the 8 a.m. sample B. Perform the test only if the serum container was tightly capped C. Perform the assay on the 8 a.m. sample only if it was refrigerated D. Request a new sample

28. D Cai is pH dependent. Heparinized blood is preferred because it can be assayed immediately. Serum may be used, but the specimen must remain tightly capped while clotting and centrifuging, and analyzed as soon as possible

29. A flameless atomic absorption spectrophotometer dehydrates and atomizes a sample using: A. A graphite capillary furnace B. An electron gun C. A thermoelectric semiconductor D. A thermospray platform

29. A Flameless atomic absorption uses a hollow tube of graphite with quartz ends. The tube is heated in stages by an electric current to successively dry, ash, and atomize the sample. During the ash and atomization steps, argon is injected into the tube to distribute the atoms. The furnace is more sensitive than a flame atomizer and more efficient in atomizing thermostable salts. However, it is prone to greater matrix interference and is slower than the flame atomizer because it must cool down before introduction of the next sample

29. Quantitation of a drug by gas chromatography-mass spectroscopy (GC-MS) is usually performed in which mode? A. Total ion chromatography B. Selective ion monitoring C. Ion subtraction D. Selective reaction monitoring

29. B Most GC-MS instruments use an electron beam to split the drug emerging from the column into its component ions. These are drawn into the mass analyzer, usually a vacuum chamber containing two pairs of charged rods (a positive pair and a negative pair) called a quadrupole analyzer.

29. Which of the following plots is best for comparison of precision and accuracy among laboratories? A. Levy-Jennings B. Tonks-Youden C. Cusum D. Linear regression

29. B The Tonks-Youden plot is used for interlaboratory comparison of monthly means. The method mean for level 1 is at the center of the y axis and mean for level 2 at the center of the x axis. Lines are drawn from the means of both levels across the graph, dividing it into four equal quadrants. If a laboratory's monthly means both plot in the lower left or upper right, then systematic error (SE) exists in its method.

29. In peroxidase-coupled glucose methods, which reagent complexes with the chromogen? A. Nitroprusside B. Phenol C. Tartrate D. Hydroxide

29. B The coupling step in the Trinder glucose oxidase method uses peroxidase to catalyze the oxidation of a dye by H2O2. Dyes such as 4-aminophenozone or 4-aminoantipyrine are coupled to phenol to form a quinoneimine dye that is red and is measured at about 500 nm.

29. What is the typical time course for plasma TnI or TnT following an AMI? A. Abnormal within 3 hours; peaks within 12 hours; returns to normal in 24 hours B. Abnormal within 4 hours; peaks within 18 hours; returns to normal in 48 hours C. Abnormal within 4 hours; peaks within 24 hours; returns to normal in 1 week D. Abnormal within 6 hours; peaks within 36 hours; returns to normal in 5 days

29. C Troponin is a complex of three polypeptides that function as a regulator of actin and tropomyosin. The three subunits are designated TnC, TnI, and TnT. All are present in both cardiac and some skeletal muscles, but cardiac and skeletal isoforms of TnI and TnT can be differentiated by specific antiseras. TnI and TnT cardiac isoforms in plasma will at least double within 4-6 hours after MI, peak within 24 hours and usually remain elevated for 7-10 days.

29. Which test is the most specific for establishing a diagnosis of Cushing's disease (pituitary Cushing's)? A. Low-dose dexamethasone suppression B. High-dose dexamethasone suppression C. Twenty-four-hour urinary free cortisol D. Petrosal sinus sampling following corticotropin-releasing hormone stimulation Chemistry/Correlate clinical and laboratory

29. D Although dexamethasone suppression tests have a high sensitivity, some patients without Cushing's syndrome have indeterminate results (e.g., values between 5 and 10 μg/dL) or abnormal results owing to medications or other conditions. When corticotropin-releasing hormone is given intravenously, patients with Cushing's disease have an exaggerated ACTH response

3. Which of the following enzymes is increased in persons with prostate and small-cell lung cancer? A. Creatine kinase-1 (CK-1) B. Gamma glutamyl transferase (GGT) C. Amylase D. Lactate dehydrogenase

3. A CK-1 (CK-BB) is not normally found in plasma or serum except in neonates. It may be present in persons with central nervous system damage and some other disorders but its presence is often associated with various malignancies, especially prostate cancer and small-cell carcinoma of the lung

3. The term pharmacodynamics is an expression of the relationship between: A. Dose and physiological effect B. Drug concentration at target sites and physiological effect C. Time and serum drug concentration D. Blood and tissue drug levels

3. B Pharmacodynamics is the relationship between the drug concentration at the receptor site (tissue concentration) and the response of the tissue to that drug. For example, the relationship between lidocaine concentration in the heart muscle and the duration of the action potential of Purkinje fibers

3. Which statement regarding creatinine is true? A. Serum levels are elevated in early renal disease B. High serum levels result from reduced glomerular filtration C. Serum creatine has the same diagnostic utility as serum creatinine D. Serum creatinine is a more sensitive measure of renal function than creatinine clearance

3. B Serum creatinine is a specific but not a sensitive measure of glomerular function. About 60% of the filtration capacity of the kidneys is lost when serum creatinine becomes elevated. Because urine creatinine diminishes as serum creatinine increases in renal disease, the creatinine clearance is more sensitive than serum creatinine in detecting glomerular disease. A creatinine clearance below 60 mL/min indicates loss of about 50% functional nephron capacity and is classified as moderate (stage 3) chronic kidney disease

3. Which of the following statements regarding enzymatic reactions is true? A. The enzyme shifts the equilibrium of the reaction to the right B. The enzyme alters the equilibrium constant of the reaction C. The enzyme increases the rate of the reaction D. The enzyme alters the energy difference between reactants and products

3. C An enzyme will accelerate the rate of a reaction, reducing the time required to reach equilibrium. The concentration of reactants and products at equilibrium will be the same with or without the enzyme.

3. Which statement about the biuret reaction for total protein is true? A. It is sensitive to protein levels below 0.1 mg/dL B. It is suitable for urine, exudates, and transudates C. Polypeptides and compounds with repeating imine groups react D. Hemolysis will not interfere

3. C The biuret reaction is not sensitive to protein levels below 0.1 g/dL and, therefore, is not sensitive enough for assays of total protein in CSF, urine, or transudates. Slight hemolysis does not cause falsely high results, if the absorbance of the Cu+2-protein complexes is measured bichromatically. However, frankly hemolyzed samples contain sufficient globin to cause positive interference

3. A lipemic sample gives a sodium of 130 mmol/L on an analyzer that uses a 1:50 dilution of serum or plasma before introducing it to the ion selective electrodes. The same sample gives a sodium of 142 mmol/L using a direct (undiluted) ion selective electrode. Assuming acceptable quality control, which of the following is the most appropriate course of action? A. Report a sodium result of 136 mmol/L B. Ultracentrifuge the sample and repeat by ISE C. Dilute the sample 1:4 and repeat by ISE D. Report the undiluted ion selective electrode result

3. D Lipemic samples give lower results for sodium (pseudohyponatremia) when diluted prior to measurement because the H2O phase is mostly diluent and a significant component of the sample volume is displaced by lipid. Direct ISEs measure sodium in the plasma water, more accurately reflecting patient status

3. Select the hormone which when elevated is associated with galactorrhea, pituitary adenoma, and amenorrhea. A. E2 B. Progesterone C. Follicle-stimulating hormone (FSH) D. Prolactin

3. D Serum prolactin may be increased from hypothalamic dysfunction or pituitary adenoma. When levels are greater than five times the URL, a pituitary tumor is suspected. Prolactin is measured by enzyme immunoassay (EIA).

3. Which of the following is characteristic of type 1 diabetes mellitus? A. Requires an oral glucose tolerance test for diagnosis B. Is the most common form of diabetes mellitus C. Usually occurs after age 40 D. Requires insulin replacement to prevent ketosis

3. D Type 1, or juvenile, diabetes is also called insulindependent diabetes because patients must be given insulin to prevent ketosis. Type 1 accounts for only about 10%-20% of cases of diabetes mellitus, and is usually diagnosed by a fasting plasma glucose. Two consecutive results ≥126 mg/dL is diagnostic. Approximately 95% of patients produce autoantibodies against the beta cells of the pancreatic islets. Other autoantibodies may be produced against insulin, glutamate decarboxylase, and tyrosine phosphorylase IA2. There is genetic association between type 1 diabetes and human leukocyte antigens (HLA) DR3 and DR4

30. Which of the following statements about the diagnosis of Addison's disease is true? A. Patients with primary Addison's disease show a normal response to ACTH stimulation B. Primary and secondary Addison's disease can often be differentiated by plasma ACTH C. Twenty-four-hour urinary free cortisol is normal in Addison's disease D. Pituitary ACTH reserves are normal in secondary Addison's disease

30. B ACTH (Cortrosyn) stimulation is used as a screening test for Addison's disease. A 250-μg dose of Cortrosyn is given intravenously. Normal patients show a 2-5 times increase in serum cortisol. A subnormal response occurs in both primary and secondary Addison's disease. Plasma ACTH is high in primary but is low in secondary Addison's disease. Patients with secondary Addison's disease (pituitary failure) do not respond to metyrapone because their ACTH reserve is diminished.

30. Point-of-care-tests (POCTs) for whole-blood glucose monitoring are based mainly on the use of: A. Glucose oxidase as the enzyme B. Amperometric detection C. Immunochromatography D. Peroxidase coupling reactions

30. B All POCT devices for monitoring blood glucose use either glucose dehydrogenase (GDH) or glucose oxidase and are amperometric. For glucose oxidase methods, the electrons derive from the oxidation of hydrogen peroxide

30. A gastric fluid from a patient suspected of having taken an overdose of amphetamine is sent to the laboratory for analysis. The technologist should: A. Perform an EMIT assay for amphetamine B. Refuse the sample and request serum or urine C. Dilute 1:10 with H2O and filter; perform TLC for amphetamines D. Titrate to pH 7.0, then follow procedure for measuring amphetamine in urine

30. C The gastric sample can be measured by TLC, but such a sample should not be used in place of serum or urine without documentation of acceptability by the reagent manufacturer or laboratory. A positive amphetamine result by a screening test such as TLC or immunoassay may be caused by a related drug which interferes, and therefore, the result should be confirmed by GC-MS if there is a medicolegal implication.

30. SITUATION: A urine sample is received in the laboratory with the appropriate custody control form, and a request for drug of abuse screening. Which test result would be cause for rejecting the sample? A. Temperature after collection 95°F B. pH 5.0 C. Specific gravity 1.005 D. Creatinine 5 mg/dL

30. D Approximately 5 per 1,000 urine samples received for DAU testing have been adulterated by either dilution, substitution, or addition of substances such as glutaraldehyde that interfere with testing

31. Which of the following conditions is classified as normochloremic acidosis? A. Diabetic ketoacidosis B. Chronic pulmonary obstruction C. Uremic acidosis D. Diarrhea

31. A Bicarbonate deficit will lead to hyperchloremia unless the bicarbonate is replaced by an unmeasured anion. In diabetic ketoacidosis, acetoacetate and other ketoacids replace bicarbonate. The chloride remains normal or low and there is an increased anion gap

31. SITUATION: Results of biochemistry tests are: Na = 138 mmol/L K = 4.2 mmol/L Cl = 94 mmol/L TCO2 = 20 mmol/L glucose = 100 mg/dL T bili = 1.2 mg/dL BUN = 6.8 mg/dL creat = 1.0 mg/dL albumin = 4.8 g/dL T protein = 5.1 g/dL What should be done next? A. Request a new specimen B. Repeat the total protein C. Repeat all tests D. Perform a protein electrophoresis

31. B All results are normal except total protein. The albumin level cannot be 94% of the total protein, and a random error in total protein measurement should be assumed

31. What is the recommended troponin T and I cutoff (upper limit of normal) for detecting myocardial infarction? A. The cutoff varies with the method of assay but should be no lower than 0.2 ng/mL B. The upper 99th percentile or lowest level that can be measured with 10% CV C. The concentration corresponding to the lowest level of calibrator used D. The highest value fitting under the area of the curve for the 95% confidence interval

31. B The American College of Cardiology recommends the cutoff for an abnormal troponin test be set at the 99th percentile of the normal population, 0.013 ng/mL, or if the assay precision at this level is >10% then the cutoff should be the lowest value measurable with a CV of 10% (typically 0.03 ng/mL).

31. Which of the following statements regarding the catecholamines is true? A. They are derived from tryptophan B. They are produced by the zona glomerulosa of the adrenal cortex C. Plasma levels show both diurnal and pulsed variation D. They are excreted in urine primarily as free catecholamines

31. C Catecholamines—epinephrine, norepinephrine, and dopamine—are produced from the amino acid tyrosine by the chromaffin cells of the adrenal medulla. Plasma and urinary catecholamines are measured in order to diagnose pheochromocytoma. Symptoms include hypertension, headache, sweating, and other endocrine involvement

31. SITUATION: A sample for ammonia assay is taken from an IV line that had been capped and injected with lithium heparin (called a heparin lock). The sample is drawn in a syringe containing lithium heparin, and immediately capped and iced. The plasma is separated and analyzed within 20 minutes of collection, and the result is 50 μg/dL higher than one measured 4 hours before. What is the most likely explanation of these results? A. Significantly greater physiological variation is seen with patients having systemic, hepatic, and gastrointestinal diseases B. The syringe was contaminated with ammonia C. One of the two samples was collected from the wrong patient D. Stasis of blood in the line caused increased ammonia

31. D Falsely elevated blood ammonia levels are commonly caused by improper specimen collection. Venous stasis and prolonged storage cause peripheral deamination of amino acids, causing a falsely high ammonia level. Plasma is the sample of choice since ammonia levels increase with storage.

31. What effect does hematocrit have on POCT tests for whole-blood glucose monitoring? A. Low hematocrit decreases glucose readings on all devices B. High hematocrit raises glucose readings on all devices C. The effect is variable and dependent on the enzyme/coenzyme system D. Low hematocrit raises readings and high hematocrit lowers readings unless corrected

31. D Hematocrit affects POCT glucose measurements. High hematocrit lowers the glucose because RBC glucose concentration is lower than plasma concentration.

31. All of the following are requirements for a QC material except: A. Long-term stability B. The matrix is similar to the specimens being tested C. The concentration of analytes reflects the clinical range D. Analyte concentration must be independent of the method of assay

31. D Quality control materials are stable, made of the same components as the specimen, cover the dynamic linear range of the assay, and can be used for multiple analytes. The target mean for QC samples is determined from replicate assays by the user's method, not the "true" concentration of the analyte. Out-of-control results are linked to analytic performance rather than to the inherent accuracy of the method.

31. Which substance has the longest detection time? A. Amphetamines B. Cocaine C. Benzodiazepines D. Marijuana

31. D Some drugs have a long half-life, and can be detected for longer periods after use, but the detection window also depends on other variables such as dosage, frequency of use, and method sensitivity. Marijuana is stored in fatty tissue and is metabolized slowly.

32. Examine the Levy-Jennings chart at the bottom of the previous page and identify the QC problem that occurred during the first half of the month. A. Shift B. Trend C. Random error D. Kurtosis

32. B A trend is characterized by six consecutive decreasing or increasing control results. The value for both controls becomes progressively higher from day 4 to day 9. Trends are caused by changes to the test system that increase over time, such as deterioration of reagents or calibrators, progressive changes in temperature, evaporation, light exposure, and bacterial contamination.

(see pic) 32. The following chart compares the monthly total bilirubin mean of Laboratory A to the monthly mean of Laboratory B, which uses the same control materials, analyzer, and method. Both laboratories performed controls at the beginning of each shift using commercially prepared liquid QC serum stored at -20°C. Which of the following conditions would explain these differences? A. Improper handling of the control material by Laboratory A resulted in loss of bilirubin due to photodegradation B. The laboratories used a different source of bilirubin calibrator C. Laboratory B obtained higher results because its precision was poorer D. Carryover from another reagent falsely elevated the results of Laboratory B Chemistry/Evaluate data to dete

32. B Interlaboratory variation in bilirubin results is often caused by differences in the assigned value of the calibrator used. Bilirubin calibrators are either serum-based material that have been reference assayed or unconjugated bilirubin stabilized by addition of alkali and albumin. C

32. Which of the following cardiac markers is consistently increased in persons who exhibit unstable angina? A. Troponin C B. Troponin T C. CK-MB D. Myoglobin

32. B Persons with unstable angina (angina at rest) who have an elevated TnT or TnI are at eight times greater risk of having an MI within the next 6 months. This property is being used to identify short-term risk patients who should be considered for coronary angioplasty. The reference range for troponin is very low (0-0.03 ng/mL); persons with unstable angina usually have values between 0.04 and 0.1 ng/mL without clinical evidence of AMI

32. All of the following are required when measuring magnesium by atomic absorption spectrophotometry except: A. A hollow cathode lamp with a magnesium cathode B. A chopper to prevent optical interference from magnesium emission C. A monochromator to isolate the magnesium emission line at 285 nm D. A 285-nm reference beam to correct for background absorption

32. D Atomic absorption requires a lamp with a cathode made from the metal to be assayed. The lamp emits the line spectrum of the metal, providing the wavelength that the atoms can absorb. The chopper pulses the source light, allowing it to be discriminated from light emitted by excited atoms. A monochromator eliminates light emitted by the ideal gas in the lamp. Deuterium (wide bandpass light) or Zeeman correction (splitting the incident light into side bands by a magnetic field) may be used to correct for background absorption

32. Which assay using 24-hour urine is considered the best single screening test for pheochromocytoma? A. Total urinary catecholamines B. VMA C. Homovanillic acid (HVA) D. Metanephrines

32. D Catecholamines are metabolized to metanephrines and VMA. Urinary catecholamines are increased by exercise and dietary ingestion. Measurement of 24-hour urinary metanephrine is about 95% sensitive for pheochromocytoma, and is the best single test. Specificity and sensitivity for detecting pheochromocytoma approach 100% when both VMA and metanephrines are measured.

32. Which of the following is classified as a mucopolysaccharide storage disease? A. Pompe's disease B. von Gierke disease C. Hers' disease D. Hurler's syndrome

32. D Hurler's syndrome is an autosomal recessive disease resulting from a deficiency of iduronidase. Glycosaminoglycans (mucopolysaccharides) accumulate in the lysosomes

33. Which of the following statements about blood alcohol measurement is correct? A. Symptoms of intoxication usually begin when the level exceeds 0.05% w/v B. The skin puncture site should be disinfected with isopropanol C. The reference method is based upon enzymatic oxidation of ethanol by alcohol dehydrogenase D. Gas chromatography methods require extraction of ethanol from serum

33. A Alcohol dehydrogenase is not specific for ethanol, and in vitro interference can occur with some ADH methods when skin is disinfected with other alcohols.

33. Which of the following conditions is associated with hyperuricemia? A. Renal failure B. Chronic liver disease C. Xanthine oxidase deficiency D. Paget's disease of the bone

33. A Excessive retention of uric acid results from renal failure and diuretics (or other drugs) that block uric acid excretion. Hyperuricemia may result from overproduction of uric acid in primary essential gout or excessive cell turnover associated with malignancy and chemotherapy.

33. Which statement best describes immunofixation electrophoresis (IEF)? A. Proteins are separated by electrophoresis followed by overlay of monospecific anti-immunoglobulins B. Proteins react with monospecific antisera followed by electrophoresis C. Antisera are electrophoresed, then diffused against patient's serum D. Serum is electrophoresed; the separated immunoglobulins diffuse against specific antisera placed into troughs

33. A Immunofixation electrophoresis (IFE) is used to identify monoclonal bands in serum or urine. Electrophoresis is performed on the serum or urine sample in the same manner as for protein electrophoresis, except that six lanes are used for the same sample. After the proteins are separated, a different monospecific antiserum is applied across the surface of each lane

33. Referring to the Levy-Jennings chart, what is the first day in the month when the run should be rejected and patient results should be repeated? A. Day 6 B. Day 7 C. Day 8 D. Day 9

33. C Although the trend is apparent across QC levels by day 7, the patient results would not be rejected until day 8 when the 41s rule is broken. An advantage to plotting control data is that trends can be identified before results are out of control and patient data must be rejected.

33. A patient has the following arterial blood gas results: pH = 7.56 PCO2 = 25 mm Hg PO2 = 100 mm Hg HCO3 - = 22 mmol/L These results are most likely the result of which condition? A. Improper specimen collection B. Prolonged storage C. Hyperventilation D. Hypokalemia

33. C The pH is alkaline (reference range 7.35-7.45) and this can be caused by either low PCO2 or increased bicarbonate. This patient has a normal bicarbonate (reference range 22-26 mmol/L) and a low PCO2 (reference range 35-45 mm Hg). Low PCO2 is always caused by hyperventilation, and therefore, this is a case of uncompensated respiratory alkalosis. The acute stages of respiratory disorders are often uncompensated. P

33. When measuring calcium by atomic absorption spectrophotometry, which is required? A. An organic extraction reagent to deconjugate calcium from protein B. An internal standard C. A magnesium chelator D. Lanthanum oxide to chelate phosphates

33. D An acidic diluent such as hydrochloric acid (HCl) will displace calcium bound to albumin. However, calcium forms a thermostable bond with phosphate that causes chemical interference in atomic absorption. Lanthanum displaces calcium, forming lanthanum phosphate, and eliminates interference from phosphates. Unlike in some colorimetric methods for calcium (e.g., o-cresolphthalein complexone), magnesium does not interfere because it does not absorb the 422.7 nm emission line from the calcium-hollow cathode lamp

33. After installing a new analyzer and reviewing the results of patients for 1 month, the lead technologist notices a greater frequency of patients with abnormally high triglyceride results. Analysis of all chemistry profiles run the next day indicated that triglyceride results are abnormal whenever the test is run immediately after any sample that is measured for lipase. These observations point to which type of error? A. Specificity of the triglyceride reagents B. Precision in pipetting of lipemic samples C. Bias caused by sequence of analysis D. Reagent carryover

33. D Carryover errors are usually attributed to interference caused by a sample with a very high concentration of analyte preceding a normal sample. However, reagent carryover may also occur on automated systems that use common reagent delivery lines or reusable cuvettes.

33. A patient has a plasma myoglobin of 10 μg/L at admission. Three hours later, the myoglobin is 14 μg/L and the troponin I is 0.02 μg/L (reference range 0-0.03 μg/L). These results are consistent with which condition? A. Skeletal muscle injury B. Acute myocardial infarction C. Unstable angina D. No evidence of myocardial or skeletal muscle injury

33. D This person displays very low plasma myoglobin (reference range for females is approximately 17-75 μg/L). The TnI result is also within normal limits. These results are consistent with baseline levels and no evidence of cardiac or skeletal muscle injury. TnI cutpoints for diagnosis of AMI are dependent upon the method, and may be higher than the upper limit of normal

34. Referring to the Levy-Jennings chart, what analytical error is present during the second half of the month? A. Shift B. Trend C. Random error D. Kurtosis

34. A A shift is characterized by six consecutive points lying on the same side of the mean. This occurs from day 15 to day 20. Shifts are caused by a change in the assay conditions that affect the accuracy of all results, such as a change in the concentration of the calibrator; change in reagent; a new lot of reagent that differs in composition; or improper temperature setting, wavelength, or sample volume

34. Why are three levels used for quality control of pH and blood gases? A. Systematic errors can be detected earlier than with two controls B. Analytical accuracy needs to be greater than for other analytes C. High, normal, and low ranges must always be evaluated D. A different level is needed for pH, PCO2, and PO2

34. A Error detection occurs sooner when more controls are used. Some errors, such as those resulting from temperature error and protein coating of electrodes, are not as pronounced near the calibration point, as in the acidosis and alkalosis range. The minimum requirement for blood gas QC is one sample every 8 hours and three levels (acidosis, normal, alkalosis) every 24 hours. T

34. Which specimen is the sample of choice for lead screening? A. Whole blood B. Hair C. Serum D. Urine

34. A Lead accumulates in RBCs, bones, and neural tissues, and whole blood, hair, and urine are suitable for demonstrating lead toxicity. Greatest sensitivity is obtained by using whole blood, which can detect exposure over time.

34. A patient has a plasma CK-MB of 14 μg/L at admission and a total CK of 170 IU/L. Serum myoglobin is 130 μg/L and TnI is 1.6 μg/L. Three hours later, the TnI is 3.0 μg/L. Which statement best describes this situation? A. This patient has had an AMI and further testing is unnecessary B. A second CK-MB and myoglobin test should have been performed at 3 hours postadmission to confirm AMI C. These results are consistent with skeletal muscle damage associated with a crush injury that elevated the CK-MB D. Further testing 6-12 hours postadmission is required to establish a diagnosis of AMI

34. A Results on admission indicate strongly that the patient has suffered an MI. The 3-hour TnI confirms this and rules out the possibility of a sample collection or transcription error for the admission sample. Repeat testing of other cardiac markers at 3 hours was not necessary because admission results were significantly increased for all three markers. Skeletal muscle damage or crush injury does not cause an increase in cardiac TnI

34. In double immunodiffusion reactions, the precipitin band is: A. Invisible before the equivalence point is reached B. Concave to the protein of greatest molecular weight C. Closest to the well containing the highest level of antigen D. Located in an area of antibody excess

34. B In double immunodiffusion (Ouchterlony), the molecules of lower molecular weight move fastest through the gel, causing a visible precipitin arc when antigen and antibody approach equivalence. At equivalence the precipitin arc remains stationary. If the concentration of antisera is constant, the distance of the precipitin arc from the antigen well is proportional to antigen concentration

34. Ion selective analyzers using undiluted samples have what advantage over analyzers that use a diluted sample? A. Can measure over a wider range of concentration B. Are not subject to pseudohyponatremia caused by high lipids C. Do not require temperature equilibration D. Require less maintenance

34. B Ion-selective analyzers measure the electrolyte dissolved in the fluid phase of the sample in millimoles per liter of plasma water. When undiluted blood is assayed, the measurement is independent of colloids such as protein and lipid. Hyperlipemic samples cause falsely low sodium measurements when assayed by flame photometry and ion-selective analyzers requiring dilution because lipids displace plasma water containing the electrolytes. One drawback to undiluted or direct measuring systems is that the electrodes require more frequent deproteinization and usually have a shorter duty cycle.

34. Which statement regarding the measurement of urinary catecholamines is true? A. An increased excretion of total urinary catecholamines is specific for pheochromocytoma B. Twenty-four-hour urinary catecholamine assay avoids pulse variations associated with measurement of plasma catecholamines C. Total urinary catecholamine measurement provides greater specificity than measurement of urinary free catecholamines D. Total urinary catecholamines are not affected by exercise

34. B Measurement of total urinary catecholamines is not a specific test for pheochromocytoma. Urine levels may be increased by exercise and in muscular diseases. Catecholamines in urine may also be derived from dietary sources rather than endogenous production. Most catecholamines are excreted as the glucuronide, and the urinary free catecholamines increase only when there is increased secretion

34. SITUATION: A digoxin result from a stable patient with a normal electrocardiogram (EKG) is reported as 7.4 ng/mL (URL 2.6 ng/mL) using an immunofluorescent method. Renal function tests were normal and the patient was not taking any other medications. The assay was repeated and results were the same. The sample was frozen and sent to a reference laboratory for confirmation. The result was 1.6 ng/mL measured by a competitive chemiluminescent procedure. Which best explains the discrepancy in results? A. The fluorescent immunoassay was performed improperly B. Digoxin was lower by the chemiluminescent method because it is less sensitive C. An interfering substance was present that cross-reacted with the antibody in the fluorescent immunoassay D. Freezing the specimen caused lower results by converting the digoxin to an inactive metabolite

34. C An error was suspected because there was a discrepancy between the test result and the patient's clinical status (i.e., signs of digoxin toxicity such as ventricular arrhythmia were not present.) Some substances called DLIFs (digoxin-like immunologic factors) can cross-react with antibodies used to measure digoxin

34. Which of the following abnormal laboratory results is found in von Gierke's disease? A. Hyperglycemia B. Increased glucose response to epinephrine administration C. Metabolic alkalosis D. Hyperlipidemia

34. D Von Gierke's disease (type 1 glycogen storage disease) results from a deficiency of glucose-6-phosphatase. This blocks the hydrolysis of glucose-6-PO4 to glucose and Pi, preventing degradation of glycogen to glucose.

(see pic) 35. The following results are reported on an adult male patient being evaluated for chest pain. What is the most likely cause of these results? A. The wrong sample was assayed for the first myoglobin B. The patient did not suffer an MI until after admission C. Hemolysis caused interference with the 3-hour and 6-hour myoglobin result D. The patient is experiencing unstable angina

35. A Myoglobin is the first cardiac marker to rise outside the URL following an MI (2-3 hours) followed by TnI (4-6 hours) and CK-MB (4-8 hours). The admission TnI and CK-MB are both elevated, and they continue to rise in all three samples.

35. Which uric acid method is associated with negative bias caused by reducing agents? A. Uricase coupled to the Trinder reaction B. Ultraviolet uricase reaction coupled to catalase and alcohol dehydrogenase reactions C. Measurement of the rate of absorbance decrease at 290 nm after addition of uricase D. Phosphotungstic acid using a protein-free filtrate

35. A The peroxidase-coupled uricase reaction is the most common method for measuring uric acid in serum or plasma. Uricase methods form allantoin, carbon dioxide, and hydrogen peroxide from the oxidation of uric acid. When peroxide is used to oxidize a Trinder dye (e.g., a phenol derivative and 4-aminoantipyrine), some negative bias may occur when high levels of ascorbate or other reducing agents are present

35. The D-xylose absorption test is used for the differential diagnosis of which two diseases? A. Pancreatic insufficiency from malabsorption B. Primary from secondary disorders of glycogen synthesis C. Type 1 and type 2 diabetes mellitus D. Generalized from specific carbohydrate intolerance

35. A Xylose is a pentose that is absorbed without the help of pancreatic enzymes and is not metabolized. In normal adults, more than 25% of the dose is excreted into the urine after 5 hours. Low blood or urine levels are seen in malabsorption syndrome, sprue, Crohn's disease, and other intestinal disorders, but not pancreatitis

35. Which method is most often used to measure fractionated catecholamines (epinephrine, norepinephrine, and dopamine)? A. Measurement of fluorescence following oxidation by potassium ferricyanide B. Measurement by HPLC with electrochemical detection C. Measure of radioactivity after conversion by catechol-O-methyltransferase (COMT) to tritiated metanephrines D. Measurement by HPLC with fluorescence detection

35. B HPLC-ECD separates catecholamines by reversephase chromatography, then detects them by oxidizing the aromatic ring at +0.8 V to a quinone ring. Current is proportional to epinephrine and norepinephrine concentration. Fluorescent methods employing ferricyanide (trihydroxyindole method) or ethylenediamine (EDA method) show interference by Aldomet and several other drugs and are obsolete

35. Which of the following statements regarding the identification of monoclonal proteins by IFE is true? A. The monoclonal band must be present in the γ region B. When testing for a monoclonal gammopathy, both serum and urine must be examined C. A diagnosis of monoclonal gammopathy is based upon quantitation of IgG, IgA, and IgM D. A monoclonal band always indicates a malignant disorder

35. B Quantitation of IgG, IgA, IgM, or IgD indicates the concentration of each class of immunoglobulin but does not distinguish monoclonal from polyclonal gammopathies. Monoclonal characteristics are determined by demonstrating restricted electrophoretic mobility, indicating that all immunoglobulins in the band are of the same amino acid sequence. Monoclonal light chains can be demonstrated in about 60% of monoclonal gammopathies.

35. What is the first day in the second half of the month that patient results would be rejected? A. Day 16 B. Day 17 C. Day 18 D. Day 19

35. B The 41s rule is broken across QC levels on day 17. This means that four consecutive controls are greater than ±1s from the mean. QC rules that are sensitive to SE are applied across both runs and levels to increase the probability of error detection. These are 22s, 41s, and 10×

35. SITUATION: An EDTA sample for TnI assay gives a result of 0.04 ng/mL (reference range 0-0.03 ng/mL). The test is repeated 3 hours later on a new specimen and the result is 0.06 ng/mL. A third sample collected 6 hours later gives a result of 0.07 ng/mL. The EKG showed no evidence of ST segment elevation (STEMI). What is the most likely explanation? A. A false-positive result occurred due to matrix interference B. Heparin should have been used instead of EDTA, which causes false positives C. The patient has suffered cardiac injury D. The patient has had an ischemic episode without cardiac injury

35. C EDTA is the additive of choice for troponin assays because it avoids microclots that can lead to false positive results when serum or heparinized plasma is used. Spurious false positives caused by matrix effects usually revert to normal when the test is repeated on a new sample. An AMI will cause the TnI to increase in subsequent tests. Results between 0.04-0.10 ng/mL are the result of cardiac injury, and indicate either AMI or an increased short-term risk of AMI

35. Which of the following enzymes can be used to measure plasma or serum salicylate? A. Peroxidase B. Salicylate esterase C. Salicylate hydroxylase D. p-Aminosalicylate oxidase

35. C The enzymatic assay of salicylate uses salicylate hydroxylase, which reduces salicylate with NADH and forms catechol and NAD+. Salicylate can also be measured by HPLC and various immunoassays including EMIT

35. Select the equation describing the potential that develops at the surface of an ion-selective electrode. A. van Deemter equation B. van Slyke equation C. Nernst equation D. Henderson-Hasselbalch equation

35. C The van Deemter equation describes the relation between the velocity of mobile phase to column efficiency in gas chromatography. The Henderson-Hasselbalch equation is used to determine the pH of a solution containing a weak acid and its salt. van Slyke developed an apparatus to measure CO2 and O2 content using a manometer

36. Which of the following statements about carbohydrate intolerance is true? A. Galactosemia results from deficiency of galactose-1-phosphate (galactose-1-PO4) uridine diphosphate transferase B. Galactosemia results in a positive glucose oxidase test for glucose in urine C. Urinary galactose is seen in both galactosemia and lactase deficiency D. A galactose tolerance test is used to confirm a diagnosis of galactosemia

36. A Galactose is metabolized to galactose-1-PO4 by the action of galactokinase. Galactose-1-PO4 uridine diphosphate (UDP) transferase converts galactose-1-PO4 to glucose. Deficiency of either enzyme causes elevated blood and urine galactose

36. The reference potential of a silver-silver chloride electrode is determined by the: A. Concentration of the potassium chloride filling solution B. Surface area of the electrode C. Activity of total anion in the paste covering the electrode D. The concentration of silver in the paste covering the electrode Chemistry/Define

36. A The activity of any solid or ion in a saturated solution is unity. For a silver electrode covered with silver chloride paste, the Nernst equation is E = E° - RT/nF × 2.3 log10 [Ag° × Cl-]/[AgCl]. Because silver and silver chloride have an activity of 1.0, and all components except chloride are constants, the potential of the reference electrode is determined by the chloride concentration of the filling solution. E = Eo - RT/nF × 2.3 log10[Cl-] = E° - 59.2 mV × log[Cl-] (at room temperature

36. Which of the following statements regarding paraproteins is true? A. Oligoclonal banding is seen in the CSF of greater than 90% of multiple sclerosis cases B. The Bence-Jones protein heat test is confirmatory for monoclonal light chains C. Light chains found in urine are always derived from monoclonal protein D. The IgA band is usually cathodal to the IgG precipitin band

36. A The α heavy chain is more acidic than γ or μ chains, giving IgA a greater net negative charge at alkaline pH. The IgA precipitin band is anodal to the IgG or IgM band. In hepatic cirrhosis, the β-γ bridging observed on serum protein electrophoresis results from increased IgA.

36. In which condition would hypochloremia be expected? A. Respiratory alkalosis B. Metabolic acidosis C. Metabolic alkalosis D. All of these options

36. C Chloride is the major extracellular anion and is retained or lost to preserve electroneutrality. Low chloride will occur in metabolic alkalosis because excess bicarbonate is present. Low chloride also will occur in partially compensated respiratory acidosis because the kidneys compensate by increased retention of bicarbonate

36. Analysis of normal and abnormal QCs performed at the beginning of the evening shift revealed a 22s error across levels for triglyceride. Both controls were within the 3s limit. The controls were assayed again, and one control was within the acceptable range and the other was slightly above the 2s limit. No further action was taken and the patient results that were part of the run were reported. Which statement best describes this situation? A. Appropriate operating procedures were followed B. Remedial evaluation should have been taken, but otherwise, the actions were appropriate C. Corrective action should have been taken before the controls were repeated D. The controls should have been run twice before reporting results

36. C Quality control limits are chosen to achieve a low probability of false rejection. For example, a 22s error occurs only once in 1,600 occurrences by chance. Therefore, such an error can be assumed to be significant. However, this does not mean the error will occur if the controls are repeated again.

36. Which statement about sample collection for catecholamines and metabolites is true? A. Blood for catecholamines is collected in the usual manner following a 12-hour fast B. Twenty-four-hour urine for vanillylmandelic acid, catecholamines, or metanephrines is collected in 1 mL of boric acid C. Twenty-four-hour urine creatinine should be measured with vanillylmandelic acid, homovanillic acid, or metanephrines D. There is no need to discontinue medications if a 24-hour urine collection is used

36. C Stress, exercise, and an upright position induce catecholamine elevation, and therefore, patients must be resting supine for at least 30 minutes prior to blood collection. The preferred method of collection is catheterization, so that the anxiety of venipuncture is not a factor. A 4-hour fast is also recommended. Many drugs contain epinephrine, which may falsely elevate catecholamine measurements

36. Which of the following laboratory tests is a marker for ischemic heart disease? A. CK-MB isoforms B. Myosin light chain 1 C. Albumin cobalt binding D. Free fatty acid binding protein

36. C When heart muscle suffers reversible damage as a result of oxygen deprivation, free radicals are released from the cells and bind to circulating albumin. The albumin is modified at the N-terminus, causing a reduced ability to bind certain metals. This ischemia-modified albumin can be measured by its inability to bind cobalt. An excess of cobalt is incubated with plasma followed by addition of dithiothreitol

(see pic) 36. Given the following QC chart, identify the day in which a violation of the R4s QC rule occurs. A. Day 3 B. Day 8 C. Day 10 D. Day 15

36. D An R4s error is defined as the algebraic difference between two controls within the same run. In this Levy-Jennings plot, on day 15, Level 1 is above the +2s limit (approximately +2.5s) and Level 2 is below the -2s limit (approximately -2.5s). These controls are approximately 5s apart (+2.5s minus -2.5s = +5s

36. Which of the following tests is least essential to the operation of an emergency department at a general hospital? A. Carboxyhemoglobin B. Osmolality C. Salicylate D. Lead

36. D The vast majority of acute toxicology situations seen in the emergency department (ED) involve poisoning with alcohol, acetaminophen, salicylate, abuse substances, or carbon monoxide

37. A biochemical profile routinely performed bimonthly on a renal dialysis patient showed a decreased serum calcium and decreased PTH level. Such a lab result may be explained by which of the following circumstances? A. Malignancy B. Aluminum toxicity C. Hypervitaminosis D D. Acidosis

37. B Aluminum present in medications and dialysis bath fluid can cause aluminum toxicity in patients receiving dialysis. Renal failure patients often display high PTH levels owing to poor retention of calcium, and are at risk of developing osteitis fibrosa (soft bones) as a result.

37. The term RT/nF in the Nernst equation defines the: A. Potential at the ion-selective membrane B. Slope of the electrode C. Decomposition potential D. Isopotential point of the electrode

37. B In the term RT/nF, R = the molar gas constant, T = temperature in degrees Kelvin, F = Faraday's constant, and n = the number of electrons donated per atom of reductant. The slope depends upon the temperature of the solution and the valence of the reductant. At room temperature, the slope is 59.2 mV for a univalent ion and 29.6 mV for a divalent ion

37. Given the following serum electrolyte data, determine the anion gap. Na = 132 mmol/L Cl = 90 mmol/L HCO3 - = 22 mmol/L A. 12 mmol/L B. 20 mmol/L C. 64 mmol/L D. Cannot be determined from the information provided

37. B The anion gap is defined as unmeasured anions minus unmeasured cations. It is calculated by subtracting the measured anions (bicarbonate and chloride) from the serum sodium (or sodium plus potassium). A normal anion gap is approximately 8-16 mmol/L (12-20 mmol/L when potassium is used). Anion gap = Na - (HCO3 + Cl) Anion gap = 132 - (90 + 22) = 20 mmol/L

37. Which statement regarding IFE is true? A. Serum containing a monoclonal protein should have a κ:λ ratio of 0.5 B. A monoclonal band seen with monospecific antiserum should not be visible in the lane where polyvalent antiserum or sulfosalicylic acid was added C. CSF should be concentrated 50- to100-fold before performing IFE D. When oligoclonal bands are seen in the CSF, they must also be present in serum to indicate multiple sclerosis

37. C Any monoclonal precipitin band formed when heavy- or light-chain-specific antiserum reacts with a sample should also be found in the same position when sample is fixed with sulfosalicylic acid or reacted with polyvalent antihuman Ig. The normal free κ:λ ratio can vary between 0.26 and 1.65. I

37. What is the minimum requirement for performing QC for a total protein assay? A. One level assayed every 8 hours B. Two levels assayed within 8 hours C. Two levels assayed within 24 hours D. Three levels assayed within 24 hours

37. C The minimum requirement for frequency of quality control for a general chemistry analyte (based upon the Clinical Laboratory Improvement Act, 1988) is two levels of control assayed every 24 hours

37. Which test becomes abnormal in the earliest stage of the acute coronary syndrome? A. Myosin light chain 1 B. CK-MB isoforms C. Myoglobin D. High-sensitivity C-reactive protein

37. D The acute coronary syndrome (ACS) refers to the evolution of coronary artery events that lead up to AMI. Coronary artery disease (CAD) begins with formation of a plaque comprised of lipid from dead endothelium that proliferates into the artery lumen. The plaque becomes disrupted and the vessel wall inflamed in the asymptomatic stage of CAD. If platelet activation occurs and results in thrombosis, blood flow becomes significantly reduced, resulting in angina. This

37. Which of the following trace elements is considered an essential micronutrient? A. Thallium B. Aluminum C. Mercury D. Selenium

37. D Trace elements can be divided into two categories, those that have no known biological purpose and those that do. The former include thallium, mercury, lead, cadmium, and aluminum.

37. Which of the following statements applies to both measurement of VMA and metanephrines in urine? A. Both can be oxidized to vanillin and measured at 360 nm without interference from dietary compounds B. Both can be measured immunochemically after hydrolysis and derivatization C. Both require acid hydrolysis prior to measurement D. Both can be measured by specific HPLC and MS assays

37. D VMA and metanephrines can both be measured as vanillin after oxidation with periodate. However, these methods are affected by dietary sources of vanillin; coffee, chocolate, bananas, and vanilla must be excluded from the diet. Metanephrines, VMA, and HVA are most often measured by HPLC-EDC.

38. When measuring trace metals in blood other than lead, what type of tube should be used? A. Navy blue top B. Green top C. Purple top D. Red top

38. A In order to avoid trace contamination by metals present in the stopper lubricants, a tube with a navy blue top is used for measuring trace metals. These tubes are validated for most but not all trace metals.

38. Which of the following statistical tests is used to compare the means of two methods? A. Student's t test B. F distribution C. Correlation coefficient (r) D. Linear regression analysis

38. A Student's t test is the ratio of mean difference to the standard error of the mean difference (bias/random error) and tests for a significant difference in means. The F test is the ratio of variances and determines if one method is significantly less precise

38. Urinary HVA is most often assayed to detect: A. Pheochromocytoma B. Neuroblastoma C. Adrenal medullary carcinoma D. Psychiatric disorders such as manic depression

38. B HVA is the major metabolite of dopa, and urinary HVA is elevated in more than 75% of neuroblastoma patients. Neuroblastomas also usually produce VMA from norepinephrine. Thus, HVA and VMA are assayed together and this increases the sensitivity of detection to around 90%.

38. Which of the following processes occurs when iron is in the oxidized (Fe3+) state? A. Absorption by intestinal epithelium B. Binding to transferrin and incorporation into ferritin C. Incorporation into protoporphyrin IX to form functional heme D. Reaction with chromogens in colorimetric assays

38. B Intestinal absorption occurs only if the iron is in the reduced (Fe+2) state. After absorption, Fe+2 is oxidized to Fe+3 by gut mucosal cells.

38. Which set of the following laboratory results is most likely from a patient who has suffered an AMI? Reference intervals are in parenthesis (see pic for answers)

38. B Results shown in C and D can be excluded because the CK-MB is not increased. Results shown in A and B have CK-MB levels above the URL. However, patient A has a CK index under 2.5% and a 5- to 10-fold elevation of total CK.

38. Which statement best describes the clinical utility of B-type natriuretic peptide (BNP)? A. Abnormal levels may be caused by obstructive lung disease B. A positive test indicates prior myocardial damage caused by AMI that occurred within the last 3 months C. A normal test result (<100 pg/mL) helps rule out congestive heart failure in persons with symptoms associated with coronary insufficiency D. A level above 100 pg/mL is not significant if evidence of congestive heart failure is absent

38. C B-type natriuretic peptide is a hormone produced by the ventricles in response to increased intracardiac blood volume and hydrostatic pressure. It is formed in the heart from a precursor peptide (preproBNP) by enzymatic hydrolysis, first forming proBNP followed by BNP and NT (N-terminal) proBNP which is not physiologically active.

38. The ion-selective membrane used to measure potassium is made of: A. High-borosilicate glass membrane B. Polyvinyl chloride dioctylphenyl phosphonate ion exchanger C. Valinomycin gel D. Calomel

38. C Valinomycin is an antibiotic with a highly selective reversible-binding affinity for potassium ions. Sodium electrodes are usually composed of a glass membrane with a high content of aluminum silicate. Calcium and lithium ion-selective electrodes are made from organic liquid ion exchangers called neutral carrier ionophores. Calomel is made of mercury covered with a paste of mercurous chloride (Hg°/Hg2Cl2) and is used as a reference electrode for pH.

38. Which of the following conditions will cause an increased anion gap? A. Diarrhea B. Hypoaldosteronism C. Hyperkalemia D. Renal failure

38. D An increased anion gap occurs when there is production or retention of anions other than bicarbonate or chloride (measured anions). For example, in renal failure, retention of phosphates and sulfates (as sodium salts) increases the anion gap. Other common causes of metabolic acidosis with an increased anion gap are diabetic ketoacidosis and lactate acidosis. The anion gap may also be increased in the absence of an acid-base disorder. Common causes include hypocalcemia, drug overdose, and laboratory error when measuring electrolytes

39. Which of the following is associated with low serum iron and high total iron-binding capacity (TIBC)? A. Iron deficiency anemia B. Hepatitis C. Nephrosis D. Noniron deficiency anemias

39. A Iron-deficiency anemia is the principal cause of low serum iron and high TIBC because it promotes increased transferrin. Pregnancy without iron supplementation depletes maternal iron stores and also results in low serum iron and high TIBC

39. Alcoholism, liver failure, and hypoxia induce acidosis by causing: A. Depletion of cellular NAD+ B. Increased excretion of bicarbonate C. Increased retention of PCO2 D. Loss of carbonic anhydrase Chemistry/Apply knowledge

39. A Oxygen debt and liver failure block oxidative phosphorylation, preventing NADH from being oxidized back to NAD+. Oxidation of ethanol to acetate results in accumulation of NADH. When NAD+ is depleted, glycolysis cannot proceed. It is regenerated by reduction of pyruvate to lactate, causing lactate acidosis

(see pic) 39. Two freezing point osmometers are compared by running 40 paired patient samples one time on each instrument, and the following results are obtained. If the critical value for F = 2.8, then what conclusion can be drawn regarding the precision of the two instruments? A. There is no statistically significant difference in precision B. Osmometer A demonstrates better precision that is statistically significant C. Osmometer B demonstrates better precision that is statistically significant D. Precision cannot be evaluated statistically when single measurements are made on samples

39. A The F test determines whether there is a statistically significant difference in the variance of the two sampling distributions. Assuming the samples are collected and stored in the same way and the analysis is done by a technologist who is familiar with the instrument, then differences in variance can be attributed to a difference in instrument precision.

39. Which statement best describes the clinical utility of plasma homocysteine? A. Levels are directly related to the quantity of LDL cholesterol in plasma B. High plasma levels are associated with atherosclerosis and increased risk of thrombosis C. Persons who have an elevated plasma homocysteine will also have an increased plasma Lp(a) D. Plasma levels are increased only when there is an inborn error of amino acid metabolism

39. B Homocysteine includes the monomeric amino acid as well as the dimers such as homocystine that contain homocysteine. Plasma levels are measured as an independent risk factor for coronary artery disease. High levels of homocysteine are toxic to vascular endothelium and promote inflammation and plaque formation. Plasma levels are independent of LDL and other cholesterol fractions and help explain why approximately 35% of people with first-time AMI have LDL cholesterol levels < 130 mg/dL

39. The response of a sodium electrode to a 10-fold increase in sodium concentration should be: A. A 10-fold drop in potential B. An increase in potential of approximately 60 mV C. An increase in potential of approximately 10 mV D. A decrease in potential of approximately 10 mV

39. B The Nernst equation predicts an increase of approximately 60 mV per 10-fold increase in sodium activity. For sodium

(see pic) 39. Hemoglobin electrophoresis performed on agarose at pH 8.8 gives the following results. All components of the Hgb C, S, F, A control hemolysate were within the acceptable range. What is the most likely cause of this patient's result? A. HgbLepore B. Hgb S-β-thalassemia (Hgb S/β+) C. Hgb SC disease post-transfusion D. Specimen contamination

39. C HemoglobinLepore results from a hybridization of the β and δ genes and produces a pattern that is similar to Hgb S trait (AS), except that the quantity of HgbLepore at the Hgb S position is below 20%. Hemoglobin S-β-thalassemia minor results in an increase in Hgb A2 (and possibly Hgb F) because there is reduced transcription of the structurally normal β chain. However, the Hgb S should be greater than the Hgb A, and the amount at the Hgb A2 is far too high.

39. Thyroid hormones are derived from the amino acid: A. Phenylalanine B. Methionine C. Tyrosine D. Histidine

39. C Thyroid hormones are derived from the enzymatic modification of tyrosine residues on thyroglobulin. Tyrosine is halogenated enzymatically with iodine, forming monoiodotyrosine (MIT) and diiodotyrosine (DIT). Enzymatic coupling of these residues form T3 (3,5,3´-triiodothyronine) and T4 (3,5,3´,5´-tetraiodothyronine). These are hydrolyzed from thyroglobulin, forming active hormones

39. Which test is the most useful way to evaluate the response to treatment for multiple myeloma? A. Measure of total immunoglobulin B. Measurement of 24-hour urinary light chain concentration (Bence-Jones protein) C. Capillary electrophoresis of M-protein recurrence D. Measurement of serum-free light chains

39. D Unlike electrophoresis methods, serum free light chain assays are quantitative and an increase in free light chain production with an abnormal kappa:lambda ratio occurs earliest in recurrence of myeloma. Light chains have a shorter plasma half life than intact Ig and therefore, the reduction in free light chain concentration is an earlier indicator of treatment effect than measurement of intact Ig.

4. Which wavelength would be absorbed strongly by a red-colored solution? A. 450 nm B. 585 nm C. 600 nm D. 650 nm

4. A A solution transmits light corresponding in wavelength to its color, and usually absorbs light of wavelengths complementary to its color. A red solution transmits light of 600-650 nm and strongly absorbs 400-500 nm light

4. Which of the following formulas is the correct expression for creatinine clearance? A. Creatinine clearance = U/P X V X 1.73/A B. Creatinine clearance = P/V X U X A/1.73 C. Creatinine clearance = P/V X U X 1.73/A D. Creatinine clearance = U/V X P X 1.73/A

4. A Clearance is the volume of plasma that contains the same quantity of substance that is excreted in the urine in 1 minute. Creatinine clearance is calculated as the ratio of urine creatinine to plasma creatinine in milligrams per deciliter. This is multiplied by the volume of urine produced per minute and corrected for lean body mass by multiplying by 1.73/A, where A is the patient's body surface area in square meters. Separate reference ranges are needed for males, females, and children because each has a different percentage of lean muscle mass

4. Which of the following best represents the reference (normal) range for arterial pH? A. 7.35-7.45 B. 7.42-7.52 C. 7.38-7.68 D. 6.85-7.56

4. A The reference range for arterial blood pH is 7.35-7.45 and is only 0.03 pH units lower for venous blood owing to the buffering effects of hemoglobin (Hgb) known as the chloride isohydric shift. Most laboratories consider less than 7.20 and greater than 7.60 the critical values for pH

4. Zollinger-Ellison (Z-E) syndrome is characterized by great (e.g., 20-fold) elevation of: A. Gastrin B. Cholecystokinin C. Pepsin D. Glucagon Chemistry/Correlate

4. A Z-E syndrome is caused by a pancreatic or intestinal tumor secreting gastrin (gastrinoma), and results in greatly increased gastric acid production. A serum gastrin level 10-fold greater than the URL in a person with hyperacidity and stomach or duodenal ulcers is diagnostic. Confirmation of gastric hyperacidity is demonstrated using the basal acid output (BAO) test

4. The study of pharmacogenomics involves which type of testing? A. Family studies to determine the inheritance of drug resistance B. Testing drugs with cell cultures to determine the minimum toxic dosage C. Testing for single nucleotide polymorphisms known to affect drug metabolism D. Comparison of dose-response curves between family members

4. C Pharmacogenomics refers to the study of genes that affect the performance of a drug in an individual. One method is to test for single nucleotide polymorphisms (SNPs) using DNA microarrays in genes such as those that code for the cytochrome P450 enzymes involved in the metabolism of many drugs. Genetic variations of one such enzyme may account for individual pharmacokinetic differences and can be used to predict the efficacy of the drug

4. Which of the following is characteristic of type 2 diabetes mellitus? A. Insulin levels are consistently low B. Most cases require a 3-hour oral glucose tolerance test to diagnose C. Hyperglycemia is often controlled without insulin replacement D. The condition is associated with unexplained weight loss

4. C Type 2, or late-onset diabetes, is associated with a defect in the receptor site for insulin. Insulin levels may be low, normal, or high. Patients are usually obese and over 40 years of age, although the incidence is increasing in both children and young adults. The American Diabetes Association (ADA) recommends screening all adults for diabetes who are overweight and have one additional risk factor and all adults over age 45, and to retest them every 3 years, if negative. Patients do not require insulin to prevent ketosis and hyperglycemia can be controlled in most patients by diet and drugs that promote insulin release. Type 2 accounts for 80%-90% of all diabetes mellitus.

4. Which statement about enzymes is true? A. An enzyme alters the Gibb's free energy of the reaction B. Enzymes cause a reaction with a positive free energy to occur spontaneously C. An enzyme's natural substrate has the highest Km D. A competitive inhibitor will alter the apparent Km of the reaction

4. D Enzymes alter the energy of activation by forming a metastable intermediate, the enzyme substrate complex. Enzymes do not alter the free energy or direction of a reaction. Competitive inhibitors bind to the active site where the enzyme binds substrate and are overcome by increasing the substrate concentration

40. Which of the following is the primary mechanism causing respiratory alkalosis? A. Hyperventilation B. Deficient alveolar diffusion C. Deficient pulmonary perfusion D. Parasympathetic inhibition

40. A Hyperventilation via stimulation of the respiratory center (or induced by a respirator) is the mechanism of respiratory alkalosis. Causes include low PO2, anxiety, fever, and drugs that stimulate the respiratory center. Acute respiratory alkalosis is often uncompensated because renal compensation is not rapid. Uncompensated respiratory alkalosis is characterized by an elevated pH and a low PCO2 with normal bicarbonate.

40. Which of the electrodes below is a currentproducing (amperometric) rather than a voltage-producing (potentiometric) electrode? A. Clark electrode B. Severinghaus electrode C. pH electrode D. Ionized calcium electrode

40. A The Clark electrode is composed of two half cells that generate current, not voltage. It is used to measure partial pressure of oxygen (PO2), and is based upon an amperometric method called polarography. When -0.8 V is applied to the cathode, O2 is reduced, causing current to flow. Current is proportional to the PO2 of the sample

40. What are the likely laboratory findings in a person suspected of having Wilson's disease? A. Blood copper and ceruloplasmin low, urinary copper excretion high B. Blood and urine copper concentration high, ceruloplasmin low C. Blood and urine copper concentration high, ceruloplasmin high D. Blood and urine copper concentration low, ceruloplasmin low

40. A Wilson's disease is an autosomal recessive disease in which copper transport is abnormal. The gene causing the disease codes for an ATPase (called Wilson's protein or ATP7B) that is needed to excrete copper into bile and incorporate copper into ceruloplasmin

40. Which statement regarding thyroid hormones is true? A. Circulating levels of T3 and T4 are about equal B. T3 is about 10-fold more active than T4 C. The rate of formation of monoiodotyrosine and diiodotyrosine is about equal D. Most of the T3 present in plasma is from its direct release from thyroid storage sites

40. B The rate of DIT synthesis is twice that of MIT and the rate of coupling favors formation of T4. Levels of T4 are about 50 times those of T3, but T3 is approximately 10 times more active physiologically. Eighty percent of circulating T3 is derived from enzymatic conversion of T4 by T4 5´-deiodinase

(see pic) 40. Two consecutive serum samples give the results shown in the table above (at the top of this page) for a metabolic function profile. The instrument is a random access analyzer that uses two sample probes. The first probe aspirates a variable amount of serum for the spectrophotometric chemistry tests, and the second probe makes a 1:50 dilution of serum for electrolyte measurements. What is the most likely cause of these results? A. Both patients have renal failure B. There is an insufficient amount of sample in both serum tubes C. There is a fibrin strand in the probe used for the spectrophotometric chemistry tests D. The same patient's sample was accidentally run twice

40. C Electrolyte results for both patients are within the physiological range but are distinctly different. The first results indicate a high potassium and increased anion gap, and one would expect the BUN, uric acid, and creatinine to be elevated. However, the results for BUN and glucose are unlikely for any patient, and the creatinine and uric acid signals are below the detection limit of the analyzer, indicating that little or no sample was added.

40. Which condition is associated with the lowest percent saturation of transferrin? A. Hemochromatosis B. Anemia of chronic infection C. Iron deficiency anemia D. Noniron deficiency anemia

40. C Percent saturation = Serum Fe × 100/TIBC. Normally, transferrin is one-third saturated with iron. In iron deficiency states, the serum iron falls but transferrin rises. This causes the numerator and denominator to move in opposite directions, resulting in very low percent saturation (about 10%).

40. Which of the following cardiac markers derived from neutrophils predicts an increased risk for myocardial infarction? A. Phospholipase A2 (PLA2) B. Glycogen phosphorylase BB (GPBB) C. Soluble CD40 ligand (sCD40l) D. Myeloperoxidase (MPO

40. D All of the answer choices are markers for acute coronary syndrome and increased risk of AMI. MPO is released from neutrophils and is thought to destabilize the arterial plaque by oxidizing both LDL and HDL and reducing nitric oxide levels in the coronary arteries. Levels in the upper third quartile predict an increased risk of a coronary event even when troponin is normal.

40. Which of the following is more commonly associated with a nonmalignant form of monoclonal gammopathy (MGUS)? A. Bone marrow plasma cells comprise 20% of nucleated cells B. Monoclonal protein (M-protein) concentration is 3.5 g/dL C. M-protein is IgG D. Age greater than 60 at the time of monoclonal protein discovery

40. D MGUS is the most common cause of monoclonal gammopathy. About 3% of the U.S. population at age 50 and 5% at age 70 have MGUS. The absence of bone lesions and organ damage, plasma cells below 10% of nucleated bone marrow cells, and M-protein below 3.0 g/dL are characteristic of MGUS as opposed to myeloma or other malignant gammopathy.

(see pic) 40. Two methods for total cholesterol are compared by running 40 paired patient samples in duplicate on each instrument. The following results are obtained. Assuming the samples are collected and stored in the same way and the analysis done by a technologist who is familiar with both methods, what is the bias of method y? A. 0.4 B. 7.2 C. 10.6 D. 11.0

40. D The bias is defined as the difference between the means of the two methods and is calculated using the formula: bias = y - ×. The bias is an estimate of SE. The student's t test is used to determine if bias is statistically significant. The t statistic is the ratio of bias to the standard error of the mean difference. The greater the bias, the higher the t score.

41. When the magnitude of error increases with increasing sample concentration, it is called: A. Constant error B. Proportional error C. Random error D. Bias

41. B Proportional error (slope or percent error) results in greater absolute error (deviation from the target value) at higher sample concentration. Constant error refers to a difference between the target value and the result, which is independent of sample concentration.

(see pic) 41. SITUATION: A blood sample in a red-stoppered tube is delivered to the laboratory for electrolytes, calcium, and phosphorus. The tube is approximately half full and is accompanied by a purple-stoppered tube for a complete blood count that is approximately three-quarters full. The chemistry results are as follows. What is the most likely explanation of these serum calcium results? A. Severe hemolysis during sample collection B. Laboratory error in the calcium measurement C. The wrong order of draw was used for vacuum tube collection D. Some anticoagulated blood was added to the red-stoppered tube

41. D The potassium and the calcium results are above and below physiological limit values, respectively. Although hemolysis could explain the high potassium, hemolysis does not cause a significant change in serum calcium. The wrong order of draw could result in the falsely low calcium value but would not be sufficient to cause a result that is incompatible with life (and does not explain a grossly elevated potassium

41. Which condition can result in acidosis? A. Cystic fibrosis B. Vomiting C. Hyperaldosteronism D. Excessive O2 therapy

41. D When O2 saturation of venous blood is greatly elevated, Hgb cannot release O2. Oxyhemoglobin cannot bind CO2 or hydrogen ions and acidosis results. Pure O2 may cause neurological damage, leading to convulsion and blindness, especially in infants. It can induce respiratory failure by causing pulmonary hemorrhage, edema, and hyalinization. The other three conditions cause alkalosis. Vomiting and cystic fibrosis cause loss of chloride, resulting in hypovolemia and intestinal bicarbonate absorption. Hyperaldosteronism causes hypokalemia; this results in increased renal H+ excretion and a shift of H+ into cells in exchange for K+.

42. Select the products formed from the forward reaction of AST. A. Alanine and α-ketoglutarate B. Oxaloacetate and glutamate C. Aspartate and glutamine D. Glutamate and NADH

42. B AST forms oxaloacetate and glutamate from aspartate and α-ketoglutarate (2-oxoglutarate). Both transaminases use α-ketoglutarate and glutamate as a common substrate and product pair. Both aspartate and alanine can be used to generate glutamate in the central nervous system, where it acts as a neurotransmitter

42. Select the order of mobility of lipoproteins electrophoresed on cellulose acetate or agarose at pH 8.6. A. - Chylomicrons→pre-β →β→α+ B. - β→pre-β→α→chylomicrons + C. - Chylomicrons →β→pre-β→α + D. - α→β→pre-β→chylomicrons +

42. C Although pre-β lipoprotein is lower in density than β lipoprotein, it migrates faster on agarose or cellulose acetate owing to its more negative apoprotein composition. When lipoproteins are separated on polyacrylamide gel, pre-β moves slower than β lipoprotein. Molecular sieving causes migration to correlate with lipoprotein density when PAGE is used

42. Which of the following is likely to occur first in iron deficiency anemia? A. Decreased serum iron B. Increased TIBC C. Decreased serum ferritin D. Increased transferrin

42. C Body stores must be depleted of iron before serum iron falls. Thus, serum ferritin falls in the early stages of iron deficiency, making it a more sensitive test than serum iron in uncomplicated cases. Ferritin levels are low only in iron deficiency. However, concurrent illness such as malignancy, infection, and inflammation may promote ferritin release from the tissues, causing the serum ferritin to be normal in iron deficiency

42. In polarography, the voltage needed to cause depolarization of the cathode is called the: A. Half-wave potential B. Isopotential point C. Decomposition potential D. Polarization potential

42. C In polarography, a minimum negative voltage must be applied to the cathode to cause reduction of metal ions (or O2) in solution. This is called the decomposition potential. It is concentration dependent (dilute solutions require greater negative voltage), and can be determined using the Nernst equation

(see pic) 42. Which explanation is the best interpretation of the following BUN bias plot? A. The new method consistently overestimates the BUN by a constant concentration B. The new method is greater than the reference method but not by a statistically significant margin C. The new method is lower than the reference method by 5 mg/dL D. The new method is lower than the reference and the magnitude is concentration dependent

42. D A bias plot compares the bias (candidate method minus reference method) to the result of the reference method. Ideally, points should be scattered equally on both sides of the zero line. When the majority of points is below the zero line, the candidate method is negatively biased (lower than the reference).

42. Which of the following conditions is associated with an increase in ionized calcium (Cai) in the blood? A. Alkalosis B. Hypoparathyroidism C. Hyperalbuminemia D. Malignancy

42. D Increased Cai occurs in hyperparathyroidism, malignancy, and acidosis. Cai is elevated in primary hyperparathyroidism due to resorption of calcium from bone. Many nonparathyroid malignancies create products called parathyroid hormone-related proteins that stimulate the parathyroid receptors of cells. Acidosis alters the equilibrium between bound and free calcium, favoring ionization. Hyperalbuminemia increases the total calcium by increasing the proteinbound fraction, but does not affect the Cai

42. Which of the following conditions will increase total T4 by increasing TBG? A. Acute illness B. Anabolic steroid use C. Nephrotic syndrome D. Pregnancy or estrogens

42. D Pregnancy and estrogens are the most common cause of increased TBG. Other causes include hepatitis, morphine, and clofibrate therapy. Acute illness, anabolic steroids, and nephrotic syndrome decrease the level of TBG. Normal pregnancy causes an elevated serum total T4. Suitable assays are available that estimate free T4 and T3 and these should be used instead of total hormone assays

42. SITUATION: A patient previously diagnosed with primary hypothyroidism and started on thyroxine replacement therapy is seen for follow-up testing after 2 weeks. The serum-free T4 is normal but the TSH is still elevated. What is the most likely explanation for these results? A. Laboratory error in measurement of free T4 B. Laboratory error in measurement of TSH C. In vitro drug interference with the free T4 assay D. Results are consistent with a euthyroid patient in the early phase of therapy Chemistry/Evaluate laboratory

42. D Results of thyroid tests (especially in hospitalized patients) may sometimes appear discrepant because medications and nonthyroid illnesses can affect test results. The pituitary is slow to respond to thyroxine replacement, and 6-8 weeks are usually required before TSH levels fall back to normal

(see pic) 43. Serum samples collected from hospitalized patients over a 2-week period are split into two aliquots and analyzed for prostate specific antigen (PSA) by two methods. Each sample was assayed by both methods within 30 minutes of collection by a technologist familiar with both methods. The reference method is method × (upper reference limit = 4.0 μg/L). Linear regression analysis was performed by the least-squares method, and results are as follows: Which statement best characterizes the relationship between the methods? A. There is a significant bias caused by constant error B. There is a significant proportional error C. There is no disagreement between the methods because the correlation coefficient approaches 1.0 D. There is no systematic error, but the random error of the new method is unacceptable

43. A The linear regression analysis is the most useful statistic to compare paired patient results because it estimates the magnitude of specific errors. The y intercept of the regression line is a measure of constant error, and the slope is a measure of proportional error.

43. Persistent noise from an ion-selective electrode is most often caused by: A. Contamination of sample B. Blocked junction at the salt bridge C. Overrange from high concentration D. Improper calibration

43. B Electrode noise most often results from an unstable junction potential. Most reference electrodes contain a high concentration of KCl internal solution used to produce the reference potential. This forms a salt bridge with the measuring half cell by contacting sample, but is kept from equilibrating via a barrier called a junction. When this junction becomes blocked by salt crystals, the reference potential will be unstable, resulting in fluctuation in the analyzer readout

43. Which formula provides the best estimate of serum TIBC? A. Serum transferrin in mg/dL × 0.70 = TIBC (μg/dL) B. Serum transferrin in mg/dL × 1.43 = TIBC (μg/dL) C. Serum iron (μg/dL)/1.2 + 0.06 = TIBC (μg/dL) D. Serum Fe (μg/dL) × 1.25 = TIBC (μg/dL)

43. B Transferrin, a β-globulin, has a molecular size of about 77,000. Transferrin is the principal iron transport protein, and TIBC is determined by the serum transferrin concentration. One mole of transferrin binds two moles of Fe+3, so the transferrin concentration can be used to predict the TIBC

43. SITUATION: A 6-year-old child being treated with phenytoin was recently placed on valproic acid for better control of seizures. After displaying signs of phenytoin toxicity including ataxia, a stat phenytoin is determined to be 15.0 mg/L (reference range 10-20 mg/L). A peak blood level drawn 5 hours after the last dose is 18.0 mg/L. The valproic acid measured at the same time is within therapeutic limits. Quality control is within acceptable limits for all tests, but the physician questions the accuracy of the results. What is the most appropriate next course of action? A. Repeat the valproic acid level using the last specimen B. Repeat the phenytoin on both trough and peak samples using a different method C. Recommend measurement of free phenytoin using the last specimen D. Recommend a second trough level be measured

43. C Phenytoin levels must be monitored closely because toxic drug levels can occur unexpectedly due to changing pharmacokinetics. Phenytoin follows a nonlinear rate of elimination, which means that clearance decreases as blood levels increase. At high blood levels, saturation of the hepatic hydroxylating enzymes can occur, causing an abrupt increase in the blood level from a small increase in dose

43. Select the most appropriate single screening test for thyroid disease. A. Free thyroxine index B. Total T3 assay C. Total T4 D. TSH assay

43. D TSH is produced by the anterior pituitary in response to low levels of free T4 or T3. A normal TSH rules out thyroid disease. TSH is low in primary hyperthyroidism and high in primary hypothyroidism

44. The serum TSH level is almost absent in: A. Primary hyperthyroidism B. Primary hypothyroidism C. Secondary hyperthyroidism D. Euthyroid sick syndrome

44. A Low TSH and a high T3 (and usually T4) occur in primary hyperthyroidism, but may also occur in systemic nonthyroid illnesses where T4 has been converted to T3. A 2-fold increase in free hormone can produce a 100-fold decrease in TSH. In primary hyperthyroidism, the TSH will be within a range of 0-0.02 mU/mL, while in nonthyroid illnesses it will be 0.03 mU/mL or higher

44. Which of the statements below regarding the methods of Henry for AST and ALT is correct? A. Hemolysis will cause positive interference in both AST and ALT assays B. Loss of activity occurs if samples are frozen at -20°C C. The absorbance at the start of the reaction should not exceed 1.0 A D. Reaction rates are unaffected by addition of P-5´-P to the substrate

44. A RBCs are rich in AST and to a lesser extent in ALT. Hemolysis causes positive interference in both assays, although the effect on AST is greater. Samples are stable for up to 24 hours at room temperature and up to 3 days at 4°C, and should be frozen if kept longer. The starting absorbance should be at least 1.5 A for both assays.

44. Which of the following conditions is associated with hypophosphatemia? A. Rickets B. Multiple myeloma C. Renal failure D. Hypervitaminosis D

44. A Rickets can result from dietary phosphate deficiency, vitamin D deficiency, or an inherited disorder of either vitamin D or phosphorus metabolism. Vitamin D-dependent rickets (VDDR) can be reversed by megadoses of vitamin D. Type 1 is caused by a deficiency in renal cells of 1-α-hydroxylase, an enzyme that converts 25 hydroxyvitamin D to the active form, 1,25 hydroxyvitamin D.

44. Select the lipoprotein fraction that carries most of the endogenous triglycerides. A. VLDL B. LDL C. HDL D. Chylomicrons

44. A The VLDL is formed in the liver largely from chylomicron remnants and hepatic-derived triglycerides. Therefore, the VLDL transports the majority of endogenous triglycerides, while the triglycerides of chylomicrons are derived entirely from dietary absorption

44. Which element is reduced at the cathode of a Clark polarographic electrode? A. Silver B. Oxygen C. Chloride D. Potassium

44. B The Clark electrode is designed to measure oxygen. O2 diffuses through a gas-permeable membrane covering the electrode. It is reduced at the cathode, which is made of platinum or other inert metal. Electrons are supplied by the anode, which is made of silver. The net reaction is: 4 KCl + 2 H2O + O2 + 4 Ag° →4 AgCl + 4 KOH

(see pic) 44. Which statement best summarizes the relationship between the new BUN method and reference method based upon the following linear regression scatterplot? A. The methods agree very well but show a high standard error of estimate B. There is little or no constant error, but some proportional error C. There will be a significant degree of uncertainty in the regression equation D. There is significant constant and proportional error but little random error

44. B The scatterplot shows that each sample produces a coordinate (x corresponds to the reference result and y to the candidate method result) that is very close to the regression line. This means that the variance of regression is low and there is a high degree of certainty that the predicted value of y will be close to its measured value.

(see pic) 44. The results shown in the table above are obtained from three consecutive serum samples using an automated random access analyzer that samples directly from a bar-coded tube. Calibration and QC performed at the start of the shift are within the acceptable range, and no error codes are reported by the analyzer for any tests on the three samples. Upon results verification, what is the most appropriate course of action? A. Report the results and proceed with other tests since no analytical problems are noted B. Repeat the controls before continuing with further testing, but report the results C. Check sample identification prior to reporting D. Do not report BUN results for these patients or continue BUN testing

44. D BUN is elevated 5- to 10-fold for three consecutive patients in the absence of any other laboratory evidence of renal disease. The glucose results show conclusively that the samples are not from the same patient. Therefore, the BUN results must be caused by a systematic error, and should not be reported. Further testing for BUN should cease until the analytical components of the BUN assay are completely evaluated and the cause of these results identified and corrected.

45. Which statement about iron methods is true? A. Interference from Hgb can be corrected by a serum blank B. Colorimetric methods measure binding of Fe2+ to a ligand such as ferrozine C. Atomic absorption is the method of choice for measurement of serum iron D. Serum iron can be measured by potentiometry

45. B Atomic absorption is not the method of choice for serum iron because matrix error and variation of iron recovered by extraction cause bias and poor precision. Most methods use HCl to deconjugate Fe3+ from transferrin followed by reduction to Fe2

45. Which of the following statements accurately characterizes the coulometric titration of chloride? A. The indicator electrodes generate voltage B. Constant current must be present across the generator electrodes C. Silver ions are formed at the generator cathode D. Chloride concentration is inversely proportional to titration time

45. B The Cotlove chloridometer is based upon the principle of coulometric titration with amperometric detection. Charge in the form of silver ions is generated by oxidation of silver wire at the generator anode. Silver ions react with chloride ions, forming insoluble silver chloride (AgCl). When all of the chloride is titrated, free silver ions are detected by reduction back to elemental silver, which causes an increase in current across the indicator electrodes (a pair of silver electrodes with a voltage difference of about 1.0 V DC). Charge or titration time is directly proportional to chloride concentration as long as the rate of oxidation remains constant at the generator anode

45. Select the coupling enzyme used in the kinetic AST reaction of Henry. A. LD B. Malate dehydrogenase C. Glutamate dehydrogenase D. G-6-PD

45. B The method of Henry for AST uses malate dehydrogenase (MD) to reduce oxaloacetate to malate. The electrons come from NADH forming NAD+. Aspartate + α-ketoglutarate AST Oxaloacetate + Glutamate Oxaloacetate + NADH + H+ MD Malate + NAD+

45. The protein composition of HDL is what percentage by weight? A. Less than 2% B. 25% C. 50% D. 90% Chemistry

45. C About 50% of the weight of HDL is protein, largely apo A-I and apo A-II. The HDL is about 30% phospholipid and 20% cholesterol by weight. The HDL binds and esterifies free cholesterol from cells and transports it to the liver, where it can be eliminated in the bile

(see pic) 45. A new method for BUN is evaluated by comparing the results of 40 paired patient samples to the urease-UV method. Normal and high controls were run on each shift for 5 days, five times per day. The results are as follows sample having a concentration of 50 mg/dL? A. -2.2 mg/dL B. -2.8 mg/dL C. -7.5 mg/dL D. -10.0 mg/dL

45. C Linear regression analysis gives an estimate of SE, which is equal to (ŷ - xc) where xc is the expected concentration, and ŷ is the value predicted by the linear regression equation SE = [-0.3 + (0.9 × 50 mg/dL)] - 50.0 mg/dL = 44.7-50.0 = -5.3 mg/dL The standard deviation of the new method for the high control is used to estimate the RE because the mean of this control is nearest to the expected concentration of 50 mg/dL. RE is estimated by ±1.96 × s. RE = 1.96 × 1.12 = ± 2.2 mg/dL Total analytical error (TE) is equal to the sum of SE and RE. TE = SE + RE = -5.3 mg/dL + (-2.2 mg/dL) = -7.5 mg/dL

45. Which assay is used to confirm difficult cases of hypothyroidism? A. Free T3 assay B. Free thyroxine index C. Thyrotropin-releasing hormone (TRH) stimulation test D. TBG assay

45. C The TRH stimulation test is used to confirm borderline cases of abnormal thyroid function. In normal patients, intravenous injection of 500 μg of TRH causes a peak TSH response within 30 minutes. In patients with primary hypothyroidism, there is an exaggerated response (>30 mU/L). Patients with hyperthyroidism do not show the expected rise in TSH after TRH stimulation

45. Which of the following tests is consistently abnormal in osteoporosis? A. High urinary calcium B. High serum Pi C. Low serum calcium D. High urine or serum N-telopeptide of type 1 collagen

45. D Commonly used markers for other bone diseases such as serum or urinary calcium, inorganic phosphorus, total alkaline phosphatase (ALP), and vitamin D are neither sensitive nor specific for osteoporosis. Calcium and phosphorus are usually within normal limits. Although estrogen deficiency reduces formation of 1,25 hydroxyvitamin D (1,25 hydroxycholecalciferol), promoting postmenopausal osteoporosis, the 1,25 hydroxyvitamin D is low in only 30%-35% of cases, and low levels may be caused by other bone disorders. S

46. Which of the following statements regarding the TIBC assay is correct? A. All TIBC methods require addition of excess iron to saturate transferrin B. All methods require the removal of unbound iron C. Measurement of TIBC is specific for transferrinbound iron D. The chromogen used must be different from the one used for measuring serum iron

46. A All TIBC methods require addition of excess iron to saturate transferrin. Excess iron is removed by ion exchange or alumina gel columns or precipitation with MgCO3 and the bound iron is measured by the same procedure as is used for serum iron. Alternatively, excess iron in the reduced state can be added at an alkaline pH. Under these conditions, transferrin will bind Fe2+ and the unbound Fe2+ can be measured directly.

46. Which apoprotein is inversely related to risk of coronary heart disease? A. Apoprotein A-I B. Apoprotein B100 C. Apoprotein C-II D. Apoprotein E4

46. A Apoprotein A-I and apo A-II are the principal apoproteins of HDL, and low apo A-I has a high correlation with atherosclerosis. Conversely, apo-B100 is the principal apoprotein of LDL, and an elevated level is a major risk factor in developing coronary heart disease. Apoprotein assays are not recommended as screening tests because they are not as well standardized as LDL cholesterol assays

46. What is the purpose of LD in the kinetic method of Henry for AST? A. Forms NADH, enabling the reaction to be monitored at 340 nm B. Rapidly exhausts endogenous pyruvate in the lag phase C. Reduces oxaloacetate, preventing product inhibition D. Generates lactate, which activates AST

46. B Patients with liver disease often have high levels of pyruvate and LD. The LD can catalyze the reaction of pyruvate with NADH in the substrate, forming NAD+ and lactate. This would give a falsely high rate for AST because NAD+ is the product measured. Adding LD to the substrate causes pyruvate to be depleted in the first 30 seconds, before AST and MD reactions reach steady state

46. In the coulometric chloride titration: A. Acetic acid in the titrating solution furnishes the counter ion for reduction B. The endpoint is detected by amperometry C. The titrating reagent contains a phosphate buffer to keep pH constant D. Nitric acid (HNO3) is used to lower the solubility of AgCl

46. B Reduction of Ag+ back to Ag° generates the current, which signals the endpoint. The titrating reagent contains HNO3, acetic acid, H2O, and either gelatin or polyvinyl alcohol. The HNO3 furnishes nitrate, which is reduced at the generator cathode, forming ammonium ions. The ammonium becomes oxidized back to nitrate at the indicator anode. Gelatin or polyvinyl alcohol is needed to prevent pitting of the generator anode. Acetic acid lowers the solubility of AgCl, preventing dissociation back to Ag

(see pic) 46. SITUATION: Biochemistry tests are performed 24 hours apart on a patient and delta-check flag is reported for inorganic phosphorus by the laboratory information system. Given the results shown in the table above, identify the most likely cause. A. Results suggest altered metabolic status caused by poor insulin control B. The patient was not fasting when the sample was collected on day 2 C. The samples were drawn from two different patients D. The delta-check limit is invalid when samples are collected 24 or more hours apart

46. B The delta check compares the difference of the patient's two most recent laboratory results within a 3-day period to a delta limit usually determined as a percentage difference. The purpose of the delta check is to detect sample identification errors. A delta-check flag can also be caused by random analytical errors and interfering substances such as hemolysis, icterus, and lipemia, and by metabolic changes associated with disease or treatment

46. In addition to the number of true negatives (TN), which of the following measurements is needed to calculate specificity? A. True positives B. Prevalence C. False negatives D. False positives

46. D The clinical specificity of a laboratory test is defined as the true negatives divided by the sum of true negatives and false positives (FP). % Specificity = TN × 100 TN + FP Specificity is defined as the percentage of disease-free people who have a negative test result. The probability of false positives is calculated from the specificity as: 1-(% specificity)

47. In familial β dyslipoproteinemia (formerly type III hyperlipoproteinemia), which lipoprotein accumulates? A. Chylomicrons B. VLDL C. IDL D. VLDL

47. C IDLs have roughly equal amounts of cholesterol and triglyceride. The IDL has a density of about 1.006-1.020, causing it to float on the 1.063 density potassium bromide solution used to recover LDL by ultracentrifugation. IDL has faster electrophoretic mobility on agarose than beta lipoprotein.

47. What role do CTx and NTx play in the management of osteoporosis? A. Increased urinary excretion is diagnostic of early stage disease B. Increased levels indicate a low risk of developing osteoporosis C. Decreased urinary excretion indicates a positive response to treatment D. The rate of urinary excretion correlates with the stage of the disease

47. C Markers for both bone formation and resorption are used to monitor treatment for osteoporosis. Serum and urinary measurements of CTx and NTx and urinary deoxypyridinoline are used to monitor medications such as biphosphonates that inhibit bone resorption. Levels fall with successful treatment.

47. A new tumor marker for ovarian cancer is evaluated for sensitivity by testing serum samples from patients who have been diagnosed by staging biopsy as having malignant or benign lesions. The following results were obtained: Number of malignant patients who are positive for CA 125 = 21 out of 24 Number of benign patients who are negative for CA 125 = 61 out of 62 What is the sensitivity of the new CA 125 test? A. 98.4% B. 95.3% C. 87.5% D. 85.0%

47. C Sensitivity is defined as the percentage of persons with the disease who have a positive test result. It is calculated as true positives (TP) divided by the sum of TP and false negatives (FN).

47. Which of the following statements regarding the metabolism of bilirubin is true? A. It is formed by hydrolysis of the α methene bridge of urobilinogen B. It is reduced to biliverdin prior to excretion C. It is a by-product of porphyrin production D. It is produced from the destruction of RBCs

47. D Synthesis of porphyrins results in production of heme and metabolism of porphyrins other than protoporphyrin IX yields uroporphyrins and coproporphyrins, not bilirubin. Reticuloendothelial cells in the spleen digest Hgb and release the iron from heme. The tetrapyrrole ring is opened at the α methene bridge by heme oxygenase, forming biliverdin. Bilirubin is formed by reduction of biliverdin at the γ methene bridge. It is complexed to albumin and transported to the liver.

48. A patient presents to the emergency department with symptoms of intoxication including impaired speech and movement. The plasma osmolality was measured and found to be 330 mOs/kg. The osmolal gap was 40 mOsm/Kg. A blood alcohol was measured by the alcohol dehydrogenase method and found to be 0.15% w/v (150 mg/dL). Electrolyte results showed an increased anion gap. Ethylene glycol intoxication was suspected because the osmolal gap was greater than could be explained by ethanol alone, but gas chromatography was not available. Which of the following would be abnormal if this suspicion proved correct? A. Arterial blood gases B. Lactic acid C. Urinary ketones D. Glucose

48. A Ethylene glycol is sometimes used as a substitute for ethanol by alcoholics. It is metabolized to formic acid and glycolic acid by the liver, resulting in metabolic acidosis and an increased anion gap. Lactic acid, glucose, and urinary ketones would be useful in ruling out other causes of metabolic acidosis, but would not be abnormal as a result of ethylene glycol intoxication

48. All of the following compounds contribute to the osmolality of plasma except: A. Lipids B. Creatinine C. Drug metabolites D. Glucose

48. A Osmolality is the concentration (in moles) of dissolved solute per kilogram solvent. Proteins and lipids are not in solution, and do not contribute to osmolality. The nonionized solutes such as glucose and urea contribute 1 osmole per mole per kilogram water, whereas dissociated salts contribute 1 osmole per mole of each dissociated ion or radical

48. Which statement accurately describes serum transaminase levels in AMI? A. ALT is increased 5- to 10-fold after an AMI B. AST peaks 24-48 hours after an AMI and returns to normal within 4-6 days C. AST levels are usually 20-50 times the upper limit of normal after an AMI D. Isoenzymes of AST are of greater diagnostic utility than the total enzyme level

48. B ALT may be slightly elevated after an AMI. AST levels can be up to 5-10 times the URL after AMI, but elevations of this range are also seen in patients with muscular dystrophy, crush injury, pulmonary embolism, infectious mononucleosis, and cancer of the liver.

48. Which of the following mechanisms accounts for the elevated plasma level of β lipoproteins seen in familial hypercholesterolemia (formerly type II hyperlipoproteinemia)? A. Hyperinsulinemia B. ApoB-100 receptor defect C. ApoC-II activated lipase deficiency D. ApoE3 deficiency

48. B The production of excess insulin leads to hypertriglyceridemia and is one mechanism responsible for familial endogenous hypertriglyceridemia. ApoC-II is an activator of lipoprotein lipase, and a homozygous deficiency results in high plasma chylomicrons and VLDL. ApoE3 deficiency is synonymous with inheritance of two apo-E2 alleles that lead to β dyslipoproteinemia

48. A serum thyroid panel reveals an increase in total T4, normal TSH, and normal free T4. What is the most likely cause of these results? A. Primary hyperthyroidism B. Secondary hyperthyroidism C. Euthyroid with increased thyroxine-binding protein D. Subclinical hypothyroidism

48. C Patients with a normal TSH are euthyroid, and most commonly an increase in total T4 in these patients is caused by an increase in TBG. An increase in TBG causes an increase in total T4 but not free T4. Subclinical hypothyroidism is usually associated with a high TSH, but normal free T3 and free T4. When TSH is indeterminate, the diagnosis is made by demonstrating an exaggerated response to the TRH stimulation test

48. A new test for prostate cancer is found to have a sensitivity of 80.0% and a specificity of 84.0%. If the prevalence of prostate cancer is 4.0% in men over 42 years old, what is the predictive value of a positive test result (PV+) in this group? A. 96.0% B. 86.0% C. 32.4% D. 17.2%

48. D The predictive value of a positive test (PV+) is defined as the percentage of persons with a positive test result who will have the disease or condition. It is dependent upon the sensitivity of the test and the prevalence of the disease in the population tested. PV+ is calculated by multiplying the true positives by 100, then dividing by the sum of true positives and false positives

48. What role does vitamin D measurement play in the management of osteoporosis? A. Vitamin D deficiency must be demonstrated to establish the diagnosis B. Vitamin D is consistently elevated in osteoporosis C. A normal vitamin D level rules out osteoporosis D. Vitamin D deficiency is a risk factor for developing osteoporosis

48. D Vitamin D assay is not used to diagnose osteoporosis. Vitamin D deficiency is a cause of secondary osteoporosis, and together with low PTH, calcium, and estrogen are important risk factors. If one or more of these is abnormal, then bone resorption or remodeling may be abnormal, predisposing one to osteoporosis.

49. What measurement in addition to true negatives and prevalence is required to calculate the predictive value of a negative test result (PV-)? A. False negatives B. Variance C. True positives D. False positives

49. A The PV- is defined as the probability that a person with a negative test result is free of disease. A high PV- is a characteristic of a good screening test. The predictive value of a negative test is calculated by multiplying the true negatives by 100, then dividing by the sum of the true negatives and false negatives

49. Which condition gives rise to the highest serum level of transaminases? A. Acute hepatitis B. Alcoholic cirrhosis C. Obstructive biliary disease D. Diffuse intrahepatic cholestasis

49. A The transaminases usually reach 20-50 times the URL in acute viral and toxic hepatitis. Both transaminases are moderately increased (5-10 × URL) in infectious mononucleosis, toxic hepatitis, diffuse intrahepatic obstruction, lymphoma, and cancer of the liver, and slightly increased (2-5 × URL) in cirrhosis and extrahepatic obstruction

49. One mole per kilogram H2O of any solute will cause all of the following except: A. Lower the freezing point by 1.86°C B. Raise vapor pressure by 0.3 mm Hg C. Raise the boiling point by 0.52°C D. Raise osmotic pressure by 22.4 atm

49. B Both freezing point and vapor pressure are lowered by increasing solute concentration. Boiling point and osmotic pressure are raised. Increasing solute concentration of a solution opposes a change in its physical state and lowers the concentration of H2O molecules

49. Which enzyme deficiency is most commonly associated with familial hypertriglyceridemia associated with fasting plasma cholomicrons (formerly type I hyperlipoproteinemia)? A. β Glucocerebrosidase deficiency B. Post-heparin-activated lipoprotein lipase deficiency C. Apo-B deficiency D. Apo-C-III deficiency

49. B Deficiency of capillary endothelial lipase is the most common cause of fasting chylomicronemia. This lipase is also known as post-heparinactivated lipase and apo C-II-activated lipase. β Glucocerebrosidase deficiency results in accumulation of glucocerebrosides and is the cause of Gaucher's disease

49. Which statement about TSH and T4 in early pregnancy is correct? A. TSH and thyroid hormones fall B. TSH falls and thyroid hormones rise C. TSH and thyroid hormones both rise D. TSH rises and thyroid hormones fall

49. B Estrogens released in pregnancy cause an increase in TBG, which causes an increase in total T4 and T3. In early pregnancy, the hCG produced by the placenta stimulates the thyroid, causing an increase in free thyroid hormones. This suppresses TSH production. In the second trimester, as hCG diminishes, free T4 levels fall, and may be lower than 0.8 ng/dL, the lower limit of the adult reference range due to expansion of the blood volume.

(see pic) 49. Given the serum protein electrophoresis pattern shown, which transaminase results would you expect? A. Within normal limits for both B. Marked elevation of both (20-50-fold normal) C. Mild elevations of both (2-5-fold normal) D. Marked elevation of AST but normal ALT

49. C The protein electrophoresis and densitometric scan show a significantly reduced albumin and polyclonal gammopathy. The densitometric scan shows beta-gamma bridging that supports a diagnosis of hepatic cirrhosis. In this condition one would expect two- to fivefold increases of both transaminases with an ALT:AST ratio below 1

49. Which statement best describes testing recommendations for vitamin D? A. Vitamin D testing should be reserved only for those persons who demonstrate hypercalcemia of an undetermined cause B. Vitamin D testing should be specific for the 1,25(OH)D3 form C. Testing should be for total vitamin D when screening for deficiency D. Vitamin D testing should not be performed if the patient is receiving a vitamin D supplement

49. C Vitamin D deficiency is far more common than vitamin D excess, and screening for vitamin D deficiency is advocated especially for dark-skinned persons and people who do not get adequate sunlight. Provitamin D is a steroid, and vitamin D is now considered a hormone rather than a vitamin.

49. In the liver, bilirubin is conjugated by addition of: A. Vinyl groups B. Methyl groups C. Hydroxyl groups D. Glucuronyl groups

49. D The esterification of glucuronic acid to the propionyl side chains of the inner pyrrole rings (I and II) makes bilirubin water soluble. Conjugation is required before bilirubin can be excreted via the bile

5. Which of the following statements regarding proteins is true? A. Total protein and albumin are about 10% higher in ambulatory patients B. Plasma total protein is about 20% higher than serum levels C. Albumin normally accounts for about one-third of the cerebrospinal fluid total protein D. Transudative serous fluid protein is about two-thirds of the serum total protein

5. A Water pools in the vascular bed in nonambulatory patients, lowering the total protein, albumin, hematocrit, and calcium. Plasma levels of total protein are 0.2-0.4 g/dL higher than serum (about 5%) owing to fibrinogen. Cerebrospinal fluid albumin levels are normally 10-30 mg/dL, which is approximately two-thirds of the CSF total protein. Transudates have a total protein below 3.0 g/dL and less than 50% of the serum total protein

5. A green-colored solution would show highest transmittance at: A. 475 nm B. 525 nm C. 585 nm D. 620 nm

5. B Green light consists of wavelengths from 500-550 nm. A green-colored solution with a transmittance maximum of 525 nm and a 50-nm bandpass transmits light of 525 nm and absorbs light below 475 nm and above 575 nm. A solution that is green would be quantitated using a wavelength that it absorbs strongly, such as 450 nm

5. SITUATION: A serum osmolality measured in the emergency department is 326 mOsm/kg. Two hours later, chemistry results are: Na = 135 mmol/L BUN = 18 mg/dL glucose = 72 mg/dL measured osmolality = 318 mOsm/kg What do these results suggest? A. Laboratory error in electrolyte or glucose measurement B. Drug or alcohol intoxication C. Specimen misidentification D. Successful rehydration of the patient

5. B The osmolal gap is the difference between calculated and measured osmolality. Here, the osmolal gap is 38 mOsm/kg. When the osmolal gap is greater than 10 mOsm/kg, an unmeasured solute is present or an analytical error occurred when measuring the osmolality, electrolytes, urea, or glucose. The reference range for serum osmolality is 280-295 mOsm/kg

5. Select the five pharmacological parameters that determine serum drug concentration. A. Absorption, anabolism, perfusion, bioactivation, excretion B. Liberation, equilibration, biotransformation, reabsorption, elimination C. Liberation, absorption, distribution, metabolism, excretion D. Ingestion, conjugation, integration, metabolism, elimination

5. C Liberation is the release of the drug and absorption is the transport of drug from the site of administration to the blood. The percent of drug absorption and the rate of absorption determine the bioavailable fraction, f. This is the fraction of the dose that reaches the blood. Distribution refers to the delivery of the drug to the tissues

5. Which tumor marker is associated with cancer of the urinary bladder? A. CA-19-9 B. CA-72-4 C. Nuclear matrix protein D. Cathepsin-D

5. C Nuclear matrix proteins (NMPs) are RNA-protein complexes. NMP-22 is shed into the urine in persons with bladder carcinoma and is about 25-fold higher than normal in this condition. It has a clinical sensitivity of about 70% but is likely to be negative when the tumor is low grade.

5. Convert 10.0 mg/dL calcium (atomic weight = 40.1) to International System of Units (SI). A. 0.25 B. 0.40 C. 2.5 D. 0.4

5. C The SI unit is the recommended method of reporting clinical laboratory results. The SI unit for all electrolytes is millimole per liter. To convert from milligrams per deciliter to millimoles per liter, multiply by 10 to convert to milligrams per liter, then divide by the atomic mass expressed in milligrams. 10.0 mg/dL × 10.0 dL/1.0 L = 100.0 mg/L 100.0 mg/L × 1.0 mmol/40.1 mg = 2.5 mmol/L

5. What is the normal ratio of bicarbonate to dissolved carbon dioxide (HCO3 -:dCO2) in arterial blood? A. 1:10 B. 10:1 C. 20:1 D. 30:1

5. C When the ratio of HCO3 -:dCO2 is 20:1, the log of salt/acid becomes 1.3. Substituting this in the Henderson-Hasselbalch equation and solving for pH gives pH = 6.1 + log 20; pH = 6.1 + 1.3 = 7.4. Acidosis results when this ratio is decreased, and alkalosis when it is increased

50. A laboratory is establishing a reference range for a new analyte and wants the range to be determined by the regional population of adults age 18 and older. The analyte concentration is known to be independent of race and gender. Which is the most appropriate process to follow? A. Determine the mean and standard deviation of the analyte from 40 healthy adults and calculate the ±2s limit B. Measure the analyte in 120 healthy adults and calculate the central 95th percentile C. Measure the analyte in 120 healthy adults and use the lowest and highest as the reference range limits D. Measure the analyte in 60 healthy adults and 60 adults with conditions that affect the analyte concentration; calculate the concentration of least overlap

50. B Since the concentration of an analyte may not be normally distributed in a population, the reference range should not be determined from the standard deviation. It is more appropriate to determine the central 95th percentile (the range that encompasses 95% of the results). A

50. Which enzyme is responsible for the conjugation of bilirubin? A. β-Glucuronidase B. UDP-glucuronyl transferase C. Bilirubin oxidase D. Biliverdin reductase

50. B UDP-glucuronyl transferase esterifies glucuronic acid to unconjugated bilirubin, making it water soluble. Most conjugated bilirubin is diglucuronide; however, the liver makes a small amount of monoglucuronide and other glycosides. β-Glucuronidase hydrolyzes glucuronide from bilirubin, hormones, or drugs

50. In which case might a very low plasma TSH result not correlate with thyroid status? A. Euthyroid sick syndrome B. Congenital hypothyroidism C. When TBG is elevated D. After high-dose corticosteroid treatment

50. D In persons with severe chronic diseases or who have hCG-secreting tumors, TSH production may be suppressed. Some drugs, especially high doses of corticosteroids, will suppress TSH production. Low TSH levels not matching thyroid status can also be seen in patients who have recently been treated for hyperthyroidism because there is a delay in the pituitary response. High-sensitivity TSH assays that can measure as little as 0.01 mIU/L and free T4 and T3 can help differentiate these conditions from clinical hyperthyroidism.

50. Serial TnI assays are ordered on a patient at admission, 3 hours, and 6 hours afterwards. The samples were collected in heparinized plasma separator tubes. Following are the results (reference range 0-0.03 μg/L) Admission = 3 hours = 6 hours = 0.03 μg/L 0.07 μg/L 0.02 μg/L These results indicate: A. A positive test for acute myocardial infarction B. Unstable angina C. Cardiac injury of severity less than myocardial infarction D. Random error with the 3-hour sample

50. D Troponin assays produce very little fluorescence or chemiluminescence when plasma levels are within the reference range and near the minimum detection limit of the assay. Fibrin, tube additives, and heterophile antibodies have been known to cause spurious elevations, and this result should be treated as a random error because the result before and after are both normal

51. Which of the following is the most accurate measurement of Pi in serum? A. Rate of unreduced phosphomolybdate formation at 340 nm B. Measurement of phosphomolybdenum blue at 680 nm C. Use of aminonaptholsulfonic acid to reduce phosphomolybdate D. Formation of a complex with malachite green dye

51. A The colorimetric method (Fiske and SubbaRow) used previously for Pi reacted ammonium molybdate with Pi, forming ammonium phosphomolybdate (NH4)3[PO4(MoO3)12]. A reducing agent, aminonaptholsulfonic acid (ANS), was added, forming phosphomolybdenum blue

51. Which of the following liver diseases produces the highest levels of transaminases? A. Hepatic cirrhosis B. Obstructive jaundice C. Hepatic cancer D. Alcoholic hepatitis

51. C Elevation of transaminases is greatest in acute hepatitis (20-50 × URL). Levels are moderately elevated (5-10 × URL) in hepatic cancer. They are slightly elevated (2-5 × URL) in chronic hepatitis, hepatic cirrhosis, alcoholic hepatitis, and obstructive jaundice

51. The term δ-bilirubin refers to: A. Water-soluble bilirubin B. Free unconjugated bilirubin C. Bilirubin tightly bound to albumin D. Direct-reacting bilirubin

51. C HPLC separates bilirubin into four fractions: α = unconjugated, β = monoglucuronide, γ = diglucuronide, and δ = irreversibly albumin bound. δ Bilirubin is a separate fraction from the unconjugated bilirubin, which is bound loosely to albumin. δ Bilirubin and conjugated bilirubin react with diazo reagent in the direct bilirubin assay

51. What type of measuring circuit is used in a freezing point osmometer? A. Electrometer B. Potentiometer C. Wheatstone bridge D. Thermal conductivity bridge

51. C The resistance of the thermistor is measured using a network of resistors called a Wheatstone bridge. When the sample is frozen, the bridge is balanced using a calibrated variable resistor, so that no current flows to the readout. The resistance required to balance the meter is equal to the resistance of the thermistor

51. Which of the following is associated with Tangier disease? A. Apoprotein C-II deficiency B. Homozygous apo-B100 deficiency C. Apoprotein C-II activated lipase D. Apoprotein A-I deficiency

51. D Deficiency of apo A-I is seen in Tangier disease, a familial hypocholesterolemia. Heterozygotes have about half of the normal level of HDL (familial hypoalphalipoproteinemia) and homozygotes have almost no detectable HDL. Tangier disease is caused by a mutation of the ATP-binding cassette gene.

51. When comparing the laboratory's monthly mean to its peer group to determine if bias is present, what statistic is most appropriate? A. F test B. Linear regression analysis C. Correlation coefficient D. Standard deviation index

51. D The standard deviation index (SDI) compares the lab's mean to the peer group's mean in terms of standard deviations instead of concentration

52. What is the percentage of serum calcium that is ionized (Cai)? A. 30% B. 45% C. 60% D. 80%

52. B Calcium exists in serum in three forms: protein bound, ionized, and complexed (as undissociated salts). Only Cai is physiologically active. Protein bound and Cai each account for approximately 45% of total calcium, and the remaining 10% is complexed

52. Which of the following processes is part of the normal metabolism of bilirubin? A. Both conjugated and unconjugated bilirubin are excreted into the bile B. Methene bridges of bilirubin are reduced by intestinal bacteria forming urobilinogens C. Most of the bilirubin delivered into the intestine is reabsorbed D. Bilirubin and urobilinogen reabsorbed from the intestine are mainly excreted by the kidneys

52. B Most of the conjugated bilirubin delivered into the intestine is deconjugated by β-glucuronidase and then reduced by intestinal flora to form three different reduction products collectively called urobilinogens. The majority of bilirubin and urobilinogen in the intestine are not reabsorbed. Most of that which is reabsorbed is re-excreted by the liver.

52. Which of the following statements regarding transaminases is true? A. ALT is often increased in muscular disease, pancreatitis, and lymphoma B. ALT is increased in infectious mononucleosis, but AST is usually normal C. ALT is far more specific for liver diseases than is AST D. Substrate depletion seldom occurs in assays of serum from hepatitis cases

52. C ALT is far more specific for liver disease than AST. High ALT may result from nonhepatic causes such as AMI, muscle injury or disease, and severe hemolysis, but nonhepatic sources can be ruled out by a high direct bilirubin. Elevated ALT (e.g., >65 IU/L) is used along with immunologic tests for hepatitis to disqualify blood donors. AST is increased in muscle disease, MI, pancreatitis, and lymphoma. Both transaminases are moderately increased in infectious mononucleosis

52. Which of the following methods is most useful in order to detect sample misidentification? A. Cumulative summation B. Critical limit C. Delta limit D. Significant change limit

52. C Cumulative summation is a statistical method used in quality assurance to detect a trend in QC results. Critical limits are used to define when medical intervention is likely needed to prevent injury or death. The

52. Which of the following statements is correct? A. Both HDL and LDL are homogenous B. There are several subfractions of LDL but not HDL C. There are several subfractions of HDL but not LDL D. There are several subfractions of both HDL and LDL

52. D There are 7 subfractions of LDL and 10 subfractions of HDL. These are grouped into subclasses defined by their molecular sizes. In general, the small, dense LDL subclasses contain more oxidized LDL and are more atherogenic than the larger LDL molecules. The larger HDL subfractions comprising the HDL-3 subclass are associated with a lower risk of coronary artery disease.

53. Which of the following is a characteristic of conjugated bilirubin? A. It is water soluble B. It reacts more slowly than unconjugated bilirubin C. It is more stable than unconjugated bilirubin D. It has the same absorbance properties as unconjugated bilirubin

53. A Conjugated bilirubin refers to bilirubin mono- and diglucuronides. Conjugated bilirubin reacts almost immediately with the aqueous diazo reagent without need for a nonpolar solvent. Historically, conjugated bilirubin has been used synonymously with direct-reacting bilirubin, although the latter includes the δ-bilirubin fraction when measured by the Jendrassik-Grof method

53. The freezing point osmometer differs from the vapor pressure osmometer in that only the freezing point osmometer: A. Cools the sample B. Is sensitive to ethanol C. Requires a thermoelectric module D. Requires calibration with aqueous standards

53. B Alcohol enters the vapor phase so rapidly that it evaporates before the dew point of the sample is reached. Therefore, ethanol does not contribute to osmolality as measured using the vapor pressure osmometer. Freezing-point osmometers measure alcohol and can be used in emergency department settings to estimate ethanol toxicity

53. Select the most sensitive marker for alcoholic liver disease. A. GLD B. ALT C. AST D. γ-Glutamyltransferase (GGT)

53. D Although AST and ALT are elevated in alcoholic hepatitis, GGT is a more sensitive indicator of alcoholic liver disease. Levels of GGT can reach in excess of 25 times the URL in alcoholic hepatitis. It is also markedly elevated in obstructive jaundice; a high GGT supports the inference that liver is the tissue source of an elevated ALP.

54. Which of the following statements regarding urobilinogen is true? A. It is formed in the intestines by bacterial reduction of bilirubin B. It consists of a single water-soluble bile pigment C. It is measured by its reaction with p-aminosalicylate D. In hemolytic anemia, it is decreased in urine and feces

54. A Urobilinogen is a collective term given to the reduction products of bilirubin formed by the action of enteric bacteria. Urobilinogen excretion is increased in extravascular hemolytic anemias and decreased in obstructive jaundice (cholestatic disease). Urobilinogen is measured using Ehrlich's reagent, an acid solution of p-dimethylaminobenzaldehyde

54. In which circumstances is a validation study (versus performing routine quality control) required? A. Instrument recalibration B. Source lamp or ion selective electrode change C. Change in reagent lot D. Change in calibrator lot

54. C All of the listed conditions except a change in the reagent lot number can be validated by assaying two levels of control material following the procedure.

54. The method for measuring iron or lead by plating the metal and then oxidizing it is called: A. Polarography B. Coulometry C. Anodic stripping voltometry D. Amperometry

54. C Anodic stripping voltometry is used to measure lead and iron. The cation of the metal is plated onto a mercury cathode by applying a negative charge. The voltage of this electrode is reversed until the plated metal is oxidized back to a cation. Current produced by oxidation of the metal is proportional to concentration.

54. What is the most appropriate fasting procedure when a lipid study of triglyceride, total cholesterol, HDL cholesterol, and LDL cholesterol tests are ordered? A. 8 hours; nothing but water allowed B. 10 hours; water, smoking, coffee, tea (no sugar or cream) allowed C. 12 hours; nothing but water allowed D. 16 hours; water, smoking, coffee, tea (no sugar or cream) allowed

54. C Lipid orders that include triglyceride and LDL cholesterol should always be performed using a plasma or serum specimen collected after a 12-14 hour fast. The patient should be instructed to drink nothing but water during this period. Fasting specimens are preferred for total and HDL cholesterol as well, but nonfasting specimens may be used for initial screening purposes.

54. Which enzyme is least useful in differentiating necrotic from obstructive jaundice? A. GGT B. ALT C. 5' Nucleotidase D. LD

54. D GGT and 5´ nucleotidase are markedly elevated in both intra- and posthepatic obstruction. ALT is slightly elevated in obstructive jaundice but is markedly elevated in necrotic jaundice. Although LD is usually greater in necrotic jaundice than in obstructive jaundice, elevations in these conditions overlap frequently and result from many other causes

54. Which of the following conditions is associated with a low serum magnesium? A. Addison's disease B. Hemolytic anemia C. Hyperparathyroidism D. Pancreatitis

54. D Low magnesium can be caused by gastrointestinal loss, as occurs in diarrhea and pancreatitis (loss of Mg and Ca as soaps). Hyperparathyroidism causes increased release of both calcium and magnesium from bone. Addison's disease (adrenocorticosteroid deficiency) may be associated with increased magnesium accompanying hyperkalemia. Hemolytic anemia causes increased release of magnesium as well as potassium from damaged red blood cells (RBCs).

56. Which of the following statements regarding ALP is true? A. In normal adults, the primary tissue source is fast-twitch skeletal muscle B. Geriatric patients have a lower serum ALP than other adults C. Serum ALP levels are lower in children than in adults D. Pregnant women have a higher level of serum ALP than other adults

56. D ALP is higher in children than in adults due to bone growth. Children and geriatric patients have higher serum ALP due to increased bone isoenzyme. Serum ALP levels are often two- or threefold higher than the URL in the third term of pregnancy. In nonpregnant normal adults, serum ALP is derived from liver and bone. Liver, bone, placental, renal, and intestinal isoenzymes of ALP can be separated by electrophoresis, and many other ALP isoenzymes have been identified by isofocusing

57. What is the primary means of solute separation in HPLC using a C18 column? A. Anion exchange B. Size exclusion C. Partitioning D. Cation exchange

57. C Stationary phases (column packings) used in HPLC separate solutes by multiple means, but in reverse-phase HPLC the relative solubility between the mobile phase and stationary phase is most important and depends upon solvent polarity, pH, and ionic strength

57. Which of the following conditions is associated with hypokalemia? A. Addison's disease B. Hemolytic anemia C. Digoxin intoxication D. Alkalosis

57. D Addison's disease (adrenocortical insufficiency) results in low levels of adrenal corticosteroid hormones, including aldosterone and cortisol. Because these hormones promote reabsorption of sodium and secretion of potassium by the collecting tubules, patients with Addison's disease display hyperkalemia and hyponatremia. Hemolytic anemia and digoxin intoxication cause release of intracellular potassium.

57. Which isoenzyme of ALP is most heat stable? A. Bone B. Liver C. Intestinal D. Placental

57. D Placental ALP and tumor-associated isoenzymes such as the Regan isoenzyme associated with lung cancer are the only isoenzymes that retain activity when serum is heated to 65° C for 10 minutes. Heat inactivation is used primarily to distinguish liver ALP from bone ALP. If less than 20% activity remains after heating serum to 56°C for 10 minutes, then bone ALP is most likely present

(see pic) 65) A patient has the following test results: Increased serum Ca levels decreased serum phosphate levels increased levels of parathyroid hormone This patient most likely has: a) hyperparathyroidism b) hypoparathyroidism c) nephrosis d) steatorrhea

a) hyperparathyroidism

33) A 68-year-old man arrives in the ER with a glucose of 722 mg/dL and serum acetone of 4+ undiluted. An ABG from this patient is likely to be: a) low pH b) high pH c) low PO2 d) high PO2

a) low pH

10) A 45-year-old woman has a fasting serum glucose concentration of 95 mg/dL and a 2-hour postprandial glucose concentration of 105 mg/dL. The statement which best describes this patient's fasting serum glucose concentration is: a) normal; reflecting glycogen breakdown by the liver b) normal; reflecting glycogen breakdown by skeletal muscle c) abnormal; indicating diabetes mellitus d) abnormal; indicating hypoglycemia

a) normal; reflecting glycogen breakdown by the liver

220) A 68-year-old female patient tells her physician of being "cold all the time" and recent weight gain, with no change in diet. The doctor orders a TSH level, and the lab reports a value of 8.7 uU/mL. (Ref. range: 0.5 - 5.0 uU/mL). This patient most likely has: a) primary hypothyroidism b) Grave disease c) a TSH-secreting tumor d) primary hyperthyroidism

a) primary hypothyroidism

305) the osmol gap is defined as measured Osm/kg minus the calculated Osm/kg. normally the osmol gap is less than a. 0 b. 2 c. 4 d. 6

a. 0

86) The following data was obtained from a cellulose acetate protein electrophoresis scan: albumin area = 75 units gamma globulin area = 30 units total area = 180 units total protein = 6.4 g/dL (65 g/L) The gamma globulin content in g/dL is: a. 1.1 g/dL (11 g/L) b. 2.7 g/dL (27 g/L) c. 3.8 g/dL (38 g/L) d. 4.9 g/dL (49 g/L)

a. 1.1 g/dL (11 g/L)

291) The following results were obtained: urine creatinine = 90 mg/dL serum creatinine = 0.90 mg/dL total body surface = 1.73 m total urine in 24 hrs. = 1500 mL Given the above data, the patient's creatinine clearance in mL/min is: a. 104 b. 124 c. 144 d. 150

a. 104

241) The screen for adrenal cortical hyperfunction with the greatest sensitivity and specificity is: a. 24 hr. urine free cortisol b. plasma cortisol c. urinary 17-hydroxycoricosteroids d. plasma corticosterone

a. 24 hr. urine free cortisol

109) in order to prepare 100 ml of 15 mg/dl BUN (5.35 mmol/l) working standard from a stock standard containing 500 mg/dl (178.5 mmol/l) of urea nitrogen, the number of ml of stock solution that should be used is a. 3 ml b. 5 ml c. 33 ml d. 75 ml

a. 3 ml

316) The buffer pH most effective at allowing amphoteric proteins to migrate toward the cathode in an electrophoretic system would be: a. 4.6 b. 7.5 c. 8.6 d. 9.5

a. 4.6

231) Night blindness is associated with deficiency of which of the following vitamins? a. A b. C c. niacin d. thiamine

a. A

173) When myocardial infarction occurs, the first enzyme to become elevated is: a. CK b. LD c. AST d. ALT

a. CK

25) What is the best method to diagnose lactase deficiency? a. H2 breath test b. plasma aldolase level c. LDH level d. d-xylose test

a. H2 breath test

178) In the immunoinhibition phase of the CK-MB procedure: a. M subunit is inactivated b. B subunit is inactivated c. MB is inactivated d. BB is inactivated

a. M subunit is inactivated

343) In the assay of lactate dehydrogenase , the reaction is dependent upon which of the following coenzyme systems? a. NAD/NADH b. ATP/ADP c. Fe++/Fe+++ d. Cu/Cu++

a. NAD/NADH

3) The preparation of a patient for standard glucose tolerance testing should include: a. a high carbohydrate diet for 3 days b. a low carbohydrate diet for 3 days c. fasting for 48 hrs. prior to testing d. bed rest for 3 days

a. a high carbohydrate diet for 3 days

226) laboratory A measures maternal serum alpha fetoprotein (MSAFP) at 16-18 weeks gestation as a screen for fetal disorders. the 16 week MSAFP median for lab A is 32 ug/ml. a 37 year old woman has an MSAFP level of 34 ug/ml at her 16th week. this result is consitent with a. a normal MSAFP for 16 week gestation b. possible neural tube defect, including spina bifida c. possible multiple birth (twins) d. possible trisomy disorder, including down syndorme

a. a normal MSAFP for 16 week gestation

230) Plasma for cortisol determinations were collected at 7 am, after waking the patient, and at 10 pm that evening. The cortisol level of the morning sample was higher that the evening sample. This is consistent with: a. a normal finding b. Cushing syndrome c. Addison disease d. hypopituitarism

a. a normal finding

(see pic) 165) given the following results. this is most consistent with. a. acute hepatitis b. chronic hepatitis c. obstructive jaundice d. liver hemangioma

a. acute hepatitis

163) The greatest activities of serum AST and ALT are seen in which of the following? a. acute viral hepatitis b. primary biliary cirrhosis c. metastatic hepatic cirrhosis d. alcoholic cirrhosis

a. acute viral hepatitis

314) Hemoglobin S can be separated from hemoglobin D by which of the following methods? a. agar gel electrophoresis at pH 5.9 b. thin layer chromatography c. alkali denaturation d. ammonium precipitation

a. agar gel electrophoresis at pH 5.9

330) Bromcresol purple at a pH of 5.2 is used in a colorimetric method to measure: a. albumin b. globulin c. Bence Jones protein d. immunoprotein

a. albumin

77) the protein that has the highest dye binding capacity is a. albumin b. alpha globulin c. beta globulin d. gamma globulin

a. albumin

82. The cellulose acetate electrophoresis at pH 8.6 of serum proteins will show an order of migration beginning with the fastest migration as follows: a. albumin, alpha 1 globulin, alpha 2 globulin, beta globulin, gamma globulin b. alpha 1 globulin, alpha 2 globulin, beta globulin, gamma globulin, albumin c. albumin, alpha 2 globulin, alpha 1 globulin, beta globulin, gamma globulin d. gamma globulin, beta globulin, alpha 2 globulin, alpha 1 globulin, albumin

a. albumin, alpha 1 globulin, alpha 2 globulin, beta globulin, gamma globulin

135) in bilirubin determinations, the purpose of adding a concentrated caffeine solution or methyl alcohol is to a. allow indirect bilirubin to react with color reagent b. dissolve conjugated bilirubin c. precipitate protein d. prevent any change in pH

a. allow indirect bilirubin to react with color reagent

249) Detection of which of the following substances is most useful to monitor the course of a patient with testicular cancer? a. alpha-fetoprotein b. carcinoembryonic antigen c. prolactin d. testosterone

a. alpha-fetoprotein

107) a critically ill patient becomes comatose. the physician belives that the coma is due to hepatic failure. the assay most helpful in this diagnosis is a. ammonia b. ALT c. AST d. GGT

a. ammonia

123) which of the following determinations is useful in prenatal diagnosis of open neural tube defect a. amniotic fluid alpha fetoprotein b. amniotic fluid estriol c. maternal serum estradiol d. maternal serum estrone

a. amniotic fluid alpha fetoprotein

156) Which of the following enzymes are used in the diagnosis of acute pancreatitis? a. amylase (AMS) and lipase (LPS) b. aspartate aminotransferase (AST) and alanine aminotransferase (ALT) c. 5'-nucleotidase (5'N) and gamma-glutamyl transferase (GGT) d. aspartate aminotransferase (AST) and lactate dehydrogenase (LD)

a. amylase (AMS) and lipase (LPS)

190) The protein portion of an enzyme complex is called the: a. apoenzyme b. coenzyme c. holoenzyme d. proenzyme

a. apoenzyme

18) Total glycosylated hemoglobin levels in a hemolysate reflect the: a. average blood glucose levels of the past 2-3 months b. average blood glucose levels for the past week c. blood glucose level at the time the sample is drawn d. hemoglobin A1c level at the time the sample is drawn

a. average blood glucose levels of the past 2-3 months

340) In the Jendrassik-Grof reaction method for the determination of serum bilirubin concentration quantitation is obtained by measuring the green color of: a. azobilirubin b. bilirubin glucuronide c. urobilin d. urobilinogen

a. azobilirubin

29) Factors that contribute to a PCO2 electrode requiring 60-120 seconds to reach equilibrium include the: a. diffusion characteristics of the membrane b. actual blood PO2 c. type of calibrating standard d. potential of the polarizing mercury cell

a. diffusion characteristics of the membrane

268) A cardiac glycoside that is used in the treatment of congenital heart failure and arrhythmias by increasing the force and velocity of myocardial contraction is: a. digoxin b. acetaminophen c. lithium d. phenytoin

a. digoxin

130) Hemoglobin S can be separated from hemoglobin D by: a. electrophoresis on a different medium and acidic pH b. hemoglobin A2 quantitation c. electrophoresis at higher voltage d. Kleihauer-Betke acid elution

a. electrophoresis on a different medium and acidic pH

353) The osmolality of a urine or serum specimen is measured by a change in the: a. freezing point b. sedimentation point c. midpoint d. osmotic pressure

a. freezing point

23) An infant with diarrhea is being evaluated for a carbohydrate intolerance. His stool yields a positive copper reduction test and a pH of 5.0. It should be concluded that: a. further tests are indicated b. results are inconsistent-repeat both tests c. the diarrhea is not due to carbohydrate intolerance d. the tests provided no useful information

a. further tests are indicated

279) the different water content of erythrocytes and plasma makes true glucose concentrations in whole blood a funciton of the a. hematocrit b. leukocyte count c. erythrocyte count d. erythrocyte indices

a. hematocrit

250) Increased concentrations of alpha-fetoprotein in adults are most characteristically associated with: a. hepatocellular carcinoma b. alcoholic cirrhosis c. chronic active hepatitis d. multiple myeloma

a. hepatocellular carcinoma

34) A patient is admitted to the emergency room in a state of metabolic alkalosis. Which of the following would be consistent with this diagnosis? a. high PCO2, increased HCO3 b. low PCO2, increased HCO3 c. high PCO2, decreased HCO3 d. low PCO2, decreased HCO3

a. high PCO2, increased HCO3

37) Respiratory acidosis is described as a(n): a. increase in CO2 content and PCO2 with a decreased pH b. decrease in CO2 content with an increased pH c. increase in CO2 content with an increased pH d. decrease in CO2 content and PCO2 with a decreased pH

a. increase in CO2 content and PCO2 with a decreased pH

309) Which of the following methods is susceptible to the solvent displacing effect that results in falsely decreased electrolyte values? a. indirect ion selective electrodes b. direct ion selective electrodes c. spectrophotometric d. fluorescence

a. indirect ion selective electrodes

345) in competitive inhibition of an enzyme reaction, the a. inhibitor binds to the enzyme at the same site as does the substrate b. inhibitor often has a chemical structure different to that of the substrate c. activity of the reaction can be decreased by increasing the concentration of the substrate d. activity of teh reaction can be increased by decreasing the temperature

a. inhibitor binds to the enzyme at the same site as does the substrate

259) which of the following factors is not relevant to therapeutic drug monitoring (TDM) of the aminoglycosides, antibiotics and vancomycin? a. intestinal absorption b. nephrotoxicity c. ototoxicity d. renal function

a. intestinal absorption

126) absorption of vitamin B12 requires the presence of a. intrinsic factor b. gastrin c. secretin d. folic acid

a. intrinsic factor

264) zinc protoporphyrin or free erythrocyte protoporphyrin measurements are useful to assess blood concentration of a. lead b. mercury c. arsenic d. beryllium

a. lead

205) in amniotic fluid the procedure used to determine fetal lung maturity is a. lecithin/sphingomyelin ratio b. creatinine c. measurement of absorbance at 450 nm d. alpha fetoprotein

a. lecithin/sphingomyelin ratio

160) Aspartate amino transferase (AST) is characteristically elevated in diseases of the: a. liver b. kidney c. intestine d. pancreas

a. liver

172) Of the following diseases, the one most often associated with elevations of LD isoenzymes 4 and 5 on electrophoresis is: a. liver disease b. hemolytic anemia c. myocardial infarction d. pulmonary edema

a. liver disease

198) in familial hypercholesterolemia, the hallmark finding is an elevation of a. low density lipoproteins b. chylomicrons c. high density lipoproteins d. apolipoprotein A1

a. low density lipoproteins

67) Which of the following is most likely to be ordered in addition to serum calcium to determine cause of tetany? a. magnesium b. phosphate c. sodium d. vitamin D

a. magnesium

72) The primary function of serum albumin in the peripheral blood is to: a. maintain colloidal osmotic pressure b. increase antibody production c. increase fibrinogen formation d. maintain blood viscosity

a. maintain colloidal osmotic pressure

224) In amniotic fluid, the procedure used to detect hemolytic disease of the newborn is: a. measurement of absorbance at 450 nm b. creatinine c. lecithin/sphingomyelin ratio d. estriol

a. measurement of absorbance at 450 nm

26) The expected blood gas results for a patient in chronic renal failure would match the pattern of: a. metabolic acidosis b. respiratory acidosis c. metabolic alkalosis d. respiratory alkalosis

a. metabolic acidosis

27) Severe diarrhea causes: a. metabolic acidosis b. metabolic alkalosis c. respiratory acidosis d. respiratory alkalosis

a. metabolic acidosis

175) an electrophoretic separation of lactate dehydrogenase isoenzymes that demonstrates an elevation in LD1 and LD2 in a flipped pattern is consistent with a. myocardial infarction b. viral hepatitis c. pancreatitis d. renal failure

a. myocardial infarction

272) an active metabolite of amitryptyline is a. notriptyline b. protryptyline c. butriptyline d. norbutryptyline

a. notriptyline

146) A stool specimen that appears black and tarry should be tested for the presence of : a. occult blood b. fecal fat c. trypsin d. excess mucus

a. occult blood

70) A low concentration of serum phosphorus is commonly found in: a. patients who are receiving carbohydrate hyperalimentation b. chronic renal disease c. hypoparathyroidism d. patients with pituitary tumors

a. patients who are receiving carbohydrate hyperalimentation

30) An emphysema patient suffering from fluid accumulation in the alveolar spaces is likely to be in what metabolic state? a. respiratory acidosis b. respiratory alkalosis c. metabolic acidosis d. metabolic alkalosis

a. respiratory acidosis

20) a patient with hemolytic anemia will a. show a decrease in glycated hgb value b. show an increase in glycated hgb value c. show little or no change in glycated hgb value d. demonstrate an elevated hgbA1

a. show a decrease in glycated hgb value

323) In a pH meter reference electrodes may include: a. silver-silver chloride b. quinhydrone c. hydroxide d. hydrogen

a. silver-silver chloride

(see pic) 283) Arterial blood that is collected in a heparinized syringe but exposed to room air would be most consistent with the changes in which of the following specimens? a. specimen A b. specimen B c. specimen C d. specimen D

a. specimen A

169) A common cause of a falsely increased LD1 fraction of lactic dehydrogenase is: a. specimen hemolysis b. liver disease c. congestive heart failure d. drug toxicity

a. specimen hemolysis

354) Which of the following applies to cryoscopic osmometry? a. temperature at equilibrium is a function of the number of particles in solution b. temperature plateau for a solution is horizontal c. freezing point of a sample is absolute d. initial freezing of a sample produces an immediate solid state

a. temperature at equilibrium is a function of the number of particles in solution

(see pic) 346) refer to the following illustration. the figure above shows the reciprocal of the measured velocity of an enzyme reaction plotted against the reciprocal of the substrate concentration. true statements about this figure include. a. the intercept of the line on the ordinate (y axis) can be used to calculate the Vmax b. the straight line indicates that the enzyme reaction proceeds according to zero order kinetics c. the intercept on the ordinate (y axis) can be used to calculate the Michaelis Menten constant d. the fact the substrate concentraiton is plotted on both sides of the zero point indicates that the reaction is reversible

a. the intercept of the line on the ordinate (y axis) can be used to calculate the Vmax

355) assay of transketolase activity in blood is used to detect deficiency of a. thiamine b. folic acid c. ascorbic acid d. riboflavin

a. thiamine

256) blood specimens for digoxin assays should be obtained between 8 hours or more after drug administration because a. tissue and serum levels need to reach equilibrium b. serum digoxin concentration will be falsely low prior to 6 hours c. all of the digoxin is in the cellular fraction prior to 6 hours d. digoxin protein biding interactions are minimal prior to 6 hours

a. tissue and serum levels need to reach equilibrium

196) if the LDL cholesterol is to be calculated by the friedwald formula, what are the 2 measurements that need to be carried out by the same chemical procedure? a. total cholesterol and HDL cholesterol b. total cholesterol and triglyceride c. triglyceride and chylomicrons d. apolipoprotein A and apolipoprotein B

a. total cholesterol and HDL cholesterol

252) Which of the following is useful in the detection and management of carcinoma of the prostate? a. total prostate-specific antigen b. prostatic acid phosphatase c. human chorionic gonadotropin d. alpha-fetoprotein

a. total prostate-specific antigen

278) a lipemic serum is separated is separated and frozen at -20 C for assay at a later date. one week later, prior to performing an assay for triglycerides, the specimen should be a. warmed to 37 C and mixed thoroughly b. warmed to 15 C and centrifuged c. transferred to a glycerated test tube d. discarded and new specimen obtained

a. warmed to 37 C and mixed thoroughly

59) which of the following is true about direct ion selective electrodes for electrolytes? a. whole blood specimens are acceptable b. elevated lipids cause falsely decreased results c. elevated proteins causes falsely decreased results d. elevated platelets cause falsely increased results

a. whole blood specimens are acceptable

(see pic) 41) The following laboratory results were obtained: Serum electrolytes Arterial blood sodium 136 mEq/L pH 7.32 potassium 4.4 mEq/L PCO2 79 mm Hg chloride 92 mEq/L bicarbonate 40 mEq/L These results are most compatible with: a) respiratory alkalosis b) respiratory acidosis c) metabolic alkalosis d) metabolic acidosis

b) respiratory acidosis

215) A 2-year-old child with a decreased serum T4 is described as being somewhat dwarfed, stocky, overweight, and having course features. Of the following, the most informative additional laboratory test would be the serum: a) thyroxine binding globulin (TBG) b) thyroid-stimulating hormone (TSH) c) triiodothyronine (T3) d) cholesterol

b) thyroid-stimulating hormone (TSH)

136) if the total bilirubin is 3.1 mg/dl and the conjugated bilirubin is 2.0 mg/dl, the unconjugated bilirubin is a. 0.5 mg/dl b. 1.1 mg/dl c. 2.2 mg/dl d. 5.1 mg/dl

b. 1.1 mg/dl

4) If a fasting glucose was 90 mg/dL, which of the following 2 hr. postprandial glucose result would most closely represent normal glucose metabolism? a. 55 mg/dL (3.0 mmol/L) b. 100 mg/dL (5.5 mmol/L) c. 180 mg/dL (9.9 mmol/L) d. 260 mg/dL (14.3 mmol/L)

b. 100 mg/dL (5.5 mmol/L)

295) I125 has a physical half life of 60.0 days. a sample tested today had activity of 10,000 CPM/ml. how many days from today will the count be 1250 CPM/ml? a. 60 b. 180 c. 240 d. 1250

b. 180

31) At blood pH 7.40, what is the ratio of bicarbonate to carbonic acid? a. 15:1 b. 20:1 c. 25:1 d. 30:1

b. 20:1

12) In the fasting state, the arterial and capillary blood glucose concentration varies from the venous glucose concentration by approximately how many mg/dL (mmol/L) a. 1 mg/dL (0.05 mmol/L) b. 5 mg/dL (0.27 mmol/L) c. 10 mg/dL (0.55 mmol/L) d. 15 mg/dL (0.82 mmol/L)

b. 5 mg/dL (0.27 mmol/L)

5) A healthy person with a blood glucose of 80 mg/dL (4.4 mmol/L) would have a simultaneously determined cerebrospinal fluid glucose value of: a. 25 mg/dL (1.4 mmol/L) b. 50 mg/dL (2.3 mmol/L) c. 100 mg/dL (5.5 mmol/L) d. 150 mg/dL (8.3 mmol/L)

b. 50 mg/dL (2.3 mmol/L)

225) During a normal pregnancy, quantitative human chorionic gonadotropin levels peak how many weeks after the last menstrual period? a. 2 - 4 b. 8 - 10 c. 14 - 16 d. 18 - 20

b. 8 - 10

(see pic) 121) which of the following is an example of a peptide bond? a. A b. B c. C d. D

b. B

48) What battery of tests is most useful in evaluating an anion gap of 22 mEq/L (22 mmol/L)? a. Ca++, Mg++, PO-4, and pH b. BUN, creatinine, salicylate and methanol c. AST, ALT, LD and amylase d. glucose, CK, myoglobin, and cryoglobulin

b. BUN, creatinine, salicylate and methanol

233) Scurvy is associated with deficiency of which of the following vitamins? a. A b. C c. niacin d. thiamine

b. C

170) The presence of which of the following isoenzymes indicates acute myocardial damage? a. CK-MM b. CK-MB c. CK-BB d. none

b. CK-MB

221) Which of the following is secreted by the placenta and used for the early determination of pregnancy? a. follicle stimulating hormone (FSH) b. Human chorionic gonadotropin (HCG) c. luteinizing hormone (LH) d. progesterone

b. Human chorionic gonadotropin (HCG)

335) in the hexokinase method for glucose determination, the actual end product measured is the a. amount of hydrogen peroxide produced b. NADH produced from the reduction of NAD c. amount of glucose combined with bromcresol purple d. condensation of glucose with an aromatic amine

b. NADH produced from the reduction of NAD

324) amperometry is the principle of the a. PCO2 electrode b. PO2 electrode c. pH electrode d. ionized calcium electrode

b. PO2 electrode

299) The creatinine clearance is equal to: a. urinary creatinine (mg/L) / [volume of urine (mL/min) x plasma creatinine (mg/L)] b. [urinary creatinine (mg/L) x volume (mL/min) / plasma creatinine (mg/L)] c. urinary creatinine (mg/L) / [volume of urine (mL/hour) x plasma creatinine (mg/L)] d. [urinary creatinine (mg/L) x volume (mL/hour)] / plasma creatinine (mg/L)

b. [urinary creatinine (mg/L) x volume (mL/min) / plasma creatinine (mg/L)]

326) Blood PCO2 may be measured by: a. direct colorimetric measurement of dissolved CO2 b. a self-contained potentiometric electrode c. measurement of CO2-saturated hemoglobin d. measurement of CO2 consumed at the cathode

b. a self-contained potentiometric electrode

(see pic) 277) serum from a patient with metastatic carcinoma of the prostate was separated from the clot and stored at room temperature. the following results were obtained. the technician should repeat the a. LD using diluted serum b. acid phosphatase with freshly drawn serum c. LD with fresh serum d. tests using plasma

b. acid phosphatase with freshly drawn serum

95) (see pic) refer to the following illustration. this electrophoresis pattern is consistent with a. cirrhosis b. acute inflammation c. polyclonal gammopathy (chronic inflammation) d. alpha 1 antitrypsin deficiency; severe emphysema

b. acute inflammation

245) how is primary hypocortisolism (Addison disease) differentiated from secondary hypocortisolism (of pituitary origin)? a. adrenal corticotropic hormone (ACTH) is decreased in primary and elevated in secondary b. adrenal corticotropic hormone (ACTH) is elevated in primary and decreased in secondary c. low aldosterone and hypoglycemia present with secondary hypocortisolism d. normal cortisol levels and blood pressure with primary hypocortisolism

b. adrenal corticotropic hormone (ACTH) is elevated in primary and decreased in secondary

(see pic) 78) refer to the following illustration. identify the serum protein fraction on the left of the illustration. a. gamma globulin b. albumin c. alpha 1 globulin d. alpha 2 globulin

b. albumin

216) assays for free T4 measure hormone not bound to thyroxine binding prealbumin, thyroxine binding globulin and a. thyrotropin releasing hormone b. albumin c. free T3 d. thyroid stimulating hormone

b. albumin

227) Which of the following steroids is an adrenal cortical hormone? a. angiotensinogen b. aldosterone c. epinephrine d. growth hormone

b. aldosterone

157) which of the following enzymes catalyzes the conversion of starch to glucose and maltose? a. malate dehydrogenase b. amylase c. creatine kinase d. isocitric dehydrogenase

b. amylase

7) Cerebrospinal fluid for glucose assay should be: a. refrigerated b. analyzed immediately c. heated to 56C d. stored at room temperature after centrifugation

b. analyzed immediately

269) A salicylate level is performed to detect toxicity caused by ingestion of excess: a. acetaminophen b. aspirin c. ibuprofen d. pseudoephedrine

b. aspirin

187) The most heat labile fraction of alkaline phosphatase is obtained from: a. liver b. bone c. intestine d. placenta

b. bone

142) Kernicterus is an abnormal accumulation of bilirubin in: a. heart tissue b. brain tissue c. liver tissue d. kidney tissue

b. brain tissue

243) estrogen and progesterone receptor assays are useful in identifying patients who are likely to benefit from endocrine therapy to treat which of the following? a. ovarian cancer b. breast cancer c. endometriosis d. amenorrhea

b. breast cancer

45) Osmolal gap is the difference between: a. the ideal and real osmolality values b. calculated and measured osmolality values c. plasma and water osmolality values d. molality and molarity at 4*C

b. calculated and measured osmolality values

122) 90% of the copper present in the blood is bound to: a. transferrin b. ceruloplasmin c. albumin d. cryoglobulin

b. ceruloplasmin

281) As part of a hyperlipidemia screening program, the following results were obtained on a 25 yr. old woman 6 hrs. after eating: triglycerides = 260 mg/dL cholesterol = 120 mg/dL Which of the following is the best interpretation of these results? a. both results are normal and not affected by the recent meal b. cholesterol is normal, but triglycerides are elevated, which may be attributed to the recent meal c. both results are elevated, indicating a metabolic problem in addition to the nonfasting state d. both results are below normal despite the recent meal, indicating a metabolic problem

b. cholesterol is normal, but triglycerides are elevated, which may be attributed to the recent meal

184. Given the following results: alkaline phosphatase: slight increase aspartate amino transferase: slight increase alanine amino transferase: slight increase gamma-glutamyl transferase: slight increase this is most consistent with a. acute hepatitis b. chornic hepatitis c. obstructive jaundice d. liver hemangioma

b. chornic hepatitis

(see pic) 207) a fasting serum sample from an asymptomatic 43 year old woman is examined visually and chemically with the following results. this sample contains predominantly a. chylomicrons, alone b. chylomicrons and very low density lipoproteins (VLDL) c. very low density lipoproteins (VLDL) and low density lipoproteins (LDL) d. high density lipoproteins (HDL)

b. chylomicrons and very low density lipoproteins (VLDL)

115) technical problems encountered during the collection of amniotic fluid specimen caused doubt as to whether the specimen was amniotic in origin. which one of the following procedures would best establish that the fluid is amniotic in origin? a. measurement of absorbance at 450 nm b. creatinine measuremnt c. lecithin/sphingomyelin ratio d. human amniotic placental lactogen (HPL)

b. creatinine measuremnt

73. In a pleural effusion caused by Streptococcus pneumoniae, the protein value of the pleural fluid as compared to the serum value would probably be: a. decreased by 2 b. decreased by 1/2 c. increased by 1/2 d. equal

b. decreased by 1/2

60) Sodium determination by indirect ion selective electrode is falsely decreased by: a. elevated chloride levels b. elevated lipid levels c. decreased protein levels d. decreased albumin levels

b. elevated lipid levels

222) during pregnancy, the form of estrogen measured in urine is mostly a. estradiol b. estriol c. estrone d. pregnanediol

b. estriol

161) Amino tranferase enzymes catalyze the: a. exchange of amino groups and sulfhydydryl groups between alpha-amino and sufur-containing acids b. exchange of amino and keto groups between alpha-amino and alpha-keto acids c. hydrolysis of amino acids and keto acids d. reversible transfer of hydrogen from amino acids to coenzyme

b. exchange of amino and keto groups between alpha-amino and alpha-keto acids

149) which of the following elevates carboxyhemoglobin a. nitrite poisoning b. exposure to carbon monoxide c. sulfa drug toxicity d. sickle cell anemia

b. exposure to carbon monoxide

14) which one of the following values obtained during a glucose tolerance test are diagnostic of diabetes mellitus? a. 2 hour specimen = 150 mg/dl (8.3 mmol/l) b. fasting plasma glucose = 126 mg/dl (6.9 mmol/l) c. fasting plasma glucose = 110 mg/dl (6.1 mmol/l) d. 2 hour specimen = 180 mg/dl (9.9 mmol/l)

b. fasting plasma glucose = 126 mg/dl (6.9 mmol/l)

80) in electrophoresis of proteins, when the sample is placed in an electric field connected to a buffer of pH 8.6, all of the proteins a. have a positive charge b. have a negative charge c. are electrically neutral d. migrate toward the cathode

b. have a negative charge

351) Which of the following methods for quantitation of high-density lipoprotein is most suited for clinical laboratory use? a. Gomori procedure b. homogeneous c. column chromatography d. agarose gel electrophoresis

b. homogeneous

212) A patient has the following thyroid profile: total T4 = decreased free T4 = decreased thyroid peroxisase antibody = positive TSH = increased This patient most probably has: a. hyperthyroidism b. hypothyroidism c. a normal thyroid d. Graves disease

b. hypothyroidism

310) upon development of a thin layer chromatogram for drug analysis all drug spots (including the standards had migrated with the solvent front. the most probable cause for this would be a. environmental temperature too warm b. incorrect aqueous to nonaqueous solvent mixture c. too much sample applied d. chromatogram dried too quickly

b. incorrect aqueous to nonaqueous solvent mixture

236) The major action of angiotensin II is: a. increase pituitary secretion of vasopressin b. increase vasoconstriction c. increase parathormone secretion by the parathyroid d. decrease adrenal secretion of aldosterone

b. increase vasoconstriction

158) which of the following sets of results would be consistent with macroamylasemia? a. normal serum amylase and elevated urine amylase values b. increased serum amylase and normal urine amylase values c. increased serum and urine amylase values d. normal serum and urine amylase values

b. increased serum amylase and normal urine amylase values

103) decreased serum iron associated with an increased TIBC is compatible with which of the following disease states? a. anemia of chronis infection b. iron deficiency anemia c. chronic liver disease d. nephrosis

b. iron deficiency anemia

(see pic) 348) the illustration below represents a Lineweaver Burk plot of 1/v vs 1/[S] in an enzyme reaction and the following assumptions should be made. the enzyme concentration was the same for reactions A and B, the substrate concentration was in excess for reactions A and B, reaction A occurred under ideal conditions. which of the following statements about reaction B is true? a. it illustrates non-competititve inhibition b. it illustrates competitive inhibition c. it illustrates neither competitive nor noncompetitive inhibition d. it could be the result of heavy metal contamination

b. it illustrates competitive inhibition

9) Serum levels that define hypoglycemia in pre-term or low birth weight infants are: a. the same as adults b. lower than adults c. the same as a normal full-term infant d. higher than a normal full-term infant

b. lower than adults

75) when performing a manual protein analysis on a xanthochromic spinal fluid, the technician should a. perform the test as usual b. make the patient blank c. centrifuge the specimen d. dilute the specimen with deionized water

b. make the patient blank

248) Clinical assays for tumor markers are most important for: a. screening for the presence of cancer b. monitoring the course of a known cancer c. confirming the absence of disease d. identifying patients at risk for cancer

b. monitoring the course of a known cancer

312) reverse phase high performance liquid chromatography is being increasingly utilized in therapeutic drug monitoring. the term reverse phase implies that the column eluant is a. pumped up the column b. more polar than the stationary phase c. always nonpolar d. less polar than the stationary phase

b. more polar than the stationary phase

97) analysis of CSF for oligoclonal bands is used to screen for which of the following disease states a. multiple myeloma b. multiple sclerosis c. myasthneia gravis d. von williebrand disease

b. multiple sclerosis

258) free therapeutic drug levels are usually higher when serum protein concentrations are below normal. in which of the following conditions would this most likely occur? a. acute inflammation b. nephrotic syndrome c. pregnancy d. multiple myeloma

b. nephrotic syndrome

(see pic) 124) below are the results of a protein electrophoresis. these results are consistent with a a. normal serum protien pattern b. normal CSF protein pattern c. abnormal serum protein pattern d. abnormal CSF protein pattern

b. normal CSF protein pattern

(see pic) 223) refer to the following graph. the HCG levels shown in the above graph most probably represent a. hydatidiform mole following miscarriage at 4 months b. normal pregnancy c. development of hydatiform mole d. miscarriage at 2 months with retained placenta

b. normal pregnancy

325) Most automated blood gas analyzers directly measure: a. pH, HCO3 and %O2 saturation b. pH, PCO2 and PO2 c. HCO3, PCO2, and PO2 d. pH, PO2 and % O2 saturation

b. pH, PCO2 and PO2

64) The regulation of calcium and phosphorous metabolism is accomplished by which of the following glands? a. thyroid b. parathyroid c. adrenal gland d. pituitary

b. parathyroid

63) Calcium concentration in the serum is regulated by: a. insulin b. parathyroid hormone c. thyroxine d. vitamin C

b. parathyroid hormone

211) TSH is produced by the: a. hypothalamus b. pituitary gland c. adrenal cortex d. thyroid

b. pituitary gland

153) which of the following enzymes of heme biosynthesis is inhibited by lead? a. aminolevulinate synthase b. porphobilinogen synthase c. uroporphyrinogen synthase d. bilirubin synthase

b. porphobilinogen synthase

317) on electrophoresis, transient bisalbuminemia or a grossly widened albumin zone is associated with a. dirty applicators b. presence of therapeutic drugs in serum sample c. endosmosis d. prestaining with tracer dye

b. presence of therapeutic drugs in serum sample

(see pic) 206) refer to the following illustration. the class of phospholipid surfactants represented by the dotted line on teh amniotic fluid analysis shown above is thought to originate in what fetal organ system? a. cardiovascular b. pulmonary c. hepatic d. placental

b. pulmonary

108) A serum sample demonstrates an elevated result when tested with the Jaffe reaction. This indicates: a. prolonged hypothermia b. renal functional impairment c. pregnancy d. arrhythmia

b. renal functional impairment

117) which of the following substances is the biologically active precursor of a fat soluble vitamin? a. biotin b. retinol c. folic acid d. ascorbic acid

b. retinol

57) The solute that contributes the most to the total serum osmolality is : a. glucose b. sodium c. chloride d. urea

b. sodium

204) a 9 month old boy from israel has gradually lost the ability to sit up, and develops seizures. he has an increased among of a phospholipid called GM2 ganglioside in his neurons, and e lacks the enzyme hexosaminidase A in his leukocytes. these findings suggest a. Neimann Pick disease b. tay sachs diseaes c. phenylketonuria d. hurler syndrome

b. tay sachs diseaes

76) the direction in which albumin migrates (ie toward anode or cathode) during electrophoresis separation of serum proteins, at pH 8.6, is determined by a. the ionization of the amine groups, yielding a net positive charge b. the ionization of the carboxyl groups, yielding a net negative charge c. albumin acting as a zwitterion d. the density of the gel layer

b. the ionization of the carboxyl groups, yielding a net negative charge

138) A serum sample was assayed for bilirubin at 10 AM, and the result was 12 mg/dL (205.6 umol/L). The same sample was retested at 3 PM. The result now is 8 mg/dL (136.8 umol/L). The most likely explanation for this discrepancy is: a. the reagent has deteriorated b. the sample was exposed to light c. a calculation error in the first assay d. the sample was not refrigerated

b. the sample was exposed to light

118) the troponin complex consists of a. troponin T, calcium and tropomyosin b. troponin C, troponin I, and troponin T c. troponin I, actin, and tropomyosin d. troponin C, myoglobin, and actin

b. troponin C, troponin I, and troponin T

162) Aspartate aminotransferase and alanine aminotransferase are both elevated in which of the following diseases? a. muscular dystrophy b. viral hepatitis c. pulmonary emboli d. infectious mononucleosis

b. viral hepatitis

62) Which percentage of total serum calcium is nondiffusible protein bound? a. 80% - 90% b. 51% - 60% c. 40% - 50% d. 10% - 30 %

c. 40% - 50%

197) the chemical composition of HDL cholesterol corresponds to (see pic for answer choices)

c. 5%; 15%; 50%

61) A physician requested that electrolytes on a multiple myeloma patient specimen be run by direct ISE and not indirect ISE because: a. excess protein binds Na in indirect ISE b. Na falsely increased by indirect ISE c. Na is falsely decreased by indirect ISE d. excess protein reacts with diluent in indirect ISE

c. Na is falsely decreased by indirect ISE

328) Which blood gas electrode is composed of silver/silver chloride reference electrode and glass? a. PO2 b. pH c. PCO2 d. HCO3

c. PCO2

88) which of the following serum protein fractions is most likely to e elevated in patients with nephrotic syndrome? a. alpha 1 globulin b. albumin c. alpha 2 globulin d. beta globulin and gamma globulin

c. alpha 2 globulin

356) in amniotic fluid, the procedure used to detect Rh isosensitization is a. human amniotic placental lactogen (HPL) b. alpha fetoprotein c. measurement of absorbance at 450 nm d. creatinine

c. measurement of absorbance at 450 nm

90) (see pic) refer to the following illustration. this electrophoresis pattern is consistent with a. cirrhosis b. acute inflammation c. monoclonal gammopathy d. polyclonal gammopathy (eg chronic inflammation)

c. monoclonal gammopathy

174) A scanning of a CK isoenzyme fractionation revealed 2 peaks: A slow cathodic peak (CK-MM) and an intermediate peak (CK-MB). A possible interpretation for this pattern is: a. brain tumor b. muscular dystrophy c. myocardial infarction d. viral hepatitis

c. myocardial infarction

179) The presence of increased CK-MB activity on a CK electrophoresis pattern is most likely found in a patient suffering from: a. acute muscular stress following strenuous exercise b. malignant liver disease c. myocardial infarction d. severe head injury

c. myocardial infarction

260) The drug procainamide is prescribed to treat cardiac arrhythmia. What biologically active liver metabolite of procainamide is often measured simultaneously? a. phenobarbital b. quinidine c. n-acetyl procainamide d. lidocaine

c. n-acetyl procainamide

242) a patient has signs and symptoms suggestive of acromegaly. the diagnosis would be confirmed if the patient had which of the following? a. an elevated serum phosphate concentration b. a decreased serum growth hormone releasing factor concentration c. no decrease in serum growth hormone concentration 90 min after oral glucose administration d. an increased serum somatostatin concentration

c. no decrease in serum growth hormone concentration 90 min after oral glucose administration

(see pic) 344) refer to the following illustration. this illustration represents the change in absorbance at 340 nm over a period of 8 minutes in an assay for lactate dehydrogenase. true statements about this figure include a. the reaction following zero order kinetics btw 5 and 8 min b. the reaction is proceeding from lactate to pyruvate c. nonlinearity after 6 min is due to substrate exhaustion d. the change in absorbance is due to reduction of NAD to NADH

c. nonlinearity after 6 min is due to substrate exhaustion

47) Quantitation of Na+ and K+ by ion selective electrode is the standard method because: a. dilution is required for flame photometry b. there is no lipoprotein interference c. of advances in electrochemistry d. of the absence of an internal standard

c. of advances in electrochemistry

275) Blood received in the lab for blood gas analysis must meet which of the following requirements? a. on ice, thin fibrin strands only, no air bubbles b. on ice, no clots, fewer than 4 air bubbles c. on ice, no clots, no air bubbles d. room temp, no clots, no air bubbles

c. on ice, no clots, no air bubbles

289) if the PKa is 6.1, the CO2 content is 25 mM/L, the salt equals the total CO2 content minus the carbonic acid; the carbonic acid equals 0.03 x PCO2 and PCO2 = 40 mmHg, it may be concluded that (see pic for answers)

c. pH = 6.1 + log [(25-1.2)/1.2]

238) Which of the following hormones regulates normal blood calcium levels? a. thyroxine b. estriol c. parathyroid hormone d. growth hormone

c. parathyroid hormone

79) the biuret reaction for the analysis of serum protein depends on the number of a. free amino groups b. free carboxyl groups c. peptide bonds d. tyrosine residues

c. peptide bonds

102) an elevated serum iron with normal iron binding capacity is most likely associated with a. iron deficiency anemia b. renal damage c. pernicious anemia d. septicemia

c. pernicious anemia

96) (see pic) refer to the following illustration. this electrophoresis pattern is consistent with a. cirrhosis b. acute inflammation c. polyclonal gammopathy (chronic inflammation) d. alpha 1 antitrypsin deficiency; severe emphysema

c. polyclonal gammopathy (chronic inflammation)

152) a fresh urine sample is recieved for analysis for oprphyrins or phorphria without further information or specifications. initial analysis should include a. porphyrin screen and quantitative total porphyrin b. quantitative total porphyrin and porphobiliogen screen c. porphyrin and porphobilogen screen d. porphobilinogen screen and ion exchange analysis for porphobilinogen

c. porphyrin and porphobilogen screen

327) Valinomycin enhances the selectivity of the electrode used to quantitate: a. sodium b. chloride c. potassium d. calcium

c. potassium

276) after a difficult venipuncture requiring prolonged application of the tourniquet, the serum K was found to be 6.8 mEq/L (6.8 mmol/l). the best course of action is to a. repeat the test using the same specimen b. adjust the value based on the current serum Na c. repeat the test using freshly drawn serum d. cancel the test

c. repeat the test using freshly drawn serum

105) To assure an accurate ammonia level result, the specimen should be: a. incubated at 37 C prior to testing b. spun and separated immediately, tested as routine c. spun, separated, iced, and tested immediately d. stored at room temp. until tested

c. spun, separated, iced, and tested immediately

201) which of the following diseases results from a familial absence of high lipoprotein? a. krabbe disease b. gaucher disease c. tangier disease d. tay sachs disease

c. tangier disease

99) Total iron binding capacity measures the serum iron transporting capacity of: a. hemoglobin b. ceruloplasmin c. transferrin d. ferritin

c. transferrin

214) The majority of thyroxine (T4) is converted into the more biologically active hormone: a. thyroglobulin b. thyroid-stimulating hormone c. triiodothyronine d. thyrotropin-releasing hormone

c. triiodothyronine

111) the principle exretory form of nitrogen is a. amino acids b. creatinine c. urea d. uric acid

c. urea

114) A blood creatinine value of 5.0 mg/dL (442.0 umol/L) is most likely to be found with which of the following blood values? a. osmolality: 292 mOsm/kg b. uric acid: 8 mg/dL c. urea nitrogen: 80 mg/dL d. ammonia: 80 ug/dL

c. urea nitrogen: 80 mg/dL

84) maple syrup urine disease is characterized by an increase in which of the following urinary amino acids? a. phenylalamine b. tyrosine c. valine, leucine, and isoleucine d. cystine and cysteine

c. valine, leucine, and isoleucine

274) Testing for the diagnosis of lead poisoning should include: a. erythrocyte protoporphyrin b. urine delta-aminolevulinic acid c. whole blood lead d. zinc protoporphyrin

c. whole blood lead

271) serum and urine copper levels are assayed on a hospital patient with the following results. this is most consistent with a. normal copper levels b. wilms tumor c. wilson disease d. addison disease

c. wilson disease

40) A blood gas sample was sent to the lab on ice, and a bubble was present in the syringe. The blood had been exposed to room air for at least 30 mins. The following change in blood gases will occur: a. CO2 content increased/PCO2 decreased b. CO2 content and PO2 increased/pH increased c. CO2 content and PCO2 decreased/pH decreased d. PO2 increased/HCO3 decreased

d. PO2 increased/HCO3 decreased

92) a characteristic of the bence jones protein that is used to distinguish it from other urinary proteins is its solubility a. in ammonium sulfate b. in sulfuric acid c. at 40-60 C d. at 100 C

d. at 100 C

337) Before unconjugated bilirubin can react with Ehrlich diazo reagent, which of the following must be added? a. acetone b. ether c. distilled water d. caffeine

d. caffeine

137) the principle of the tablet test for bilirubin in urine or feces is a. the reaction btw bile and 2,4 dichloronitrobenzene to yellow color b. the liberation of oxygen by bile to oxidize orthotolidine to a blue purple color c. chemical coupling of bile with a diazonium salt to form a brown color d. chemical coupling of bilirubin with a diazonium salt to form a purple color

d. chemical coupling of bilirubin with a diazonium salt to form a purple color

53) The buffering capacity of blood is maintained by a reversible exchange process between bicarbonate and: a. sodium b. potassium c. calcium d. chloride

d. chloride

228) What common substrate is used in the biosynthesis of adrenal steroids, including androgens and estrogens? a. cortisol b. catecholamines c. progesterone d. cholesterol

d. cholesterol

285) Unless blood gas measurements are made immediately after sampling, in vitro glycolysis of the blood causes a: a. rise in pH and PCO2 b. fall in pH and a rise in PO2 c. rise is pH and a fall in PO2 d. fall in pH and a rise in PCO2

d. fall in pH and a rise in PCO2

87) a patient is admitted with biliary cirrhosis. if a serum protein electrophoresis is performed, which of the following globulin fractions will be most elevated? a. alpha 1 b. alpha 2 c. beta d. gamma

d. gamma

113) Creatinine clearance is used to estimate the: a. tubular secretion of creatinine b. glomerular secretion of creatinine c. renal glomerular and tubular mass d. glomerular filtration rate

d. glomerular filtration rate

286) Which of the following serum constituents is unstable if a blood specimen is left standing at room temp for 8 hrs. before processing: a. cholesterol b. triglycerides c. creatinine d. glucose

d. glucose

11) Pregnant women with symptoms of thirst, frequent urination or unexplained weight loos should have which of the following tests performed? a. tolbutamide test b. lactose tolerance test c. epinephrine tolerance test d. glucose tolerance test

d. glucose tolerance test

13) The conversion of glucose or other hexoses into lactate or pyruvate is called: a. glycogenesis b. glycogenolysis c. gluconeogenesis d. glycolysis

d. glycolysis

16) Monitoring long term glucose control in patients with adult onset diabetes mellitus can best be accomplished by measuring: a. weekly fasting 7 am serum glucose b. glucose tolerance testing c. 2 hr. postprandial serum glucose d. hemoglobin A1c

d. hemoglobin A1c

294) the bicarbonate and carbonic acid ratio is calculated from an equiation by a. siggaard andersen b. gibbs donnan c. natelson d. henderson hasselbalch

d. henderson hasselbalch

38) A common cause of respiratory alkalosis is: a. vomiting b. starvation c. asthma d. hyperventilation

d. hyperventilation

247) in developing the reference for a new EIA and CEA, the range for the normal population was broader than that published by the vendor. controls are acceptable with a narrow coefficient of variation. this may be explained by a. positive interference by another tumor marker b. population skewed to a younger age c. improper temperature control during assay d. inclusion of nonsmokers and smokers in the study population

d. inclusion of nonsmokers and smokers in the study population

134) Urobilinogen is formed in the: a. kidney b. spleen c. liver d. intestine

d. intestine

(see pic) 104) a patient has the following results. the best conclusion is that this patient has a. normal iron status b. iron deficiency anemia c. chronic disease d. iron hemochromatosis

d. iron hemochromatosis

270) Lithium therapy is widely used in the treatment of: a. hypertension b. hyperactivity c. aggression d. manic-depressive disorder

d. manic-depressive disorder

262) The metabolite 11-nor-tetrahydrocannabinol-9-COOH can be detected by immunoassay 3 -5 days after a single use of: a. methamphetamine b. cocaine c. benzodiazepine d. marijuana

d. marijuana

74) The first step in analyzing a 24 hr. urine specimen for quantitative urine protein is: a. subculture the urine for bacteria b. add the appropriate preservative c. screen for albumin using a dipstick d. measure the total volume

d. measure the total volume

235) Pellagra is associated with deficiency of which of the following vitamins? a. A b. B1 c. thiamine d. niacin

d. niacin

120) oligoclonal bands are present on electrophoresis of concentrated CSF and also on concurrently tested serum of teh same patient. the proper interpretation is a. diagnostic for primary CNS tumor b. diagnostic for multiple sclerosis c. CNA involvement by acute leukemia d. nondiagnostic for multiple sclerosis

d. nondiagnostic for multiple sclerosis

164) malic dehydrogenase is added to the aspartate aminotransaminase (AST) reaction to catalyze the conversion of a. alpha ketoglutarate to aspartate b. alpha ketoglutarate to malate c. aspartate to oxaloacetate d. oxaloacetate to malate

d. oxaloacetate to malate

347) The International Federation for Clinical Chemistry (IFCC) recommends the use of methods such as the Bessey-Lowry-Brock method for determining alkaline phosphatase activity. The substrate used in this type of method is: a. monophosphate b. phenylphosphate c. disodium phenylphosphate d. para-nitrophenylphosphate

d. para-nitrophenylphosphate

166) Which of the following clinical disorders is associated with the greatest elevation of lactate dehydrogenase isoenzyme? a. pneumonia b. glomerulonephritis c. pancreatitis d. pernicious anemia

d. pernicious anemia

128) the prinicple of the occult blood test depends upon the a. coagulase ability of blood b. oxidative power of atmospheric oxygen c. hydrogen peroxide in hemoglobin d. peroxidase like activity of hemoglobin

d. peroxidase like activity of hemoglobin

266) An antiepileptic used to control seizure disorders is: a. digoxin b. acetaminophen c. lithium d. phenytoin

d. phenytoin

186) Regan isoenzyme has the same properties as alkaline phosphatase that originates in the: a. skeleton b. kidney c. intestine d. placenta

d. placenta

54) In respiratory acidosis, a compensatory mechanism is the increase in: a. respiration rate b. ammonia formation c. blood PCO2 d. plasma bicarbonate concentration

d. plasma bicarbonate concentration

213) A 45 yr. old woman complains of fatigue, heat intolerance, and hair loss. Total and free T4 are abnormally low. If the TSH showed marked elevation, this would be consistent with: a. Graves disease b. an adenoma of the thyroid c. thyrotoxicosis d. primary hypothyroidism

d. primary hypothyroidism

331) Magnesium carbonate is added in an iron binding capacity determinations in order to: a. allow color to develop b. precipitate protein c. bind with hemoglobin iron d. remove excess unbound iron

d. remove excess unbound iron

287) An arterial blood specimen submitted for blood gas analysis was obtained at 8:30 am but was not received in the lab until 11 am. The technologist should: a. perform test immediately upon receipt b. perform the test only if the specimen was submitted in ice water c. request a venous blood specimen d. request a new arterial specimen be obtained

d. request a new arterial specimen be obtained

100) The first step in the quantitation of serum iron is: a. direct reaction with appropriate chromogen b. iron saturation of transferrin c. free iron precipitation d. separation of iron from transferrin

d. separation of iron from transferrin

307) stray light can be detected in a spectrophotometer by utilizing a a. mercury vapor lamp b. holmium oxide glass c. potassium dichromate solution d. sharp cutoff filter

d. sharp cutoff filter

195) the most consistent analytical error involved in the routine determination of HDL cholesterol is caused by a. incomplete precipitation of LDL cholesterol b. coprecipitation of HDL and LDL cholesterol c. inaccurate protein estimation of HDL cholesterol d. small concentration of apoB containing lipoproteins after precipitation

d. small concentration of apoB containing lipoproteins after precipitation

55) which of the following electrolytes is the chief plasma cation whose main function is maintaining osmotic pressure? a. chloride b. calcium c. potassium d. sodium

d. sodium

189) Isoenzyme assays are performed to improve: a. precision b. accuracy c. sensitivity d. specificity

d. specificity

218) The screening test for congenital hypothyroidism is based upon: a. TSH level in the newborn b. thyroid-binding globulin level in the newborn c. iodine level in the newborn d. total thyroxine (T4) level in the newborn

d. total thyroxine (T4) level in the newborn

253) Which of the following statements most correctly describes the utility of clinical laboratory assays for tumor markers? a. tumor markers are useful to screen asymptomatic patients for tumors b. tumor markers are highly specific c. tumor markers indicate the likelihood of an individual developing a tumor d. tumor markers are useful in tracking the efficacy of treatment

d. tumor markers are useful in tracking the efficacy of treatment

116) which of the following represents the end product of purine metabolism in humans a. AMP and GMP b. DNA and RNA c. allantoin d. uric acid

d. uric acid

44) The degree to which the kidney concentrates the glomerular filtrate can be determined by: a. urine creatine b. serum creatinine c. creatinine clearance d. urine to serum osmolality ratio

d. urine to serum osmolality ratio

237) The urinary excretion product measured as an indicator of epinephrine production is: a. dopamine b. dihydroxyphenylalanine c. homovanillic acid d. vanillylmandelic acid

d. vanillylmandelic acid

(see pic) 249. A 68-year-old male in an unconscious state is transported to the emergency department after being involved in a one-car crash, where he drove off the road and hit a tree. Because he was alone at the time and there was no apparent cause for the accident, it is assumed that he blacked out, which caused him to lose control of the car. He was not wearing a seat belt and has a broken leg, multiple contusions, and cuts. Blood samples were drawn upon arrival to the ED and in 3-hour intervals for 12 hours; all control values were within acceptable range. Selected test results follow: What do these test results suggest? A. The man had a myocardial infarction, which caused the accident. B. The elevated results are from the skeletal muscle injuries sustained in the car crash. C. The elevated results are a combination of the car crash injuries and a myocardial infarction. D. The elevated total CK and CK-MB results indicate that the man had a stroke.

B. The elevated results are from the skeletal muscle injuries sustained in the car crash.

414. Free drug levels can generally be determined by analyzing what body fluid? A. Whole blood B. Ultrafiltrate of plasma C. Urine D. Protein-free filtrate of plasma

B. Ultrafiltrate of plasma

139. What is the end product of purine catabolism in humans? A. Urea B. Uric acid C. Allantoin D. Ammonia

B. Uric acid

263. What breakdown product of bilirubin metabolism is produced in the colon from the oxidation of urobilinogen by microorganisms? A. Porphobilinogen B. Urobilin C. Stercobilinogen D. Protoporphyrin

B. Urobilin

430. The term "lipid" encompasses a wide variety of compounds characterized as being insoluble in water but soluble in nonpolar solvents. Which of the following vitamins is not classified as fat soluble? A. Vitamin A B. Vitamin C C. Vitamin D D. Vitamin E

B. Vitamin C

110. In cases of hepatoma, which protein not normally found in adult serum is synthesized by liver cells? A. cq-Acid glycoprotein B. aj-Fetoprotein C. a2-Macroglobulin D. Carcinoembryonic antigen

B. aj-Fetoprotein

86. Which of the following is a low-weight protein that is found on the cell surfaces of nucleated cells? A. C-reactive protein B. p2-Microglobulin C. Ceruloplasmin D. G^-Macroglobulin

B. p2-Microglobulin

345. Given the following information, calculate the blood pH. PCO2 = 44 mm Hg Total CO2 = 29 mmol/L A. 6.28 B. 6.76 C. 7.42 D. 7.44

C. 7.42

1 1 . In spectrophotometry, which of the following is a mathematical expression of the relationship between absorbance and transmittance? A. A = abc B. Au/Cu = As/Cs C. A = 2 - log %T D. A = log %T

C. A = 2 - log %T

348. Blood gases are drawn on a 68-year-old asthmatic who was recently admitted for treatment of a kidney infection. Blood gas results are as follows: pH — 7.25, PCO2 — 56mmHg, HCOJ = 16 mmol/L .What condition is indicated by these results? A. Metabolic alkalosis, partially compensated B. Respiratory acidosis, uncompensated C. A dual problem of acidosis D. An error in one of the blood gas measurements

C. A dual problem of acidosis

193. The term "lipid storage diseases" is used to denote a group of lipid disorders, the majority of which are inherited as autosomal recessive mutations. What is the cause of these diseases? A. Excessive dietary fat ingestion B. Excessive synthesis of chylomicrons C. A specific enzyme deficiency or nonfunctional enzyme form D. An inability of adipose tissue to store lipid materials

C. A specific enzyme deficiency or nonfunctional enzyme form

246. Which of the following is not associated with assessment of an AMI? A. Elevated serum cTnl level B. Elevated serum CK-MB level C. Abnormal serum alkaline phosphatase isoenzyme pattern D. Blood collected upon presentation and serially in 3- to 6-hour intervals

C. Abnormal serum alkaline phosphatase isoenzyme pattern

22. What is the function of the flame in atomic absorption spectroscopy? A. Absorb the energy emitted from the metal analyte in returning to ground state B. Supply the thermal energy needed to excite the metal analyte C. Bring the metal analyte to its ground state D. Supply the light that is absorbed by the metal analyte

C. Bring the metal analyte to its ground state

156. Which of the following is not associated with insulin? A. Synthesized from proinsulin B. Synthesized by (3-cells in the pancreas C. C-peptide is active form D. Two-chain polypeptide

C. C-peptide is active form

240. Which of the following sets of tests would be the most useful in diagnosing an AMI? A. AST, LD, CK-MB B. LD, CK-MB, troponin C. CK-MB, troponin, myoglobin D. LD, troponin, myoglobin

C. CK-MB, troponin, myoglobin

274. Indirect-reacting bilirubin may be quantified by reacting it initially in which reagent? A. Dilute hydrochloric acid B. Dilute sulfuric acid C. Caffeine-sodium benzoate D. Sodium hydroxide

C. Caffeine-sodium benzoate

3. Which of the following is not descriptive of a photomultiplier tube? A. Emits electrons proportionally to initial light absorbed B. Must be shielded from stray light C. Cannot be used with a chopper D. Amplifies the initial signal received

C. Cannot be used with a chopper

327. The measurement of the pressure of dissolved CO2 (PCO2) in the blood is most closely associated with the concentration of what substance? A. pH B. Bicarbonate (HCOs) C. Carbonic acid (H2CO3) D. PO2

C. Carbonic acid (H2CO3)

431. Measuring which of the following compounds is useful in the diagnosis of steatorrhea? A. Vitamin B12 B. Vitamin C C. Carotenoids D. Folic acid

C. Carotenoids

391. Why are the total thyroxine (T4) levels increased in pregnant women and women who take oral contraceptives? A. Inappropriate iodine metabolism B. Changes in tissue use C. Changes in concentration of thyroxine-binding globulin (TBG) D. Changes in thyroglobulin synthesis

C. Changes in concentration of thyroxine-binding globulin (TBG)

20. Which of the following best describes chemiluminescence? A. Electron excitation caused by radiant energy B. Enzymatic oxidation of a substrate produces light emission C. Chemical energy excites electrons that emit light upon return to ground state D. Employs a fluorescent label that produces light

C. Chemical energy excites electrons that emit light upon return to ground state

69. Which of the following is not quantified using an immunoassay method? A. Vitamins B. Hormones C. Electrolytes D. Drugs

C. Electrolytes

50. Which of the following instruments has a sample-introduction system, solventdelivery system, column, and detector as components? A. Atomic absorption spectrometer B. Mass spectrometer C. High-performance liquid chromatograph D. Nephelometer

C. High-performance liquid chromatograph

242. A 53-year-old female presents with fatigue, pruritus, and an enlarged, nontender liver. The physician orders a series of blood tests. Based on the following serum test results, what is the most likely diagnosis? Alkaline phosphatase—markedly elevated Alanine aminotransferase—slightly elevated Lactate dehydrogenase—slightly elevated Gamma-glutamyltransferase— markedly elevated Total bilirubin—slightly elevated A. Alcoholic cirrhosis B. Infectious mononucleosis C. Intrahepatic cholestasis D. Viral hepatitis

C. Intrahepatic cholestasis

80. When quantifying serum total proteins, upon what is the intensity of the color produced in the biuret reaction dependent? A. Molecular weight of the protein B. Acidity of the medium C. Number of peptide bonds D. Nitrogen content of the protein

C. Number of peptide bonds

335. How would blood gas parameters change if a sealed specimen is left at room temperature for 2 or more hours? A. PO2 increases, PCO2 increases, pH increases B. PO2 decreases, PCO2 decreases, pH decreases C. PO2 decreases, PCO2 increases, pH decreases D. PO2 increases, PCO2 increases, pH decreases

C. PO2 decreases, PCO2 increases, pH decreases

168. An individual has a plasma glucose level of 110 mg/dL. What would be the approximate glucose concentration in this patient's cerebrospinal fluid? A. 33 mg/dL B. 55 mg/dL C. 66 mg/dL D. 110 mg/dL

C. The reference interval for glucose in CSF is 60% of the normal plasma value. For a plasma glucose of 110 mg/dL, the expected CSF glucose level would be 66 mg/dL.

134. The creatinine clearance test is routinely used to assess the glomerular filtration rate. Given the following information for an average-size adult, calculate a creatinine clearance. Urine creatinine—120 mg/dL Plasma creatinine—1.2 mg/dL Urine volume for 24 hours—1520 mL A. 11 mL/min B. 63 mL/min C. 95 mL/min D. 106 mL/min

D. 106 mL/min The following general mathematical formula is used to calculate creatinine clearance: U/P X V = Creatinine clearance (mL/min) where U = urine creatinine concentration in milligrams per deciliter, P = plasma creatinine concentration in milligrams per deciliter, and V = volume of urine per minute, with volume expressed in milliliters and 24 hours expressed as 1440 minutes. Applying this formula to the problem presented in the question: (120 / 1.2) x (1520/1440) = 106 ml/min

(see pic) 216. Given the following information for a rate reaction, calculate the activity of a serum specimen for alanine aminotransferase in international units per liter (IU/L). A. 186 B. 198 C. 1857 D. 1869

D. 1869 An international unit (IU) is defined as the enzyme activity that catalyzes the conversion of 1umol of substrate in 1 minute under standard used: (deltaAxvolumex10^6)/(Absorptivityxlight pathxspecimen) = IU/L (0.77x3.02x10^6) / (6.22x10^3x0.02) = 1869 IU/L

93. Of the five immunoglobulin classes, IgG is the most structurally simple, consisting of how many light chains/heavy chains, respectively? A. 5/2 B. 1/1 C. 2/5 D. 2/2

D. 2/2

324. Given the following information, calculate the plasma osmolality in milliosmoles per kilogram: sodium—142 mmol/L; glucose—130 mg/dL; urea nitrogen— 18mg/dL. A. 290 B. 291 C. 295 D. 298

D. 298 Use the following equation: Calculated osmolality (mOsm/kg) = 2.0 Na+(mmol/L) + Glucose (mmol/L) + Urea nitrogen (mmol/L) = 2.0 (142 mmol/L) + (0.056 X 130 mg/dL) + (0.36 X 18 mg/dL) =284 + 7.3 + 6.5 = 298 mOsm/kg

368. What percentage decrease in plasma or urinary estriol, in comparison with the previous day's level, is considered significant during pregnancy? A. 5 B. 10 C. 25 D. 40

D. 40

303. Which of the following does not have an effect on plasma calcium levels? A. Parathyroid hormone B. Vitamin D C. Calcitonin D. Aldosterone

D. Aldosterone

369. Which of the following compounds is not a precursor of the estrogens? A. Progesterone B. Testosterone C. Cholesterol D. Aldosterone

D. Aldosterone

159. Which of the following statements applies to the preferred use of plasma or serum, rather than whole blood, for glucose determination? A. Glucose is more stable in separated plasma or serum. B. Specificity for glucose is higher with most methods when plasma or serum is used. C. It is convenient to use serum or plasma with automated instruments because whole blood requires mixing immediately before sampling. D. All the above

D. All the above

61. For which of the following laboratory instalments should preventive maintenance procedures be performed and recorded? A. Analytical balance B. Centrifuge C. Chemistry analyzer D. All the above

D. All the above

30. What are the principles of operation for a chloride analyzer that generates silver ions as part of its reaction mechanism? A. Potentiometry and amperometry B. Amperometry and polarography C. Coulometry and potentiometry D. Amperometry and coulometry

D. Amperometry and coulometry

349. A mother brings her daughter, a 22-yearold medical technology student, to her physician. The patient is hyperventilating and has glossy eyes. The mother explains that her daughter is scheduled to take her final course exam the next morning. The patient had been running around frantically all day in a worried state and then started to breathe heavily. Blood gases are drawn in the office with the following results: pH = 7.58, PCO2 = 55 mm Hg, HCOj = 18 mmol/L .What do these data indicate? A. Metabolic alkalosis, partially compensated B. Respiratory acidosis, uncompensated C. A dual problem of acidosis D. An error in one of the blood gas measurements

D. An error in one of the blood gas measurements

425. Which of the following classes of compounds has a sedative effect and as such is used to treat anxiety? A. Amphetamines B. Opiates C. Cannabinoids D. Benzodiazepines

D. Benzodiazepines

260. What is the immediate precursor of bilirubin formation? A. Mesobilirubinogen B. Verdohemoglobin C. Urobilinogen D. Biliverdin

D. Biliverdin

83. Which term describes a congenital disorder that is characterized by a split in the albumin band when serum is subjected to electrophoresis? A. Analbuminemia B. Anodic albuminemia C. Prealbuminemia D. Bisalbuminemia

D. Bisalbuminemia

116. Although serum elevations are not generally seen in early stages, which of the following tumor markers are elevated in more advanced stages of breast cancer? A. CEA and AFP B. AFP and C A 125 C. PSA and CA 15-3 D. CA 15-3 and CA 549

D. CA 15-3 and CA 549

154. A sample of blood is collected for glucose in a sodium fluoride tube before the patient has had breakfast. The physician calls 2 hours later and requests that determination of blood urea nitrogen (BUN) be performed on the same sample rather than obtaining another specimen. The automated analyzer in your laboratory utilizes the urease method to quantify BUN. What should you tell the physician? A. Will gladly do the test if sufficient specimen remains B. Could do the test using a micromethod C. Can do the BUN determination on the automated analyzer D. Cannot perform the procedure

D. Cannot perform the procedure

405. Anticoagulated whole blood is the preferred specimen in determining exposure to what compound? A. Methanol B. Mercury C. Acetaminophen D. Carbon monoxide

D. Carbon monoxide

39. Which of the following is not a type of support media used for serum protein electrophoresis? A. Agarose gel B. Cellulose acetate C. Acrylamide D. Celite

D. Celite

104. The abnormal metabolism of several of the amino acids has been linked with disorders classified as inborn errors of metabolism. What technique is used to differentiate among several different amino acids? A. Electrophoresis B. Microbiological analysis C. Enzyme immunoassay D. Chromatography

D. Chromatography

7. In spectrophotometric analysis, what is the purpose of the reagent blank? A. Correct for interfering chromogens B. Correct for lipemia C. Correct for protein D. Correct for color contribution of the reagents

D. Correct for color contribution of the reagents

398. When screening urine for toxic concentrations of certain substances, which of the following will not be identified by the Reinsch test? A. Bismuth B. Arsenic C. Mercury D. Cyanide

D. Cyanide

376. A 4-year-old female presents with a palpable abdominal mass, pallor, and petechiae. Based on family history, clinical findings, and the patient's physical examination, neuroblastoma is suspected. Which of the following does not support such a diagnosis? A. Increased blood dopamine levels B. Increased blood epinephrine levels C. Increased urinary homovanillic acid D. Decreased urinary vanillylmandelic acid

D. Decreased urinary vanillylmandelic acid

279. What may be the cause of neonatal physiological jaundice of the hepatic type? A. Hemolytic episode caused by an ABO incompatibility B. Stricture of the common bile duct C. Hemolytic episode caused by an Rh incompatibility D. Deficiency in the bilirubin conjugation enzyme system

D. Deficiency in the bilirubin conjugation enzyme system

128. Which of the following disorders is not associated with an elevated blood ammonia level? A. Reye syndrome B. Renal failure C. Chronic liver failure D. Diabetes mellitus

D. Diabetes mellitus

100. Bence Jones proteinuria is a condition characterized by the urinary excretion of what type of light chain? A. Kappa light chains B. Lambda light chains C. Both kappa and lambda light chains D. Either kappa or lambda light chains

D. Either kappa or lambda light chains

232. Which statement concerning gammaglutamyltransferase is false? A. Present in almost all cells of the body B. Elevated in liver and some pancreatic diseases C. Elevated in chronic alcoholism D. Elevated in bone disease

D. Elevated in bone disease

252. A 10-year-old female presents with varicella. The child has been experiencing fever, nausea, vomiting, lethargy, and disorientation. A diagnosis of Reye syndrome is determined. Which of the following laboratory results is not consistent with the diagnosis? A. Elevated serum AST B. Elevated serum ALT C. Elevated plasma ammonia D. Elevated serum bilirubin

D. Elevated serum bilirubin

44. What is the purpose of using ampholytes in isoelectric focusing? A. Maintain the polyacrylamide gel in a solid state B. Maintain the protein sample in a charged state C. Maintain the pH of the buffer solution D. Establish a pH gradient in the gel

D. Establish a pH gradient in the gel

339. Which of the following is a cause of metabolic alkalosis? A. Late stage of salicylate poisoning B. Uncontrolled diabetes mellitus C. Renal failure D. Excessive vomiting

D. Excessive vomiting

239. Which of the following is false about cardiac troponin I (cTnl) as it relates to AMI? A. Increase above reference interval seen in 3 to 6 hours B. Measure initially and serially in 3- to 6-hour intervals C. Remains elevated 5 to 10 days D. Expressed in regenerating and diseased skeletal muscle and cardiac muscle disorders

D. Expressed in regenerating and diseased skeletal muscle and cardiac muscle disorders

145. What does hydrolysis of sucrose yield? A. Glucose only B. Galactose and glucose C. Maltose and glucose D. Fructose and glucose

D. Fructose and glucose

223. What abbreviation has been used in the past to designate alanine aminotransf erase? A. AST B. A AT C. GOT D. GPT

D. GPT

92. There are five immunoglobulin classes: IgG, IgA, IgM, IgD, and IgE. With which globulin fraction do these immunoglobulins migrate electrophoretically? A. Alphapglobulins B. Alpha2-globulins C. Betapglobulins D. Gamma-globulins

D. Gamma-globulins

58. Which chromatography system is commonly used in conjunction with mass spectrometry? A. High-performance liquid B. Ion-exchange C. Partition D. Gas-liquid

D. Gas-liquid

396. Which of the following methods would yield reliable quantification of ethanol in the presence of isopropanol? A. Reaction with permanganate and chromotropic acid B. Conway diffusion followed by dichromate reaction C. Alcohol dehydrogenase reaction D. Gas-liquid chromatography

D. Gas-liquid chromatography

151. Which test may be performed to assess the average plasma glucose level that an individual maintained during a previous 2- to 3-month period? A. Plasma glucose B. Two-hour postprandial glucose C. Oral glucose tolerance D. Glycated hemoglobin

D. Glycated hemoglobin

300. Which of the following conditions is not associated with hyponatremia? A. Addison disease B. Diarrhea C. Diuretic therapy D. Gushing syndrome

D. Gushing syndrome

238. Which of the following disorders is not characterized by an elevated serum myoglobin? A. Renal failure B. Vigorous exercise C. Acute myocardial infarction D. Hepatitis

D. Hepatitis

71. The substance to be measured reacts with a specific macromolecule of limited binding capacity. Which of the following assays does not employ this principle? A. Chemiluminescence immunoassay B. Enzyme-multiplied immunoassay technique C. Fluorescent polarization immunoassay D. High-performance liquid chromatography

D. High-performance liquid chromatography

82. Which disorder is not associated with an elevated protein level in cerebrospinal fluid? A. Bacterial meningitis B. Multiple sclerosis C. Cerebral infarction D. Hyperthyroidism

D. Hyperthyroidism

158. Which of the following is not characteristic of severe hyperglycemia? A. Polyuria B. Ketonuria C. Glycosuria D. Hypoglucagonemia

D. Hypoglucagonemia

95. Which immunoglobulin class is able to cross the placenta from the mother to the fetus? A. IgA B. IgD C. IgE D. IgG

D. IgG

183. To produce reliable results, when should blood specimens for lipid studies be drawn? A. Immediately after eating B. Anytime during the day C. In the fasting state, approximately 2 to 4 hours after eating D. In the fasting state, approximately 9 to 12 hours after eating

D. In the fasting state, approximately 9 to 12 hours after eating

304. Which of the following is an effect of increased parathyroid hormone secretion? A. Decreased blood calcium levels B. Increased renal reabsorption of phosphate C. Decreased bone resorption D. Increased intestinal absorption of calcium

D. Increased intestinal absorption of calcium

299. Primary aldosteronism results from a tumor of the adrenal cortex. How would the extracellular fluid be affected? A. Normal sodium, decreased potassium levels B. Decreased sodium, decreased potassium levels C. Decreased sodium, increased potassium levels D. Increased sodium, decreased potassium levels

D. Increased sodium, decreased potassium levels

254. Which of the following is not characteristic of cystic fibrosis? A. Decreased bicarbonate concentration in duodenal fluid B. Decreased lipase activity in duodenal fluid C. Decreased amylase activity in duodenal fluid D. Increased trypsin in feces

D. Increased trypsin in feces

21. In assaying an analyte with a single-beam atomic absoiption spectrophotometer, what is the instrument actually measuring? A. Intensity of light emitted by the analyte on its return to the ground state B. Intensity of light that the analyte absorbs from the hollow-cathode lamp C. Intensity of light that the analyte absorbs from the flame D. Intensity of the beam from the hollowcathode lamp after it has passed through the analyte-containing flame

D. Intensity of the beam from the hollowcathode lamp after it has passed through the analyte-containing flame

62. Which of the following is not the reason that preventive maintenance schedules are required? A. Keep instrument components clean B. Replace worn parts C. Extend the life of the equipment D. Keep personnel busy when the laboratory work is slow

D. Keep personnel busy when the laboratory work is slow

75. Singlet oxygen reacting with a precursor chemiluminescent compound to form a decay product whose light energizes a fluorophore best describes A. Fluorescent polarization immunoassay B. Enzyme-multiplied immunoassay technique C. Electrochemiluminescence immunoassay D. Luminescent oxygen channeling immunoassay

D. Luminescent oxygen channeling immunoassay

294. Which of the following is not associated with potassium? A. Has no renal threshold B. Increased serum level in acidosis C. Hemolysis causes false increase in serum levels D. Major anion of intracellular fluid

D. Major anion of intracellular fluid

435. A deficiency in which of the following leads to increased clotting time and may result in hemorrhagic disease in infancy? A. Riboflavin B. Pyridoxine C. Tocopherols D. Menaquinone

D. Menaquinone

55. Which of the following does not apply to gas-liquid chromatography? A. Separation depends on volatility of the sample. B. Separation depends on the sample's solubility in the liquid layer of the stationary phase. C. Stationary phase is a liquid layer adsorbed on the column packing. D. Mobile phase is a liquid pumped through the column.

D. Mobile phase is a liquid pumped through the column.

426. What is the active metabolite of the antiarrhythmic drug procainamide? A. Pronestyl B. Disopyramide C. PEMA D. NAPA

D. NAPA

77. Proteins, carbohydrates, and lipids are the three major biochemical compounds of human metabolism. What is the element that distinguishes proteins from carbohydrate and lipid compounds? A. Carbon B. Hydrogen C. Oxygen D. Nitrogen

D. Nitrogen

356. A patient is suspected of having Addison disease. His symptoms are weakness, fatigue, loss of weight, skin pigmentation, and hypoglycemia. His laboratory tests show low serum sodium and chloride, elevated serum potassium, and elevated urine sodium and chloride levels. The serum cortisol level is decreased and the plasma ACTH is increased. To make a definitive diagnosis, the physician orders an ACTH stimulation test, and serum cortisol levels are measured. If the patient has primary hypoadrenocortical function (Addison disease), what would be the expected level of serum cortisol following stimulation? If the patient has hypopituitarism and secondary hypoadrenocortical function, what would be the expected level of serum cortisol following stimulation? A. Increase from baseline; decrease from baseline B. Decrease from baseline; increase from baseline C. Slight increase from baseline; no change from baseline D. No change from baseline; slight increase from baseline

D. No change from baseline; slight increase from baseline

155. Which of the following does not properly describe type 1 diabetes mellitus? A. Insulin deficiency B. Associated with autoimmune destruction of pancreatic (3-cells C. Ketoacidosis prone D. Occurs more frequently in adults

D. Occurs more frequently in adults

302. Measuring the tubular reabsorption of phosphate is useful in diagnosing diseases that affect which of the following organs? A. Liver B. Adrenal gland C. Thyroid gland D. Parathyroid gland

D. Parathyroid gland

417. What is the major active metabolite of the anticonvulsant drug primidone? A. Phenytoin B. Acetazolamide C. NAPA D. Phenobarbital

D. Phenobarbital

5. Which type of photodetector employs a linear arrangement that allows it to respond to a specific wavelength resulting in complete UV/visible spectrum analysis? A. Photomultiplier tube B. Phototube C. Barrier layer cell D. Photodiode array

D. Photodiode array

43. When electrophoresis is performed, holes appear in the staining pattern, giving the stained protein band a doughnut-like appearance. What is the probable cause of this problem? A. Protein denatured and will not stain properly B. Ionic strength of the buffer was too high C. Protein reached its isoelectric point and precipitated out D. Protein concentration was too high

D. Protein concentration was too high

256. What compound chelates iron and is the immediate precursor of heme formation? A. Porphobilinogen B. Protopoiphyrinogen IX C. Uroporphyrinogen III D. Protoporphyrin IX

D. Protoporphyrin IX

79. What is the basis for the Kjeldahl technique for the determination of serum total protein? A. Quantification of peptide bonds B. Determination of the refractive index of proteins C. Ultraviolet light absorption by aromatic rings at 280 nm D. Quantification of the nitrogen content of protein

D. Quantification of the nitrogen content of protein

127. Which of the following statements can be associated with the enzymatic assay of ammonia? A. Increase in absorbance monitored at 340 nm B. Nicotinamide-adenine dinucleotide (NAD+) required as a cofactor C. Ammonium ion isolated from specimen before the enzymatic step D. Reaction catalyzed by glutamate dehydrogenase

D. Reaction catalyzed by glutamate dehydrogenase

340. Which of the following statements is true about partially compensated respiratory alkalosis? A. PCC>2 is higher than normal. B. HCO^ is higher than normal. C. More CC>2 is eliminated through the lungs by hyperventilation. D. Renal reabsorption of HCC>3 is decreased.

D. Renal reabsorption of HCC>3 is decreased.

157. Which of the following statements may be associated with the activity of insulin? A. Increases blood glucose levels B. Decreases glucose uptake by muscle and fat cells C. Stimulates release of hepatic glucose into the blood D. Stimulates glycogenesis in the liver

D. Stimulates glycogenesis in the liver

323. Which of the following describes the basis for the freezing point osmometer? A. The freezing point depression is directly proportional to the amount of solvent present. B. The freezing point depression varies as the logarithm of the concentration of solute. C. The freezing point is raised by an amount that is inversely proportional to the concentration of dissolved particles in the solution. D. The freezing point is lowered by an amount that is directly proportional to the concentration of dissolved particles in the solution.

D. The freezing point is lowered by an amount that is directly proportional to the concentration of dissolved particles in the solution.

177. Laboratory tests are performed on a postmenopausal, 57-year-old female as part of an annual physical examination. The patient's casual plasma glucose is 220 mg/dL, and the glycated hemoglobin (Hb AIC) is 11%. Based on this information, how should the patient be classified? A. Normal glucose tolerance B. Impaired glucose tolerance C. Gestational diabetes mellitus D. Type 2 diabetes mellitus

D. Type 2 diabetes mellitus

143. In gout, what analyte deposits in joints and other body tissues? A. Calcium B. Creatinine C. Urea D. Uric acid

D. Uric acid

210. A healthy, active 10-year-old boy with no prior history of illness comes to the lab after school for a routine chemistry screen in order to meet requirements for summer camp. After centrifugation, the serum looks cloudy. The specimen had the following results: blood glucose = 135 mg/dL, total cholesterol =195 mg/dL, triglyceride =185 mg/dL. What would be the most probable explanation for these findings? The boy A. Is at risk for coronary artery disease B. Has type 1 diabetes mellitus that is undiagnosed C. Has an inherited genetic disease causing a lipid imbalance D. Was most likely not fasting when the specimen was drawn

D. Was most likely not fasting when the specimen was drawn

257. Which of the following is a qualitative screening test for porphobilinogen that may be performed to aid in the diagnosis of the porphyrias? A. Caraway test B. Gutmantest C. Jendrassik-Grof test D. Watson-Schwartz test

D. Watson-Schwartz test

1. Which formula correctly describes the relationship between absorbance and %T ? A. A = 2 - log %T B. A = log 1/T C. A = -log T D. All of these options

1. D Absorbance is proportional to the inverse log of transmittance. A = -log T = log 1/T

10. Which of the following factors is most likely to cause a falsely low result when using the BCG dye-binding assay for albumin? A. The presence of penicillin B. An incubation time of 120 seconds C. The presence of bilirubin D. Lipemia

10. A BCG and BCP are not significantly affected by bilirubin or hemolysis, although negative interference caused by free Hgb has been reported with some BCG methods. Lipemic samples may cause positive interference, which can be eliminated by serum blanking. Incubation times as long as 2 minutes result in positive interference from globulins, which react with the dye. Penicillin and some other anionic drugs bind to albumin at the same site as the dye, causing falsely low results

11. What is the pH of a 0.05 M solution of acetic acid? Ka = 1.75 × 10-5, pKa = 4.76 A. 1.7 B. 3.0 C. 4.3 D. 4.6

11. B Weak acids are not completely ionized, and pH must be calculated from the dissociation constant of the acid (in this case 1.75 × 10-5).

16. Urea concentration is calculated from the BUN by multiplying by a factor of: A. 0.5 B. 2.14 C. 6.45 D. 14 Chemistry/Calculate

16. B BUN is multiplied by 2.14 to give the urea concentration in mg/dL. BUN (mg/dL) = urea × (% N in urea ÷ 100) Urea = BUN × 1/(% N in urea ÷100) Urea = BUN × (1/0.467) = 2.14

2. A solution that has a transmittance of 1.0 %T would have an absorbance of: A. 1.0 B. 2.0 C. 1% D. 99%

2. B A = 2.0 - log %T A = 2.0 - log 1.0 The log of 1.0 = 0 A = 2.0

22. Which instrument requires a primary and secondary monochromator? A. Spectrophotometer B. Atomic absorption spectrophotometer C. Fluorometer D. Nephelometer

22. C A fluorometer uses a primary monochromator to isolate the wavelength for excitation, and a secondary monochromator to isolate the wavelength emitted by the fluorochrome.

23. In addition to polarography, what other electrochemical method can be used to measure glucose in plasma? A. Conductivity B. Potentiometry C. Anodic stripping voltammetry D. Amperometry

23. D In some critical care analyzers, amperometric measurement of glucose is used. The glucose oxidase is impregnated into the membrane covering the electrode. It reacts with glucose in the sample, forming H2O2

25. Which of the following is the mechanism causing Cushing's disease? A. Excess secretion of pituitary ACTH B. Adrenal adenoma C. Treatment with corticosteroids D. Ectopic ACTH production by tumors

25. A Cushing's disease refers to adrenal hyperplasia resulting from misregulation of the hypothalamic-pituitary axis. It is usually caused by small pituitary adenomas. Cushing's syndrome may be caused by Cushing's disease, adrenal adenoma or carcinoma, ectopic ACTH-producing tumors, or excessive corticosteroid administration. The cause of Cushing's syndrome can be differentiated using the ACTH and dexamethasone suppression tests.

33. Identify the enzyme deficiency responsible for type 1 glycogen storage disease (von Gierke's disease). A. Glucose-6-phosphatase B. Glycogen phosphorylase C. Glycogen synthetase D. β-Glucosidase

33. A Type 1 glycogen storage disease (von Gierke's disease) is an autosomal recessive deficiency of glucose-6-phosphatase. Glycogen accumulates in tissues, causing hypoglycemia, ketosis, and fatty liver

34. Orders for uric acid are legitimate stat requests because: A. Levels above 10 mg/dL cause urinary tract calculi B. Uric acid is hepatotoxic C. High levels induce aplastic anemia D. High levels cause joint pain

34. A Uric acid calculi form quickly when the serum uric acid level reaches 10 mg/dL. They are translucent compact stones that often lodge in the ureters, causing postrenal failure.

39. Which whole-blood level is suggestive of excessive exposure to lead in children but not adults? A. 4 μg/dL B. 14 μg/dL C. 28 μg/dL D. 32 μg/dL Chemistry/Evaluate

39. B Because lead exposure in children leads to learning impairment, the cutoff for exposure recommended by the Centers for Disease Control is 5 μg/dL in venous whole blood. Values

43. Select the products formed from the forward reaction of ALT. A. Aspartate and alanine B. Alanine and α-ketoglutarate C. Pyruvate and glutamate D. Glutamine and NAD

43. C Because glutamate is a common product for transaminases, pyruvate (a three-carbon ketoacid) and glutamate would be generated from the transamination reaction between alanine and α-ketoglutarate

44. Which statement regarding the diagnosis of iron deficiency is correct? A. Serum iron levels are always higher at night than during the day B. Serum iron levels begin to fall before the body stores become depleted C. A normal level of serum ferritin rules out iron deficiency D. A low serum ferritin is diagnostic of iron deficiency

44. D Serum iron levels are falsely elevated by hemolysis and subject to diurnal variation. Levels are highest in the morning and lowest at night, but this pattern is reversed in persons who work at night. A low ferritin is specific for iron deficiency. However, only about 1% of ferritin is in the vascular system. Any disease that increases ferritin release may mask iron deficiency

45. An AFP measured on a 30-year-old pregnant woman at approximately 12 weeks gestation is 2.5 multiples of the median (MOM). What course of action is most appropriate? A. Repeat the serum AFP in 2 weeks B. Recommend AFP assay on amniotic fluid C. Repeat the AFP using the same sample by another method D. Repeat the AFP using the sample by the same method

45. A The analytical sensitivity of immunochemical AFP tests is approximately 5 ng/mL. The maternal serum AFP at 12 weeks' gestation is barely above the analytical detection limit. Therefore, to achieve the needed sensitivity, the test should be repeated at 14 weeks. If the result is still equal to or greater than 2.5 MOM, then ultrasound should be performed to verify last menstrual period dating

50. The serum level of which of the following laboratory tests is decreased in both VDDR and VDRR? A. Vitamin D B. Calcium C. Pi D. Parathyroid hormone

50. C Persons with VDDR and VDRR have a low Pi. However, persons with VDDR have decreased serum calcium, as well. Parathyroid hormone (PTH) is increased in persons with VDDR because calcium is the primary stimulus for PTH release, but not in persons with VDRR

52. Which measurement principle is employed in a vapor pressure osmometer? A. Seebeck B. Peltier C. Hayden D. Darlington

52. A The Seebeck effect refers to the increase in voltage across the two junctions of a thermocouple caused by a difference in the temperature at the junctions. Increasing osmolality lowers the dew point of a sample. When sample is cooled to its dew point, the voltage change across the thermocouple is directly proportional to osmolality

53. Which of the following total quality management tools can be used to calculate the analytical error rate for an analyte in the clinical laboratory? A. LEAN B. Six sigma C. ISO 9000 D. Laboratory information system

53. B All four of the answer choices are total quality management (TQM) tools used in the clinical laboratory to improve performance. Six sigma is a measurement of the frequency of product defects

53. Which of the following conditions will cause erroneous Cai results? Assume that the samples are collected and stored anaerobically, kept at 4°C until measurement, and stored for no longer than 1 hour. A. Slight hemolysis during venipuncture B. Assay of whole blood collected in sodium oxalate C. Analysis of serum in a barrier gel tube stored at 4°C until the clot has formed D. Analysis of whole blood collected in sodium heparin, 20 U/mL (low-heparin tube) Chemistry/Apply knowledge to recognize sources

53. B Unlike Pi, the intracellular calcium level is not significantly different from plasma calcium, and calcium is not greatly affected by diet. Whole blood collected with 5-20 U/mL heparin and stored on ice no longer than 2 hours is the sample of choice for Cai. Blood gas syringes prefilled with 100 U/mL heparin should not be used because the high heparin concentration will cause low results. Citrate, oxalate, and ethylenediaminetetraacetic acid (EDTA) must not be used because they chelate calcium.

53. What is the lipid testing protocol for adults recommended by the National Cholesterol Education Program (NCEP) to evaluate risk for atherosclerosis beginning at age 20? A. Total cholesterol, fasting or nonfasting every year B. Total cholesterol, fasting, every 2 years C. Lipid profile, fasting, every 5 years D. LDL cholesterol, fasting, every 2 years

53. C Because LDL cholesterol, HDL cholesterol, VLDL cholesterol, and triglycerides are all risk factors for coronary artery disease, NCEP recommends a fasting lipid profile to include triglycerides, total cholesterol, HDL cholesterol, and LDL cholesterol be performed every 5 years beginning at age 20. However, because LDL cholesterol is the target of treatment, therapeutic goals are based on the LDL cholesterol. New guidelines recommend an LDL cholesterol goal below 70 mg/dL for the highest-risk persons

(see pic) 55. The following plot represents a study of a screening test for malignant prostate cancer using plasma PSA (ng/mL). The outcome measured was positive cytology results obtained by biopsy. What concentration gives the highest sensitivity with the least number of unnecessary biopsies? A. 2.6 B. 3.6 C. 3.8 D. 5.2

55. B A receiver operating characteristic (ROC) curve is used to identify the test result, giving the highest sensitivity with the least number of false-positive results.

56. Which electrolyte measurement is least affected by hemolysis? A. Potassium B. Calcium C. Pi D. Magnesium

56. B Potassium, phosphorus, and magnesium are the major intracellular ions, and even slight hemolysis will cause falsely elevated results. Serum samples with visible hemolysis (20 mg/dL free Hgb) should be redrawn.

57. An EDTA blood sample is collected from a nonfasting person for a CBC. The physician collected the sample from the femoral vein because venipuncture from the arm was unsuccessful. He called the lab 15 minutes after the sample arrived and requested a lipid study including triglyceride, total cholesterol, HDL cholesterol, and LDL cholesterol. Which test results should be used to evaluate the patient's risk for coronary artery disease? A. Total cholesterol and LDL cholesterol B. LDL cholesterol and triglyceride C. Total cholesterol and HDL cholesterol D. Total cholesterol and triglyceride

57. C NCEP recommends a 12-hour fasting sample when screening persons for risk of coronary artery disease. However, if a fasting sample is unavailable, NCEP recommends performing the total cholesterol and HDL cholesterol because these tests are least affected by recent ingestion of food. If

59. Which statement regarding bone-specific ALP is true? A. The bone isoenzyme can be measured immunochemically B. Bone ALP is increased in bone resorption C. Bone ALP is used for the diagnosis of osteoporosis D. There are two distinct bone isoenzymes

59. A Bone ALP assays (Ostase and Alkphase-B) use monoclonal antibodies to measure the bone isoenzyme in mass units. The assays may be used to monitor bone remodeling by osteoblasts in osteoporosis, and thus, are useful for following treatment. Bone specific ALP is not sufficiently sensitive to diagnose osteoporosis, and antibodies may cross-react with other ALP isoenzymes, depending on their source

59. Which of the following values is the threshold critical value (alert or action level) for low plasma potassium? A. 1.5 mmol/L B. 2.0 mmol/L C. 2.5 mmol/L D. 3.5 mmol/L

59. C The reference range for potassium is 3.6-5.4 mmol/L. However, values below 2.5 mmol/L require immediate intervention because below that level there is a grave risk of cardiac arrhythmia, which can lead to cardiac arrest. The upper alert level for potassium is usually 6.5 mmol/L, except for neonatal and hemolyzed samples. Above this level, there is danger of cardiac failure

6. Which route of administration is associated with 100% bioavailability? A. Sublingual B. Intramuscular C. Oral D. Intravenous

6. D When a drug is administered intravenously, all the drug enters the bloodstream, and therefore, the bioavailable fraction is 1.0. All other routes of administration require absorption through cells, and this process reduces the bioavailable fraction

60. In gas chromatography, the elution order of volatiles is usually based upon the: A. Boiling point B. Molecular size C. Carbon content D. Polarity

60. A The order of elution is dependent upon the velocity of the analyte. Usually, the lower the boiling point of the compound, the greater its velocity or solubility in carrier gas

63. Which substrate is used in the Bowers-McComb method for ALP? A. p-Nitrophenyl phosphate B. β-Glycerophosphate C. Phenylphosphate D. α-Naphthylphosphate

63. A The method of Bowers-McComb (Szasz modification) is the IFCC-recommended method for ALP. This method uses 2-amino-2-methyl-1-propanol, pH 10.15, and measures the increase in absorbance at 405 nm as p-nitrophenyl phosphate is hydrolyzed to p-nitrophenol

64. Which of the following conditions is associated with total body sodium excess? A. Renal failure B. Hyperthyroidism C. Hypoparathyroidism D. Diabetic ketoacidosis

64. A Total body sodium excess often occurs in persons with renal failure, congestive heart failure, and cirrhosis of the liver. When water is retained along with sodium, total body sodium excess results rather than hypernatremia. Heart failure causes sodium and water retention by reducing blood flow to the kidneys

65. A serum ALP level greater than twice the elevation of GGT suggests: A. Misidentification of the specimen B. Focal intrahepatic obstruction C. Acute alcoholic hepatitis D. Bone disease or malignancy

65. D In obstructive jaundice, GGT is elevated more than ALP. A disproportionate increase in ALP points to a nonhepatic source of ALP, often bone disease. GGT is the most sensitive marker of acute alcoholic hepatitis, rising about fivefold higher than ALP or transaminases

68. Which of the following amylase substrates is recommended by the IFCC? A. Starch B. Maltodextrose C. Maltotetrose D. Blocked maltohepatoside Chemistry/Apply knowledge

68. D Amylase is commonly measured using synthetic substrates. In the IFCC-recommended method, p-nitrophenyl maltohepatiside is used. One end of the polymer is covalently linked to p-nitrophenol and the other is linked to 4,6 ethylidine to prevent its hydrolysis by α-glucosidase

69. In the enzymatic assay of bilirubin, how is measurement of both total and direct bilirubin accomplished? A. Using different pH for total and direct assays B. Using UDP glucuronyl transferase and bilirubin reductase C. Using different polarity modifiers D. Measuring the rate of absorbance decrease at different time intervals

69. A Enzymatic methods use bilirubin oxidase to convert bilirubin back to biliverdin, and measure the decrease in absorbance that results. At pH 8, both conjugated, unconjugated, and delta bilirubin react with the enzyme, but at pH 4 only the conjugated form reacts

70. What is the principle of the transcutaneous bilirubin assay? A. Conductivity B. Amperometric inhibition C. Multiwavelength reflectance photometry D. Infrared spectroscopy

70. C Measurement of bilirubin concentration through the skin requires the use of multiple wavelengths to correct for absorbance by melanin and other light-absorbing constituents of skin and blood

78. Which of the following enzymes is usually depressed in liver disease? A. Elastase-1 B. GLD C. Pseudocholinesterase D. Aldolase Chemistry/Correlate

78. C Pseudocholinesterase is found mainly in the liver and functions to hydrolyze acetylcholine. It is depressed by organophosphate insecticides and drugs that function as cholinesterase inhibitors and the serum assay is used to presumptively identify cases of insecticide poisoning. Levels of pseudocholinesterase are decreased in patients with liver disease as a result of depressed synthesis

8. When pituitary adenoma is the cause of decreased estrogen production, an increase of which hormone is most frequently responsible? A. Prolactin B. FSH C. LH D. Thyroid-stimulating hormone (TSH) Chemistry/Correlate clinical and laboratory data

8. A Prolactinoma can result in anovulation because high levels of prolactin suppress release of LHRH (gonadotropin-releasing hormone), causing suppression of growth hormone (GH), FSH, and estrogen. Prolactinoma is the most commonly occurring pituitary tumor accounting for 40%-60%. Adenomas producing FSH have a frequency of about 20%, while those pituitary tumors secreting LH and TSH are rare.

8. Which of the following enzymes allows creatinine to be measured by coupling the creatinine amidohydrolase (creatininase) reaction to the peroxidase reaction? A. Glucose-6-phosphate dehydrogenase B. Creatinine iminohydrolase C. Sarcosine oxidase D. Creatine kinase

8. C The peroxidase-coupled enzymatic assay of creatinine is based upon the conversion of creatinine to creatine by creatinine amidohydrolase (creatininase). The enzyme creatinine amidinohydrolase (creatinase) then hydrolyzes creatine to produce sarcosine and urea.

8. Which of the following 2-hour glucose challenge results would be classified as impaired glucose tolerance (IGT)? Two-hour serum glucose: A. 130 mg/dL B. 135 mg/dL C. 150 mg/dL D. 204 mg/dL

8. C With the exception of pregnant females, impaired glucose tolerance is defined by the ADA as a serum or plasma glucose at 2 hours following a 75-g oral glucose load of ≥140 mg/dL and < 200 mg/dL. Persons who have a fasting plasma glucose of ≥100 but < 126 mg/dL are classified as having impaired fasting glucose (IFG). Both IGT and IFG are risk factors for developing diabetes later in life. Such persons are classified as having prediabetes and should be tested annually.

8. Which monochromator specification is required in order to measure the true absorbance of a compound having a natural absorption bandwidth of 30 nm? A. 50-nm bandpass B. 25-nm bandpass C. 15-nm bandpass D. 5-nm bandpass

8. D Bandpass refers to the range of wavelengths passing through the sample. The narrower the bandpass, the greater the photometric resolution. Bandpass can be made smaller by reducing the width of the exit slit. Accurate absorbance measurements require a bandpass less than one-fifth the natural bandpass of the chromophore.

8. Which of the following conditions is most commonly associated with an elevated level of total protein? A. Glomerular disease B. Starvation C. Liver failure D. Malignancy

8. D Malignant disease is usually associated with increased immunoglobulin and acute-phase protein production. However, nutrients required for protein synthesis are consumed, causing reduced hepatic albumin production. Glomerular damage causes albumin and other low molecular weight proteins to be lost through the kidneys. Liver failure and starvation result in decreased protein synthesis

8. The increase in the level of serum enzymes used to detect cholestatic liver disease is caused mainly by: A. Enzyme release from dead cells B. Leakage from cells with altered membrane permeability C. Decreased perfusion of the tissue D. Increased production and secretion by cells

8. D The amount of enzyme in the serum can be increased by necrosis, altered permeability, secretion, or synthesis. It is also dependent upon tissue perfusion, enzyme half-life, molecular size, and location of the enzyme within the cell. Most enzymes are liberated by necrosis, but a few, such as ALP and γ-glutamyltransferase, are produced and secreted at a greater rate in obstructive liver disease.

9. Select the primary reagent used in the Jaffe method for creatinine. A. Alkaline copper II sulfate B. Saturated picric acid and NaOH C. Sodium nitroprusside and phenol D. Phosphotungstic acid

9. B The Jaffe method uses saturated picric acid, which oxidizes creatinine in alkali, forming creatinine picrate. The reaction is nonspecific; ketones, ascorbate, proteins, and other reducing agents contribute to the final color. Alkaline CuSO4 is used in the biuret method for protein

9. Which of the following enzymes is considered most tissue specific? A. Creatine kinase (CK) B. Amylase C. Alkaline phosphatase (ALP) D. Alcohol dehydrogenase (ADH)

9. D No enzyme is truly tissue specific and diagnostic accuracy depends upon recognizing changes in plasma levels that characterize different diseases. This includes the mass or activity of enzyme released, its rise, peak, and return to normal, the isoenzyme(s) released, and the concomitant changes of other enzymes. Alanine aminotransferase and alcohol dehydrogenase are primarily increased in necrotic liver disease.

9. Which photodetector is most sensitive to low levels of light? A. Barrier layer cell B. Photodiode C. Diode array D. Photomultiplier tube

9. D The photomultiplier tube uses dynodes of increasing voltage to amplify the current produced by the photosensitive cathode. It is 10,000 times as sensitive as a barrier layer cell, which has no amplification. A photomultiplier tube requires a DC-regulated lamp because it responds to light fluctuations caused by the AC cycle.

121. What endogenous substance may cause a positive interference in the urease/glutamate dehydrogenase assay? A. Ammonia B. Creatinine C. Glucose D. Cholesterol

A. Ammonia

10. Which of the following formulas is an expression of the Beer-Lambert law that is routinely applied to spectrophotometric analysis? Cv A. Au X Cs/As = Cu B. Cu X Cs/As = Au C. As X Cs/Cu = Au D. A = 2 - log %T

A. Au X Cs/As = Cu

36. Which of the following statements is false about anodic stripping voltammetry (ASV)? A. Based on potentiometry B. Occurs in an electrochemical cell C. Involves preconcentration of the analyte by electroplating D. Used to measure lead

A. Based on potentiometry

437. Which disorder is associated with thiamin deficiency? A. Beriberi B. Pellagra C. Rickets D. Dermatitis

A. Beriberi

326. Which is the most predominant buffer system in the body? A. Bicarbonate/carbonic acid B. Acetate/acetic acid C. Phosphate/phosphorous acid D. Hemoglobin

A. Bicarbonate/carbonic acid

266. In the condition kernicterus, the abnormal accumulation of bilirubin occurs in what tissue? A. Brain B. Liver C. Kidney D. Blood

A. Brain

113. Which of the following is a sialylated Lewis blood group antigen associated with colorectal carcinoma? A. CA 19-9 B. CA 15-3 C. CA549 D. CEA

A. CA 19-9

381. Which of the following would be elevated in the blood in medullary carcinoma of the thyroid? A. Calcitonin B. Thyroxine C. Catecholamines D. Secretin

A. Calcitonin

319. Which of the following constituents normally present in serum must be chemically eliminated so that it will not interfere with the measurement of serum magnesium? A. Calcium B. Chloride C. Iron D. Potassium

A. Calcium

97. Which of the following is a copper transport protein that migrates as an alpha2-globulin? A. Ceruloplasmin B. Haptoglobin C. Transferrin D. Fibrinogen

A. Ceruloplasmin

184. Which of the following lipid tests is least affected by the fasting status of the patient? A. Cholesterol B. Triglyceride C. Fatty acid D. Lipoprotein

A. Cholesterol

142. Which of the following disorders is best characterized by laboratory findings that include increased serum levels of inorganic phosphorus, magnesium, potassium, uric acid, urea, and creatinine and decreased serum calcium and erythropoietin levels? A. Chronic renal failure B. Renal tubular disease C. Nephrotic syndrome D. Acute glomerulonephritis

A. Chronic renal failure

295. Which of the following is a spectrophotometric method for quantifying serum chloride? A. Ferric perchlorate B. Ammonium molybdate C. Bathophenanthroline D. Cresolphthalein complexone

A. Ferric perchlorate

146. In what form is glucose stored in muscle and liver? A. Glycogen B. Maltose C. Lactose D. Starch

A. Glycogen

250. If elevated, which of the following is associated with increased risk for coronary heart disease? A. Homocysteine B. Vitamin B6 C. Myoglobin D. pro-BNP

A. Homocysteine

24. In potentiometry, which of the following is considered the standard electrode? A. Hydrogen electrode B. Calcium electrode C. Potassium electrode D. Copper electrode

A. Hydrogen electrode

173. What would an individual with Gushing syndrome tend to exhibit? A. Hyperglycemia B. Hypoglycemia C. Normal blood glucose level D. Decreased 2-hour postprandial glucose

A. Hyperglycemia

94. Which immunoglobulin class, characterized by its possession of a secretory component, is found in saliva, tears, and body secretions? A. IgA B. IgD C. IgG D. IgM

A. IgA

57. Which parameter is used in mass spectrometry to identify a compound? A. Ion mass-to-charge ratio B. Molecular size C. Absorption spectrum D. Retention time

A. Ion mass-to-charge ratio

265. What term is used to describe the accumulation of bilirubin in the skin? A. Jaundice B. Hemolysis C. Cholestasis D. Kernicterus

A. Jaundice

149. What is the glucose concentration in fasting whole blood? A. Less than the concentration in plasma or serum B. Greater than the concentration in plasma or serum C. Equal to the concentration in plasma or serum D. Meaningless because it is not stable

A. Less than the concentration in plasma or serum

342. Which of the following will cause a shift of the oxygen dissociation curve to the right, resulting in a decreased affinity of hemoglobin for C"2? A. Low plasma pH level B. Low PCO2 level C. Low concentration of 2,3-bisphosphoglycerate D. Low temperature

A. Low plasma pH level

215. When measuring enzyme activity, if the instrument is operating 5°C lower than the temperature prescribed for the method, how will the results be affected? A. Lower than expected B. Higher than expected C. Varied, showing no particular pattern D. All will be clinically abnormal

A. Lower than expected

346. A 75-year-old woman comes to her physician complaining of abdominal pain. She says she has had a sore stomach for the last 3 weeks and has been taking increasing doses of antacid pills to control it. Now she is taking a box of pills a day. Blood gases are drawn with the following results: pH = 7.49, PCO2 = 59 mm Hg, HCOJ = 25 mmol/L. What do these data indicate? A. Metabolic alkalosis, partially compensated B. Respiratory acidosis, uncompensated C. A dual problem of acidosis D. An error in one of the blood gas measurements

A. Metabolic alkalosis, partially compensated

374. In a patient who is suspected of having pheochromocytoma, measurement of which of the following would be most useful? A. Metanephrine B. Homovanillic acid C. 5-Hydroxyindoleacetic acid D. Homogentisic acid

A. Metanephrine

85. Identification of which of the following is useful in early stages of glomerular dysfunction? A. Microalbuminuria B. Ketonuria C. Hematuria D. Urinary light chains

A. Microalbuminuria

165. Which of the following is not a reagent required in an enzymatic serum glucose method? A. NAD+ B. Glucose oxidase C. Peroxidase D. Reduced chromogen

A. NAD+

225. Which of the following does not accurately describe properties associated with lactate dehydrogenase? A. Optimum pH for the catalysis of lactate to pyruvate is 7.4-7.8. B. LD is increased in a hemolyzed serum specimen. C. LD catalyzes the oxidation of lactate to pyruvate with mediation of nicotinamide-adenine dinucleotide. D. LD-4 and LD-5 are labile in the cold

A. Optimum pH for the catalysis of lactate to pyruvate is 7.4-7.8.

334. If a blood gas specimen is left exposed to air, which of the following changes will occur? A. PO2 and pH increase; PCO2 decreases B. PO2 and pH decrease; PCO2 increases C. PO2 increases; pH and PCO2 decrease D. PO2 decreases; pH and PCO2 increase

A. PO2 and pH increase; PCO2 decreases

297. Using the following data: Na+ = 143 mmol/L; K+ = 4.9 mmol/L; Cl" = 105 mmol/L; and HCO^ = 25 mmol/L, which of the following statements is false? A. Patient results are not acceptable. B. Anion gap is useful in detecting some disease states. C. Anion gap equals 18 mmol/L. D. Anion gap is useful for checking analytical error

A. Patient results are not acceptable.

384. How is the majority of reverse T3 (rT3) made? A. Peripheral deiodination of T4 B. Peripheral deiodination of T3 C. From T3 in the thyroid gland D. From thyroglobulin in the thyroid gland

A. Peripheral deiodination of T4

409. Which of the following is an example of a long-acting barbiturate? A. Phenobarbital B. Amobarbital C. Secobarbital D. Pentobarbital

A. Phenobarbital

424. What is the recommended name for diphenylhydantoin? A. Phenytoin B. Nalorphine C. Primidone D. Carbamazepine

A. Phenytoin

137. Which of the following reagents is not utilized in a coupled enzymatic reaction method to quantify serum creatinine? A. Picric acid B. Chromogenic dye C. Creatinine amidohydrolase D. Sarcosine oxidase

A. Picric acid

16. Which of the following may be associated with fluorescence polarization? A. Plane-polarized light is used for sample excitation. B. Small molecular complexes show a greater amount of polarization. C. It is a heterogeneous technique employed in fluorophore-ligand immunoassays. D. Polarized light detected is directly proportional to concentration of ligand in sample.

A. Plane-polarized light is used for sample excitation.

305. The following laboratory results are obtained on a 60-year-old woman who is complaining of anorexia, constipation, abdominal pain, nausea, and vomiting: Ionized serum calcium—elevated Serum inorganic phosphate— decreased Urine calcium—elevated Urine phosphate—elevated What do these results suggest? A. Primary hyperparathyroidism B. Vitamin D deficiency C. Hypoparathyroidism D. Paget disease

A. Primary hyperparathyroidism

76. Which of the following stimulates the production of singlet oxygen at the surface of the sensitizer particle in a luminescent oxygen channeling immunoassay? A. Radiant energy B. Heat energy C. Enzymatic reaction D. Fluorescent irradiation

A. Radiant energy

354. What effect would a low-salt diet, upright position, and diuretics have on the following test results? A. Renin inc, aldosterone inc, hypernatremia, hypokalemia B. Renin inc aldosterone dec, hypernatremia, hypokalemia C. Renin dec, aldosterone dec, hyponatremia, hyperkalemia D. Renin dec, aldosterone inc, hyponatremia, hyperkalemia

A. Renin inc, aldosterone inc, hypernatremia, hypokalemia

289. Which of the following results is least consistent with a diagnosis of viral hepatitis? A. Seaim total bilirubin 7.5 mg/dL, direct bilirubin 5.5 mg/dL, indirect bilirubin 2.0 mg/dL B. Urine urobilinogen increased C. AST increased 10 times the upper limit of the reference range D. ALT increased 13 times the upper limit of the reference range

A. Seaim total bilirubin 7.5 mg/dL, direct bilirubin 5.5 mg/dL, indirect bilirubin 2.0 mg/dL

397. Which of the following tests would be particularly useful in determining isopropanolexposure? A. Serum osmolality and urine acetone B. Urine osmolality and serum osmolality C. Urine acetone and urine osmolality D. Serum sodium and serum acetone

A. Serum osmolality and urine acetone

133. When the Jaffe reaction is employed as a kinetic assay to quantify serum creatinine, which of the following is used in the analysis? A. Serum sample used directly B. Folin-Wu filtrate C. Somogyi-Nelson filtrate D. Trichloroacetic acid filtrate

A. Serum sample used directly

124. To maintain acid-base balance, it is necessary that the blood ammonia level be kept within narrow limits. This is accomplished primarily by which of the following? A. Synthesis of urea from ammonia B. Synthesis of glutamine from ammonia C. Excretion of ammonia in the bile D. Excretion of ammonia in the stools

A. Synthesis of urea from ammonia

427. Which of the following drugs is used as a bronchodilator? A. Theophylline B. Phenytoin C. Amikacin D. Clozapine

A. Theophylline

54. Which of the following chromatography systems is characterized by a stationary phase of silica gel on a piece of glass and a moving phase of liquid? A. Thin-layer B. Ion-exchange C. Gas-liquid D. Partition

A. Thin-layer

382. What is the predominant form of thyroid hormone in the circulation? A. Thyroxine B. Triiodothyronine C. Diiodotyrosine D. Monoiodotyrosine

A. Thyroxine

328. What is the term that describes the sum of carbonic acid and bicarbonate in plasma? A. Total CO2 B. Standard bicarbonate C. Buffer base D. Base excess

A. Total CO2

63. Which globin chains compose hemoglobin Aj? A. Two alpha chains and two beta chains B. Two alpha chains and two delta chains C. Two alpha chains and two gamma chains D. Two beta chains and two delta chains

A. Two alpha chains and two beta chains

278. Which of the following is characteristic of hemolytic jaundice? A. Unconjugated serum bilirubin level increased B. Urinary bilirubin level increased C. Urinary urobilinogen level decreased D. Fecal urobilin level decreased

A. Unconjugated serum bilirubin level increased

144. During chemotherapy for leukemia, which of the following analytes would most likely be elevated in the blood? A. Uric acid B. Urea C. Creatinine D. Ammonia

A. Uric acid

90. Which total protein method requires copper sulfate, potassium iodide in sodium hydroxide, and potassium sodium tartrate in its reagent system? A. Kjeldahl B. Biuret C. Folin-Ciocalteu D. Ultraviolet absorption

B. Biuret

383. Once synthesized, the thyroid hormones are stored as a component of thyroglobulin in what area of the thyroid gland? A. Epithelial cell wall of the follicle B. Colloid in the follicle C. Isthmus of the thyroid gland D. Extracellular space of the thyroid gland

B. Colloid in the follicle

281. Which of the following characterizes hepatic dysfunction in the early stage of viral hepatitis? A. Elevation in urobilinogen and urobilin excretion in the feces B. Elevation in the serum unconjugated bilirubin fraction C. Depression in the serum conjugated bilirubin fraction D. Depression in urinary urobilinogen excretion

B. Elevation in the serum unconjugated bilirubin fraction

394. When performing parathyroid surgery for adenoma resection, parathyroid hormone is quantified at three points relative to the surgical procedure: baseline prior to incision, second baseline with gland exposure, and third sample at postexcision. Which of the following is not correct in assessing the PTH values? A. The second baseline value should be higher than the first baseline. B. The first baseline value should be the highest value of the three samples. C. The post-excision value should be at least 50% of or lower than the second baseline. D. The lack of decrease in the PTH value post-excision indicates possible multigland disease

B. The first baseline value should be the highest value of the three samples.

141. In the ultraviolet procedure for quantifying uric acid, what does the reaction between uric acid and uricase cause? A. Production of reduced nicotinamideadenine dinucleotide (NADH) B. The formation of allantoin C. An increase in absorbance D. A reduction of phosphotungstic acid

B. The formation of allantoin

390. What is the major carrier protein of the thyroid hormones in the blood? A. Albumin B. Thyroxine-binding globulin C. Thyroxine-binding prealbumin D. Thyroglobulin

B. Thyroxine-binding globulin

120. In the diacetyl method, what does diacetyl react with to form a yellow product? A. Ammonia B. Urea C. Uric acid D. Nitrogen

B. Urea

406. What is the approximate number of half-life periods required for a serum drug concentration to reach 97-99% of the steady state? A. 1-3 B. 2-4 C. 5-7 D. 7-9

C. 5-7

337. In order to maintain electrical neutrality in the red blood cell, bicarbonate leaves the red blood cell and enters the plasma through an exchange mechanism with what electrolyte? A. Sodium B. Potassium C. Chloride D. Phosphate

C. Chloride

179. Bile acids that are synthesized in the liver are derived from what substance? A. Bilirubin B. Fatty acid C. Cholesterol D. Triglyceride

C. Cholesterol

180. The turbid, or milky, appearance of serum after fat ingestion is termed postprandial lipemia, which is caused by the presence of what substance? A. Bilirubin B. Cholesterol C. Chylomicron D. Phospholipid

C. Chylomicron

188. Exogenous triglycerides are transported in the plasma in what form? A. Phospholipids B. Cholesteryl esters C. Chylomicrons D. Free fatty acids

C. Chylomicrons

196. The quantification of the high-density lipoprotein cholesterol level is thought to be significant in the risk assessment of what disease? A. Pancreatitis B. Cirrhosis C. Coronary artery disease D. Hyperlipidemia

C. Coronary artery disease

138. An endogenous substance assayed to assess the glomerular filtration rate may be described as being filtered by the glomeruli, not reabsorbed by the tubules, and only secreted by the tubules when plasma levels become elevated. What is this frequently assayed substance? A. Inulin B. Uric acid C. Creatinine D. Urea

C. Creatinine

432. Which of the following is another name for vitamin Bj2? A. Retinol B. Pyridoxine C. Cyanocobalamin D. Riboflavin

C. Cyanocobalamin

321. Which of the following disorders is characterized by increased production of chloride in sweat? A. Multiple myeloma B. Hypoparathyroidism C. Cystic fibrosis D. Wilson disease

C. Cystic fibrosis

363. During pregnancy in the second trimester, human chorionic gonadotropin (hCG) levels ______ and progesterone and estriol levels _____ A. Increase,increase B. Increase, decrease C. Decrease, increase D. Decrease, decrease

C. Decrease, increase

84. In what condition would an increased level of serum albumin be expected? A. Malnutrition B. Acute inflammation C. Dehydration D. Renal disease

C. Dehydration

273. Which of the following reagent systems contains the components sulfanilic acid, hydrochloric acid, and sodium nitrite? A. Jaffe B. Zimmerman C. Diazo D. Lowry

C. Diazo

373. The adrenal medulla secretes which of the following in the greatest quantity? A. Metanephrine B. Norepinephrine C. Epinephrine D. Dopamine

C. Epinephrine

331. Which of the following characterizes respiratory acidosis? A. Excess of bicarbonate B. Deficit of bicarbonate C. Excess of dissolved carbon dioxide (PCO2) D. Deficit of dissolved carbon dioxide (PCO2)

C. Excess of dissolved carbon dioxide (PCO2)

181. Cholesterol ester is formed through the esterification of the alcohol cholesterol with what substance? A. Protein B. Triglyceride C. Fatty acid D. Digitonin

C. Fatty acid

313. What is the primary storage form of iron? A. Apotransferrin B. Myoglobin C. Ferritin D. Hemosiderin

C. Ferritin

187. Enzymatic methods for the determination of total cholesterol in serum utilize a cholesterol oxidase-peroxidase method. In this method, cholesterol oxidase reacts specifically with what? A. Free cholesterol and cholesteryl ester B. Free cholesterol and fatty acid C. Free cholesterol only D. Cholesteryl ester only

C. Free cholesterol only

74. The enzyme activity measured in the EMIT is the result of the reaction between the substrate and coenzyme with A. Free antibody B. Free unlabeled antigen C. Free labeled antigen D. Labeled antigen-antibody complexes

C. Free labeled antigen

60. Checking instrument calibration, temperature accuracy, and electronic parameters are part of A. Preventive maintenance B. Quality control C. Function verification D. Precision verification

C. Function verification

371. What would be an example of ectopic hormone production? A. Prolactin production by pituitary tumors B. Calcitonin production by thyroid tumors C. Growth hormone production by lung tumors D. Cortisol production by adrenal tumors

C. Growth hormone production by lung tumors

211. A mother brings her obese, 4-year-old child who is a known type 1 diabetic to the laboratory for a blood workup. She states that the boy has been fasting for the past 12 hours. After centrifugation the tech notes that the serum looks turbid. The specimen had the following results: blood glucose = 150 mg/dL, total cholesterol = 250 mg/dL, HDL cholesterol = 32 mg/dL, triglyceride = 395 mg/dL. What best explains these findings? The boy A. Is a low risk for coronary artery disease B. Is a good candidate for a 3-hour oral glucose tolerance test C. Has secondary hyperlipidemia due to the diabetes D. Was not fasting when the specimen was drawn

C. Has secondary hyperlipidemia due to the diabetes

167. Which glucose method is considered to be the reference method? A. Glucose oxidase B. o-Toluidine C. Hexokinase D. Glucose dehydrogenase

C. Hexokinase

31. When quantifying glucose using an amperometric glucose electrode system, which of the following is not a component of the system? A. Product oxidation produces a current B. Hydrogen peroxide formed C. Hexokinase reacts with glucose D. Platinum electrode

C. Hexokinase reacts with glucose

108. A male patient, 48 years old, mentions during his annual physical that he has been having difficulty urinating. The physician performs a rectal examination, and he orders a total prostate-specific antigen (PSA) and free PSA. The patient has the tests done the following week, and the total PSA result is 3.1 ng/mL and the free PSA is 0.3 ng/mL. What do these results suggest? A. Both are normal, no disease present B. Benign prostatic hypertrophy C. Increased risk of prostate cancer D. Free PSA is low and does not correlate with total PSA

C. Increased risk of prostate cancer

218. Which of the following is a true statement concerning serum enzymes? A. The presence of hemolyzed red cells is of no significance for an accurate assay of most serum enzymes. B. Serum asparate transaminase (AST), but not serum lactate dehydrogenase (LD), is usually elevated in acute myocardial infarction. C. Increased serum alkaline phosphatase may be found in bone disease. D. Aspartate transaminase was formerly known as glutamate pyruvate transaminase

C. Increased serum alkaline phosphatase may be found in bone disease.

386. In a patient with suspected primary hyperthyroidism associated with Graves disease, one would expect the following laboratory serum results: free thyroxine (FT4) _____ thyroid hormone binding ratio (THBR) ______ and thyroid-stimulating hormone (TSH) ______ A. Increased, decreased, increased B. Increased, decreased, decreased C. Increased, increased, decreased D. Decreased, decreased, increased

C. Increased, increased, decreased

317. Which of the following disorders is best characterized by these laboratory results? Serum iron—decreased Total iron-binding capacity— increased Transferrin saturation—decreased Serum ferritin—decreased Free erythrocyte protoporphyrin— increased A. Anemia of chronic disease B. Thalassemia C. Iron-deficiency anemia D. Hemochromatosis

C. Iron-deficiency anemia

298. A patient presents with Addison disease. Serum sodium and potassium analyses are performed. What would the results reveal? A. Normal sodium, low potassium levels B. Low sodium, low potassium levels C. Low sodium, high potassium levels D. High sodium, low potassium levels

C. Low sodium, high potassium levels

49. To achieve the best levels of sensitivity and specificity, to what type of detector system could a gas chromatograph be coupled? A. UV spectrophotometer B. Bichromatic spectrophotometer C. Mass spectrometer D. Fluorescence detector

C. Mass spectrometer

227. Lactate dehydrogenase (LD) catalyzes the following reaction: LD Lactate+ NADH pyruvate + NADH As the reaction is written, which of the following techniques can be used to assess LD activity? A. Measure the colorimetric product pyruvate. B. Measure the colorimetric product NADH. C. Measure the increase in absorbance at 340 nm as NADH is produced. D. Measure the decrease in absorbance at 340 nm as NADH is produced

C. Measure the increase in absorbance at 340 nm as NADH is produced.

41. Which of the following characteristics will a protein have at its isoelectric point? A. Net negative charge B. Net positive charge C. Net zero charge D. Mobility

C. Net zero charge

45. Which of the following is not associated with silver stains? A. Reactive to nanogram concentrations of proteins B. Polypeptides stain a variety of colors C. Not as sensitive as Coomassie brilliant blue D. Preconcentration of CSF not necessary

C. Not as sensitive as Coomassie brilliant blue

126. Which of the following does not need to be done when collecting, handling, and using a specimen for ammonia analysis? A. Avoid using a hemolyzed specimen. B. Collect blood in EDTA or heparin evacuated tubes. C. Place specimen in a 37°C water bath immediately. D. Advise patient not to smoke for 8 hours before blood collection

C. Place specimen in a 37°C water bath immediately.

411. Which of the following is an example of an antiarrhythmic drug that has a metabolite with the same action? A. Quinidine B. Digoxin C. Procainamide D. Nortriptyline

C. Procainamide

364. Which of the following is not quantified in the triple test for Down syndrome? A. cq-Fetoprotein B. Unconjugated estriol C. Progesterone D. Human chorionic gonadotropin

C. Progesterone

380. The secretion of which of the following is controlled by growth hormone? A. Growth hormone-releasing hormone B. Corticotropin-releasing hormone C. Somatomedin D. Somatostatin

C. Somatomedin

153. A 30-year-old pregnant woman has a gestational diabetes mellitus screening test performed at 26 weeks of gestation. Her physician chooses to order a 50-g oral glucose load. Her serum glucose level is 150 mg/dL at 1 hour. What should occur next? A. This confirms diabetes mellitus; give insulin. B. This confirms diabetes mellitus; dietary intake of carbohydrates should be lessened. C. This is suspicious of diabetes mellitus; an oral glucose tolerance test should be performed. D. This is an expected glucose level in a pregnant woman

C. This is suspicious of diabetes mellitus; an oral glucose tolerance test should be performed.

385. Which of the following is an autoantibody that binds to TSH receptor sites on thyroid cell membranes, preventing thyroidstimulating hormone from binding? A. Antithyroglobulin antibodies B. Thyroid antimicrosomal antibodies C. Thyrotropin-receptor antibodies D. Antithyroid peroxidase antibodies

C. Thyrotropin-receptor antibodies

212. What does an increase in the serum enzyme levels indicate? A. Decreased enzyme catabolism B. Accelerated enzyme production C. Tissue damage and necrosis D. Increased glomerular filtration rate

C. Tissue damage and necrosis

198. The VLDL fraction primarily transports what substance? A. Cholesterol B. Chylomicron C. Triglyceride D. Phospholipid

C. Triglyceride

13. Which of the following may be associated with reflectance spectrophotometry as it relates to the dry reagent slide technique? A. Light projected to the slide at 180-degree angle B. Dye concentration directly proportional to reflectance C. Unabsorbed, reflected light detected by photodetector D. Reflectance values are linearly proportional to transmission values

C. Unabsorbed, reflected light detected by photodetector

271. Which of the following methods is not used for the quantification of serum bilirubin? A. Bilirubinometer B. Jendrassik and Grof C. Zimmerman D. Bilirubin oxidase

C. Zimmerman

98. Which of the following proteins is normally produced by the fetus but is found in increased amounts in the amniotic fluid in cases of spina bifida? A. «|-Antitrypsin B. ai-Acid glycoprotein C. aj-Fetoprotein D. a2-Macroglobulin

C. aj-Fetoprotein

367. During pregnancy, estriol is synthesized in the placenta from ______ formed in the _________ A. Estradiol, mother B. Estradiol, fetus C. 16a-Hydroxy-DHEA-S,mother D. 16a-Hydroxy-DHEA-S, fetus

D. 16a-Hydroxy-DHEA-S, fetus

150. Of the following blood glucose levels, which would you expect to result in glucose in the urine? A. 60mg/dL B. 120mg/dL C. 150mg/dL D. 225mg/dL

D. 225mg/dL

118. Express 30 mg/dL of urea nitrogen as urea. A. 14 mg/dL B. 20 mg/dL C. 50 mg/dL D. 64 mg/dL

D. 64 mg/dL When it is necessary to convert urea nitrogen values to urea, the concentration may be calculated easily by multiplying the urea nitrogen value by 2.14.

130. A serum creatinine was found to be 6.0 mg/dL. Which of the following urea nitrogen serum results would support the same pathological condition? A. 6 mg/dL B. 20 mg/dL C. 35 mg/dL D. 70 mg/dL

D. 70 mg/dL

291. Of the total serum osmolality, sodium, chloride, and bicarbonate ions normally contribute approximately what percent? A. 8 B. 45 C. 75 D. 92

D. 92

88. In a healthy individual, which protein fraction has the greatest concentration in serum? A. Alphai-globulin B. Beta-globulin C. Gamma-globulin D. Albumin

D. Albumin

161. Ingestion of which of the following drugs may cause hypoglycemia? A. Ethanol B. Propranolol C. Salicylate D. All the above

D. All the above

314. A serum ferritin level may not be a useful indicator of iron-deficiency anemia in patients with what type of disorder? A. Chronic infection B. Malignancy C. Viral hepatitis D. All the above

D. All the above

59. Which of the following may be a sampling source of error for an automated instrument? A. Short sample B. Air bubble in bottom of sample cup C. Fibrin clot in sample probe D. All the above

D. All the above

125. When a blood ammonia determination is performed, the blood specimen must be treated in a manner that will ensure that A. The deamination process continues in vitro B. Glutamine formation in vitro is avoided C. The transamination process continues in vitro D. Ammonia formation in vitro is avoided

D. Ammonia formation in vitro is avoided

67. Because of similar electrophoretic mobilities, several hemoglobins cannot be differentiated on cellulose acetate medium. Electrophoresis of hemoglobins at pH 6.2 on agar gel may be useful in differentiating which hemoglobins? A. AI from A2 B. A! from D C. A, from E D. CfromA2

D. CfromA2

423. Which of the following is an example of a phenothiazine drug? A. Cyclosporine B. Theophylline C. Phenytoin D. Chlorpromazine

D. Chlorpromazine

395. Levels of 8-9% carboxyhemoglobin saturation of whole blood are commonly found in which of the following situations? A. Fatal carbon monoxide poisoning B. Acute carbon monoxide poisoning C. Nonsmoking residents of rural areas D. Cigarette smokers

D. Cigarette smokers

258. What compound may be detected by observing its orange-red fluorescence in acid solution? A. Porphobilinogen B. Uroporphyrinogen C. Aminolevulinic acid D. Coproporphyrin

D. Coproporphyrin

132. What analyte is measured using the Jaffe reaction? A. Urea B. Uric acid C. Ammonia D. Creatinine

D. Creatinine

136. What compound normally found in urine may be used to assess the completeness of a 24-hour urine collection? A. Urea B. Uric acid C. Creatine D. Creatinine

D. Creatinine

72. Which of the following is not associated with the enzyme-multiplied immunoassay technique (EMIT)? A. Is a homogeneous enzyme immunoassay B. Determines antigen concentration C. Employs a labeled reactant D. Enzyme reacts with drug in serum sample

D. Enzyme reacts with drug in serum sample

70. Which of the following is a homogeneous immunoassay where separation of the bound from the free labeled species is not required? A. Radioimmunoassay B. Enzyme-linked immunosorbent assay C. Immunoradiometric assay D. Enzyme-multiplied immunoassay technique

D. Enzyme-multiplied immunoassay technique

64. Which hemoglobin may be differentiated from other hemoglobins on the basis of its resistance to denature in alkaline solution? A. A, B. A2 C. C D. F

D. F

283. Which of the following disorders is characterized by an inability to transport bilirubin from the sinusoidal membrane into the hepatocyte? A. Carcinoma of the common bile duct B. Crigler-Najjar syndrome C. Dubin-Johnson syndrome D. Gilbert syndrome

D. Gilbert syndrome

91. Which of the following plasma proteins is not manufactured by the liver? A. Albumin B. Haptoglobin C. Fibrinogen D. IgG

D. IgG

171. Which of the following hormones does not promote an increase in blood glucose levels? A. Growth hormone B. Cortisol C. Glucagon D. Insulin

D. Insulin

293. Which of the following is not a component of the total anion content of serum? A. Acetoacetate B. Protein C. Lactate D. Iron

D. Iron

4. Which of the following is false about a photomultiplier tube? A. Converts radiant energy (light) to electrical energy (current) B. Amplifies the current significantly C. Has a very rapid response time D. Is composed of an iron plate and a layer of selenium

D. Is composed of an iron plate and a layer of selenium

408. Acetaminophen is particularly toxic to what organ? A. Heart B. Kidney C. Spleen D. Liver

D. Liver

366. The placenta secretes numerous hormones both protein and steroid. Which of the following hormones is not secreted by the placenta? A. Human chorionic gonadotropin (hCG) B. Estrogen C. Human placental lactogen (HPL) D. Luteinizing hormone (LH)

D. Luteinizing hormone (LH)

224. When measuring CK-MB, which of the following would provide the most sensitive method? A. Electrophoretic B. Colorimetric C. Kinetic D. Mass immunoassay

D. Mass immunoassay

23. Most atomic absorption spectrophotometers incorporate a beam chopper and a tuned amplifier. The purpose of these components is to avoid errors that would be caused by A. Variations in flame temperature B. Deterioration of the hollow-cathode lamp C. Stray light from the hollow-cathode lamp D. Measurement of light emitted by the analyte

D. Measurement of light emitted by the analyte

330. In the plasma, an excess in the concentration of bicarbonate without a change in PCO2 from normal will result in what physiological state? A. Respiratory acidosis B. Respiratory alkalosis C. Metabolic acidosis D. Metabolic alkalosis

D. Metabolic alkalosis

241. A physician orders several laboratory tests on a 55-year-old male patient who is complaining of pain, stiffness, fatigue, and headaches. Based on the following serum test results, what is the most likely diagnosis? Alkaline phosphatase—significantly increased Gamma-glutamyltransferase—normal A. Biliary obstruction B. Cirrhosis C. Hepatitis D. Osteitis deformans

D. Osteitis deformans

320. In the collection of plasma specimens for lactate determinations, which of the following anticoagulants would be more appropriate? A. Sodium heparin B. Sodium citrate C. EDTA D. Oxalate plus fluoride

D. Oxalate plus fluoride

14. Fluorometers are designed so that the path of the exciting light is at a right angle to the path of the emitted light. What is the purpose of this design? A. Prevent loss of emitted light B. Prevent loss of the excitation light C. Focus emitted and excitation light upon the detector D. Prevent excitation light from reaching the detector

D. Prevent excitation light from reaching the detector

268. Which of the following factors will not adversely affect the accurate quantification of bilirubin in serum? A. Lipemia B. Hemolysis C. Exposure to light D. Specimen refrigeration

D. Specimen refrigeration

190. Which of the following is most associated with the membrane structure of nerve tissue? A. Cholesterol B. Triglyceride C. Phospholipids D. Sphingolipids

D. Sphingolipids

255. Which compounds originally condense to form aminolevulinic acid? A. Oxoglutarate and aspartate B. Isocitrate and coenzyme II C. Oxalacetate and malate D. Succinyl coenzyme A and glycine

D. Succinyl coenzyme A and glycine

360. Which of the following is the most potent androgen? A. Androstenedione B. Dehydroepiandrosterone C. Androsterone D. Testosterone

D. Testosterone

182. Which of the following tests would most likely be included in a routine lipid profile? A. Total cholesterol, triglyceride, fatty acid, chylomicron B. Total cholesterol, triglyceride, HDL cholesterol, phospholipid C. Triglyceride, HDL cholesterol, LDL cholesterol, chylomicron D. Total cholesterol, triglyceride, HDL cholesterol, LDL cholesterol

D. Total cholesterol, triglyceride, HDL cholesterol, LDL cholesterol

219. Enzymes that catalyze the transfer of groups between compounds are classified as belonging to which enzyme class? A. Hydrolases B. Eyases C. Oxidoreductases D. Transferases

D. Transferases

204. A patient's total cholesterol/HDL cholesterol ratio is 10.0. What level of risk for coronary heart disease does this result indicate? A. No risk B. Half average risk C. Average risk D. Twice average risk

D. Twice average risk

32. To calibrate the pH electrode in a pH/ blood gas analyzer, it is necessary that A. The barometric pressure be known and used for adjustments B. Calibrating gases of known high and low concentrations be used C. The calibration be performed at room temperature D. Two buffer solutions of known pH be used

D. Two buffer solutions of known pH be used

117. What is the compound that comprises the majority of the nonprotein-nitrogen fractions in serum? A. Uric acid B. Creatinine C. Ammonia D. Urea

D. Urea

140. When mixed with phosphotungstic acid, what compound causes the reduction of the former to a tungsten blue complex? A. Urea B. Ammonia C. Creatinine D. Uric acid

D. Uric acid

28. When measuring K+ with an ion-selective electrode by means of a liquid ionexchange membrane, what antibiotic will be incorporated into the membrane? A. Monactin B. Nonactin C. Streptomycin D. Valinomycin

D. Valinomycin

6. When performing spectrophotometer quality assurance checks, what is the holmium oxide glass filter used to assess? A. Linearity B. Stray light C. Absorbance accuracy D. Wavelength accuracy

D. Wavelength accuracy

336. The bicarbonate ion concentration may be calculated from the total CO2 and PCO2 blood levels by using which of the following formulas? A. 0.03 x (PCO2 - total CO2) B. (total CO2 + 0.03) x PCO2 C. 0.03 x (total CO2 - PO2) D. total CO2 - (0.03 x PCO2)

D. total CO2 - (0.03 x PCO2)

292) A 45-year-old male of average height and weight was admitted to the hospital for renal function studies. He had the following lab results: urine creatinine: 120 mg/dL serum creatinine 1.5 mg/dL totol urine volume/24 hours: 1800 mL Calculate the creatinine clearance for this patient in mL/mn. a) 100 b) 144 c) 156 d) 225

a) 100

290) A 24-hour urine specimen (total volume = 1,136 mL) is submitted to the lab for quantitative urine protein. Calculate the amount of protein excreted per day, if the total protein is 52 mg/dL. a) 591 mg b) 487 mg c) 220 mg d) 282 mg

a) 591 mg 0.52 mg/ml / 1136 = 591mg

342) In the assay of lactate dehydrogenase, which of the following products is actually measured? a. NADH b. ATP c. lactic acid d. pyruvic acid

a. NADH

349) Which of the following is a glycolytic enzyme that catalyzes the cleavage of fructose-1, 6-diphosphate to glyceraldehyde-3-phosphate and dihydroxyacetone phosphate? a. aldolase b. phosphofructokinase c. pyruvate kinase d. glucose-6-phosphate dehydrogenase

a. aldolase

332) The most specific method for the assay of glucose utilizes: a. hexokinase b. glucose oxidase c. glucose-6-phosphatase d. glucose dehydrogenase

a. hexokinase

273) phenobarbital is a metabolite of a. primidone b. phenytoin c. amobarbital d. secobarbital

a. primidone

320) Hydrogen ion concentration (pH) in blood is usually determined by means of which of the following electrodes? a) silver b) glass c) platinum d) platinum-lactate

b) glass

129) a breakdown product of hemoglobin in a. lipoprotein b. bilirubin c. hematoxylin d. bence jones protein

b. bilirubin

68) A reciprocal relationship exists between: a. sodium and potassium b. calcium and phosphate c. chloride and CO2 d. calcium and magnesium

b. calcium and phosphate

181) Increased serum lactic dehydrogenase activity due to elevation of fast fraction (1 and 2) on electrophoretic separation is caused by: a. nephrotic syndrome b. hemolytic anemia c. pancreatitis d. hepatic damage

b. hemolytic anemia

284) specimens for blood gas determination should be drawn into syringe containing a. no preservative b. heparin c. EDTA d. oxalate

b. heparin

171) In which of the following conditions would a normal level of creatine kinase be found? a. acute myocardial infarct b. hepatitis c. progressive muscular dystrophy d. intramuscular injection

b. hepatitis

148) a condition in which erythrocyte protoporphyrin is increased is a. acute intermittent porphyria b. iron deficiency anemia c. porphyria cutanea tarda d. acute porphyric attack

b. iron deficiency anemia

(see pic) 139) Serial bilirubin determinations are charted below. The best explanation for the results is: a. sample hemolysis and hemoglobin deterioration b. sample exposure to light c. sample left in warm location d. reagent deterioration

b. sample exposure to light

185) What specimen preparation is commonly used to perform the alkaline phosphatase isoenzyme determinations? a. serum is divided into 2 aliquots, one is frozen and the other is refrigerated b. serum is divided into 2 aliquots, one is heated at 56 C and the other is unheated c. no preparation is necessary since the assay used EDTA plasma d. protein-free filtrate is prepared first

b. serum is divided into 2 aliquots, one is heated at 56 C and the other is unheated

93) the electrophoretic pattern of plasma sample as compared to a serum sample shows a a. broad prealbumin peak b. sharp fibrinogen peak c. diffuse pattern becasue of the presence of anticoagulants d. decreased globulin fraction

b. sharp fibrinogen peak

217) The recommended initial thyroid function test for either a healthy, asymptomatic patient, or a patient with symptoms which may be related to a thyroid disorder is: a. free thyroxine (free T4) b. thyroid stimulating hormone (TSH) c. total thyroxine (T4) d. triiodothyronine (T3)

b. thyroid stimulating hormone (TSH)

203) which of the following lipid results would be expected to be falsely elevated on a serum specimen from a nonfasting patient? a. cholesterol b. triglyceride c. HDL d. LDL

b. triglyceride

6) A 25-year-old man became nauseated and vomited 90 minutes after receiving a standard 75g carbohydrate dose for an oral glucose tolerance test. The best course of action is to: a) give the patient a glass of orange juice and continue the test b) start the test over immediately with a 50g carbohydrate dose c) draw blood for glucose and discontinue test d) place the patient in a recumbent position, reassure him and continue the test

c) draw blood for glucose and discontinue test

(see pic) 43) A patient had the following serum results: Which osmolality is consistent with these results? a. 188 b. 204 c. 270 d. 390

c. 270

341) In the Jendrassik-Grof reaction for total bilirubin, bilirubin reacts with diazotized sulfanilic acid to form: a. diazo bilirubin b. biliverdin c. azobilirubin d. bilirubin glucuronide

c. azobilirubin

42) Select the test which evaluates renal tubular function: a. IVP b. creatinine clearance c. osmolarity d. microscopic urinalysis

c. osmolarity

(see pic) 49) A patient with a myeloproliferative disorder has the following values: The serum ICE should be confirmed by: a. repeat testing of the original serum b. testing freshly drawn serum c. testing heparinized plasma d. atomic absorption spectrometry

c. testing heparinized plasma

(see pic) 301) refer to the following illustration. the above figure shows the reciprocal of the measured velocity of an enzyme reaction plotted against the reciprocal of the substrate concentration. true statements about the figure include. a. the intercept of the line on the abscissa (x axis) can be used to calculate the V max b. the straight line indicates that the enzyme reaction proceeds according to zero order kinetics c. the intercept on the abscissa (x axis) can be used to calculate the Michaelis Metnen constant d. the fact that the substrate concentration is plotted on both sides of the zero point indicates that the reaction is reversible

c. the intercept on the abscissa (x axis) can be used to calculate the Michaelis Metnen constant

251) Carcinoembryonic antigen (CEA) is most likely to be produced in a malignancy involving the: a) brain b) testes c) bone d) colon

d) colon

132) the hemoglobin that is resistant to alkali (KOH) denaturation is a. A b. A2 c. C d. F

d. F

155) The most specific enzyme test for acute pancreatitis is: a. acid phosphatase b. trypsin c. amylase d. lipase

d. lipase

267) A drug that relaxed the smooth muscles of the bronchial passages is: a. acetaminophen b. lithium c. phenytoin d. theophylline

d. theophylline

15. Which of the following assays is recommended as a screening test for colorectal cancer in persons over 50 years old? A. CEA B. AFP C. Occult blood D. Fecal trypsin Chemistry/Correlate

15. C Bleeding in the gastrointestinal tract occurs during the early stages of colorectal cancer when treatment can be most effective. Although occult blood can be caused by many other GI problems, it is not associated with benign polyps and has a sensitivity of over 80% for detection of colorectal cancer.

1. Creatinine is formed from the: A. Oxidation of creatine B. Oxidation of protein C. Deamination of dibasic amino acids D. Metabolism of purines

1. A Creatinine is produced at a rate of approximately 2% daily from the oxidation of creatine mainly in skeletal muscle. Creatine can be converted to creatinine by addition of strong acid or alkali or by the enzyme creatine hydroxylase

1. How many grams of sodium hydroxide (NaOH) are required to prepare 150.0 mL of a 5.0% w/v solution? A. 1.5 g B. 4.0 g C. 7.5 g D. 15.0 g

1. C A percent solution expressed in w/v (weight/volume) refers to grams of solute per 100.0 mL of solution. To calculate, multiply the percentage (as grams) by the volume needed (mL), then divide by 100.0 (mL). (5.0 g × 150.0 mL) ÷ 100.0 mL = 7.5 g To prepare the solution, weigh 7.5 g of NaOH pellets and add to a 150.0-mL volumetric flask. Add sufficient deionized H2O to dissolve the NaOH. After the solution cools, add deionized H2O to the 150.0-mL line on the flask and mix again

1. Which of the following biochemical processes is promoted by insulin? A. Glycogenolysis B. Gluconeogenesis C. Lipolysis D. Uptake of glucose by cells

1. D Insulin reduces blood glucose levels by increasing glucose uptake by cells. It promotes lipid and glycogen production, induces synthesis of glycolytic enzymes, and inhibits formation of glucose from pyruvate and Krebs cycle intermediates

10. Which tumor marker is used to determine the usefulness of trastuzumab (Herceptin) therapy for breast cancer? A. PR B. CEA C. HER-2/neu D. Myc

10. C Trastuzumab is an antibody to the HER-2/neu gene product, a tyrosine kinase receptor protein. HER- 2/neu is an oncogene that is overexpressed in some breast cancers. Overexpression is associated with a more aggressive clinical course but responds to treatment with trastuzumab, which blocks the attachment of growth factor to the receptor

11. Which of the following enzymes is a transferase? A. ALP B. CK C. Amylase D. LD

11. B Enzymes are identified by a numeric system called the EC (Enzyme Commission) number. The first number refers to the class of the enzyme. There are six classes; in order, these are oxidoreductases, transferases, hydrolases, lyases, isomerases, and ligases. Dehydrogenases are oxidoreductases, whereas kinases and transaminases are transferases. CK is EC number 2.7.3.2, which distinguishes it from other kinases.

12. If too small a peak-trough difference is seen for a drug given orally, then: A. The dose should be decreased B. Time between doses should be decreased C. Dose interval should be increased D. Dose per day and time between doses should be decreased

12. C Increasing the dosing interval will reduce the trough concentration of the drug, and increasing the dose will increase the peak concentration of the drug, resulting in a greater peak-trough difference. The peak-trough ratio is usually adjusted to 2 with the dose interval set to equal the drug half-life. Under these conditions, both peak and trough levels often fall within the therapeutic range

12. Which type of filter is best for measuring stray light? A. Wratten B. Didymium C. Sharp cutoff D. Neutral density

12. C Sharp cutoff filters transmit almost all incident light until the cutoff wavelength is reached. At that point, they cease to transmit light. Because they give an "all or none effect," only stray light reaches the detector when the selected wavelength is beyond the cutoff.

12. SITUATION: The following lab results are reported. Which result is most likely to be erroneous? Arterial blood gases: pH = 7.42 pO2 = 90 mm Hg pCO2 = 38.0 mm Hg bicarbonate = 24 mmol/L. Plasma electrolytes: Na = 135 mmol/L Cl = 98 mmol/L K = 4.6 mmol/L TCO2 = 33 mmol/L A. pH B. Na C. K D. TCO2

12. D The pH, pCO2, and bicarbonate are normal, and therefore, agree. The electrolytes are normal also, but the TCO2 is increased significantly. The reference range for venous TCO2 is 22-28 mmol/L.

15. Which of the following proteins has the highest pI? A. Albumin B. Transferrin C. Ceruloplasmin D. IgG

15. D Albumin is the fastest migrating protein toward the anode at pH 8.6 followed by α1-, α2-, β-, and γ-globulins. Because albumin is fastest, it has the greatest net negative charge and lowest pI (about 4.6). γ-Globulins are predominantly immunoglobulins and have the highest pI (about 7.2

16. Which of the following assays is used to determine the risk of developing cancer? A. Epidermal growth factor receptor (EGF-R) B. Squamous cell carcinoma antigen (SCC) C. c-erb B-2 gene expression D. p53 gene mutation

16. D The p53 gene (tumor suppressor gene) is located on chromosome 17 and produces a protein that down-regulates the cell cycle. A mutation of p53 is associated with an increased incidence of many cancers.

18. Select the elimination model that best describes most oral drugs. A. One compartment, linear first-order elimination B. Michaelis-Menton or concentration-dependent elimination C. Two compartment with a biphasic elimination curve D. Logarithmic elimination

18. A Most drugs given orally distribute uniformly through the tissues reaching rapid equilibrium, so both blood and tissues can be viewed as a single compartment. Elimination according to Michaelis-Menton kinetics is nonlinear because at high concentrations, the hepatic enzyme system becomes saturated, reducing the elimination efficiency

18. In the Oliver-Rosalki method, the reverse reaction is used to measure CK activity. The enzyme(s) used in the coupling reactions is (are): A. Hexokinase and G-6-PD B. Pyruvate kinase and LD C. Luciferase D. Adenylate kinase

18. A The Oliver-Rosalki method for CK is based upon the formation of ATP from creatine phosphate. Hexokinase catalyzes the phosphorylation of glucose by ATP. This produces glucose-6-PO4 and adenosine diphosphate (ADP). The glucose-6-PO4 is oxidized to 6-phosphogluconate as NADP+ is reduced to NADPH. ATP + glucose Hexokinase ADP + glucose-6-PO4 glucose-6-PO4 + NADP+ G-6-PD 6-phosphogluconate + NADPH + H+

18. Which of the following quality control (QC) rules would be broken 1 out of 20 times by chance alone? A. 12s B. 22s C. 13s D. 14s Chemistry

18. A The notation 12S means that one control is outside ±2 standard deviation units. QC results follow the bellshaped curve called the Gaussian (normal) distribution. If a control is assayed 100 times, 68 out of 100 results would fall within +1 s and -1 s of the mean. Ninetyfive (95.4) out of 100 results would fall within +2 s and -2 s. This leaves only 5 out of 100 results (1:20) that fall outside the ±2 s limit. Also, 99.7 out of 100 results fall within ±3 s of the mean

19. Which of the following is most often elevated in hypercalcemia associated with malignancy? A. Parathyroid-derived PTH B. Ectopic PTH C. Parathyroid hormone-related protein (PTHRP) D. Calcitonin

19. C PTHRP is a peptide produced by many tissues and normally present in the blood at a very low level. The peptide has an N-terminal sequence of eight amino acids that are the same as found in PTH and that will stimulate the PTH receptors of bone. Some malignancies (e.g., squamous, renal, bladder, and ovarian cancers) secrete PTHRP, causing hypercalcemia-associated malignancy. Because the region shared with PTH is small and poorly immunoreactive, the peptide does not cross-react in most assays for PTH

19. Which instrument requires a highly regulated DC power supply? A. A spectrophotometer with a barrier layer cell B. A colorimeter with multilayer interference filters C. A spectrophotometer with a photomultiplier tube D. A densitometer with a photodiode detector

19. C When AC voltage regulators are used to isolate source lamp power, light output fluctuates as the voltage changes. Because this occurs at 60 Hz, it is not detected by eyesight or slow-responding detectors. Photomultiplier tubes are sensitive enough to respond to the AC frequency and require a DC-regulated power supply

2. The term pharmacokinetics refers to the: A. Relationship between drug dose and the drug blood level B. Concentration of drug at its sites of action C. Relationship between blood concentration and therapeutic response D. The relationship between blood and tissue drug levels Chemistry/

2. A Pharmacokinetics is the mathematical expression of the relationship between drug dose and drug blood level. When the appropriate formula is applied to quantitative measures of drug dose, absorption, distribution, and elimination, the blood concentration can be accurately determined

2. Upon which principle is the biuret method based? A. The reaction of phenolic groups with CuIISO4 B. Coordinate bonds between Cu+2 and carbonyl and imine groups of peptide bonds. C. The protein error of indicator effect producing color when dyes bind protein D. The reaction of phosphomolybdic acid with protein

2. B Biuret is a compound with two carbonyl groups and three amino groups and forms coordinate bonds with Cu+2 in the same manner as does protein. Therefore, proteins and peptides are both measured in the biuret reaction. The biuret reagent consists of an alkaline solution of copper II sulfate. Tartrate salts are added to keep the copper in solution and prevent turbidity. Potassium iodide prevents autoreduction of Cu+2

2. The syndrome of inappropriate antidiuretic hormone secretion (SIADH) causes: A. Low serum vasopressin B. Hypernatremia C. Urine osmolality to be lower than plasma D. Low serum electrolytes

2. D SIADH results in excessive secretion of vasopressin (ADH) from the posterior pituitary, causing fluid retention and low plasma osmolality, sodium, potassium, and other electrolytes by hemodilution. It is suspected when urine osmolality is higher than plasma, but urine sodium concentration is normal or increased. Patients with sodium depletion have a urine osmolality higher than plasma, but low urine sodium

20. Which testing situation is appropriate for the use of point-of-care whole-blood glucose methods? A. Screening for type 2 diabetes mellitus B. Diagnosis of diabetes mellitus C. Monitoring of blood glucose control in type 1 and type 2 diabetics D. Monitoring diabetics for hyperglycemic episodes only

20. C The ADA does not recommend the use of whole-blood glucose monitors for establishing a diagnosis of diabetes or screening persons for diabetes

21. In the ultraviolet enzymatic method for BUN, the urease reaction is coupled to a second enzymatic reaction using: A. AST B. Glutamate dehydrogenase C. Glutamine synthetase D. Alanine aminotransferase (ALT)

21. B BUN is most frequently measured by the urease-UV method in which the urease reaction is coupled to the glutamate dehydrogenase reaction, generating NAD+. Urea + H2O Urease 2NH3 + CO2 2-Oxoglutarate + NH3 + NADH + H+ GLD Glutamate + NAD+ + H2O

22. Which product is measured in the coupling step of the urease-UV method for BUN? A. CO2 B. Dinitrophenylhydrazine C. Diphenylcarbazone D. NAD+

22. D In the urease-UV method, urease is used to hydrolyze urea, forming CO2 and ammonia. Glutamate dehydrogenase catalyzes the oxidation of NADH, forming glutamate from 2-oxoglutarate and ammonia. The glutamate dehydrogenase reaction is used for measuring both BUN and ammonia.

23. A trend in QC results is most likely caused by: A. Deterioration of the reagent B. Miscalibration of the instrument C. Improper dilution of standards D. Electronic noise

23. A A trend occurs when six or more consecutive quality control results either increase or decrease in the same direction; however, this is not cause for rejection until a multirule is broken. Trends are systematic errors (affecting accuracy) linked to an unstable reagent, calibrator, or instrument condition.

23. Which enzyme deficiency is responsible for phenylketonuria (PKU)? A. Phenylalanine hydroxylase B. Tyrosine transaminase C. p-Hydroxyphenylpyruvic acid oxidase D. Homogentisic acid oxidase

23. A PKU is an overflow aminoaciduria resulting from the accumulation of phenylalanine. It is caused by a deficiency of phenylalanine hydroxylase, which converts phenylalanine to tyrosine. Excess phenylalanine accumulates in blood. This is transaminated, forming phenylpyruvic acid, which is excreted in the urine

(see pic) 23. The electrophoretic pattern shown in the following densitometric tracing most likely indicates: A. α1-Antitrypsin deficiency B. Infection C. Nephrosis D. Systemic sclerosis

23. A This pattern shows a marked decrease in the α1-globulin (slightly less than one-fifth of the expected peak area). Staining of the α1-globulin fraction is predominately determined by the α1-antitrypsin level. A value of less than 20% of normal (0.2-0.4 g/dL) is usually caused by homozygous α1-antitrypsin deficiency

23. Which of the following statements regarding total CK is true? A. Levels are unaffected by strenuous exercise B. Levels are unaffected by repeated intramuscular injections C. Highest levels are seen in Duchenne's muscular dystrophy D. The enzyme is highly specific for heart injury

23. C Total CK is neither sensitive nor specific for AMI. An infarct can occur without causing an elevated total CK. Exercise and intramuscular injections cause a significant increase in total CK. Crush injuries and muscular dystrophy can increase the total CK up to 50 times the URL

17. The reagent blank corrects for absorbance caused by: A. The color of reagents B. Sample turbidity C. Bilirubin and hemolysis D. All of these options

17. A When a spectrophotometer is set to 100%T with the reagent blank instead of water, the absorbance of reagents is automatically subtracted from each unknown reading. The reagent blank does not correct for absorbance caused by interfering chromogens in the sample such as bilirubin, hemolysis, or turbidity

17. According to American Diabetes Association criteria, which result is consistent with a diagnosis of impaired fasting glucose? A. 99 mg/dL B. 117 mg/dL C. 126 mg/dL D. 135 mg/dL

17. B Impaired fasting glucose is defined as a plasma glucose ≥100 but <126 mg/dL. A fasting glucose of 126 or higher on two consecutive occasions indicates diabetes. A fasting glucose of 99 mg/dL is considered normal.

17. A chromatogram for blood alcohol (GC) gives broad trailing peaks and increased retention times for ethanol and internal standard. This is most likely caused by: A. A contaminated injection syringe B. Water contamination of the column packing C. Carrier gas flow rate that is too fast D. Oven temperature that is too high

17. B Increased oven temperature or gas flow rate will shorten retention times and decrease peak widths. Syringe contamination may cause the appearance of ghost peaks. Water in a PEG column such as Carbowax used for measuring volatiles causes longer retention times and loss of resolution

24. Which of the following components is not needed in a chemiluminescent immunoassay analyzer? A. Source lamp B. Monochromator C. Photodetector D. Wash station

24. A Chemiluminescence is the production of light following a chemical reaction. Immunoassays based upon chemiluminescence generate light when the chemiluminescent molecule becomes excited; therefore, a light source is not used. In immunoassay platforms, chemiluminescent molecules such as acridinium can be used to label antigens or antibodies. Alternatively, chemiluminescent substrates such as luminol or dioxetane phosphate may be used. Light will be emitted when the enzyme-labeled molecule reacts with the substrate. In such assays, free and bound antigen separation is required and is usually accomplished using paramagnetic particles bound to either antibody or reagent antigen

24. Which of the following conditions is classified as a renal-type aminoaciduria? A. Fanconi syndrome B. Wilson's disease C. Hepatitis D. Homocystinuria

24. A Fanconi syndrome is an inherited disorder characterized by anemia, mental retardation, rickets, and aminoaciduria. Because the aminoaciduria results from a defect in the renal tubule, it is classified as a (secondary-inherited) renal-type aminoaciduria. Wilson's disease (inherited ceruloplasmin deficiency) causes hepatic failure

24. Which of the following statements regarding the clinical use of CK-MB (CK-2) is true? A. CK-MB becomes elevated before myoglobin after an AMI B. CK-MB levels are usually increased in cases of cardiac ischemia C. CK-MB is more specific than myoglobin D. An elevated CK-MB is always accompanied by an elevated total CK

24. C Serum myoglobin becomes abnormal within 1-2 hours after an acute myocardial infarction (AMI) before troponin and CK-MB. CK-MB becomes abnormal shortly after troponin I (TnI) or troponin T (TnT) when a URL of 4 μg/L is used, and peaks at around the same time following AMI.

28. Which statement is true regarding particle-enhanced turbidimetric inhibition immunoassay methods for therapeutic drugs? A. Drug concentration is proportional to light scatter B. Magnetic separation is needed to remove unbound conjugate C. When particle-bound drug binds to antibody, light scattering is increased D. Two antibodies to the drug are needed

28. C Particle-enhanced turbidimetric inhibition immunoassays are homogenous immunoassays frequently used to measure proteins and therapeutic drugs in serum or plasma. Polystyrene-modified latex particles conjugated to the drug (particle-bound drug) compete with drug in the sample for a limited number of antibodies. If drug concentration is low, more of the antibody binds to the particle-bound drug, increasing the turbidity of the reaction. Therefore, light scattering is inversely proportional to the drug concentration

29. Enzymatic measurement of ammonia requires which of the following substrates and coenzymes? (see pic for answers)

29. A Enzymatic assays of ammonia utilize glutamate dehydrogenase (GLD). This enzyme forms glutamate from α-ketoglutarate (2-oxoglutarate) and ammonia, resulting in oxidation of NADH. The rate of absorbance decrease at 340 nm is proportional to ammonia concentration when the reaction rate is maintained under first-order conditions

29. Which of the following abnormal types of Hgb migrates to the same position as Hgb S on agarose or cellulose acetate at pH 8.6? A. Hgb C B. Hgb DPunjab C. Hgb OArab D. Hgb E Chemistry

29. B Hgb D Punjab migrates with Hgb S on cellulose acetate or agarose at pH 8.6-9.2. Hgb C, E, OArab, and CHarlem migrate to the same position as Hgb A2 on cellulose acetate or agarose at pH 8.6-9.2. Hgb S may be differentiated from Hgb D Punjab using citrate (acid) agar at pH 6.2. Using this technique, Hgb S migrates further toward the anode than Hgb D Punjab.

29. SITUATION: A patient's biochemistry results are: Na = 125 mmol/L Cl = 106 mmol/L K = 4.5 mmol/L TCO2 = 19 mmol/L chol = 240 mg/dL triglyceride = 640 mg/dL glucose = 107 mg/dL AST = 16 IU/L ALT = 11 IU/L amylase = 200 U/L Select the most likely cause of these results. A. The sample is hemolyzed B. Serum was not separated from cells in sufficient time C. Lipemia is causing in vitro interference D. The specimen is contaminated

29. C The triglyceride level is about five times normal, causing the sample to be lipemic. This will cause pseudohyponatremia (unbalanced electrolytes). Lipemia may cause a falsely high rate reaction when amylase is measured by turbidimetry; however, the high amylase may be associated with pancreatitis, which results in hyperlipidemia

29. Which of the following conditions is associated with both metabolic and respiratory alkalosis? A. Hyperchloremia B. Hypernatremia C. Hyperphosphatemia D. Hypokalemia

29. D Hypokalemia is both a cause and result of alkalosis. In alkalosis, hydrogen ions may move from the cells into the extracellular fluid and potassium into the cells. In hypokalemia caused by overproduction of aldosterone, hydrogen ions are secreted by the renal tubules. This increase in net acid excretion results in metabolic alkalosis.

3. A biuret reagent requires preparation of a stock solution containing 9.6 g of copper II sulfate (CuSO4) per liter. How many grams of CuSO4 • 5H2O are needed to prepare 1.0 L of the stock solution? Atomic weights: H = 1.0; Cu = 63.6; O = 16.0; S = 32.1 A. 5.4 g B. 6.1 g C. 15.0 g D. 17.0 g

3. C Determine the mass of CuSO4 • 5H2O containing 9.6 g of anhydrous CuSO4. First, calculate the percentage of CuSO4 in the hydrate, then divide the amount needed (9.6 g) by the percentage. % CuSO4 = molecular weight CuSO4 ÷ molecular weight CuSO4 • 5H2O × 100 = (159.7 ÷ 249.7) × 100 = 63.96% Grams CuSO4 • 5H2O = 9.6 g ÷ 0.6396 = 15.0 g A convenient formula to use is: g hydrate = (MW hydrate ÷ MW anhydrous salt) × g anhydrous salt

3. What is the blood pH when the partial pressure of carbon dioxide (PCO2) is 60 mm Hg and the bicarbonate concentration is 18 mmol/L? A. 6.89 B. 7.00 C. 7.10 D. 7.30

3. C Solve using the Henderson-Hasselbalch equation. pH = pK´ + log HCO3 -/(0.03 × PCO2), where pK´, the negative logarithm of the combined hydration and dissociation constants for dissolved CO2 and carbonic acid, is 6.1 and 0.03 is the solubility coefficient for CO2 gas. pH = 6.1 + log 18/(0.03 × 60) = 6.1 + log 18/1.8 pH = 6.1 + log 10. Because log 10 = 1, pH = 7.10

3. In absorption spectrophotometry: A. Absorbance is directly proportional to transmittance B. Percent transmittance is directly proportional to concentration C. Percent transmittance is directly proportional to the light path length D. Absorbance is directly proportional to concentration

3. D Beer's law states that A = a × b × c, where a is the absorptivity coefficient (a constant), b is the path length, and c is concentration. Absorbance is directly proportional to both b and c. Doubling the path length results in incident light contacting twice the number of molecules in solution. This causes absorbance to double, the same effect as doubling the concentration of molecules.

24. SITUATION: Results of an iron profile are: serum Fe = 40 μg/dL TIBC = 400 μg/dL ferritin = 40μg/L transferrin = 300 mg/dL (reference range 15-200) These results indicate: A. Error in calculation of TIBC B. Serum iron falls before ferritin in iron deficiency C. A defect in iron transport and not Fe deficiency D. Excess release of ferritin caused by injury

24. D Serum ferritin levels fall before iron or TIBC in iron deficiency, and a low level of serum ferritin is diagnostic. However, low tissue levels of ferritin may be masked by increased release into the blood in liver disease, infection, and acute inflammation. Although this patient's serum ferritin is within reference limits, serum iron is low and percent saturation is only 10%. Note that the TIBC and transferrin results are both elevated and agree. TIBC can be estimated by multiplying the serum transferrin by 1.4. These results point to iron deficiency

25. Which of the following mechanisms is responsible for metabolic acidosis? A. Bicarbonate deficiency B. Excessive retention of dissolved CO2 C. Accumulation of volatile acids D. Hyperaldosteronism Chemistry/Apply knowledge

25. A Metabolic acidosis is caused by bicarbonate deficiency and metabolic alkalosis by bicarbonate excess. Respiratory acidosis is caused by PCO2 retention (defective ventilation), and respiratory alkalosis is caused by PCO2 loss (hyperventilation). Important causes of metabolic acidosis include renal failure, diabetic ketoacidosis, lactate acidosis, and diarrhea

25. Select the coupling enzyme used in the hexokinase method for glucose. A. Glucose-6-phosphate dehydrogenase B. Peroxidase C. Glucose dehydrogenase D. Glucose-6-phosphatase

25. A The hexokinase reference method uses a protein-free filtrate prepared with barium hydroxide (BaOH) and zinc sulfate (ZnSO4).

25. SITUATION: Results of an iron profile are: Serum Fe = TIBC = ferritin = 40 μg/dL 400 μg/dL 50 μg/L All of the following tests are useful in establishing a diagnosis of Fe deficiency except: A. Protein electrophoresis B. Erythrocyte zinc protoporphyrin C. Serum transferrin D. Hgb electrophoresis

25. D Electrophoresis may show an elevated β-globulin (transferrin) characteristic of iron deficiency, or inflammation that would help explain a normal ferritin. Zinc protoporphyrin is elevated in iron deficiency and in lead poisoning. Hemoglobinopathies and thalassemias are not associated with iron deficiency

25. Which serum protein should be measured in a patient suspected of having Wilson's disease? A. Hemopexin B. Alpha-1 antitrypsin C. Haptoglobin D. Ceruloplasmin

25. D α-1 antitrypsin, haptoglobin, and ceruloplasmin are acute phase proteins and will be increased in inflammatory diseases. Ceruloplasmin is an α-2 globulin that binds the majority of the serum copper. Levels are low in almost all patients with Wilson's disease, an autosomal recessive disorder caused by accumulation of copper in liver, brain, kidney, and other tissues

26. A patient with hemolytic-uremic syndrome associated with septicemia has a haptoglobin level that is normal, although the plasma free hemoglobin is elevated and hemoglobinuria is present. Which test would be more appropriate than haptoglobin to measure this patient's hemolytic episode? A. Hemopexin B. Alpha-1 antitrypsin C. C-reactive protein D. Transferrin

26. A Hemopexin is a small β globulin that binds to free heme. Haptoglobin is an α-2 globulin that binds to free hemoglobin and disappears from the serum when intravascular hemolysis produces more than 3 grams of free plasma hemoglobin. However, haptoglobin is an acute phase protein, and hepatic production and release are increased in response to acute infections. The normal serum haptoglobin is most likely the result of increased synthesis and would not accurately estimate the hemolytic episode in this patient

27. Of the methods used to measure amino acids, which is capable of measuring fatty acids simultaneously? A. Tandem-mass spectroscopy B. High-performance liquid chromatography C. Capillary electrophoresis D. Two-dimensional thin-layer chromatography

27. A All four methods are able to separate each amino acid (up to 40 species); however, tandem-mass spectroscopy with electrospray ionization can measure amino acids, organic acids such as methylmalonic acid, and fatty acids

27. Given the following data, calculate the coefficient of variation for glucose. Analyte Mean Standard Deviation Glucose 76 mg/dL 2.3 A. 3.0% B. 4.6% C. 7.6% D. 33.0%

27. A The coefficient of variation is calculated by dividing the standard deviation by the mean and multiplying by 100.

27. SITUATION: Hgb electrophoresis is performed and all of the Hgbs have greater anodal mobility than usual. A fast Hgb (Hgb H) is at the edge of the gel and bands are blurred. The voltage is set correctly, but the current reading on the ammeter is too low. Select the course of action that would correct this problem. A. Reduce the voltage B. Dilute the buffer and adjust the pH C. Prepare fresh buffer and repeat the test D. Reduce the running time

27. C Increased mobility, decreased resolution, and low current result from low ionic strength. Reducing voltage will slow migration but will not improve resolution. Diluting the buffer will reduce the current, resulting in poorer resolution.

28. The following conditions are all causes of alkalosis. Which condition is associated with respiratory (rather than metabolic) alkalosis? A. Anxiety B. Hypovolemia C. Hyperaldosteronism D. Hypoparathyroidism

28. A Respiratory alkalosis is caused by hyperventilation, which leads to decreased PCO2. Anxiety and drugs such as epinephrine that stimulate the respiratory center are common causes of respiratory alkalosis. Excess aldosterone increases net acid excretion by the kidneys. Low parathyroid hormone causes increased bicarbonate reabsorption, resulting in alkalosis. Hypovolemia increases the relative concentration of bicarbonate. This is common and is called dehydrational alkalosis, chloride responsive alkalosis, or alkalosis of sodium deficit

28. Blood ammonia levels are usually measured in order to evaluate: A. Renal failure B. Acid-base status C. Hepatic coma D. Gastrointestinal malabsorption

28. C Hepatic coma is caused by accumulation of ammonia in the brain as a result of liver failure. The ammonia increases central nervous system pH and is coupled to glutamate, a central nervous system neurotransmitter, forming glutamine. Blood and cerebrospinal fluid ammonia levels are used to distinguish encephalopathy caused by cirrhosis or other liver disease from nonhepatic causes and to monitor patients with hepatic coma

28. Select the correct order of Hgb migration on agarose or cellulose acetate at pH 8.6. A. - C→F→S→A + B. - S→C→A→F + C. - C→S→F→A + D. - S→F→A→C +

28. C Hgb A2 is the slowest of the normal Hgbs, and Hgb A is the fastest. Hgb F migrates just behind Hgb A. Hgb S migrates midway between Hgb A2 and Hgb A. Hgbs C, CHarlem (Georgetown), O, and E migrate with Hgb A2. Hgbs G and DPunjab and Hgb OArab migrate with Hgb S.

30. Which Hgb is a β-δ chain hybrid and migrates to the same position as Hgb S at pH 8.6? A. Hgb CHarlem B. HgbLepore C. Hgb GPhiladelphia D. Hgb DPunjab

30. B HgbLepore results from translocation of β and δ globin genes, resulting in a polypeptide chain that migrates midway between Hgb A2 and Hgb A. The chain is transcribed more slowly than the β polypeptide chain, causing the quantity of HgbLepore to be less than 15%. HgbLepore is suspected when Hgb migrating in the "S" zone comprises less than 20% of the total Hgb. In Hgb S trait, the AS phenotype produces 20%-40% Hgb S.

30. In uncompensated metabolic acidosis, which of the following will be normal? A. Plasma bicarbonate B. PCO2 C. p50 D. Total CO2

30. B The normal compensatory mechanism for metabolic acidosis is respiratory hyperventilation. In uncompensated cases, the PCO2 is not reduced, indicating a concomitant problem in respiratory control.

30. When measuring lead in whole blood using atomic absorption spectrophotometry, what reagent is required to obtain the needed sensitivity and precision? A. Lanthanum B. Lithium C. Triton X-100 D. Chloride

30. C A graphite furnace is preferred over a flame for measuring lead because it is sufficiently sensitive to detect levels below 5 μg/dL, the cutoff needed for lead screening of children. The matrix modifier consists of Triton X -100, ammonium phosphate and nitric acid. This allows for release of Pb from the RBCs, and solubilization of cell stroma. The matrix modifier also prevents loss of Pb caused by formation of lead halides and promotes interaction between Pb and the tube wall, preventing its loss during the ashing cycle.

30. Which statement about ammonia is true? A. Normally, most of the plasma ammonia is derived from peripheral blood deamination of amino acids B. Ammonia-induced coma can result from salicylate poisoning C. Hepatic coma can result from Reye's syndrome D. High plasma ammonia is usually caused by respiratory alkalosis

30. C Ammonia produced in the intestines from the breakdown of proteins by bacterial enzymes is the primary source of plasma ammonia. Most of the ammonia absorbed from the intestines is transported to the liver via the portal vein and converted to urea. Blood ammonia levels will rise in any necrotic liver disease including hepatitis, Reye's syndrome, and drug-induced injury such as acetaminophen poisoning.

30. Which plot will give the earliest indication of a shift or trend? A. Levy-Jennings B. Tonks-Youden C. Cusum D. Histogram

30. C Cusum points are the algebraic sum of the difference between each QC result and the mean. The y axis is the sum of differences and the x axis is the run number. The center of the y axis is 0. Because QC results follow a random distribution, the points should distribute about the zero line. Results are out of control when the slope exceeds 45° or a decision limit (e.g., ±2.7s) is exceeded.

30. Which of the following is the most effective serial sampling time for ruling out AMI using both myoglobin and a cardiac specific marker in an emergency department environment? A. Admission and every hour for the next 3 hours or until positive B. Admission, 2 hours, 4 hours, and 6 hours or until positive C. Admission, 3 hours, 6 hours, and a final sample within 12 hours D. Admission and one sample every 8 hours for 48 hours

30. C Since the time between the onset of symptoms and arrival in the emergency department is often speculative, serial measurement of cardiac markers is required in order to rule out AMI. Since myoglobin is the first marker to rise after AMI, if used it should be measured on admission

31. Select the correct order of Hgb migration on citrate agar at pH 6.2. A. - F→S→C→A + B. - F→A→S→C + C. - A→S→F→C + D. - A→C→S→F +

31. B In an acid buffer, the hemoglobins are expected to migrate to the cathode, with hemoglobin A being the slowest because it has the weakest net positive charge. However, Hgb C and Hgb S bind to sulfated pectins in the agar gel, forming a complex that is negatively charged causing them to migrate toward the anode. Hgb C migrates furthest toward the anode, followed by Hgb S. Hgb F migrates furthest toward the cathode. Hgbs A, A2, DPunjab, E, G, and HgbLepore migrate slightly toward the cathode

31. Interference in atomic absorption spectrophotometry caused by differences in viscosity is called: A. Absorption interference B. Matrix effect C. Ionization interference D. Quenching

31. B Significant differences in aspiration and atomization result when the matrix of sample and unknowns differ. Differences in viscosity and protein content are major causes of matrix error. Matrix effects can be reduced by using protein-based calibrators and diluting both standards and samples prior to assay

32. Which Hgb separates from Hgb S on citrate (acid) agar, but not agarose or cellulose acetate? A. Hgb DPunjab B. Hgb E C. Hgb CHarlem (Georgetown) D. Hgb OArab Chemistry/Evaluate

32. A Hgbs OArab, E, and CHarlem migrate to the same position as Hgbs A2 and C on agarose × or cellulose acetate) at pH 8.6. Hgb DPunjab migrates to the same position as Hgb S on agarose, but moves with Hgb A on citrate agar. Agarose is a purified form of agar; it lacks the sulfated pectins required to separate Hgbs DPunjab and G from Hgb S, and Hgbs E, CHarlem, and OArab from Hgb C. Hgb CHarlem is a sickling Hgb and it migrates to the same position as Hgb S on citrate (acid) agar

32. Which PCO2 value would be seen in maximally compensated metabolic acidosis? A. 15 mm Hg B. 30 mm Hg C. 40 mm Hg D. 60 mm Hg

32. A In metabolic acidosis, hyperventilation increases the ratio of bicarbonate to dissolved CO2. The extent of compensation is limited by the rate of both gas diffusion and diaphragm contraction. The lower limit is between 10 and 15 mm Hg PCO2, which is the maximum compensatory effect

32. Uric acid is derived from the: A. Oxidation of proteins B. Catabolism of purines C. Oxidation of pyrimidines D. Reduction of catecholamines

32. B Uric acid is the principal product of purine (adenosine and guanosine) metabolism. Oxidation of proteins yields urea along with CO2, H2O, and inorganic acids. Catecholamines are oxidized, forming vanillylmandelic acid (VMA) and homovanillic acid (HVA

32. Which statement about the measurement of carboxyhemoglobin is true? A. Treatment with alkaline dithionite is used to convert carboxyhemoglobin to oxyhemoglobin B. Oxyhemoglobin has no absorbance at 540 nm, but carboxyhemoglobin does C. Bichromatic analysis is required in order to eliminate interference by oxyhemoglobin D. Carboxyhemoglobin can be measured by potentiometry

32. C The absorbance spectras of oxy- and carboxyhemoglobin pigments overlap, and bichromatic or multichromatic analysis is required in order to accurately measure carboxyhemoglobin

35. A single-point calibration is performed between each blood gas sample in order to: A. Correct the electrode slope B. Correct electrode and instrument drift C. Compensate for temperature variance D. Prevent contamination by the previous sample

35. B Calibration using a single standard corrects the instrument for error at the labeled value of the calibrator but does not correct for analytic errors away from the set point. A two-point calibration adjusts the slope response of the electrode, eliminating proportional error caused by poor electrode performance

37. Which of the following statements regarding iron metabolism is correct? A. Iron absorption is decreased by alcohol ingestion B. Normally, 40%-50% of ingested iron is absorbed C. The daily requirement is higher for pregnant and menstruating women D. Absorption increases with the amount of iron in the body stores

37. C For adult men and nonmenstruating women, approximately 1-2 mg/day of iron is needed to replace the small amount lost mainly by exfoliation of cells. Because 5%-10% of dietary iron is absorbed normally, the daily dietary requirement in this group is 10-20 mg/day.

38. Which test is the most sensitive in detecting early monoclonal gammopathies? A. High-resolution serum protein electrophoresis B. Urinary electrophoresis for monoclonal light chains C. Capillary electrophoresis of serum and urine D. Serum-free light chain immunoassay

38. D Immunonephelometric free light chain assays can detect monoclonal protein production before the mass is sufficient to cause a monoclonal spike on protein electrophoresis or capillary electrophoresis, but will be positive only in cases where monoclonal light chain production occurs.

33. Which metabolite is most often increased in carcinoid tumors of the intestine? A. 5-Hydroxyindolacetic acid (5-HIAA) B. 3-Methoxy-4-hydroxyphenylglycol (MHPG) C. 3-Methoxydopamine D. HVA

33. A 5-HIAA is a product of serotonin catabolism. Excess levels are found in urine of patients with carcinoid tumors composed of argentaffin cells. Carcinoid tumors are usually found in the intestine or lung, and produce serotonin and 5-hydroxytryptophan, a serotonin precursor

4. SITUATION: A 22S QC error occurs for serum calcium by atomic absorption. Fresh standards prepared in 5.0% w/v albumin are found to be linear, but repeating the controls with fresh material does not improve the QC results. Select the most likely cause of this problem. A. Matrix effect caused by a viscosity difference between the standards and QC sera B. Chemical interference caused incomplete atomization C. Incomplete deconjugation of protein-bound calcium D. Ionization interference caused by excessive heat

4. B Poor recovery of calcium by atomic absorption is often caused by failure to break thermostable bonds between calcium and phosphate (a form of chemical interference). This may be caused by failure to add lanthanum to the diluent or by low atomizer temperature. The use of 5.0 % w/v albumin in the calibrator produces viscosity and protein-binding characteristics similar to plasma, helping to eliminate matrix interference

4. Which of the following protein methods has the highest analytical sensitivity? A. Refractometry B. Folin-Lowry C. Turbidimetry D. Direct ultraviolet absorption

4. B The Folin-Lowry (Lowry's) method uses both biuret reagent and phosphotungstic and molybdic acids to oxidize the aromatic side groups on proteins. The acids oxidize the phenolic rings of tyrosine and tryptophan. These, in turn, reduce the Cu+2 in the biuret reagent, increasing sensitivity about 100-fold.

4. Which of the following is the best analyte to monitor for recurrence of ovarian cancer? A. CA-15-3 B. CA-19-9 C. CA-125 D. CEA

4. C CA-125 is an oncofetal antigen, meaning that it is produced by genes that are active during fetal development but minimally active after birth except in malignant tissues. This group includes α-fetoprotein (AFP), CEA, PSA, and the carbohydrate-associated antigens (CA

4. How many milliliters of HNO3 (purity 68.0%, specific gravity 1.42) are needed to prepare 1.0 L of a 2.0 N solution? Atomic weights: H = 1.0; N = 14.0; O = 16.0 A. 89.5 mL B. 126.0 mL C. 130.5 mL D. 180.0 mL

4. C The molecular weight of HNO3 is 63.0 g. Because the valance of the acid is 1 (1 mol of hydrogen is produced per mole of acid), the equivalent weight is also 63.0 g. The mass is calculated by multiplying the normality (2.0 N) by the equivalent weight (63.0 g) and volume (1.0 L); therefore, 126.0 g of acid are required. Because the purity is 68.0% and the specific gravity 1.42, the amount of HNO3 in grams per milliliter is 0.68 × 1.42 g/mL or 0.9656 g/mL. The volume required to give 126.0 g is calculated by dividing the mass needed (grams) by the grams per milliliter. mL HNO3 = 126.0 g ÷ 0.9656 g/mL = 126.0 g × 1.0 mL/0.9656 g = 130.5 mL

41. Capillary electrophoresis differs from agarose gel electrophoresis in which respect? A. A stationary support is not used B. An acidic buffer is used C. A low voltage is used D. Electroendosmosis does not occur

41. A Capillary electrophoresis is a rapid automated procedure for separating serum or body fluid proteins. Instead of a stationary support, the proteins migrate based upon their charge/mass ratio inside a small-bore silica capillary tube (20-200 μm). The cations in the buffer are attracted to the negatively charged silicates and migrate to the cathode rapidly when voltage is applied

41. Which of the following statements about the aminotransferases (AST and ALT) is true? A. Isoenzymes of AST and ALT are not found in humans B. Both transfer an amino group to α-ketoglutarate C. Both require NADP+ as a coenzyme D. Both utilize four carbon amino acids as substrates

41. B ALT catalyzes the transfer of an amino group from alanine, a three-carbon amino acid, to α-ketoglutarate (2-oxoglutarate), forming pyruvate. AST catalyzes the transfer of an amino group from aspartate (four carbons) to α-ketoglutarate, forming oxaloacetate.

41. Which of the following would cause a "response" error from an ion-selective electrode for sodium when measuring serum but not the calibrator? A. Interference from other electrolytes B. Protein coating the ion-selective membrane C. An overrange in sodium concentration D. Protein binding to sodium ions

41. B Response is the time required for an electrode to reach maximum potential. Ion-selective analyzers use a microprocessor to monitor electrode response, slope, drift, and noise. When an electrode gives an acceptable response time when measuring an aqueous calibrator, but not when measuring serum, the cause is often protein buildup on the membrane

41. Which of the following statements regarding thyroid hormones is true? A. Both protein-bound and free T3 and T4 are physiologically active B. Total T3 and T4 are influenced by the level of thyroxine-binding globulin C. Variation in thyroxine-binding protein levels affects both free T3 and T4 D. An elevated serum total T4 and T3 is diagnostic of hyperthyroidism

41. B Total serum T4 and T3 are dependent upon both thyroid function and the amount of thyroxine-binding proteins such as thyroxine-binding globulin (TBG). Total T4 or T3 may be abnormal in a patient with normal thyroid function, if the TBG level is abnormal. For this reason, free T3 and T4 are more specific indicators of thyroid function than are measurements of total hormone. Only free hormone is physiologically active

41. Which condition is most often associated with a high serum iron level? A. Nephrosis B. Chronic infection or inflammation C. Polycythemia vera D. Noniron deficiency anemias

41. D Anemia associated with chronic infection causes a low serum iron, but unlike iron deficiency, causes a low (or normal) TIBC and does not cause low ferritin. Noniron deficiency anemias such as pernicious anemia and sideroblastic anemia produce high serum iron and low TIBC.

43. Which of the following laboratory results is consistent with primary hypoparathyroidism? A. Low calcium; high inorganic phosphorus Pi B. Low calcium; low Pi C. High calcium; high Pi D. High calcium; low Pi Chemistry/Correlate clinical

43. A Parathyroid hormone deficiency causes reduced resorption of calcium from bone, increased renal excretion of calcium, and decreased renal excretion of phosphorus. It is distinguished from other causes of hypocalcemia by Cai, which is reduced only by primary hypoparathyroidism and alkalosis

43. Following ultracentrifugation of plasma, which fraction correlates with pre-β lipoprotein? A. Very low-density lipoprotein (VLDL) B. Low-density lipoprotein (LDL) C. High-density lipoprotein (HDL) D. Chylomicrons Chemistry/Apply

43. A The VLDL (very low-density lipoprotein) migrates in the pre-β zone. The VLDL is about 50% triglyceride, whereas LDL is only 10% triglyceride by weight. LDL is formed from VLDL in the circulation. The process is initiated by apoC-II on VLDL activating peripheral lipoprotein lipase. Hydrolysis of triglycerides and transfer of apoproteins from VLDL to HDL result in formation of IDL. Larger IDLs are returned to the liver as remnant lipoproteins. Further hydrolysis of triglycerides, transfer cholesterol esters from HDL, and transfer of apoproteins to HDL convert IDL to LDL.

46. Which of the following is a marker for bone formation? A. Osteocalcin B. Tartrate resistant acid phosphatase (TRAP) C. Urinary pyridinoline and deoxypyridinoline D. Urinary C-telopeptide and N-telopeptide crosslinks (CTx and NTx) Chemistry/Select tests/Bone disorde

46. A Biochemical markers for osteoporosis are classified as either markers for bone formation or resorption. Osteocalcin is a protein hormone that stimulates osteoblasts and increases bone mineralization. Pyridinoline is formed when hydroxylysine groups on adjacent fibrils are joined together, and deoxypyridinoline when hydroxylysine and lysine groups are joined.

46. Which of the following statements is true regarding reverse T3 (rT3)? A. Formed in the blood by degradation of T4 B. Physiologically active, but less than T3 C. Decreased in euthyroid sick syndrome D. Interferes with the measurement of serum T3

46. A Reverse T3 is formed from the deiodination of T4 in the blood. It is an inactive isomer of T3, (3,3´,5´-triiodothyronine). Reverse T3 is increased in acute and chronic illness and is used to identify patients with euthyroid sick syndrome

47. A patient has an elevated serum T3 and free T4 and undetectable TSH. What is the most likely cause of these results? A. Primary hyperthyroidism B. Secondary hyperthyroidism C. Euthyroid with increased thyroxine-binding proteins D. Euthyroid sick syndrome

47. A An undetectable TSH with increased T3 is caused by primary hyperthyroidism (suppression via high free thyroid hormone). In secondary hyperthyroidism, the TSH will be elevated in addition to at least the T3. Patients with an increased thyroxine-binding protein level will have an increase in total T3 but not free T4 or TSH. Patients with euthyroid sick syndrome usually have a low total T3 due to deficient conversion of T4 to T3, normal free T4, and a normal or slightly elevated TSH.

47. Which of the following compounds can interfere with the coulometric chloride assay? A. Bromide B. Ascorbate C. Acetoacetate D. Nitrate

47. A Chloride assays based upon either coulometric or chemical titration are subject to positive interference from other anions and electronegative radicals that may be titrated instead of chloride ions. These include other halogens such as bromide, cyanide, and cysteine.

47. Which of the following statements regarding the naming of transaminases is true? A. Serum glutamic oxaloacetic transaminase (SGOT) is the older abbreviation for ALT B. Serum glutamic pyruvic transaminase (SGPT) is the older abbreviation for AST C. SGPT is the older abbreviation for ALT D. SGOT is the newer abbreviation for AST

47. C SGOT refers to the products measured in the in vitro reaction, and is more correctly named AST for the four-carbon amino acid substrate aspartate. SGPT is the older name referring to the products of the reaction for ALT. SGPT is more correctly named ALT for the three-carbon amino acid substrate alanine

47. A quantitative sandwich enzyme immunoassay for intact serum hCG was performed on week 4 and the result was 40,000 mIU/mL (reference range 10,000-80,000 mIU/mL). The physician suspected a molar pregnancy and requested that the laboratory repeat the test checking for the hook effect. Which process would identify this problem? A. Obtain a new plasma specimen and heat inactivate before testing B. Obtain a urine specimen and perform the assay C. Perform a qualitative pregnancy test D. Perform a serial dilution of the sample and repeat the test

47. D The hook effect is the result of excessive antigen concentration and results in a dose response (calibration) curve that reverses direction at very high antigen concentrations. It occurs in two-site double antibody sandwich assays when both the capture antibody and the enzyme-conjugated antibody are incubated with the antigen at the same time.

48. Bilirubin is transported from reticuloendothelial cells to the liver by: A. Albumin B. Bilirubin-binding globulin C. Haptoglobin D. Transferrin

48. A Albumin transports bilirubin, haptoglobin transports free Hgb, and transferrin transports ferric iron. When albumin binding is exceeded, unbound bilirubin, called free bilirubin, increases. This may cross the blood-brain barrier, resulting in kernicterus.

5. Which substrate concentration is needed to achieve zero-order conditions? A. Greater than 99 × Km B. [S] = Km C. Less than 10 × Km D. [S] = 0

5. A A zero-order reaction rate is independent of substrate concentration because there is sufficient substrate to saturate the enzyme. V = Vmax × [S]/Km + [S] where V = velocity, Vmax = maximum velocity, [S] = substrate concentration, and Km = substrate concentration required to give 1/2 Vmax. If [S] >>> Km, then the Km can be ignored. V = Vmax × [S]/[S] = Vmax × [S]° or velocity approaches maximum and is independent of substrate concentration

5. Which statement about multiple endocrine neoplasia (MEN) is true? A. It is associated with hyperplasia or neoplasia of at least two endocrine organs B. Insulinoma is always present when the pituitary is involved C. It is inherited as an autosomal recessive disorder D. Plasma hormone levels from affected organs are elevated at least 10-fold

5. A Multiple-endocrine neoplasia syndrome is inherited as an autosomal dominant disease involving excess production of hormones from several endocrine glands. MEN I results from adenomas (usually benign) of at least two glands, including the pituitary, adrenal cortex, parathyroid, and pancreas. The parathyroid gland is the organ most commonly involved, and in those patients an elevated Cai is an early sign

5. Which of the following conditions is most likely to cause a falsely high creatinine clearance result? A. The patient uses the midstream void procedure when collecting his or her urine B. The patient adds tap water to the urine container because he or she forgets to save one of the urine samples C. The patient does not empty his or her bladder at the conclusion of the test D. The patient empties his or her bladder at the start of the test and adds the urine to the collection

5. D Urine in the bladder should be eliminated and not saved at the start of the test because it represents urine formed prior to the test period. The other conditions (choices A-C ) will result in falsely low urine creatinine or volume and, therefore, falsely lower clearance results. Error is introduced by incomplete emptying of the bladder when short times are used to measure clearance.

50. What component of a freezing point osmometer measures the sample temperature? A. Thermistor B. Thermocouple C. Capacitor D. Electrode

50. A A thermistor is a temperature-sensitive resistor. The resistance to current flow increases as temperature falls. The temperature at which a solution freezes can be determined by measuring the resistance of the thermistor. Resistance is directly proportional to the osmolality of the sample

50. In which liver disease is the DeRitis ratio (ALT:AST) usually greater than 1.0? A. Acute hepatitis B. Chronic hepatitis C. Hepatic cirrhosis D. Hepatic carcinoma

50. A ALT prevails over AST in hepatitis; however, AST is greater than ALT in carcinoma, alcoholic liver disease, and cirrhosis of the liver.

5. Which of the following results falls within the diagnostic criteria for diabetes mellitus? A. Fasting plasma glucose of 120 mg/dL B. Two-hour postprandial plasma glucose of 160 mg/dL C. Two-hour plasma glucose of 180 mg/dL following a 75 g oral glucose challenge D. Random plasma glucose of 250 mg/dL and presence of symptoms

5. D The American Diabetes Association recommends the following criteria for diagnosing diabetes mellitus: fasting glucose ≥ 126 mg/dL, casual (random) glucose ≥ 200 mg/dL in the presence of symptoms (polyuria, increased thirst, weight loss), glucose ≥ 200 mg/dL at 2 hours after an oral dose of 75 g of glucose, and hemoglobin A1c ≥ 6.5%. A diagnosis of diabetes mellitus is indicated if any one or combination of these four criteria is met on more than a single testing event. The fasting plasma glucose test requires at least 8 hours with no food or drink except water.

50. Which of the following conditions is most consistently associated with secondary hypercholesterolemia? A. Hypothyroidism B. Pancreatitis C. Oral contraceptive therapy D. Diabetes mellitus

50. A The conditions listed are very commonly encountered causes of secondary hyperlipoproteinemia. Oral contraceptives, pregnancy, and estrogens may cause secondary hypertriglyceridemia owing to increased VLDL and endogenous triglycerides. Hypothyroidism and obstructive hepatobiliary diseases are usually associated with secondary hypercholesterolemia owing to high LDL.

55. Treatment recommendations for patients with coronary heart disease are based upon measurement of which analyte? A. HDL cholesterol B. Apo-B100 C. LDL cholesterol D. Total cholesterol

55. C NECP has identified LDL cholesterol as the target of therapy for reducing the risk of heart attack because lowering LDL cholesterol has proven to be an effective intervention. The greater the risk of coronary heart disease, the lower the cutpoint for intervention. For persons at high risk (a 10-year risk of heart attack > 20%) the cutpoint is ≥ 100 mg/dL for initiation of statin therapy. For highest-risk persons (those that have acute coronary syndrome, and multiple or uncontrolled risk factors) the treatment goal is LDL cholesterol below 70 mg/dL

56. The term reverse phase is used in HPLC to indicate that the mobile phase is: A. More polar than the stationary phase B. Liquid and the stationary phase is solid C. Organic and the stationary phase is aqueous D. A stronger solvent than the stationary phase

56. A In reverse-phase HPLC, the separation takes place using a nonpolar sorbent (stationary phase) such as octadecylsilane (C18). Solutes that are nonpolar are retained longer than polar solutes. Most clinical separations of drugs, hormones, and metabolites use reverse phase because aqueous mobile phases are far less toxic and flammable

56. What is the HDL cholesterol cutpoint recommend by NCEP? A. <30 mg/dL B. <40 mg/dL C. <30 mg/dL for males and < 40 mg/dL for females D. <45 mg/dL for males and < 50 mg/dL for females

56. B The HDL cholesterol cutpoint recommended by NCEP is < 40 mg/dL regardless of sex. A result below 40 mg/dL counts as a risk factor for coronary artery disease. Conversely, if the HDL cholesterol is ≥ 60 mg/dL, then one risk factor is subtracted from the total number. The therapeutic goal for someone with low HDL cholesterol is still reduction of LDL cholesterol (if elevated), weight loss, and increased exercise

56. Which condition is caused by deficient secretion of bilirubin into the bile canaliculi? A. Gilbert's disease B. Neonatal hyperbilirubinemia C. Dubin-Johnson syndrome D. Crigler-Najjar syndrome

56. C Dubin-Johnson syndrome is an autosomal recessive condition arising from mutation of an ABC transporter gene. It produces mild jaundice from accumulation of conjugated bilirubin that is not secreted into the bile canaliculi

57. In hepatitis, the rise in serum conjugated bilirubin can be caused by: A. Secondary renal insufficiency B. Failure of the enterohepatic circulation C. Enzymatic conversion of urobilinogen to bilirubin D. Extrahepatic conjugation

57. B Conjugated bilirubin is increased in hepatitis and other causes of hepatic necrosis due to failure to re-excrete conjugated bilirubin reabsorbed from the intestine. Increased direct bilirubin can also be attributed to accompanying intrahepatic obstruction, which blocks the flow of bile

58. Which of the following conditions is most likely to produce an elevated plasma potassium? A. Hypoparathyroidism B. Cushing's syndrome C. Diarrhea D. Digitalis overdose

58. D Digitalis toxicity causes potassium to leave the cells and enter the extracellular fluid, resulting in hyperkalemia. Renal failure, hemolytic anemia and Addison's disease are other frequent causes of hyperkalemia. Hypoparathyroidism indirectly causes hypokalemia by inducing alkalosis via increased renal retention of phosphate and bicarbonate

6. Hyperalbuminemia is caused by: A. Dehydration syndromes B. Liver disease C. Burns D. Gastroenteropathy

6. A A high serum albumin level is caused only by dehydration or administration of albumin. Liver disease, burns, gastroenteropathy, nephrosis, starvation, and malignancy cause hypoalbuminemia.

55. Which statement regarding bilirubin metabolism is true? A. Bilirubin undergoes rapid photo-oxidation when exposed to daylight B. Bilirubin excretion is inhibited by barbiturates C. Bilirubin excretion is increased by chlorpromazine D. Bilirubin is excreted only as the diglucuronide

55. A Samples for bilirubin analysis must be protected from direct sunlight. Drugs may have a significant in vivo effect on bilirubin levels. Barbiturates lower serum bilirubin by increasing excretion. Other drugs that cause cholestasis, such as chlorpromazine, increase the serum bilirubin. Although most conjugated bilirubin is in the form of diglucuronide, some monoglucuronide and other glycosides are excreted. In glucuronyl transferase deficiency, some bilirubin is excreted as sulfatides

6. A person presents with a cushingoid appearance and an elevated 24-hour urinary cortisol level. The plasma adrenocotropic hormone (ACTH) is very elevated, and the physician suspects the cause is ectopic ACTH production. Which test would be most useful in substantiating this diagnosis? A. Plasma cortisol B. CA-50 C. Alkaline phosphatase isoenzymes D. AFP

6. C Most often, ectopic ACTH production occurs in lung cancer. Tumors of the lung are often associated with the production of placental-like alkaline phosphatase, and a positive finding would support the diagnosis of an ectopic (nonpituitary) source of ACTH

6. When calibrating a pH meter, unstable readings occur for both pH 7.00 and 4.00 calibrators, although both can be set to within 0.1 pH unit. Select the most appropriate course of action. A. Measure the pH of the sample and report to the nearest 0.1 pH B. Replace both calibrators with unopened buffers and recalibrate C. Examine the reference electrode junction for salt crystals D. Move the electrodes to another pH meter and calibrate

6. C Noise in pH measurements often results from a blocked junction between the reservoir of the reference electrode and test solution. This occurs when salt crystals collect at the junction or when KCl concentration in the reservoir increases due to evaporation of water. The fluid in the reference electrode should be replaced with warm deionized water. After the crystals have dissolved, the water is replaced with fresh reference electrolyte solution

6. Select the most appropriate adult reference range for fasting blood glucose. A. 40-105 mg/dL (2.22-5.82 mmol/L) B. 60-140 mg/dL (3.33-7.77 mmol/L) C. 65-99 mg/dL (3.61-5.50 mmol/L) D. 75-150 mg/dL (4.16-8.32 mmol/L)

6. C Reference ranges vary slightly depending upon method and specimen type. Enzymatic methods specific for glucose have an upper limit of normal no greater than 99 mg/dL. This is the cutoff value for impaired fasting plasma glucose (prediabetes) recommended by the American Diabetes Association. Although 65 mg/dL is considered the 2.5 percentile, a fasting level below 50 mg/dL is often seen without associated clinical hypoglycemia, and neonates have a lower limit of approximately 40 mg/dL owing to maternal insulin

55. Which of the following statements about the phosphatases is true? A. They hydrolyze adenosine triphosphate and related compounds B. They are divided into two classes based upon pH needed for activity C. They exhibit a high specificity for substrate D. They are activated by Pi

55. B Phosphatases are classified as either alkaline or acid depending upon the pH needed for optimum activity. The phosphatases hydrolyze a wide range of monophosphoric acid esters. ALP is inhibited by phosphorus (product inhibition). The International Federation of Clinical Chemistry (IFCC) recommended method employs 2-amino-2-methyl-1-propanol, a buffer that binds Pi.

55. When measuring calcium with the complexometric dye o-cresolphthalein complexone, magnesium is kept from interfering by: A. Using an alkaline pH B. Adding 8-hydroxyquinoline C. Measuring at 450 nm D. Complexing to EDTA

55. B o-Cresolphthalein complexone can be used to measure either magnesium or calcium. Interference in calcium assays is prevented by addition of 8-hydroxyquinoline, which chelates magnesium. When magnesium is measured, ethyleneglycol bistetraacetic acid (EGTA) or EDTA is used to chelate calcium. Two other dyes that can be used for both magnesium and calcium assays are calmagite and methylthymol blue. Arsenazo III dye is commonly used to measure calcium.

55. The term isocratic is used in high-performance liquid chromatography (HPLC) to mean the: A. Mobile phase is at constant temperature B. Stationary phase is equilibrated with the mobile phase C. Mobile phase consists of a constant solvent composition D. Flow rate of the mobile phase is regulated

55. C An isocratic separation uses a single mobile phase of constant composition, pH, and polarity, and requires a single pump. Some HPLC separations use a gradient mobile phase to increase distance between peaks. Gradients are made by mixing two or more solvents using a controller to change the proportions of solvent components.

62. What is the purpose of the saponification step used in the Abell-Kendall method for cholesterol measurement? A. Remove phospholipids B. Reduce sterol molecules structurally similar to cholesterol C. Convert cholesterol esters to free cholesterol D. Remove proteins that can interfere with color formation

62. C The Abell-Kendall method is the reference method for cholesterol assay because differences in esterase activity and interference in the peroxidase step are potential sources of error in enzymatic assays. Saponification is performed to hydrolyze the fatty acid esters of cholesterol, forming free cholesterol. This is required because the reagents react more intensely with cholesterol esters than with free cholesterol.

62. Which condition is least likely to be associated with increased serum ALP? A. Osteomalacia B. Biliary obstruction C. Hyperparathyroidism and hyperthyroidism D. Osteoporosis

62. D ALP is elevated in osteomalacia (rickets), bone cancer, and bone disease secondary to hyperthyroidism and hyperparathyroidism, but total ALP it is high in less than 30% of osteoporosis patients. Pancreatic disease associated with biliary obstruction, such as cancer at the head of the pancreas, is associated with elevated ALP.

63. Which reagent is used in thin-layer chromatography (TLC) to extract cocaine metabolites from urine? A. Acid and sodium chloride B. Alkali and organic solvent C. Chloroform and sodium acetate D. Neutral solution of ethyl acetate Chemistry/Apply principles of special procedures

63. B Alkaline drugs such as cocaine, amphetamine, and morphine are extracted at alkaline pH. Ideally, the pH for extracting alkaline drugs into an organic solvent should be 2 pH units greater than the negative log of dissociation constant (pKa) of the drug. More than 90% of the drug will be nonionized and will extract in ethyl acetate or another organic solvent.

63. Which form of jaundice occurs within days of delivery and usually lasts 1-3 weeks, but is not due to normal neonatal hyperbilirubinemia or hemolytic disease of the newborn? A. Gilbert syndrome B. Lucey -Driscoll syndrome C. Rotor syndrome D. Dubin-Johnson syndrome

63. B Lucey-Driscoll syndrome is a rare form of jaundice caused by unconjugated bilirubin that presents within 2-4 days of birth and can last several weeks. It is caused by an inhibitor of UDP-glucuronyl transferase in maternal plasma that crosses the placenta. Jaundice is usually severe enough to require treatment

63. Which of the following values is the threshold critical value (alert or action level) for high plasma sodium? A. 150 mmol/L B. 160 mmol/L C. 170 mmol/L D. 180 mmol/L

63. B The adult reference range for plasma sodium is approximately 135-145 mmol/L. Levels in excess of 160 mmol/L are associated with severe dehydration, hypovolemia, and circulatory and heart failure. The threshold for the low critical value for sodium is 120 mmol/L. This is associated with edema, hypervolemia, and circulatory overload

63. Which of the following methods for HDL cholesterol is the reference method? A. Manganese-heparin B. Magnesium-phosphotungstate C. Magnesium-dextran D. Ultracentrifugation

63. D Ultracentrifugation of plasma in a potassium bromide solution with a density of 1.063 is used to separate HDL from LDL and VLDL. The HDL fraction is transferred from the bottom of the tube and assayed for cholesterol content by the Abell-Kendall method. The remaining three methods rely upon selective precipitation of lipoproteins containing apoprotein B using a polyanionic solution.

64. What is the purpose of an internal standard in HPLC and GC methods? A. To compensate for variation in extraction and injection B. To correct for background absorbance C. To compensate for changes in flow rate D. To correct for coelution of solutes

64. A Internal standards should have the same affinity as the analyte for the extraction reagents. Dividing peak height (or area) of all samples (standards and unknowns) by the peak height (or area) of the internal standard reduces error caused by variation in extraction recovery and injection volume.

60. Which electrolyte is least likely to be elevated in renal failure? A. Potassium B. Magnesium C. Inorganic phosphorus D. Sodium

60. D Reduced glomerular filtration coupled with decreased tubular secretion causes accumulation of potassium, magnesium, and inorganic phosphorus. Poor tubular reabsorption of sodium offsets reduced glomerular filtration. Unfiltered sodium draws both chloride and water, causing osmotic equilibration between filtrate, serum, and the tissues. In renal disease, serum sodium is often normal, although total body sodium is increased owing to fluid and salt retention.

61. Which statement regarding total and direct bilirubin levels is true? A. Total bilirubin level is a less sensitive and specific marker of liver disease than the direct level B. Direct bilirubin exceeds 3.5 mg/dL in most cases of hemolytic anemia C. Direct bilirubin is normal in cholestatic liver disease D. The ratio of direct to total bilirubin exceeds 0.40 in hemolytic anemia

61. A Direct bilirubin measurement is a sensitive and specific marker for hepatic and posthepatic jaundice because it is not elevated by hemolytic anemia

61. In which condition would an elevated serum alkaline phosphatase be likely to occur? A. Small cell lung carcinoma B. Hemolytic anemia C. Prostate cancer D. Acute myocardial infarction

61. A The primary diagnostic utility of ALP is to help differentiate necrotic jaundice (↑ALT) from obstructive jaundice (↑ ALP). ALP is also increased in several bone diseases. Large increases are seen in Paget's disease, moderate increases in bone cancer, and slight increases in rickets

61. Which of the following is the primary mechanism for vasopressin (ADH) release? A. Hypovolemia B. Hyperosmolar plasma C. Renin release D. Reduced renal blood flow

61. B ADH is released by the posterior pituitary in response to increased plasma osmolality. Normally, this is triggered by release of aldosterone caused by ineffective arterial pressure in the kidney. Aldosterone causes sodium reabsorption, which raises plasma osmolality; release of ADH causes reabsorption of water, which increases blood volume and restores normal osmolality

61. Select the reagent needed in the coupling enzyme reaction used to generate a colored product in the cholesterol oxidase method for cholesterol. A. Cholestahexaene B. H2O2 C. 4-Aminoantipyrine D. Cholest-4-ene-3-one

61. C In the cholesterol oxidase method, cholesterol ester hydrolase converts cholesterol esters to free cholesterol by hydrolyzing the fatty acid from the C3-OH group. Cholesterol oxidase catalyzes the oxidation of free cholesterol at the C3-OH group forming cholest-4-ene-3-one and hydrogen peroxide.

62. In thin-layer chromatography (TLC), the distance the solute migrates divided by the distance the solvent migrates is the: A. tR B. Kd C. Rf D. pK

62. C Rf is the distance migrated by the solute divided by the distance migrated by the solvent. The tR refers to the retention time of the solute in HPLC or gas chromatography (GC). The Kd is the partition coefficient, and is a measure of the relative affinity of solutes for the stationary phase. The solute with the greater Kd will be retained longer. The pK is the negative logarithm of K, the ionization constant, and is a measure of ionization

7. The phrase "first-pass hepatic metabolism" means that: A. One hundred percent of a drug is excreted by the liver B. All drug is inactivated by hepatic enzymes after one pass through the liver C. Some drug is metabolized from the portal circulation, reducing bioavailability D. The drug must be metabolized in the liver to an active form

7. C Drugs given orally enter the blood via the portal circulation and are transported directly to the liver. Some drugs are excreted by the liver, and a fraction will be lost by hepatic excretion before the drug reaches the general circulation.

70. A person has a fasting triglyceride level of 240 mg/dL. The physician wishes to know the patient's non-HDL cholesterol level. What cholesterol fractions should be measured? A. Total cholesterol and HDL cholesterol B. Total cholesterol and LDL cholesterol C. HDL cholesterol and LDL cholesterol D. Total cholesterol and chylomicrons

70. A When the HDL cholesterol is subtracted from the total cholesterol, the result is called the non-HDL cholesterol. This result, the sum of LDL cholesterol and VLDL cholesterol, represents the fraction with atherogenic remnant lipoproteins as well as LDL cholesterol

7. Which of the following statements about enzymatic reactions is true? A. NADH has absorbance maximas at 340 and 366 nm B. Enzyme concentration must be in excess to achieve zero-order kinetics C. Rate is proportional to substrate concentration in a zero-order reaction D. Accumulation of the product increases the reaction rate

7. A Most enzymes are measured by monitoring the rate of absorbance change at 340 nm as NADH is produced or consumed. This rate will be proportional to enzyme activity when substrate is in excess. When the enzyme is present in excess, the initial reaction rate will be proportional to substrate concentration. This condition, called a first-order reaction, is needed when the enzyme is used as a reagent to measure a specific analyte

7. In the Henderson-Hasselbalch expression pH = 6.1 + log HCO3 -/dCO2, the 6.1 represents: A. The combined hydration and dissociation constants for CO2 in blood at 37°C B. The solubility constant for CO2 gas C. The dissociation constant of H2O D. The ionization constant of sodium bicarbonate (NaHCO3) Chemistry/Apply

7. A The equilibrium constant, Kh, for the hydration of CO2 (dCO2 + H2O →H2CO3) is only about 2.3 × 10-3M, making dCO2 far more prevalent than carbonic acid. The dissociation constant, Kd, for the reaction H2CO3 →H+ + HCO3 - is about 2 × 10-4 M. The product of these constants is the combined equilibrium constant, K´. The negative logarithm of K´ is the pK´, which is 6.103 in blood at 37°C.

7. What substance may be measured as an alternative to creatinine for evaluating GFR? A. Plasma urea B. Cystatin C C. Uric acid D. Potassium

7. B Although all of the analytes listed are increased in chronic kidney disease as a result of low GFR, potassium, urea, and uric acid may be increased by other mechanisms and therefore, they are not specific for glomerular function. Cystatin C is an inhibitor of cysteine proteases

7. Which of the following tumor markers is used to monitor persons with breast cancer for recurrence of disease? A. Cathepsin-D B. CA-15-3 C. Retinoblastoma gene D. Estrogen receptor (ER)

7. B CA-15-3 shares the same antigenic determinant as CA-27.29. Both are present on MUC1, a mucinous protein on the cell membrane of various tissues. The markers are used to monitor treatment and recurrence of breast cancer. However, abnormal plasma levels are seen in many nonmalignant conditions, and the test is not used for diagnostic purposes. CA

7. When preparing a patient for an oral glucose tolerance test (OGTT), which of the following conditions will lead to erroneous results? A. The patient remains ambulatory for 3 days prior to the test B. Carbohydrate intake is restricted to below 150 g/day for 3 days prior to test C. No food, coffee, tea, or smoking is allowed 8 hours before and during the test D. Administration of 75 g of glucose is given to an adult patient following a 10-12-hour fast

7. B Standardized OGTTs require that patients receive at least 150 grams of carbohydrate per day for 3 days prior to the test in order to stabilize the synthesis of inducible glycolytic enzymes. The 2-hour OGTT test is no longer recommended for screening and should be reserved for confirmation of diabetes in cases that are difficult to diagnose, such as persons who lack symptoms and signs of fasting hyperglycemia

7. How many milliliters of a 2,000.0 mg/dL glucose stock solution are needed to prepare 100.0 mL of a 150.0 mg/dL glucose working standard? A. 1.5 mL B. 7.5 mL C. 15.0 mL D. 25.0 mL

7. B To calculate the volume of stock solution needed, divide the concentration of working standard by the concentration of stock standard, then multiply by the volume of working standard that is needed. C1 × V1 = C2 × V2, where C1 = concentration of stock standard V1 = volume of stock standard C2 = concentration of working standard V2 = volume of working standard 2000.0 mg/dL × V1 = 150.0 mg/dL × 100.0 mL V1 = (150.0 ÷ 2000.0) × 100.0 mL V1 = 7.5 mL

76. The reference method for serum lipase is based upon: A. Assay of triglycerides following incubation of serum with olive oil B. Rate turbidimetry C. Titration of fatty acids with dilute NaOH following controlled incubation of serum with olive oil D. Immunochemical assay

76. C The reference method of Cherry and Crandall is based upon the titration of fatty acids formed by the hydrolysis of an emulsion of olive oil after incubation for 24 hours at 37°C. Because most of the activity occurs within the first 3 hours, the incubation time may be shortened to as little as 1 hour without loss of clinical utility

(see pic) 77. Given the following real-time PCR amplification curve, what is the threshold cycle A. 15 B. 20 C. 25 D. 30

77. C The maximum curvature of the plot approximates the threshold cycle. A line is drawn from the threshold value on the y-axis through the curve, and a perpendicular dropped to the x-axis. The Ct is determined by the intersection point on the x-axis. The threshold is usually determined by an algorithm but can be calculated manually as 10 times the average standard deviation of the RFUs for cycles 2-10.

77. The most commonly employed method of assay for plasma or serum lipase is based on: A. Hydrolysis of olive oil B. Rate turbidimetry C. Immunoassay D. Peroxidase coupling

77. D Although all of the methods cited are available, the most commonly used method for lipase assay is based upon the hydrolysis of a synthetic diglyceride substrate yielding 2-monoglyceride. This is hydrolyzed and forms glycerol, which is phosphorylated and forms glycerol-3-phosphate. This is oxidized by glycerophosphate oxidase, yielding hydrogen peroxide.

79. Which enzyme is most likely to be elevated in the plasma of a person suffering from a muscle wasting disorder? A. 5´-Nucleotidase B. Pseudocholinesterase C. Aldolase D. Glutamate dehydrogenase

79. C 5'-Nucleotidase is increased primarily in obstructive liver disease and liver cancer. When elevated along with ALP, it identifies the liver as the source of ALP. Glutamate dehydrogenase is increased in necrotic liver diseases along with transaminases, but because of its distribution it is elevated to a greater extent in toxic hepatitis and therefore is useful as a marker for halothane (anesthesia) toxicity

8. Which formula can be used to estimate dosage needed to give a desired steady-state blood level? A. Dose per hour = clearance (milligrams per hour) × average concentration at steady state ÷ f B. Dose per day = fraction absorbed - fraction excreted C. Dose = fraction absorbed × (1/protein-bound fraction) D. Dose per day = half-life × log Vd (volume distribution

8. A After a patient receives a loading dose to rapidly bring the drug level up to the desired therapeutic range, a maintenance dose must be given at consistent intervals to maintain the blood drug level at the desired concentration. The dose per hour is determined by multiplying the clearance per hour by the desired average steady-state concentration, then dividing by f (bioavailable fraction

8. What is the pH of a solution of HNO3, if the hydrogen ion concentration is 2.5 × 10-2 M? A. 1.0 B. 1.6 C. 2.5 D. 2.8

8. B For a strong acid, the pH is equal to the negative logarithm of the hydrogen ion concentration. pH = -Log H+ pH = -Log 0.025 pH = 1.6

8. Which of the following statements regarding the Philadelphia chromosome is true? A. It is seen exclusively in chronic myelogenous leukemia B. It results from a translocation C. It appears as a short-arm deletion of chromosome 21 D. It is associated with a poor prognosis

8. B The Philadelphia chromosome (Ph1) is formed by translocation of the long arms of chromosomes 9 and 22. The result is that part of the ABL gene of chromosome 9 becomes inserted into the BCR gene of chromosome 22. The ABL gene is an oncogene and the product of the hybrid gene is a tyrosine kinase that signals cell proliferation

8. SITUATION: A patient who has a positive urinalysis for glucose and ketones has a glycated Hgb of 4.0%. A fasting glucose performed the previous day was 180 mg/dL. Assuming acceptable QC, you would: A. Report the glycosylated Hgb B. Request a new specimen and repeat the glycosylated Hgb C. Perform a Hgb electrophoresis on the sample D. Perform a glucose measurement on the sample

8. B The glycated Hgb is at the lowest normal limit (4%-5.5%), but the fasting glucose indicates frank diabetes mellitus. Although the glycosylated Hgb reflects the average blood glucose 2-3 months earlier, the value reported is inconsistent with the other laboratory results. A high probability of sample misidentification or analytical error necessitates that the test be repeated

8. Which of the following contributes the most to the serum total CO2? A. PCO2 B. dCO2 C. HCO3 - D. Carbonium ion

8. C The total CO2 is the sum of the dCO2, H2CO3 (carbonic acid or hydrated CO2), and bicarbonate (as mainly NaHCO3). When serum is used to measure total CO2, the dCO2 is insignificant because all the CO2 gas has escaped into the air. Therefore, serum total CO2 is equivalent to the bicarbonate concentration. Total CO2 is commonly measured by potentiometry. An organic acid is used to release CO2 gas from bicarbonate and pCO2 is measured with a Severinghaus electrode. A

80. When calibrating a semiautomatic pipet that has a fixed delivery of 10.0 μL using a gravimetric method, what should be the average weight of deionized water transferred? A. 10.0 μg B. 100.0 μg C. 1.0 mg D. 10.0 mg

80. D Gravimetric and spectrophotometric analysis are the two methods used to verify pipet volume accuracy and precision. Since spectrophotometric analysis involves dilution, gravimetric analysis is associated with greater certainty. At 20°C, the density of pure water is 0.99821 g/mL. Therefore, each microliter weighs almost exactly 1.0 mg.

72. Which of the following statements regarding amylase methods is true? A. Requires sulfhydryl compounds for full activity B. Activity will vary depending on the method used C. Amyloclastic methods measure the production of glucose D. Overrange samples are diluted in deionized water

72. B Chloride and Ca2+ ions are required for amylase activity. Samples with high activity should be diluted with NaCl to prevent inactivation. Lipase and CK require sulfhydryl activators. Saccharogenic methods measure the production of glucose, while amyloclastic methods measure the degradation of starch.

329. To maintain a pH of 7.4 in plasma, it is necessary to maintain a A. 10:1 ratio of bicarbonate to carbonic acid B. 20:1 ratio of bicarbonate to carbonic acid C. 1:20 ratio of bicarbonate to carbonic acid D. 20:1 ratio of carbonic acid to bicarbonate

B. 20:1 ratio of bicarbonate to carbonic acid

315. Which of the following chromogens will not produce a colored complex with iron that can be measured spectrophotometrically? A. Bathophenanthroline B. 8-Hydroxyquinoline C. Tripyridyltriazine D. Ferrozine

B. 8-Hydroxyquinoline

170. At what level should a 52-year-old male diagnosed with type 2 diabetes mellitus maintain his hemoglobin A]c? A. <3% B. <7% C. <9% D. <11%

B. <7%

185. What compound is a crucial intermediary in the metabolism of triglyceride to form energy? A. Bile B. Acetyl-coenzyme A C. Acetoacetate D. Pyruvate

B. Acetyl-coenzyme A

236. For assessing carcinoma of the prostate, quantification of PSA has virtually replaced the measurement of which of the following enzymes? A. Alkaline phosphatase B. Acid phosphatase C. Alanine aminotransferase D. Trypsin

B. Acid phosphatase

26. Mercury covered by a layer of mercurous chloride in contact with saturated potassium chloride solution is a description of which of the following types of electrodes? A. Sodium B. Calomel C. Calcium D. Silver/silver chloride

B. Calomel

25. In an electrolytic cell, which of the following is the half-cell where reduction takes place? A. Anode B. Cathode C. Combination electrode D. Electrode response

B. Cathode

192. What substance is the precursor to all steroid hormones? A. Fatty acid B. Cholesterol C. Triglyceride D. Phospholipid

B. Cholesterol

351. An elevated level of which of the following hormones will inhibit pituitary secretion of adrenocorticotropic hormone (ACTH)? A. Aldosterone B. Cortisol C. 17p-Estradiol D. Progesterone

B. Cortisol

122. Which of the following methods utilizes urease and glutamate dehydrogenase for the quantification of serum urea? A. Berthelot B. Coupled enzymatic C. Conductimetric D. Indicator dye

B. Coupled enzymatic

234. Which of the following enzymes would not be useful to quantify in the assessment of liver function? A. Alanine aminotransferase B. Creatine kinase C. Alkaline phosphatase D. Gamma-glutamyltransferase

B. Creatine kinase

152. The physician determined that the patient needed an oral glucose tolerance test (OGTT) to assist in diagnosis. The patient had blood drawn for the OGTT, and the following serum glucose results were obtained. These results are indicative of what state? Fasting serum glucose 124 mg/dL 2-hour postload serum glucose 227 mg/dL A. Normal B. Diabetes mellitus C. Addison disease D. Hyperinsulinism

B. Diabetes mellitus

34. The measurement of oxygen in blood by means of a PO2 electrode involves which of the following? A. Wheatstone bridge arrangement of resistive elements sensitive to oxygen concentration B. Direct relationship between amount of oxygen in the sample and amount of current flowing in the measuring system C. Change in current resulting from an increase of free silver ions in solution D. Glass electrode sensitive to H+ ions

B. Direct relationship between amount of oxygen in the sample and amount of current flowing in the measuring system

37. Which of the following methods allows for the separation of charged particles based on their rates of migration in an electric field? A. Rheophoresis B. Electrophoresis C. Electroendosmosis D. Ion exchange

B. Electrophoresis

333. What is the anticoagulant of choice for blood gas analysis? A. EDTA B. Heparin C. Sodium fluoride D. Citrate

B. Heparin

195. What is the sedimentation nomenclature associated with alpha-lipoprotein? A. Very-low-density lipoproteins (VLDLs) B. High-density lipoproteins (HDLs) C. Low-density lipoproteins (LDLs) D. Chylomicrons

B. High-density lipoproteins (HDLs)

388. Thyroid-releasing hormone (TRH) is given to a patient. Serum thyroid-stimulating hormone (TSH) levels are taken before and after the injection, and the values are the same—low. This patient probably has which of the following disorders? A. Primary hypothyroidism B. Secondary hypothyroidism C. Tertiary hypothyroidism D. Iodine deficiency

B. Secondary hypothyroidism

267. As a reduction product of bilirubin catabolism, this compound is partially reabsorbed from the intestine through the portal circulation for reexcretion by the liver. What is this compound? A. Verdohemoglobin B. Urobilinogen C. Urobilin D. Biliverdin

B. Urobilinogen

201. What is the principle of the "direct" or "homogeneous" HDL cholesterol automated method, which requires no intervention by the laboratorian? The direct HDL method A. Quantifies only the cholesterol in HDL, whereas the precipitation HDL method quantifies the entire lipoprotein B. Utilizes polymers and detergents that make the HDL cholesterol soluble while keeping the other lipoproteins insoluble C. Uses a nonenzymatic method to measure cholesterol, whereas the other methods use enzymes to measure cholesterol D. Uses a column chromatography step to separate HDL from the other lipoproteins, whereas the other methods use a precipitation step

B. Utilizes polymers and detergents that make the HDL cholesterol soluble while keeping the other lipoproteins insoluble

392. Which of the following is the Hollander insulin test used to confirm? A. Hyperglycemia B. Vagotomy C. Pancreatectomy D. Insulinoma

B. Vagotomy

436. Which vitamin is a constituent of two redox coenzymes? A. Vitamin A B. Vitamin B2 C. Vitamin B6 D. Vitamin C

B. Vitamin B2

429. In the United States, most cases of scurvy occur in children between the ages of 7 months to 2 years. Scurvy is a disease caused by a deficiency in which of the following? A. Vitamin A B. Vitamin C C. Vitamin D D. Vitamin K

B. Vitamin C

141) in which of the following disease states is conjugated bililrubin a major serum component? a. biliary obstruction b. hemolysis c. neonatal jaundice d. erythoblastosis fetalis

a. biliary obstruction

66) A hospitalized patient is experiencing increased neuromuscular irritability (tetany). Which of the following tests should be ordered immediately? a. calcium b. phosphate c. BUN d. glucose

a. calcium

22) An increase in serum acetone is indicative of a defect in the metabolism of: a. carbohydrates b. fats c. urea nitrogen d. uric acid

a. carbohydrates

56) A potassium level of 6.8 mEq/L (6.8 mmol/L) is obtained. Before reporting the results the first step the technologist should take is to: a. check the serum for hemolysis b. rerun the test c. check the age of the patient d. do nothing, simply report out the result

a. check the serum for hemolysis

322) Coulometry is often used to measure: a. chloride in sweat b. the pH in saliva c. bicarbonate in urine d. ammonia in plasma

a. chloride in sweat

(see pic) 91) refer to the following pattern. this pattern is consistent with a. cirrhosis b. acute inflammation c. polyclonal gammopathy (chronic inflammation) d. alpha 1 antitrypsin deficiency; severe emphysema

a. cirrhosis

321) An automated method for measuring chloride which generates silver ions in the reaction is: a. coulometry b. mass spectroscopy c. chromatography d. polarography

a. coulometry

(see pic) 182) A serum sample drawn in the emergency room from a 42-year-old man yielded the following laboratory results: Which of the following conditions might account for these values? a. crush injury to the thigh b. cerebrovascular accident c. pulmonary infarction d. early acute hepatitis

a. crush injury to the thigh

83) which of the following amino acids is associated with sulfhydryl group? a. cysteine b. glycine c. serine d. tyrosine

a. cysteine

282) Blood was collected in a serum separator tube on a patient who has been fasting since midnight. The time of collection was 7 am. The lab test which should be recollected is: a. triglycerides b. iron c. LD d. sodium

a. triglycerides

151) detection of carriers of hereditary coproporphyria should include an analysis of a. 24 hour urine for porphobilinogen b. fresh morning urine for delta aminoevulinic acid c. erythrocyte protoporphyrin d. 24 hour urine for porphyrin

b. fresh morning urine for delta aminoevulinic acid

333) which of the following would be an example of a glucose specific colorimetric method? a. alkaline ferricyanide b. glucose oxidase c. hexokinase d. o-toluidine

b. glucose oxidase

350) The substance that is measured to estimate the serum concentration of triglycerides by most methods is: a. phospholipids b. glycerol c. fatty acids d. pre-beta lipoprotein

b. glycerol

293) one international unit of enzyme activity is the amount of enzyme that will, under specified reaction conditions of substrate concentration, pH and temperature, cause utilization of substrate concentration, pH and temperature, cause utilization of substrate at the rate of a. 1 mol/min b. 1 mmol/min c. 1 umol/min d. 1 nmol/min

c. 1 umol/min

(see pic) 303) in the international system of units, serum urea is expressed in millimoles per liter. a serum urea nitrogen concentration of 28 mg/dl would be equivalent to what concentration of urea? a. 4.7 mEq/L b. 5.0 mEq/L c. 10.0 mEq/L d. 20 mEq/L

c. 10.0 mEq/L

239) the most common form (95%) of congenital adrenal hyperplasia is 21 hydroxylase deficiency, which is detected by elevated plasma a. cortisol b. aldosterone c. 17-OH progesterone d. 11-deoxycortisol

c. 17-OH progesterone

257) A drug has a half-life of 6 hrs. If a dose is given every 6 hrs., a steady state drug level would usually be achieved in: a. 3 - 5 hrs. b. 10 - 12 hrs. c. 24 - 42 hrs. d. 48 - 50 hrs.

c. 24 - 42 hrs.

(see pic) 300) and adult diabetic with renal complications has the following results. on the basis of these results, the calculated serum osmolality is a. 266 mOsm/kg b. 290 mOsm/kg c. 304 mOsm/kg d. 709 mOsm/kg

c. 304 mOsm/kg

(see pic) 304) the urea nitrogen concentration of a serum sample was measured to be 15 mg/dl. the urea concentration of the same sample, in mg/dl is a. 15 b. 24 c. 32 d. 40

c. 32

15) The glycated hemoglobin value represents the integrated values of glucose concentration during the preceding: a. 1-3 weeks b. 4-5 weeks c. 6-8 weeks d. 16-20 weeks

c. 6-8 weeks

32) The reference range for the pH of arterial blood measured at 37C is: a. 7.28-7.34 b. 7.33-7.37 c. 7.35-7.45 d. 7.45-7.50

c. 7.35-7.45

306) Normally the bicarbonate concentration is about 24 mEq/L and the carbonic acid concentration is about 1.2; pK=6.1, log 20 = 1.3. Using the equation pH = pK + log[salt]/[acid]. calculate the pH: a. 7.28 b. 7.38 c. 7.40 d. 7.42

c. 7.40

315) What is the proper pH for the buffered solution used to perform serum protein electrophoresis? a. 5.6 b. 7.6 c. 8.6 d. 9.6

c. 8.6

188) The most sensitive enzyme indicator for liver damage from ethanol intake is: a. ALT b. AST c. GGT d. alkaline phosphatase

c. GGT

131) On electrophoresis at alkaline pH, which of the following is the slowest migrating hemoglobin? a. Hgb A b. Hgb S c. Hgb C d. Hgb F

c. Hgb C

52) The anion gap is useful for quality control of laboratory results for: a. amino acids and proteins b. blood gas analyzers c. Na, K, Cl, and CO2 d. Ca, Ph, Mg

c. Na, K, Cl, and CO2

319) An electrode has a silver/silver chloride anode and a platinum wire cathode. It is suspended in KCl solution and separated from the blood to be analyzed by a selectively permeable membrane. Such an electrode is used to measure which of the following? a. pH b. PCO2 c. PO2 d. HCO3

c. PO2

296) in spectrophotometric determination, which of the following is the formula for calculating the absorbance of a solution? a. (absorptivity x light path) / concentration b. (absorptivity x concentration) / light path c. absorptivity x light path x concentration d. (light path x concentration) / absorptivity

c. absorptivity x light path x concentration

191) Which of the following chemical determinations may be of help in establishing the presence of seminal fluid? a. LD b. ICD (isocitrate dehydrogenase) c. acid phosphatase d. alkaline phosphatase

c. acid phosphatase

177) A 10 yr. old child was admitted to pediatrics with an initial diagnosis of skeletal muscle disease. The best confirmatory tests would be: a. creatine kinase and isocitrate dehydrogenase b. gamma-glutamyl transferase and alkaline phosphatase c. aldolase and creatine kinase d. lactate dehydrogenase and malate dehydrogenase

c. aldolase and creatine kinase

2) (see pic) the following results are from a 21 year old patient with a back injury who appears otherwise healthy. the best interpretation of these results is that a. the whole blood and serum values are expected but the CSF value is elevated b. the whole blood glucose value should be higher than the serum value c. all values are consistent with a normal healthy individual d. the serum and whole blood values should be identical

c. all values are consistent with a normal healthy individual

192) which of the following enzyme substrates for prostatic acid phosphatase is best for continuous monitoring method? a. phenyl phosphate b. thymolphtalein monophosphate c. alpha napthyl phosphate d. beta glycerophosphate

c. alpha napthyl phosphate

119) the presence of c reactive protein in the blood is an indication of a. recent streptococcal infeciton b. recovery from a pneumococcal infection c. an inflammatory process d. a state of hypersensitivity

c. an inflammatory process

255) In addition to carcinoma of the prostate, elevated prostate-specific antigen can occur due to: a. aspirin therapy b. exogenous steroid use c. benign prostatic hyperplasia d. stain therapy (cholesterol lowering drug)

c. benign prostatic hyperplasia

39) Acidosis and alkalosis are best defined as fluctuations in blood pH and CO2 content due to changes in: a. Bohr effect b. O2 content c. bicarbonate buffer d. carbonic anhydrase

c. bicarbonate buffer

50) Most of the carbon dioxide present in blood is in the form of: a. dissolved CO2 b. carbonate c. bicarbonate ion d. carbonic acid

c. bicarbonate ion

46) The most important buffer pair in plasma is the: a. phosphate/biphosphate b. hemoglobin/imidazole c. bicarbonate/carbonic acid d. sulfate/bisulfate

c. bicarbonate/carbonic acid

154) serum haptoglobin a. is decreased in patients with tissue injury and neoplasia b. is increased in patients with prosthetic heart valves c. can be separated into distinct phenotypes by starch gel electrophoresis d. binds heme

c. can be separated into distinct phenotypes by starch gel electrophoresis

193) lactate dehydrogenase, malate dehydrogenase, isocitrate dehydrogenase, and hydroxybutyrate dehydrogenase all a. are liver enzymes b. are cardiac enzymes c. catalyze oxidation reduction reactions d. are class III enzymes

c. catalyze oxidation reduction reactions

125) serum concentrations of vitamin B12 are elevated in a. pernicious anemia in relapse b. patients on chronic hemodialysis c. chornic granulocytic leukemia d. hodgkin disease

c. chornic granulocytic leukemia

210) Turbidity in serum suggests elevation of: a. cholesterol b. total protein c. chylomicrons d. albumin

c. chylomicrons

244) Which of the following sample collections would give an accurate assessment of potential excess cortisol production? a. collect a plasma sample as a baseline, and another one 1 hr. after administration of metyrapone b. collect a plasma sample at 8 am only c. collect a 24 hr. urine free cortisol d. collect a plasma sample at 8 am and at 8 am the next day

c. collect a 24 hr. urine free cortisol

(see pic) 145) biochemical profile. the results the biochemical profile are most consistent with a. viral hepatitis b. hemolytic anemia c. common bile duct stone d. chronic active hepatitis

c. common bile duct stone

288) in monitoring glomerular function, which of the following tests has the highest sensitivity? a. urine sodium b. BUN/creatinine ratio c. creatinine clearance d. urea clearance

c. creatinine clearance

208) Chylomicrons are present in which of the following dyslipidemias? a. familial hypercholesterolemia b. hypertriglyceridemia c. deficiency in lipoprotein lipase activity d. familial hypoalphalipoproteinemia

c. deficiency in lipoprotein lipase activity

28) The following blood gas results were obtained: The patient's results are compatible with which of the following? a. fever b. uremia c. emphysema d. dehydration

c. emphysema

94) at a pH of 8.6 the gamma globulins move toward the cathode despite the fact that they are negatively charged. what is this phenomenon called? a. reverse migration b. molecular sieve c. endosmosis d. migratory inhibition factor

c. endosmosis

313) When separating serum proteins by cellulose acetate electrophoresis, using Veronal buffer at pH 8.6. beta globulin migrates: a. faster than albumin b. slower than gamma globulins c. faster than gamma globulin d. faster than alpha-2 globulin

c. faster than gamma globulin

311) To detect barbiturate abuse when analyzing urine specimens, immunoassay is the method of choice for screening. The method of choice for confirmation is: a. nephelometry b. thin layer chromatography c. gas chromatography/mass spectrometry d. ultraviolet absorption spectroscopy

c. gas chromatography/mass spectrometry

334) increased concentrations of ascorbic acid inhibit chromogen production in which of the following glucose methods? a. ferricyanide b. ortho-toluidine c. glucose oxidase (peroxidase) d. hexokinase

c. glucose oxidase (peroxidase)

150) the reason carbon monoxide is so toxic is because it a. is a protoplasmic poison b. combines with cytochrom oxide c. has 200 times the affinity of oxygen for hemoglobin binding sites d. sensitizes the myocardium

c. has 200 times the affinity of oxygen for hemoglobin binding sites

98) the identification of bence jones proteins is best accomplished by a. a sulfosalicyclic acid test b. urine reagent strips c. immunofixation d. electrophoresis

c. immunofixation

240) A diagnosis of primary adrenal insufficiency requires demonstration of: a. decreased urinary 17-keto and 17-hydroxysteroids b. decreased cortisol production c. impaired response to ACTH stimulation d. increased urinary cortisol excretion after metyrapone

c. impaired response to ACTH stimulation

(see pic) 17) a patient with type I, insulin-dependent diabetes mellitus has the following results. after reviewing these test results, the technologist concluded that the patient his in a a. steady state of metabolic control b. state the flux, progressively worsening metabolic control c. improving state of metabolic control as indicated by fructosamine d. state of flux as indicated by the fasting glucose level

c. improving state of metabolic control as indicated by fructosamine

81) the relative migration rate of proteins on cellulose acetate is based on a. molecular weight b. concentration c. ionic charge d. particle size

c. ionic charge

265) A carbonate salt used to control manic depressive disorders is: a. digoxin b. acetaminophen c. lithium d. phenytoin

c. lithium

199) premature atherosclerosis can occur when which of the following becomes elevated? a. chylomicrons b. prostaglandins c. low density lipoproteins d. high density lipoproteins

c. low density lipoproteins

200) transportation of 60-75% of the plasma cholesterol is performed by a. chylomicrons b. very low density lipoproteins c. low density lipoproteins d. high density lipoproteins

c. low density lipoproteins

69) Fasting serum phosphate concentration is controlled primarily by the: a. pancreas b. skeleton c. parathyroid glands d. small intestine

c. parathyroid glands

(see pic) 133) The following bilirubin results are obtained on a patient: Given that the controls were within range each day, what is a probable explanation for the result on day 4? a. no explanation necessary b. serum, not plasma, was used for testing c. specimen had prolonged exposure to light d. specimen was hemolyzed

c. specimen had prolonged exposure to light

147) what substance gives feces its normal color? a. uroerythrin b. urochrome c. urobilin d. urobilinogen

c. urobilin

280) In a specimen collected for plasma glucose analysis, sodium fluoride: a) serves as a coenzyme of hexokinase b) prevents reactivity of non-glucose reducing substances c) precipitates proteins d) inhibits glycolysis

d) inhibits glycolysis

263) A 3-year-old child was evaluated for abdominal pain and anorexia by a physician. A CBC revealed a hemoglobin of 9.8 g/dL and basophilic stippling of the RBCs. The doctor should order further tests to check for poisoning from: a) arsenic b) iron c) mercury d) lead

d) lead

254) Cancer antigen 125 (CA 125) is a tumor marker associated with: a) breast carcinoma b) colon cancer c) lung cancer d) ovarian and endometrial carcinoma

d) ovarian and endometrial carcinoma

(see pic) 71) The following laboratory results were obtained: Calcium Alk Phosphate Alk Phosphatase Serum: increased decreased normal/increased urine: increased increased The results are most compatible with: a) multiple myeloma b) milk-alkali syndrome c) sarcoidosis d) primary hyperparathyroidism

d) primary hyperparathyroidism

219) Which one of the following sets of results is consistent with primary hypothyroidism, (eg. Hashimoto's thyroiditis): Result TSH T4 Antimicrosomal Ab result A decreased decreased positive result B increased increased positive result C normal decreased negative result D increased decreased positive a) result A b) result B c) result C d) result D

d) result D

297) Which of the following is the formula for calculating absorbance given the percent transmittance of a solution? a. 1 - log(%T) b. log(%T)/2 c. 2 x log(%T) d. 2 - log(%T)

d. 2 - log(%T)

8) which of the following 2 hour postprandial glucose values demonstrates unequivocal hyperglycemia diagnostic for diabetes mellitus? a. 160 mg/dl (8.8 mmol/l) b. 170 mg/dl (9.4 mmol/l) c. 180 mg/dl (9.9 mmol/l) d. 200 mg/dl (11.0 mmol/l)

d. 200 mg/dl (11.0 mmol/l)

(see pic) 302) the following results were obtained in a creatinine clearance evaluation. the creatinine clearance in ml/min is a. 6 b. 22 c. 60 d. 65

d. 65

19) which of the following hemoglobins has glucose-6-phosphate on the amino-terminal valine of the beta chain? a. S b. C c. A2 d. A1c

d. A1c

110) a patient with glomerulonephritis is most likely to present with the following serum results a. creatinine decreased b. calcium increased c. phosphorous decreased d. BUN increased

d. BUN increased

318) which of the following serum proteins migrate with the beta globulins on cellulose acetate at pH 8.6? a. ceruloplasmin b. hemoglobin c. haptoglobin d. C3 component of complement

d. C3 component of complement

35) A person suspected of having metabolic alkalosis would have which of the following laboratory findings? a. CO2 content elevated and PCO2 elevated, pH decreased b. CO2 content decreased and pH elevated c. CO2 content decreased, PCO2 decreased and pH decreased d. CO2 content elevated and pH elevated

d. CO2 content elevated and pH elevated

234) Rickets is associated with deficiency of which of the following vitamins? a. B1 b. C c. niacin d. D

d. D

194) high levels of which lipprotein class are associated with decrease risk of accelerated atherosclerosis a. chylomicrons b. VLDL c. LDL d. HDL

d. HDL

21) In using ion-exchange chromatographic methods, falsely increased levels of Hgb A1c might be demonstrated in the presence of: a. iron deficiency anemia b. pernicious anemia c. thalassemias d. Hgb S

d. Hgb S

338) The most widely used methods for bilirubin measurement are those based on the: a. Jaffe reaction b. Schales and Schales method c. 8-hydroxyquinoline reaction d. Jendrassik-Grof method

d. Jendrassik-Grof method

176) increased total serum lactic dehydrogenase (LD) activity, confined to fractions 4 and 5 is most likely to be associated with a. pulmonary infarction b. hemolytic anemia c. myocardial infarction d. acute viral hepatitis

d. acute viral hepatitis

112) in the jaffe reaction, creatinine reacts with a. alkaline sulfasalazine solution to produce an orange yellow complex b. potassium iodide to form a reddish purple complex c. sodium nitroferricyanide to yield a reddish brow color d. alkaline picrate solution to yield an orange red complex

d. alkaline picrate solution to yield an orange red complex

(see pic) 89) refer to the following illustration. this electrophoresis pattern is consistent with a. cirrhosis b. monoclonal gammopathy c. polyclonal gammopathy (eg. chronic inflammation) d. alpha 1 antitrypsin deficiency; severe emphysema

d. alpha 1 antitrypsin deficiency; severe emphysema

127) the procedure used to determine the presence of neural tube defects is a. lecithin/sphingomyelin ratio b. amniotic fluid creatinine c. measurement of absorbance at 450 nm d. alpha fetoprotein

d. alpha fetoprotein

159) A physician suspects his patient has pancreatitis. Which test would be most indicative of this disease? a. creatinine b. LD isoenzymes c. beta-hydroxybutyrate d. amylase

d. amylase

246) Aldosterone is released by the adrenal cortex upon stimulation by: a. renin b. angiotensinogen c. angiotensin I d. angiotensin II

d. angiotensin II

336) Which of the following calcium procedures utilizes lanthanum chloride to eliminate interfering substances? a. o-cresolphthalein complexone b. precipitation with chloranilic acid c. chelation with EDTA d. atomic absorption spectrophotometry

d. atomic absorption spectrophotometry

261) Cocaine is metabolized to: a. carbamazepine b. codeine c. hydrocodone d. benzolylecgonine

d. benzolylecgonine

140) in the liver, bilirubin is converted to a. urobilinogen b. urobilin c. bilirubin albumin complex d. bilirubin diglucuronide

d. bilirubin diglucuronide

329) Most chemical methods for determining total protein utilize which of the following reactions? a. molybdenum blue b. ferri-ferrocyanide c. resorcinol-HCL d. biuret

d. biuret

58) A sweat chloride result of 55 mEq/L (55 mmmol/L) and a sweat sodium of 52 mEq/L (52 mmol/L) were obtained on a patient who has a history of respiratory problems. The best interpretation of these results is: a. normal b. normal sodium and an abnormal chloride test should be repeated c. abnormal results d. borderline results, the test should be repeated

d. borderline results, the test should be repeated

(see pic) 144) a 21 year old man with nausea, vomitting, and jaundice has the following lab findings. these can best be explained as representing a. unconjugated hyperbilirubinemia, probably due to hemolysis b. unconjugated hyperbilirubinemia, probably due to toxic liver damage c. conjugated hyperbilirubinemia, probable due to biliary tract disease d. conjugated hyperbilirubinemia probably due to hepatocellular obstruction

d. conjugated hyperbilirubinemia probably due to hepatocellular obstruction

202) A 1 yr. old girl with a hyperlipoproteinemia and lipase deficiency has the following lipid profile: cholesterol = 300 mg/dL LDL increased HDL decreased triglycerides = 200 mg/dL chylomicrons present A serum specimen from this patient that was refrigerated overnight would most likely be: a. clear b. cloudy c. creamy layer over cloudy serum d. creamy layer over clear serum

d. creamy layer over clear serum

167) The enzyme which exists chiefly in skeletal muscle, heart, and brain, is grossly elevated in active muscular dystrophy, and rises early in myocardial infarction is: a. lipase b. transaminase c. lactate dehydrogenase d. creatine kinase

d. creatine kinase

143) in which of the following conditions does decreased activity of glucuronyl transferase result in increased unconjugated bilirubin and kernicterus in neonates? a. gilbert disease b. rotor syndrome c. dubin johnson syndrome d. crigler najjar syndrome

d. crigler najjar syndrome

36) Metabolic acidosis is described as a: a. increase in CO2 content and PCO2 with a decrease pH b. decrease in CO2 content with an increased pH c. increase in CO2 with an increased pH d. decrease in CO2 content and PCO2 with a decreased pH

d. decrease in CO2 content and PCO2 with a decreased pH

85) increased serum albumin concentrations are seen in which of the following conditions? a. nephrotic syndrome b. acute hepatitis c. chronic inflammation d. dehydration

d. dehydration

339) In the Malloy and Evelyn method for the determination of bilirubin: the reagent that is reacted with bilirubin to form a purple azobilirubin is: a. dilute sulfuric acid b. diazonium sulfate c. sulfobromophthalein d. diazotized sulfanilic acid

d. diazotized sulfanilic acid

24) Blood samples were collected at the beginning of an exercise class and after thirty mins. of aerobic activity. Which of the following would be most consistent with the post exercise sample? a. normal lactic acid, low pyruvate b. low lactic acid, elevated pyruvate c. elevated lactic acid, low pyruvate d. elevate lactic acid, elevated pyruvate

d. elevate lactic acid, elevated pyruvate

209) the funciton of the major lipid components of the very low density lipoproteins (VLDL) is to transport a. cholesterol form peripheral cells to the liver b. cholesterol and phospholipids to peripheral cells c. exogenous triglycerides d. endogenous triglycerides

d. endogenous triglycerides

352) A chemiluminescent EIA: a. measures absorption of light b. is less sensitive than radioisotopic reactions c. is monitored by the use of a gamma counter d. is quantitated by the amount of light produced by the reactions

d. is quantitated by the amount of light produced by the reactions

168) The enzyme present in almost all tissues that may be separated by electrophoresis into 5 components is: a. lipase b. transaminase c. creatine kinase d. lactate dehydrogenase

d. lactate dehydrogenase

51) Serum anion gap is increased in patients with: a. renal tubular acidosis b. diabetes alkalosis c. metabolic acidosis due to diarrhea d. lactic acidosis

d. lactic acidosis

1) Following overnight fasting, hypoglycemia in non-diabetic adults is defined as a glucose of: a. less than 70 mg/dl (less than 3.9 mmol/l) b. less than 60 mg/dl (less than 3.3 mmol/l) c. less than 55 mg/dl (less than 3.0 mmol/l) d. less than 45 mg/dl (less than 2.5 mmol/l)

d. less than 45 mg/dl (less than 2.5 mmol/l)

183. Given the following results: alkaline phosphatase: marked increase aspartate amino transferase: slight increase alanine amino transferase: slight increase gamma-glutamyl transferase: marked increase this is most consistent with. a. acute hepatitis b. osteitis fibrosa c. chronic hepatitis d. obstructive jaundice

d. obstructive jaundice

298) Which of the following is the Henderson Hasselbach equation? a. pKa=pH+log([acid]/[salt]) b. pKa=pH+log([salt]/[acid]) c. pH=pKa+log([acid]/[salt]) d. pH=pKa+log([salt]/[acid])

d. pH=pKa+log([salt]/[acid])

106) Erroneous ammonia levels can be eliminated by all of the following except: a. assuring water and reagents are ammonia-free b. separating plasma from cells and performing test analysis as soon as possible c. drawing the specimen in a pre-chilled tube and immersing the tube in ice d. storing the specimen protected from light until the analysis is done

d. storing the specimen protected from light until the analysis is done

229) the biologically most active, naturally occurring androgen is a. androstenedione b. cortisol c. epiandrosterone d. testosterone

d. testosterone

101) a patient's blood was drawn at 8 am for a serum iron determination. the result was 85 ug/dl (15.2 mmol/l). a repeat specmiman was drawn at 8 pm; the serum was stored at 4C and run the next morning. the result was 40 ug/dl (7.2 mmol/l). these results are most likley due to a. iron deficiency anemia b. improper storage of the specimen c. possible liver damage d. the time of day teh second specimen was drawn

d. the time of day teh second specimen was drawn

232) Beriberi is associated with deficiency of vitamin: a. A b. C c. niacin d. thiamine

d. thiamine

308) in the atomic absorption method for calcium, lanthanum is used a. as an internal standard b. to bind calcium c. to eliminate protein interference d. to prevent phosphate interference

d. to prevent phosphate interference

(see pic) 180) Refer to the following illustration: Which of the following is the most likely interpretation of the LD isoenzyme scan illustrated above? a. myocardial infarction b. megaloblastic anemia c. acute pancreatitis d. viral hepatitis

d. viral hepatitis

362. Which of the following is the most potent estrogen and is considered to be the true ovarian hormone? A. Estriol (E3) B. Estrone (Ej) C. 17p-Estradiol (E2) D. 16a-Hydroxyestrone

C. 17p-Estradiol (E2)

169. What is the reference interval for fasting serum glucose in an adult expressed in SI units (International System of Units)? A. 1.7-3.3 mmol/L B. 3.3-5.6 mmol/L C. 4.1-5.5 mmol/L D. 6.7-8.3 mmol/L

C. 4.1-5.5 mmol/L

301. Of the total serum calcium, free ionized calcium normally represents approximately what percent? A. 10 B. 40 C. 50 D. 90

C. 50

9. The bandpass of a spectrophotometer is 10 nm. If an instalment is set at 540 nm, the wavelengths that are permitted to impinge on the sample will be within what wavelength range? A. 530-540 nm B. 530-550 nm C. 535-545 nm D. 540-550 nm

C. 535-545 nm

135. When it is not possible to perform a creatinine assay on a fresh urine specimen, to what pH level should the urine be adjusted? A. 3.0 B. 5.0 C. 7.0 D. 9.0

C. 7.0

387. In a patient suspected of having primary myxedema, one would expect the following serum results: free thyroxine (FT4) ______ thyroid hormone binding ratio (THBR) ______ and thyroid-stimulating hormone (TSH) ______ A. Decreased, increased, decreased B. Increased, increased, decreased C. Decreased, decreased, increased D. Increased, decreased, increased

C. Decreased, decreased, increased

269. Which bilirubin fraction is unconjugated and covalently bound to albumin? A. Alpha B. Beta C. Delta D. Gamma

C. Delta

105. Serum protein electrophoresis is routinely performed on the serum obtained from a clotted blood specimen. If a plasma specimen is substituted for serum, how will the electrophoresis be affected? A. Electrophoresis cannot be performed because the anticoagulant will retard the mobilities of the protein fractions. B. Electrophoresis cannot be performed because the anticoagulant will cause migration of the protein fractions in the direction of the cathode. C. Electrophoresis will show an extra fraction in the beta-gamma region. D. Electrophoresis will show an extra fraction in the prealbumin area.

C. Electrophoresis will show an extra fraction in the beta-gamma region.

99. The physician is concerned that a pregnant patient may be at risk for delivering prematurely. What would be the best biochemical marker to measure to assess the situation? A. Inhibin A B. ctpFetoprotein C. Fetal fibronectin D. Human chorionic gonadotropin

C. Fetal fibronectin

200. Name a commonly used precipitating reagent to separate HDL cholesterol from other lipoprotein cholesterol fractions. A. Zinc sulfate B. Trichloroacetic acid C. Heparin-manganese D. Isopropanol

C. Heparin-manganese

164. Which glucose method catalyzes the phosphorylation of glucose by adenosine triphosphate, forming glucose-6- phosphate and adenosine diphosphate with the absorbance of the NADPH product read at 340 nm? A. o-Toluidine B. Glucose oxidase C. Hexokinase D. Glucose dehydrogenase

C. Hexokinase

284. Which of the following is not characteristic of Dubin-Johnson syndrome? A. Impaired excretion of bilirubin into the bile B. Hepatic uptake of bilirubin is normal C. Inability to conjugate bilirubin D. Increased level of bilirubin in urine

C. Inability to conjugate bilirubin

341. Which is a compensatory mechanism in respiratory acidosis? A. Hypoventilation B. Decreased reabsorption of bicarbonate by the kidneys C. Increased Na+/H+ exchange by the kidneys D. Decreased ammonia formation by the kidneys

C. Increased Na+/H+ exchange by the kidneys

47. Which of the following is an electrophoretic technique employing a pH gradient that separates molecules with similar isoelectric points? A. Zone electrophoresis B. High-resolution electrophoresis C. Isoelectric focusing D. Immunoelectrophoresis

C. Isoelectric focusing

253. Which of the following enzyme activities can be determined by using a dilute olive oil emulsion substrate, whose hydrolyzed product is monitored as a decrease in turbidity or light scatter? A. Alkaline phosphatase B. Amylase C. Lipase D. Trypsin

C. Lipase

325. Which of the following may be associated with the colloid osmotic pressure (COP) osmometer? A. Utilizes a cooling bath set at -7°C B. Measures total serum osmolality C. Negative pressure on reference (saline) side equivalent to COP of sample D. Measures contribution of electrolytes to osmolality

C. Negative pressure on reference (saline) side equivalent to COP of sample

280. Which of the following laboratory results is not characteristic of a complete obstruction of the common bile duct? A. Negative urine urobilinogen B. Negative fecal urobilinogen and urobilin C. Negative urine bilirubin D. Excretion of a pale-colored stool

C. Negative urine bilirubin

421. In regard to drug distribution patterns, which of the following statements is false? A. Drug metabolism is slower in newborns than adults. B. Drug metabolism is more rapid for 6-year-old children than for adults. C. Renal clearance of drugs is faster in newborns than adults. D. Drug metabolism often changes during pubescence.

C. Renal clearance of drugs is faster in newborns than adults

230. Which of the following statements concerning creatine kinase is false? A. Rises within 4-6 hours after acute myocardial infarction B. Catalyzes the phosphorylation of creatine by ATP C. Requires Ca2+ for activity D. Found mainly in skeletal and cardiac muscles and in brain tissue

C. Requires Ca2+ for activity

407. For what colorimetric determination is the Trinder reaction widely used? A. Acetaminophen B. Propoxyphene C. Salicylate D. Barbiturate

C. Salicylate

19. Which of the following instruments is used in the clinical laboratory or in reference laboratories to detect beta and gamma emissions? A. Fluorometer B. Nephelometer C. Scintillation counter D. Spectrophotometer

C. Scintillation counter

377. Of which of the following is 5-hydroxyindoleacetic acid (5-HIAA) the primary metabolite? A. Epinephrine B. Norepinephrine C. Serotonin D. Prolactin

C. Serotonin

420. When is a blood sample for determination of the trough level of a drug appropriately drawn? A. During the absorption phase of the drug B. During the distribution phase of the drug C. Shortly before drug administration D. Two hours after drug administration

C. Shortly before drug administration

296. Which of the following statements is false about the electrolyte chloride? A. Main anion of the extracellular fluid B. Can shift from the extracellular plasma to the intracellular fluid of red blood cells C. Unable to be reabsorbed by active transport D. Measured in serum, urine, and sweat

C. Unable to be reabsorbed by active transport

357. What does the concentration of urinary free cortisol mainly reflect? A. Total serum cortisol B. Conjugated cortisol C. Unbound serum cortisol D. Protein-bound serum cortisol

C. Unbound serum cortisol

262. What enzyme catalyzes the conjugation of bilirubin? A. Leucine aminopeptidase B. Glucose-6-phosphate dehydrogenase C. Uridine diphosphate glucuronyltransferase D. Carbamoyl phosphate synthetase

C. Uridine diphosphate glucuronyltransferase

251. Which statement best describes the clinical use of measuring NT-proBNP? A. Used to assess risk of coronary heart disease B. Used to assess risk of angina C. Used to assess individuals treated with nesiritide D. Used to assess individuals treated with vitamin B

C. Used to assess individuals treated with nesiritide

226. Which test, if elevated, would provide information about risk for developing coronary artery disease? A. Troponin B. CK-MB C. hs-CRP D. Myoglobin

C. hs-CRP

35. Which of the following blood gas parameters are measured directly by the blood gas analyzer electrochemically as opposed to being calculated by the instrument? A. pH, HCC-3, total CO2 B. PCO2, HCOJ, PO2 C. pH, PCO2, K)2 D. PO2, HCOf, total CO2

C. pH, PCO2, K)2

288. A 42-year-old woman is admitted to the hospital with complaints of abdominal pain and inability to eat, which have gotten worse during the past several weeks. Although the pain had been uncomfortable, what alarmed her was noticing a slight yellow color in her eyes. Blood was drawn and the test results follow: total bilirubin 3.9 mg/dL, direct bilirubin 2.7 mg/dL, AST slightly elevated (3 times the upper limit of the reference range), ALT slightly elevated (3 times the upper limit of the reference range), alkaline phosphatase markedly elevated (6 times the upper limit of the reference range), and urine urobilinogen decreased. What diagnosis do these test results support? A. Viral hepatitis B. Cirrhosis C. Exposure to toxic chemicals D. Biliary obstruction

D. Biliary obstruction

115. Which of the following is not associated with human chorionic gonadotropin? A. (3 subunit confers immunogenic specificity B. Used to confirm pregnancy C. Used as a tumor marker D. Found in hepatoma

D. Found in hepatoma

12. Which of the following is not a problem inherent in turbidimetry? A. Variation in particle size of samples B. Variation in particle size of standards C. Rate of aggregation or settling of particles D. Need to maintain a constant and specific temperature

D. Need to maintain a constant and specific temperature

285. Which of the following disorders is not a form of hepatic jaundice? A. Cirrhosis B. Crigler-Najjar syndrome C. Hepatitis D. Neoplasm of common bile duct

D. Neoplasm of common bile duct

1 1 1 . Which of the following is false about PSA? A. Serum quantified using immunoassays B. Single-chain glycoprotein C. Used as a tumor marker D. Not elevated in benign prostatic hyperplasia

D. Not elevated in benign prostatic hyperplasia

277. What condition is characterized by an elevation of total bilirubin primarily due to an increase in the conjugated bilirubin fraction? A. Hemolyticjaundice B. Neonatal jaundice C. Crigler-Najjar syndrome D. Obstructive jaundice

D. Obstructive jaundice

208. Your lab routinely uses a precipitation method to separate HDL cholesterol. You receive a slightly lipemic specimen for HDL cholesterol. The total cholesterol and triglyceride for the specimen were 450 and 520 mg/dL, respectively. After adding the precipitating reagents and centrifuging, you notice that the supernatant still looks slightly cloudy. What is your next course of action in analyzing this specimen? A. Perform the HDL cholesterol test; there is nothing wrong with this specimen. B. Take off the supernatant and recentrifuge. C. Take off the supernatant and add another portion of the precipitating reagent to it and recentrifuge. D. Send specimen to a lab that offers other techniques to separate more effectively the HDL cholesterol.

D. Send specimen to a lab that offers other techniques to separate more effectively the HDL cholesterol.


Related study sets

Bio201 4 review questions & Critical thinking

View Set

The Biliary System Review Packet

View Set

Ch 39: Rehabilitation Nursing, Ch 38: Long-Term Care

View Set